Download as pdf or txt
Download as pdf or txt
You are on page 1of 470

PLAB 2 NOTES

ASPIRE: THE QUEST FOR EXCELLENCE


Disclaimer

This book is published and distributed solely by Aspire Education and prepared by various
authors working for Aspire Education. However, the publisher and the authors are providing
this book and its contents on an “as is” basis and make no representations or warranties of
any kind with respect to this book or its contents. The publisher and the authors disclaim all
such representations and warranties, including but not limited to warranties of healthcare for
a particular purpose and PLAB 2 exam preparatory course material. In addition, the publisher
and the author assume no responsibility for errors, inaccuracies, omissions, similarities or any
other inconsistencies herein.

The contents of this book are for informational purposes only to provide guidance for
preparation of PLAB 2 exam in the UK. The content is also not intended to diagnose, treat,
cure, or prevent any condition or disease. The publisher and the author make no guarantees
concerning the level of success you may experience by following the advice and strategies
contained in this book, and you accept the risk that results will differ for each individual. The
similarities of any content / scenario (s) will be totally coincidental. Aspire Education or any
organisation working in collaboration with Aspire will not be held responsible for the content
of this book.

The use of this book implies your acceptance of this disclaimer.

www.aspire2plab.com 2
No real PLAB2 cases discussed. All discussed scenarios are fictitious and for educational purposes only.
Table of Contents
Paediatrics......................................................................................................................... 7
History ....................................................................................................................................... 7
Immunisation Schedule ............................................................................................................. 8
Otitis Media child ........................................................................................................................................ 10
Eye Infection in Neonate (Chlamydia) ........................................................................................................ 12
Febrile Convulsion....................................................................................................................................... 16
Head Injury.................................................................................................................................................. 19
Non-Accidental Injury ................................................................................................................................. 22
Intussusception ........................................................................................................................................... 25
Pyloric Stenosis ........................................................................................................................................... 28
Constipation in Child ................................................................................................................................... 31
Neonatal Jaundice....................................................................................................................................... 33
Bronchiolitis ................................................................................................................................................ 36
Paediatrics – Telephone Conversation........................................................................................................ 39
Child Development Milestone .................................................................................................................... 41
Twin Delay................................................................................................................................................... 47
Child Waking up at Night (Night Terrors) .................................................................................................... 49
Autism ......................................................................................................................................................... 51
Temper Tantrums ....................................................................................................................................... 54
Negativism .................................................................................................................................................. 57
Primary Enuresis ......................................................................................................................................... 63
Cystic Fibrosis .............................................................................................................................................. 65
Haemangioma (Birth Mark) ........................................................................................................................ 68
UTI in Female Child ..................................................................................................................................... 70
Meconium Ileus .......................................................................................................................................... 71
Counselling ...............................................................................................................................72
MMR ........................................................................................................................................................... 72
Vaccination (Flu Jab) ................................................................................................................................... 75
8 Weeks Vaccination ................................................................................................................................... 77
Prescribing IV Fluid in a Child ...................................................................................................................... 78
Epilepsy ....................................................................................................................................................... 82

Obstetrics & Gynaecology ................................................................................................ 85


Pre-eclampsia.............................................................................................................................................. 86
PID (RIF Pain) ............................................................................................................................................... 89
PID (Lower Abdominal Pain) ....................................................................................................................... 92
Gonorrhoea ................................................................................................................................................. 95
Bacterial Vaginosis ...................................................................................................................................... 98
Ectopic Pregnancy (LIF Pain & PV Bleeding).............................................................................................. 101
Miscarriage ............................................................................................................................................... 104
Missed Abortion/Miscarriage ................................................................................................................... 107
Polycystic Ovarian Syndrome .................................................................................................................... 109
Contraception ........................................................................................................................................... 111
Combined Pill Prescription ........................................................................................................................ 119
Premenstrual Syndrome (PMS) ................................................................................................................. 124
Mood Swings (Depo-Provera) ................................................................................................................... 126
Pregnancy (16-Year-Old Vomiting) ........................................................................................................... 127
Antenatal Check-up (Rubella/RH Negative) .............................................................................................. 129
Pre – Conception Counselling ................................................................................................................... 132
Premature Ovarian Insufficiency .............................................................................................................. 134
Menopause ............................................................................................................................................... 136
Postmenopausal bleeding ......................................................................................................................... 138
Incontinence ............................................................................................................................................. 139

www.aspire2plab.com 3
No real PLAB2 cases discussed. All discussed scenarios are fictitious and for educational purposes only.
Cervical Screening (Dyskaryosis) ............................................................................................................... 141

Psychiatry ...................................................................................................................... 143


Mini Mental State Examination ................................................................................................................ 143
Psychotic Patient....................................................................................................................................... 149
Psychosis ................................................................................................................................................... 153
Alcohol Dependency ................................................................................................................................. 156
Drug Dependency ..................................................................................................................................... 161
Suicide ....................................................................................................................................................... 165
Paracetamol Overdose (Homosexual Patient) .......................................................................................... 169
Anorexia Nervosa ...................................................................................................................................... 171
Concerned Daughter MMSE ..................................................................................................................... 176
Low Mood (Lesbian Miscarriage) .............................................................................................................. 178
Depression (CBT Failed) ............................................................................................................................ 183
Depression (CBT failed 2) .......................................................................................................................... 183
Depression (Weight Loss) ......................................................................................................................... 184
Post-Partum Depression (Telephonic Consultation) ................................................................................. 185
Colleagues Consultation .......................................................................................................... 188
Alcohol Colleague ..................................................................................................................................... 189
Cocaine colleague ..................................................................................................................................... 191
Colleague Coming Late.............................................................................................................................. 194
Colleague Confidentiality Issue (Facebook Post) ...................................................................................... 196
Doctor & Patient Relationship (Nurse Concerned) ................................................................................... 198
Examination & Mannequin ............................................................................................ 199
Medicine................................................................................................................................. 199
Diabetic Foot ............................................................................................................................................. 199
Alcoholic Foot ........................................................................................................................................... 201
Cerebellar Ataxia ....................................................................................................................................... 202
Meningitis ................................................................................................................................................. 203
Cranial Nerve Examination........................................................................................................................ 204
Peripheral Field of Vision .......................................................................................................................... 204
Fundoscopy (Giant Cell Arteritis) .............................................................................................................. 205
Thyroid Annual Review ............................................................................................................................. 206
Acoustic Neuroma..................................................................................................................................... 207
Otoscopy (Otitis Media) ............................................................................................................................ 208
Otitis Externa ............................................................................................................................................ 209
Ear Wax ..................................................................................................................................................... 211
Otitis Media (Child unwell) ....................................................................................................................... 212
Cholesteatoma .......................................................................................................................................... 214
Alcohol Related Liver Disease ................................................................................................................... 216
Surgery ................................................................................................................................... 218
Whiplash Injury ......................................................................................................................................... 219
Brachial Plexus Injury ................................................................................................................................ 221
Primary Survey (Conscious) ...................................................................................................................... 223
Primary Survey (Unconscious) .................................................................................................................. 225
Testicular Examination .............................................................................................................................. 228
Epididymal Cyst ......................................................................................................................................... 229
Varicocele ................................................................................................................................................. 230
Hydrocele .................................................................................................................................................. 231
Epididymo-Orchitis.................................................................................................................................... 233
Mumps Orchitis......................................................................................................................................... 235
OBG ........................................................................................................................................ 236
Breast Lump .............................................................................................................................................. 237
Cyclical Breast Pain ................................................................................................................................... 238

www.aspire2plab.com 4
No real PLAB2 cases discussed. All discussed scenarios are fictitious and for educational purposes only.
Nipple Discharge ....................................................................................................................................... 240
Antenatal .................................................................................................................................................. 242
Cervical Smear .......................................................................................................................................... 244
Procedures ............................................................................................................................. 247
Blood Sampling (Paracetamol) .................................................................................................................. 248
Blood Culture ............................................................................................................................................ 249
IV Cannulation........................................................................................................................................... 250
Arterial Blood Gas ..................................................................................................................................... 251
Catheterisation ......................................................................................................................................... 253
Joint Examinations .................................................................................................................. 255
Hip Examination ........................................................................................................................................ 256
Knee Examination ..................................................................................................................................... 259
Elbow Examination ................................................................................................................................... 261
Hand & Wrist Examination........................................................................................................................ 264
Teaching ................................................................................................................................. 267
Basic Life Support (Adult) .......................................................................................................................... 268
Basic Life Support (Paediatric) .................................................................................................................. 274
Per Speculum ............................................................................................................................................ 276
Inguino - Scrotal Examination ................................................................................................................... 278
Teaching ECG ............................................................................................................................................ 281
Epi Pen Teaching ....................................................................................................................................... 287
Urine Dipstick ............................................................................................................................................ 290
Sub Cutaneous Injection ........................................................................................................................... 293
8th Nerve Examination Teaching .............................................................................................................. 295
Arterial Blood Gas Teaching ...................................................................................................................... 295
Thyroid Teaching ....................................................................................................................................... 295
Testicular Examination Teaching .............................................................................................................. 295
Breast Examination Teaching .................................................................................................................... 295
Abdominal Examination ............................................................................................................................ 296
Respiratory Examination ........................................................................................................................... 299
Examinations .......................................................................................................................... 303
Foot Examination ...................................................................................................................................... 304
Upper Limb Examination........................................................................................................................... 308
Cerebellar Examination ............................................................................................................................. 310
Meningitis ................................................................................................................................................. 312
Eye Examination........................................................................................................................................ 313
Ear Examination ........................................................................................................................................ 319
Thyroid Examination ................................................................................................................................. 323
Cranial Nerve Examination........................................................................................................................ 326
Testicular/ Inguino-scrotal Examination ................................................................................................... 328
Per rectal Examination .............................................................................................................................. 331
Breast Examination ................................................................................................................................... 332
Antenatal Examination.............................................................................................................................. 333
Blood Sampling/ Blood Culture ................................................................................................................. 335
IV Cannulation........................................................................................................................................... 337
ABG Sampling ............................................................................................................................................ 338
Catheterisation ......................................................................................................................................... 339
Per Speculum and Pap Smear ................................................................................................................... 340
Spacer ....................................................................................................................................................... 341
Epi Pen ...................................................................................................................................................... 342
Urine Dipstick ............................................................................................................................................ 343
Sim Mannequin ............................................................................................................. 344
Asthma ...................................................................................................................................................... 345
Anaphylaxis ............................................................................................................................................... 348

www.aspire2plab.com 5
No real PLAB2 cases discussed. All discussed scenarios are fictitious and for educational purposes only.
Post UTI Sepsis Shock ................................................................................................................................ 351
UTI after TURP........................................................................................................................................... 354
Upper GI Bleed .......................................................................................................................................... 358
Post-Partum Haemorrhage ....................................................................................................................... 362
Dizzy Spells ................................................................................................................................................ 366
Acute Limb Ischaemia ............................................................................................................................... 369
DNAR ......................................................................................................................................................... 372
Hypoglycaemia .......................................................................................................................................... 375
Heart Failure ............................................................................................................................................. 377
Haematemesis – Post Endoscopy ............................................................................................................. 379
Post CS – Pain Management ..................................................................................................................... 382
Prescription Writing ................................................................................................................ 383
MRSA......................................................................................................................................................... 389
VTE Risk Assessment ................................................................................................................................. 390
Palliative Care (1) ...................................................................................................................................... 391
Palliative Care (2) ...................................................................................................................................... 392
Community Acquired Pneumonia ............................................................................................................. 393
Non-Valvular Atrial Fibrillation ................................................................................................................. 394
Drug interaction (Lithium) ........................................................................................................................ 396
UTI (Nitrofurantoin) .................................................................................................................................. 397
Apixaban (1) .............................................................................................................................................. 398
Apixaban (2) .............................................................................................................................................. 399
Peritonsillar Abscess ................................................................................................................................. 400
Acute Pyelonephritis ................................................................................................................................. 402
Cellulitis (Prescription) .............................................................................................................................. 404

www.aspire2plab.com 6
No real PLAB2 cases discussed. All discussed scenarios are fictitious and for educational purposes only.
Paediatrics
Paediatric GCS

History

B – Birth
I – Immunisation (Jabs)
R – Development (Red Book)
D – Diet (Breast Fed/Bottle Fed)

Wee & Poo (Urine and Stool)


NAI (Who takes care of the child)

www.aspire2plab.com 7
No real PLAB2 cases discussed. All discussed scenarios are fictitious and for educational purposes only.
Immunisation Schedule

www.aspire2plab.com 8
No real PLAB2 cases discussed. All discussed scenarios are fictitious and for educational purposes only.
!

www.aspire2plab.com 9
No real PLAB2 cases discussed. All discussed scenarios are fictitious and for educational purposes only.
Otitis Media child

You are FY2 in Paediatrics. Miss Daisy Shah, 15 months old, was brought to the hospital by
her father with fever, shortness of breath and was pulling her left ear. They went to the GP
and he gave her PCM and referred her to the hospital. Her father is concerned about her
condition. On examination, there is redness over her left ear drum. Her right ear drum is
pink. Please talk to her father, take history, discuss the plan of management with the father
and address his concerns. Nurse colleagues are looking after her in the next room.

NEWS chart:
Before: Temp – 38, HR – 130 (80-130), RR – 40 (25-35)
After: Temp – 37, HR – 100 (80-130), RR – 30 (25-35)

D: Hello, what brings you to the hospital? P: How is my daughter?

D: She’s in good hands, our nurse colleagues are looking after her in the next room. So, why
did you bring your daughter to the hospital?
P: She’s been having fever and shortness of breath.
D: Since when? P: Last 2 days.
D: Did you measure her temperature? P: No Doctor.
D: What did you do for that?
P: We went to the GP and he gave her Calpol and referred us to the hospital.

D: Anything else? P: She was having a cough.

D: Any Phlegm? P: No
D: Anything else? P: Yes, she was tugging her left ear.
D: Since when? P: Last 2 days.
D: Any discharge? P: No
D: Hearing loss? P: No
D: Was she feeling sick? P: No
D: Have you noticed any jerky movements? P: No
D: Have you noticed any rash on her body? (Meningitis) P: No
D: Have you noticed your daughter being shy to light/cries whilst moving her neck? P: No

D: Any similar episodes before? P: No


D: Has she been diagnosed with any medical conditions in the past? P: No
D: Is she taking any medications including OTC or supplements? P: No
D: Any allergy to medications or food? P: No
D: Has anyone in the family been diagnosed with any medical conditions? P: No

D: How was the birth of your baby? P: Normal Vaginal Delivery.


D: Are you happy with Red Book? P: Yes
D: Is she up to date with all her jabs? P: Yes
D: Is she feeding well? P: Yes
D: Does she have any problem with her wee and poo? P: No
D: By any chance have you noticed the change in colour and smell of her wee? (UTI) P: No

www.aspire2plab.com 10
No real PLAB2 cases discussed. All discussed scenarios are fictitious and for educational purposes only.
D: Have you noticed any tummy pain or changes in her poo? P: No
D: Who looks after her? P: Me and my wife.

We have examined your daughter, she had high fever but after paracetamol it has come
down. We also checked her heart rate and respiratory rate which were high but after giving
her paracetamol, they have also settled.

On examination of her ear, there is redness over her left ear drum.
From our assessment, we suspect your child has a condition called OTITIS MEDIA.

It looks like your child has got viral infection which usually subsides on its own in the next
few days. We will prescribe her some paracetamol for her fever and pain.

Please give her plenty of fluids to drink. Hopefully, she will completely recover in the next
few days.

P: Are you going to give her antibiotics?


D: At the moment, there is no need to give her antibiotics as it is a viral condition, but if her
fever lasts for more than 4 days, we will give her antibiotics.

Please bring her back to the hospital if she develops high fever, rash, neck stiffness and
discharge from her ear.

If the fever does not subside by 4 days, then please come back to us.

Drug of choice: Amoxicillin for 5 days.


(If Allergic to Penicillin, then give Erythromycin or Clarithromycin for 5 days)

Antibiotics are usually only considered if your child:


- has a serious health condition that makes them more vulnerable to complications, such
as cystic fibrosis or congenital heart disease
- is less than three months old
- is less than two years old with an infection in both ears
- has discharge coming from their ear

www.aspire2plab.com 11
No real PLAB2 cases discussed. All discussed scenarios are fictitious and for educational purposes only.
Eye Infection in Neonate (Chlamydia)

You are an FY2 in Paediatrics. Miss Jane Jackson, aged 26, having 10 days old boy (Peter
Jackson) with (chlamydia) infection in the eyes. He was admitted and after 3 days
treatment, he was discharged with his father. Swab has been taken from the eyes and
Chloramphenicol eye drop is being given to the boy. Mother is here to discuss the results of
the swab. Talk to the mother and address her concern.

D: Hello, how can I help you today?


P: I have come for the results of the swab taken from my son’s eye.

D: Okay, I have the results with me, but before I proceed let me ask you a few questions, so
that I would be in a better position to answer everything you need.
P: Okay.

D: Could you please confirm the age of your son? P: He is 10 days old.
D: Why did you bring your son to the hospital? P: He was having red and sticky eyes
D: Is it in one eye or in both the eyes? P:
D: May I know since when? P: For the last 3 days.

D: Any other symptoms? P: No

D: Did he have any kind of discharge from the eyes?


P: Yes/No (If yes elaborate colour, quantity, smell, time of the day, early morning/especially
after sleep)

D: Any swelling of the eyelids? P: Yes/No


D: Any swelling or redness around the eyes? P: No
D: Any fever? P: No
D: Any neck stiffness or pain while moving the neck? P: No
D: Any shyness towards light? P: No
D: Is he drowsy or tired? P: No
D: Any cough or phlegm? P: No
D: Any breathing difficulty? P: No

D: Did you notice any health-related problems in these 10 days? P: No


D: Is he allergic to anything? P: No
D: Have you or his father ever had any medical conditions? P: No

D: Is he feeding well? P: Yes


D: What do you feed him? P: Just milk
D: How is his wee and poo? P: Its fine

D: Thank you for answering all my questions.

P: Could you please tell me about the results

www.aspire2plab.com 12
No real PLAB2 cases discussed. All discussed scenarios are fictitious and for educational purposes only.
D: Yes. We have done the swab to check which type of bug has caused him this eye
infection. Unfortunately, the swab results came back positive for a bacteria called
Chlamydia. We call this condition Ophthalmia Neonatorum. It can happen in the first 28
days after birth.
D: As the swab is positive for Chlamydia, we need to treat him with oral antibiotics
(Erythromycin or Azithromycin) for 2 weeks to treat the infection completely and to prevent
the complications.

P: But, why did he have this infection?


D: If you have chlamydia that's not treated while you're pregnant, there's a chance you
could pass the infection on to your baby during the delivery.

P: But I don’t have any symptoms?


D: Most people who have chlamydia don't notice any symptoms. For some people they
don't develop until many months later. Sometimes the symptoms can disappear after a few
days. Even if the symptoms disappear you may still have the infection and be able to pass it
on.

D: Chlamydia is a sexually transmitted bacterium, that means this bug is transmitted by


having unprotected sex.

D: Let me ask you a few questions to see if you have had this infection? Have you or your
partner ever been diagnosed with STIs?
P: No

D: Did you ever have symptoms like pain when urinating or unusual vaginal discharge or
pain in the tummy or pelvis?
P: No

D: Any pain during sex?


P: No

D: Any bleeding after sex or bleeding between periods?


P: No

D: Multiple sexual partners increases the risk of STIs. Have you had any other sexual
partners before?
P: No dr, I only have one sexual partner from the last 2/3 years.

P: Dr, does this mean my partner is cheating on me, because I have only one sexual partner?
D: As you told me that you only have one sexual partner, there is a possibility that you might
have got this bug from your partner. This does not necessarily mean that your partner is
cheating on you. As I told you earlier, Chlamydia can stay in our body for many months
without causing any symptoms. He might have got this infection from his previous
relationships.

www.aspire2plab.com 13
No real PLAB2 cases discussed. All discussed scenarios are fictitious and for educational purposes only.
D: We need to test you both for this infection and treat both of you even if you are positive
for Chlamydia.

(If pt says cannot bring partner – talk about partner notification programme).

D: We need to refer you to Genito-Urinary Medicine (GUM) Clinic.

D: Please bring your son immediately to the hospital if he develops any high fever,
drowsiness, neck pain/stiffness, shyness to light, cough, phlegm or breathing difficulty.

" Gonorrhoea and Chlamydia are the most common bacterial causes of neonatal
conjunctivitis.

Initial Management

● Sticky eye/blocked tear duct: 4-6 hourly eye toilet using sterile saline
● Suspected conjunctivitis:
Swab for: Gram stain and bacterial culture and sensitivities
If other suspicions of HSV (e.g. vesicles etc.), viral swab
Chlamydia swab (specific for Chlamydia PCR)

Treat with:

● frequent eye toilet as necessary


● chloramphenicol 0.5% eye drops
● Presentation within first 24 hr suggests gonococcal infection
● inform senior paediatrician

● Neisseria gonorrhoeae suspected:


Request Gram stain and culture
Assess neonate for systemic infection
● Neisseria gonorrhoeae confirmed:
Give single dose ceftriaxone 125 mg IV if IV access present, otherwise IM (40 mg/kg
for low- birth-weight babies).
If signs of systemic infection (e.g. sepsis, meningitis), give course of IV ceftriaxone.
Refer to ophthalmology.
● Chlamydia result positive:
Treat with azithromycin 20 mg/kg single dose or erythromycin 12.5 mg/kg/dose
orally 6 hourly for 2 weeks. This will treat the conjunctivitis and prevent most cases
of chlamydia pneumonitis. Monitor for signs of Pyloric stenosis as Macrolides can
cause infantile hypertrophic pyloric stenosis when given before 6 weeks.

www.aspire2plab.com 14
No real PLAB2 cases discussed. All discussed scenarios are fictitious and for educational purposes only.
Gonococcal versus chlamydial conjunctivitis:

Gonococcal Chlamydial

2-5 days incubation 5-14 days incubation


Transmission vaginal or from contaminated Transmission vaginal or from contaminated
fingers after birth fingers after birth
Mild inflammation with sero-sanguineous Varies from mild inflammation to severe
discharge swelling of eyelids with copious purulent
discharge
Progression to thick, purulent discharge with
tense oedema of eyelids
Complications include corneal ulceration and Corneas rarely affected
perforation 1 in 5 untreated will develop chlamydial
Meningitis and sepsis pneumonitis

Gonococcal or chlamydia infection detected:


• Refer mother and partner to genito-urinary medicine for immediate treatment
Gonococcal versus chlamydial conjunctivitis.

www.aspire2plab.com 15
No real PLAB2 cases discussed. All discussed scenarios are fictitious and for educational purposes only.
Febrile Convulsion

You are F2 in Paediatrics. Miss Maria Lawson, aged 2, has been brought to the hospital by
her mother because she had a fit. She has been managed in the A&E department and has
been referred to you. Her temperature is 38.9 C. On examination, there is redness over her
left eardrum. Please talk to her mother, Mrs. Diana Lawson, take history, discuss your plan
of management with the mother and address her concerns. The mother is very concerned.
The child is not in the cubicle.

D: What brought you to the hospital?


P: I was playing with my daughter and suddenly she started having jerky movements.

D: Tell me more about the jerky movements?


P: Dr, she was having shaky movements all over her body.

D: When did this happen? P: It happened 2 hours ago


D: Is it the first time? P: Yes

D: Did you notice anything strange before she had these jerky movements?
P: She became pale and sweaty.
D: Any other symptoms before the jerky movements? P: No doctor.

D: For how long did she have jerky movements? P: Around 2 mins
D: Did she become unconscious? P: No
D: Any symptoms during the fit? P: No
D: Did her eyes appear to roll backwards? P: Yes/No
D: Did she wet herself? P: No
D: Did she bite her tongue? P: No
D: How was she after the fit? P: She was sleepy and drowsy.
D: For how long? P: For few mins
D: What did you do afterwards? P: I called the ambulance and brought her to the hospital.
D: Did she injure herself? P: No.
D: Did she vomit? P: No
D: Any fever or flu like symptoms? P: Yes, she has had a fever for the last 2 days.
D: Did you measure the temperature? P: No
D: Have you noticed anything unusual? P: Yes, she was tugging her left ear.
D: Since when? P: From last 2 days
D: Any discharge from the ear? P: No
D: Have you noticed any rash? P: No (Meningitis)
D: Have you noticed that your child is shy to light or cries while moving her neck? P: No
D: Any cough? P: No (pneumonia)
D: Has she been diagnosed with any medical condition in the past? Any Diabetes?
P: No (DM)

D: Is she taking any medications including OTC or supplements? P: No


D: Any allergies from any food or medications? P: No

www.aspire2plab.com 16
No real PLAB2 cases discussed. All discussed scenarios are fictitious and for educational purposes only.
D: Has anyone in the family been diagnosed with any medical condition? P: No
D: Any family history of epilepsy? P: No
D: How was the birth of your baby? P: It was normal vaginal delivery.
D: Are you happy with the red book? P: Yes.
D: Is she up to date with all her jabs? P: Yes.
D: Has she received any recent jab? P: No
D: Is she feeding well? P: Yes. She is feeding very well.
D: Does she have any problems with her wee? P: No.
D: Have you noticed any changes in the colour or smell of her urine? (UTI) P: No
D: Have you noticed any tummy pain or change in her poo? (Gastroenteritis). P: No
D: Any diarrhoea? P: No
D: Who looks after her? P: It’s me.

We have checked your child and we found that her temperature was high.

From our assessment, we suspect your child has a condition called febrile convulsion.
Febrile convulsion is a type of fit that happens in children aged between 6 months to 5 years
due to high temperature.

On examination of the ear we found there was redness in the left ear which is known as
otitis media.

We will keep your child under observation to make sure everything is fine. If everything goes
smoothly, you can take her home.

For now, we will give her paracetamol to reduce her temperature.

We will give her antibiotics if signs of infection such as fever lasts for more than 4 days. If
we are prescribing any antibiotics, please rule out any allergy. DOC- Amoxicillin for 5 days.
If allergic to penicillin, then erythromycin or clarithromycin for 5 days.

It is not dangerous as the fit lasted for less than 5 mins and she recovered fully after the
incident.

Febrile convulsions and epilepsy are two different things. As I told you, febrile convulsions
are because of feverish conditions. However, epilepsy is some abnormal activity in the brain
without any high temperature.

It can happen again. Since your child is 2 years old and I have already mentioned that it
usually happens in children who have fever and aged between 6 months to 5 years. Most
children outgrow this condition after 5 years of age.

You can prevent febrile convulsions by keeping your child’s temperature down. You can give
her paracetamol to lower down her temperature. Keep her lightly dressed and remove
excessive clothing. Give her lots of cold fluids to drink.

www.aspire2plab.com 17
No real PLAB2 cases discussed. All discussed scenarios are fictitious and for educational purposes only.
If your child is having a febrile seizure, place her in the recovery position. Stay with your
child and try to make a note of how long the seizure lasts. Don't put anything into your
child's mouth during a seizure – including medication – as there's a slight chance they might
bite their tongue, or they aspirate.

Please call an ambulance if the fit lasts more than five minutes.

If she doesn’t improve quickly after the seizure or she has difficulty breathing, she should be
seen by a doctor.

P: Doctor, my friend’s kid had the same problem and the doctor gave her a medicine to put
through her back passage.
D: There are some medications that can be used to stop the fit.
We can prescribe it only if the child has recurrent fits or the fit lasts for more than 5 mins.
And also, if you live far from the hospital (More than 2 hours).

P: Can my son also have the same problem?


D: Tell me his age. P: He is 4 years old, doctor.
D: There is some evidence that suggests that the chance of having febrile convulsions
increases in close family members aged between 6 months and 5 years.

Please bring her back to us if she develops any high fever, rash, neck stiffness or any
bleeding or discharge from the ear. If the fever doesn’t subside by 4 days, please come back
to the hospital.

P: Oh, okay doctor. Can I take my baby home?


P: Is it dangerous?
P: Is it epilepsy?
P: Can it happen again in the future?
P: How can I prevent it?
P: Thank you doctor. What should I do if it happens again?

www.aspire2plab.com 18
No real PLAB2 cases discussed. All discussed scenarios are fictitious and for educational purposes only.
Head Injury

You are FY2 in Paediatrics. Miss Jane Xavier, aged 3, had a fall from the sofa. She is happy
and playful now. She is moving her four limbs. She has a bruise on her forehead. Please talk
to her mother and discuss your management with her and address her concerns. Your nurse
colleague is looking after the baby in the next room.

D: Hello Mrs. Hannah Xavier, I am Dr. __, are you the mother of Jane? P: Yes Dr.
D: I am going to talk to you about your child’s health today, could you please give me a
summary of what exactly happened and why did you bring your child to the hospital?
P: Doctor, I brought my little one to the hospital because she had a fall from the sofa two
hours ago.

D: How did it happen?


P: Doctor, I was changing my other kid’s nappy. Jane was behind me and I heard her crying
and turned around. I found her on the carpet.

D: Did you notice if she banged her head on the floor? P: I’m not sure, doctor.
D: Did she become unconscious? P: No
D: Any symptoms after the fall?
P: No, she was crying and went pale for a few seconds.

D: Any jerky movements? P: No


D: Did she wet herself? P: No
D: Did she bite her tongue? P: No
D: How was she after the fall? P: She was sleepy and drowsy.
D: For how long? P: For a few seconds.
D: Did she vomit? P: Oh yes doctor. She vomited.
D: How many times she vomited?
P: Just once in the ambulance.
D: Did she injure herself?
P: Oh yes doctor. There is a bruise on her forehead/at the back of her head.
D: Could you tell me how big it is?
P: It is as big as this doctor, like a coin. (She shows the size of the bruise using her fingers.)
D: Any other injuries? P: No doctor.
D: Have you noticed any injury in the neck? Was she crying while moving her neck? P: No
D: What did you do afterwards?
P: I called the ambulance and brought her to the hospital.

D: Has this happened before? P: No


D: Any fever or flu like symptoms? P: No
D: Any rash? P: No
D: Has she been diagnosed with any medical condition in the past? P: No
D: Any Diabetes/ heart disorder/ blood disorder/ epilepsy? P: No
D: Is she taking any medications including OTC or supplements? P: No
D: Any allergies from any food or medications? P: No

www.aspire2plab.com 19
No real PLAB2 cases discussed. All discussed scenarios are fictitious and for educational purposes only.
D: Has anyone in the family been diagnosed with any medical condition? P: No
D: Any family history of epilepsy? P: No

D: How was the birth of your baby? P: It was normal vaginal delivery.
D: Are you happy with the red book? P: Yes.
D: Is she up to date with all her jabs? P: Yes.
D: Has she received any recent jab? P: No
D: Is she feeding well? P: Yes. She is feeding very well.
D: Does she have any problems with her wee? P: No.
D: By any chance have you noticed any changes in the poo? P: No

D: Who looks after her? P: It’s me.


D: Is there anyone else to help you? P: Sometimes my sister is helping me.
D: How about her dad? P: I got separated from her dad. Our relationship didn’t work.
D: What do you do for a living? P: No, I left my job. I am looking after my kids.
D: How do you cope with that? P: Doctor, It is difficult but it is fine

I would like to check the vitals, general physical examination and neurology examination.

Examiner:
There is a bruise on her forehead, rest of them are normal.
From our assessment, it seems like your baby has a head injury.

On examination of your little one: we did physical examination to look for injury in any part
of her body. We also performed neurological examination to check how well her brain and
spine is working.

Fortunately, she is fine now. She is playful and is moving her four limbs. The only thing we
found was a bruise on her forehead. However, we need to keep her in the hospital. I may
ask one of my seniors to come and review your little one.

P: Doctor, if she is fine, then why are you going to keep her here?
D: I do understand your concern as a mother but is there any particular reason that is
bothering you regarding admission?

P: Doctor, I have another kid. I left him with my neighbor.


D: I understand but we need to observe her a little longer to reassess her condition and to
make sure she is fine. We will keep her under our observation and if everything goes
smooth you can take her home. However, we might need to do CT head if we feel that
needs to be done.

Fortunately, she is fine now but as you told me she was a bit drowsy/sleepy after the
incident and that’s why it is important to have your little one for a while in the hospital.
Hopefully it won’t be more than a few hours (four hours), if by any chance she looks drowsy,
or she has any further vomiting, you need to bring her to the hospital to perform a CT Scan
of her brain. I can imagine how difficult it must be for you to look after your little ones.

www.aspire2plab.com 20
No real PLAB2 cases discussed. All discussed scenarios are fictitious and for educational purposes only.
P: How long will she be here, doctor?
P: Why CT head?
P: How long do I need to stay?

Criteria for performing a CT scan for children


CT scan head should be performed within 1 hour.

1. Suspicion of NAI.
2. Post-traumatic seizure but no history of epilepsy.
3. On initial assessment GCS less than 14, or for children under 1 year GCS less than 15.
4. 2 hours after the injury, GCS less than 15.
5. Suspect of basal skull fracture or tense fontanelle.
6. Any sign of basal skull fracture.
7. Focal neurological deficit.
8. For children under 1 year, presence of bruise, swelling or laceration of more than 5 cm on
head.

NOTE
For children who have sustained a head injury and have more than one of the following risk
factors a CT scan should be performed within 1 hour.

1. LOC more than 5 mins.


2. Abnormal drowsiness.
3. Three or more discrete episodes of vomiting.
4. Dangerous mechanism of injury (high speed RTA either as pedestrian or cyclist or vehicle
occupant, fall from a height of greater than 3 meters, high speed injury from projectile or
other object)
5. Amnesia (antegrade or retrograde) lasting more than 5 mins.

NOTE
Children who have sustained a head injury and have only 1 of the risk factors should be
observed for a minimum of 4 hours after head injury.

NOTE
If during observation any of the risk factors below are identified, a CT scan head should be
performed in 1 hour.
1. GCS less than 15.
2. Further vomiting.
3. A further episode of abnormal drowsiness.
!

www.aspire2plab.com 21
No real PLAB2 cases discussed. All discussed scenarios are fictitious and for educational purposes only.
Non-Accidental Injury

You are FY2 in Paediatrics. Mr Daniel McCann, aged 4 months, has been brought to the
hospital by his mother this morning. She rushed to the hospital after she noticed a swelling
on his right arm. An X-ray has been done and a spiral fracture of right humerus has been
diagnosed. Please talk to the mother, Mrs. Diana McCann, take history and discuss the
initial plan of management with her. Your nurse colleague is looking after the child in the
next room. The mother is worried.

D: Hello Mrs. Diana McCann. Are you the mother of Daniel? P: Yes Dr.
D: I am going to talk to you about your child’s health today, could you please give me a
summary of what happened and why did you bring your child to the hospital?
P: Doctor, there was a swelling on his arm, which worried me and I brought him to the
hospital.

D: That must be very stressful for you, as you mentioned the purpose of the visit was
swelling on his arm, we examined him and did an X ray which revealed a fracture in one of
his bones in his arm unfortunately. P: Is my baby okay? Is he in pain
now?

We can disclose the fracture later on as well. It depends on the situation if you have to
disclose it in the beginning or in the last.

D: Don’t worry, he is in good hands. We are managing his broken bone. We gave him
enough pain killers and he is not in pain now.
P: Thank you doctor.

D: May I know how it happened?


P: I am a nurse by profession, and I had a shift last night. When I came back home this
morning, I realised he was crying more than often and not moving his arm as well. Later
while changing the nappy, I observed the swelling on his arm and I rushed to the hospital.
OR I was working in the kitchen when I heard my baby crying and I rushed to the room
where I saw he had fallen from the cot and I rushed to the hospital.

D: How did you come to the hospital? P: I took a bus and came to the hospital.
D: How long did it take for you to get here? P: It took me about an hour
D: Okay. So you were not present when this happened? P: No Dr.
D: Was there anyone with the child when this happened? P: Yes Dr. My partner
D: Was he looking after your child last night/ when you were in the kitchen? P: Yes
D: Does he usually take care of your little one while you are at work OR away?
P: Yes, When I am away, he is the one taking care.
D: Did you talk to your partner about this? P: No doctor.
D: May I know why?
P: I did not get the chance; he was ready to go to his work when I came back home and
later, I had to rush to the hospital.

www.aspire2plab.com 22
No real PLAB2 cases discussed. All discussed scenarios are fictitious and for educational purposes only.
D: Is he the biological father of your little one? P: No, I got separated 1 year ago.
D: How long have you been with your new partner? P: It’s been six months doctor.
D: Is there anyone else at home living with you? P: No Dr.
D: Does he get on well with your baby? P: They get on well.

D: Has it ever happened in the past? P: No doctor.


D: How has been his health been recently? P: He has been fine, doctor.
D: Any trauma recently? P: No doctor.
D: Has he ever been diagnosed with any medical condition? P: No doctor.
D: Does he have any bone disease? P: No
D: Have you noticed any bluish discoloration of your little one’s eyes/hearing problem/blood
disorder? P: No
D: Is he on any medications? P: No.
D: Has any member of your family been diagnosed with any medical condition? P: No.
D: Any bone disease in the family? P: No.

D: How was the birth of your baby? P: I had normal vaginal delivery.
D: Was there any complication or birth-related trauma? P: No doctor.
D: Did you use any medications before giving birth? (Perinatal drug history) P: No.
D: Is he up-to-date with all his jabs? P: Yes.
D: Are you happy with his red book? P: Yes.
D: Is he feeding well? P: Yes
D: Is he gaining weight? P: Yes doctor.
D: Is he bottle-fed or breast-fed? P: He is bottle-fed,
doctor.
D: Does he have any problems with his wee? P: No.
D: How about poo? P: No.

I would like to check his Vitals, do general physical examination, eyes, ear and
Musculoskeletal examination

As I mentioned, your baby has a fractured bone, so we have to keep him in the hospital.
P: But doctor, why do you want to admit my baby?
D: The fracture needs to be taken care of.

As part of initial management, we have given him painkillers and stabilised the fracture but
we will refer him to a bone specialist so that they can manage the fracture.

We also need to do some further investigations to look for the cause of fracture.

I will also ask my senior colleague to come and review your baby. Since your little one is just
4 months old, it's better for him to be reviewed by a senior doctor. We want to make sure
your little one is fine before going home.

My senior is the better person to determine what can be the possible cause of this fracture.
Moreover, we’ll be doing some further investigations which include blood tests to check the

www.aspire2plab.com 23
No real PLAB2 cases discussed. All discussed scenarios are fictitious and for educational purposes only.
levels of some minerals such as Calcium and Vitamin D and check levels of some other
substances that can be related to such a problem. (ALP)

Other than that, we need to see if there is any other fracture in his body for which we will
be doing detailed X rays. Bone scan could be another possibility. We might need to do CT
head to see if there is any head injury. There is also a possibility of injury to other organs in
the tummy so we will check how well they are working by doing some specific blood tests.
(LFT’s, S. Amylase, U&Es).

All these investigations should be done because this sort of fracture at this age is a bit
unusual. Children at this age have very flexible bones. This sort of injury usually doesn’t
happen due to simple mechanisms such as fall. Moreover, your baby is only 4 months old
and he is not mobile enough to have such incidents on his own. This sort of fracture usually
suggests an external force such as twisting.

P: Doctor, do you mean someone (my boyfriend) did this?


D: We are not sure of that yet but it is one of the possibilities. That’s why we need to do all
the investigations to rule out all the possible causes that might have led to this. We are not
trying to blame or accuse anyone. As children’s doctors, we need to follow certain protocols
and consider many possibilities. This is a possibility that we have to look for. This is a very
sensitive matter and that’s why facts need to be confirmed by a senior doctor.

If it is confirmed, as a part of hospital protocol, Social Services would be involved. They will
talk to you and your partner about the incident.

They might come to your house to do further assessment and get to the bottom of how the
incident happened.

All we are trying to do is help your baby and keep him safe.

I want to reassure you that we are trying to help and support you and your baby.

We want to make the best decision for your child. That’s why Social Services is going to be
involved. It is not their goal to take your child away from you. However, they have to do a
complete assessment and make sure that there is no risk to the baby.

P: What is going to be the next step if it is confirmed?


P: Are they going to take my child away from me?
P: Can I take him home?
P: Doctor, why do you need to do so many investigations?
P: Doctor, why is it unusual?

www.aspire2plab.com 24
No real PLAB2 cases discussed. All discussed scenarios are fictitious and for educational purposes only.
Intussusception

You are FY2 in Paediatrics. Mr Mike White, aged 20 months, was brought to the hospital
by his mother Mrs Jane White due to abdominal pain. GP has seen the child and send him
to the hospital with the following letter:

Thank you for seeing this child. I have seen this 20 month old child with abdominal
discomfort. On examination: The child looks pale. His peripheries are cold. He looks lethargic.
Abdomen is not distended. There is a mass in the right hypochondrium.
Vitals have been recorded and are as follows:
PR 140 bpm, BP 90/60 mmHg, RR 28.

Please talk to the mother, take history, give possible diagnosis to her, explain the next
steps of management and address her concern. Baby is in the next room.

D: As we understand that you took your child to GP and he has sent you to us, May I know
what made you take your child to GP? P: Doctor, my baby had tummy discomfort.

D: How did you notice that?


P: Doctor, he has been crying a lot and whenever I want to change his nappy and I touch his
tummy, he cries even more.

D: When did he start crying? P: 10 hours ago

D: Has it changed since it started?


P: It has become worse. He was crying on and off initially but then he started crying
continuously around 2 hours ago and that’s why I took him to the GP.

D: Have you noticed anything unusual other than crying?


P: He pulls his legs towards his chest and that’s why I think he has some problem in his
tummy.

D: Does he have any other symptoms?


P: He is passing loose stools. I need to change the nappy more often.
D: How long has he had this problem? P: Since yesterday doctor.
D: How many episodes? P: 4-5 times a day.
D: Have you noticed any blood in his stools? P: I have noticed some red jelly material
D: When did you first notice that? P: A few hours ago.
D: How many times? P: A few times
D: Does he have any other symptoms? P: Yes doctor, he has been vomiting also
D: How many times has he vomited? P: Around 2-3 times in the last few hours
D: Tell me about the colour of his vomit? P: Doctor, it was green.
D: Have you noticed that his mouth is dry? P: I’m not sure.
D: Is he as playful as before? P: No.
D: By any chance has he become drowsy? P: Yes.
D: Has he become floppy? P: He is pale and he is floppy and that’s why I took him to the GP.
D: Is your baby able to eat and drink?

www.aspire2plab.com 25
No real PLAB2 cases discussed. All discussed scenarios are fictitious and for educational purposes only.
P: No doctor. In the past few hours he could not tolerate any food or drink.
D: Does he have any fever? P: I haven’t noticed.
D: Did he have any recent infection such as flu or diarrhoea? P: No.
D: By any chance did he hurt himself? P: No doctor

D: Has he ever been diagnosed with any other medical conditions? P: No.
D: Any polyp/cystic fibrosis/bleeding disorder/Meckel’s diverticulum by any chance?
P: No.
D: Is he on any medications? P: No.
D: Has any member of your family been diagnosed with any medical condition? P: No.
D: Any member of the family has been diagnosed with any bowel problems such as
Polyposis? P: No doctor.
D: Has any member of your family got similar symptoms recently? (Diarrhoea, blood in the
stool, vomiting and tummy pain) P: No.

D: How was the birth of your baby? P: It was normal vaginal delivery.
D: Are you happy with the red book? P: Yes.
D: Is he up to date with all his jabs? P: Yes.
D: Has he received any recent jab? P: No
D: Is he feeding well other than recently? P: Yes. He is feeding very well.
D: Have you changed his milk recently? P: No.
D: Does he have any problems with his wee? P: No.
D: By any chance has your child taken any food from outside recently? P: No doctor.
D: Have you recently travelled anywhere with your child? P: No.
D: Who looks after him? P: It’s me

As your child was seen by the GP who performed Physical Examination and he found that
your little one looks pale, and his hands and feet are cold. He also looks lethargic. That can
be because of fluid loss as a result of diarrhoea and vomiting, which can be compensated.

On Examination of his tummy, there was a mass on the right side. We also checked your
little one’s Blood Pressure and breathing rate, which are fine, but his heart was beating a bit
fast.

I would like to send for some initial investigations including routine blood tests, Serum
Electrolytes, urine and stool tests.

From our assessment, your child has a condition called Intussusception. Intussusception
occurs when one part of the bowel slides into another part of the bowel like a telescope.
This causes the bowel to become blocked.

We did some routine blood tests to see if there are any changes in the level of chemicals in
the body. We have also sent urine test and stool sample to the lab.

We will do an abdominal X ray to check if there is any dilated gas-filled proximal bowel,
paucity of gas distally and multiple fluid levels. More importantly, we’ll be doing abdominal
ultrasound to confirm diagnosis.

www.aspire2plab.com 26
No real PLAB2 cases discussed. All discussed scenarios are fictitious and for educational purposes only.
The cause is unknown but it may be linked to infections.
This condition is the most common tummy emergency in small children, mostly between the
ages of 3 months and 2 years.

As part of initial management, we have started giving him painkillers for tummy pain and
fluids to compensate for the fluid loss. We may give him other medication such as
antibiotics. Since he is vomiting and not able to tolerate food and drinks, we need to give
this through his blood vessel as a drip.

When this happens, it is unlikely for it to be resolved by itself and medical intervention is
usually required.

A tube called ‘nasogastric tube’ is often passed from his nose into his stomach to drain any
stomach or bowel contents and get rid of any pressure that may build up due to the bowel
blockage.

If the ultrasound scan confirms the diagnosis, we will first try a treatment called ‘air enema’.
In this procedure, air is introduced through a tube into your child’s bottom, while X-ray
pictures are taken. The pressure of the air pushes back the telescoping parts of the bowel
(‘reduction’), which can be seen directly on the X-ray images. This is successful in 8-9 out of
10 patients.

If air enema is unsuccessful in reducing the intussusception (several attempts may be tried),
your child will need an operation. It is unlikely that it will subside on its own.

An operation will also be required if your child gets very unwell on admission to the
hospital, or if the doctors suspect that the bowel has perforated (burst) already.

During the operation, an incision is made to open the tummy and the bowel is exposed. The
surgeon gently separates the telescoping segments of the bowel. If any bowel tissue has
died due to lack of blood supply, or if any obvious cause (‘lead point’) of the intussusception
is found, this affected segment needs to be cut out (‘resection’). The two surrounding ends
are then stitched back together (‘Primary anastomosis’).

P: Doctor, how do you do the operation?


P: Doctor, Will it subside on its own?
P: When can I go home?
P: How is he going to be put to sleep before operation?
P: When can he start feeding after operation?

www.aspire2plab.com 27
No real PLAB2 cases discussed. All discussed scenarios are fictitious and for educational purposes only.
Pyloric Stenosis

You are FY2 in Paediatrics. Miss Katie Jones, aged 6 weeks, has been brought to the hospital
by her mother, Jane Jones, because of vomiting. Vitals have been recorded and are as
follows:

Pulse: 140, Respiratory rate: 40, Blood pressure: 100/70mmhg, SpO2: 96%, Temp: 36.8.
Please talk to the mother, take history, discuss the plan of management and address her
concerns. Baby is not in the cubicle

D: What brought you to the hospital?


P: My baby has been vomiting for 2 days every time I fed her.

D: How long does she take to vomit after you have fed her? P: After a few minutes.
(usually 30-60 minutes after feed)
D: Could you describe the vomiting? P: It’s like a fountain.
D: How much is it in amount? P: I’m breastfeeding & whatever I’m feeding she vomits it out.
D: What’s the content/colour of vomit? P: It’s just the milk that she had from me.
D: Is it digested or undigested milk? P: It is liquid.
D: Any blood in there? P: No doctor
D: Have you noticed anything else? P: No
D: Does your baby look hungry? P: Yes doctor.
D: Any dry mouth? P: I didn’t notice
D: Is your baby playful as before? P: She hasn’t been as playful as before since yesterday.
D: Does your baby seem floppy? P: Yes, since this morning.
D: Tell me about the little one’s poo. Is the baby passing stool?
P: Last time she passed stool was two days ago.
D: Have you noticed any mass in your baby’s tummy? P: No
D: Any temperature or flu like symptoms? P: No
D: Any diarrhoea? P: No
D: Have you noticed any tummy pain? P: I don’t think so

D: Has your baby been diagnosed with any medical condition? P: No


D: Any medication including OTC medicines? P: No
D: Does any member of the family have any medical conditions? P: No doctor.
D: Any member of the family had any similar complaints in childhood?
P: I am not aware of any one.

D: How was the birth? P: Normal vaginal delivery.


D: Is she up to date with her jabs? P: Yes doctor
D: Are you happy with the red book? P: Yes doctor
D: How is your baby’s feeding generally?
P: It was ok initially but since two days she has been vomiting out milk
D: How is your baby’s wee (if you forgot)? P: Doctor baby’s nappy is dry.

www.aspire2plab.com 28
No real PLAB2 cases discussed. All discussed scenarios are fictitious and for educational purposes only.
I would like to check your baby’s vitals and examine her tummy.
I would like to send for some initial investigations including routine blood test, Serum
Electrolytes, ABG.

pH: 7.5 (7.35-7.45)


pCO2: Increased (4.5-6 kPa) OR (35-45 mmHg)
pO2: Decreased (>10.6 kPa) OR (80-100 mmHg)
HCO3: 37mmol/l (19-25 mEq/L)
Na: 129mmol/l (135-145 mmol/L)
K: 2.7mmol/l (3.5-5 mmol/L)

From our assessment, it seems your child has a condition called pyloric stenosis. In this
condition, the outlet of the stomach into the small intestine is called the pylorus. Stenosis
means narrowing. Pyloric stenosis means a narrowed outlet of the stomach.

A narrowed or blocked outlet from the stomach doesn’t let milk pass from the stomach and
that’s why your little one has been vomiting undigested milk.

It is common in babies at the age of 2-8 weeks. It affects 2-4 out of 1000 new-born babies.
Let me tell you what we have done and what we will do for your little one.

We did some blood tests. Since your baby has been unwell and vomiting for a few days,
your baby has become dehydrated. Dehydration usually leads to an alteration in the
chemical balance of your baby’s blood. This condition is called metabolic alkalosis.

To correct this, we need to keep your little one in hospital. We are going to give your baby
fluids through a small tube into one of the blood vessels as a drip.

In this condition, the muscle in the wall of the outlet of the stomach into the small intestine
(pylorus) is abnormally thick. This causes the outlet to become narrowed (stenosed).
Although it is not known exactly why this occurs, genetics can have a role. To confirm the
diagnosis, we will feel your baby’s tummy. Sometimes the thickened pylorus can be felt as a
small hard lump during this examination. This is easier if the baby is feeding. This is known
as “test feed.” We may have to do this more than once to confirm the diagnosis.

We may consider doing an ultrasound scan to confirm the diagnosis.

Like I said, we are already giving your child fluids to correct the dehydration. Your baby will
need to have an operation under General Anaesthesia to cut some of the muscle fibres that
are causing the problem.

However, before your baby can be operated on, we will need to have another blood test to
see if the chemical balance in the baby’s blood has been corrected.

www.aspire2plab.com 29
No real PLAB2 cases discussed. All discussed scenarios are fictitious and for educational purposes only.
All feeds will be stopped until after the operation as your baby’s stomach needs to be empty
for the surgery. We will place a small tube into your baby’s nose and slide it down into the
stomach. It allows any fluid that collects in the stomach to be removed helping to prevent
your child from feeling sick before and after the surgery.
A small operation will normally solve the problem. This operation will be done after putting
your little one to sleep.

A small cut is made in the skin over the stomach outlet (pylorus). This operation is called
“pyloromyotomy.”

The pylorus is found and the muscle in the pylorus is then cut. This allows the pylorus to
widen into a normal size. This means that food and milk can pass easily out of the stomach
into the bowel. This operation is usually done by keyhole surgery.

This uses only a tiny cut to the skin to allow fine instruments into the tummy to cut the
pylorus muscle.

Open surgery is also a possibility, but the surgeon will discuss these options with you and
help you make the best decision for the baby. The operation is usually totally successful.
Normal feed is started again shortly after the operation, most babies recover quickly and
have no further problems.

P: Doctor, it seems like something serious.


P: Is she going to be alright?
P: How is surgery done?
P: When can I go home?

www.aspire2plab.com 30
No real PLAB2 cases discussed. All discussed scenarios are fictitious and for educational purposes only.
Constipation in Child

You are an FY2 in GP. Elizabeth Jakes mother of Daniel Jakes, aged 2 years, has come to see
you. Child was constipated. Daniel was examined. All the examinations were normal. She
was given dietary advice and was asked to come back after 2 weeks. She has come back
after 1 week. Talk to her and address her concerns.

D: What brought you to the clinic today? P: My child is having constipation, doctor.
D: I am so sorry. I understand that you came a week ago. P: Yes, doctor. His constipation is
still not resolved.
D: Don’t worry, I will ask you a few questions to see what’s going on. P: Ok
D: May I know what was told to you when you came last time?
P: They gave me dietary advice and asked me to give my child lots of fruits and vegetables
and plenty of water.

D: Were you giving him fibre diet? P: Yes.


D: Could you please elaborate what you give him throughout the day? P: Normal food.
D: Is he drinking enough water? P: I don’t think so, he doesn’t have water at all.
D: How about his physical activity? P: He is not that playful as before.
D: When was the last time he passed stool? P: 10 days ago
D: Did he pass any wind? P: Yes, this morning
D: Any overflow diarrhoea in between? P: No
D: Did he soil his clothes anytime? P: No
D: Does he know how to potty or are you training him now? P: Yes/No
D: Does he try to go to the loo at all every day?
P: Yes, he sits on the potty and cries sometimes.
D: Does he complain of any pain to defecate? P: Yes/No
D: Does he complain of any tummy pain? P: Yes/No

D: Did he have this problem in the past? P: No


D: Has your child been diagnosed with any medical condition? P: No
D: Is your child on any medication or is allergic to anything? P: No
D: Does anyone in the family have any medical conditions? P: No
D: How was the birth of your child? P: Normal delivery.
D: Are you happy with the red book? P: Yes
D: Is he up to date with all his jabs? P: Yes
D: Do you have any other children? P: No, he is my first child.
D: Who looks after the child? P: Me and my husband

D: What do you do for a living? P: Manager


D: Does he go to school/nursery? P: Yes/No
D: Does he have any stress or trouble going to school? P: No

I would like to examine your child. Check his vitals, do head to toe examination and tummy
examination.
Examination: All examinations normal.

www.aspire2plab.com 31
No real PLAB2 cases discussed. All discussed scenarios are fictitious and for educational purposes only.
From my assessment, your child has constipation. As you told me that he cries sometimes
when he is on the potty, he might be having hard stools due to constipation. Constipation is
common in childhood, particularly when children are being potty trained at around two to
three years old. If your child is constipated, they may find it painful to poo. This can create a
vicious circle: the more it hurts, the more they hold back. The more constipated they get,
the more it hurts, and so on.
Your child may be constipated because they:
- aren't eating enough high-fibre foods like fruit and veg
- aren't drinking enough water
- are having problems with potty (or toilet) training
- are worried or anxious about something, such as moving to a new house, starting
nursery or the arrival of a new baby

Laxatives are often recommended for children who are eating solid foods, alongside diet
and lifestyle changes. It may take several months for the treatment to work but keep trying
until they do. Laxative treatment may make your child have overflow soiling, before it gets
better. We will reassess your child after the constipation is resolved and if needed, we may
have to prescribe him laxatives for a longer duration. Try to stay calm as this can be stressful
for the both of you.
D: How to prevent constipation:
- Make sure your child has plenty to drink
- Give your child a variety of foods, including plenty of fruit and vegetables, which are
a good source of fibre.
- Encourage your child to be physically active.
- Get your child into a routine of regularly sitting on the potty or toilet, after meals or
before bed, and praise them whether or not they poo. This is particularly important
for potty-trained boys, who may forget about pooing once they are weeing standing
up.
- Make sure your child can rest their feet flat on the floor or a step when they're using
the potty or toilet, to get them in a good position for pooing.
- Ask if they feel worried about using the potty or toilet – some children don't want to
poo in certain situations, such as at nursery or school.
- Stay calm and reassuring, so that your child doesn't see going to the toilet as a
stressful situation – you want your child to see pooing as a normal part of life, not
something to be ashamed of.

Please come back to the hospital if your child develops any severe tummy pain, tummy
distension or vomiting.

P: Why is he still constipated?


P: What are you going to do?
P: Are you going to give him any medications?
P: I don’t want my child to have these medications for life long.

www.aspire2plab.com 32
No real PLAB2 cases discussed. All discussed scenarios are fictitious and for educational purposes only.
Neonatal Jaundice

You are an FY2 in the Paediatric ward. Mrs. Layla Hussain, the mother of a 15-day old child,
was referred to the paediatrics ward by midwife. Weight of the child is 3.7 kg. Talk to the
mother, assess the child and discuss the initial plan of management.

D: How can I help you?


P: My midwife observed that my child has yellow discoloration of the eyes. She told me he
has jaundice.

D: Could you please tell me more about it? P: What would you like to know?
D: Since when? P: I don’t know, the midwife noticed it
D: Did you notice any other symptoms? P: No doctor.
D: Have you noticed yellowish discoloration of the palms or soles? P: No
D: Have you noticed any yellow discoloration inside the mouth? P: No
D: Any change in the color of urine and stool? P: No.
D: Any fever or flu-like illness? P: No
D: Any vomiting? P: No / Yes
D: Diarrhea? Constipation? P: No
D: Did you notice any abdominal distension? P: No
D: Is she playful? P: Yes
D: Is she feeding well? P: Yes. She is feeding very well.
D: Is she breast fed? P: Yes
D: Any change in the amount of milk he is taking? P: No

D: Has she been diagnosed with any medical condition in the past? P: No
D: Is she taking any medications including OTC or supplements? P: No
D: Any allergies from any food or medications? P: No
D: Has anyone in the family been diagnosed with any medical condition? P: No

D: How was the birth of your baby? P: It was normal vaginal delivery.
D: Was your baby delivered at term or post term? P: At term
D: Are you happy with the red book? P: Yes.
D: Is she up to date with all her jabs? P: Yes.
D: Has she received any recent jab? P: No

D: Who looks after her? P: I do.


D: Do you have other kids? P: Yes
D: Did any of them have such a condition? P: No

I would like to examine the child. I would like to check vitals, GPE, eye examination and
gums, tummy. We will also look at the colour of the urine and poo.

I would like to do some routine blood investigation to see if there is any bug. We will also do
blood grouping, LFT, direct and indirect bilirubin level in the blood of your child and
depending on that we will give him the treatment.

www.aspire2plab.com 33
No real PLAB2 cases discussed. All discussed scenarios are fictitious and for educational purposes only.
Examiner:
Abdominal examination is normal.
Bilirubin Level is below the treatment level.
There is yellow discoloration of the eyes and skin.

We have examined your child and his tummy is fine, and there is yellow discoloration of the
eyes and skin. It is called Neonatal Jaundice.

Newborn babies have a large number of red blood cells in their blood. They are broken
down & replaced frequently. When the red blood cells are broken down a yellow substance
is produced called bilirubin, which is removed from the body by the liver. The liver of
newborn babies is not fully developed so cannot remove all the bilirubin from the body thus
it gives a yellow colour to the skin & eyes. When the baby is older, the liver works better &
starts removing the bilirubin & thus the yellowness of skin disappears.

Treatment is usually only necessary if your baby has high levels of a substance called
bilirubin in their blood. We have checked the level of bilirubin in your child’s blood which is
below the treatment level.

Most babies with jaundice don't need treatment because the level of bilirubin in their blood
is found to be low. In these cases, the condition usually gets better within a few weeks and
won't cause any harm to your baby.

If treatment is felt to be unnecessary, you should continue to breastfeed or bottle feed your
baby regularly, waking them up for feeds if necessary. If your baby's condition gets worse or
doesn't disappear after two weeks, contact your midwife, health visitor or GP.

Please keep your child under observation. If you notice any change in the colour of the skin,
eyes, wee or poo then please immediately come back to us.

If your baby's jaundice doesn't improve over time or tests show high levels of bilirubin in
their blood, they may be admitted to hospital and treated with phototherapy or an
exchange transfusion.

Breast milk jaundice can last for 3-12 weeks after birth, but as long as the baby is feeding
well and bilirubin levels are monitored, it rarely leads to serious complications.
Breast milk jaundice must also be differentiated from breastfeeding jaundice, which is
jaundice resulting from an insufficient intake of milk.

Normal physiologic jaundice of the newborn typically appears between the 2nd and 5th
days of life and clears on its own within two weeks.

Further blood tests may need to be carried out if your baby's jaundice lasts longer than two
weeks or treatment is needed. The blood is analyzed to determine:

www.aspire2plab.com 34
No real PLAB2 cases discussed. All discussed scenarios are fictitious and for educational purposes only.
- the baby's blood group – this is to see if it's incompatible with the mother's
- whether any antibodies (infection-fighting proteins) are attached to the baby's red blood
cells
- the number of cells in the baby's blood
- whether there's any infection
- whether there's an enzyme deficiency

These tests help determine whether there's another underlying cause for the raised levels of
bilirubin.

Phototherapy:
Phototherapy is treatment with light. It is used in some cases of newborn jaundice to lower
the bilirubin levels in your baby's blood through a process called photo-oxidation.
Photo-oxidation adds oxygen to the bilirubin so it dissolves easily in water. This makes it
easier for your baby's liver to break down and remove the bilirubin from their blood.

Exchange transfusion:
A blood transfusion, known as an exchange transfusion, may be recommended if your baby
has particularly high levels of bilirubin in their blood or if phototherapy hasn't been effective.
During an exchange transfusion, small amounts of your baby's blood are removed through a
thin plastic tube placed into blood vessels in their umbilical cord, arms or legs. The blood is
then replaced with blood from a suitable matching donor (someone with the same blood
group). As the new blood won't contain bilirubin, the overall level of bilirubin in your baby's
blood will fall quickly.

Your baby will be monitored throughout the transfusion process, which can take several
hours to complete. Any problems that may arise, such as bleeding, will be treated.
Your baby's blood will be tested within two hours of treatment to check if it's been
successful. If the level of bilirubin in your baby's blood remains high, the procedure may need
to be repeated.
!

www.aspire2plab.com 35
No real PLAB2 cases discussed. All discussed scenarios are fictitious and for educational purposes only.
Bronchiolitis

You are an FY2 in Paediatric emergency. Mrs Saira Melik, has brought her 4 month old child
Abdullah for an emergency appointment. Please talk to her and address her concerns.

D: How can I help you today? P: Doctor, my son has been snotty for the last 2-3 days.
D: Can you tell me more about it? P: What do you want to know?
D: Can you please tell me what happened?
P: He was fine 3 days ago, and then he just became snotty. I gave him paracetamol syrup
but that didn’t help. Now I feel that his chest is also full.
D: Does he have a cough? Sputum? P: No (CF)
D: Any fever? P: He does feel a little hot, but I haven’t
checked
D: Is he crying? P: Yes, a lot.
D: Have you noticed any rash? P: No (Meningitis)
D: Have you noticed that your child is shy to light or cries while moving his neck? P: No
D: Have you noticed any difficulty in breathing? P: I just feel that his chest is full
D: Any vomiting? P: No
D: Do you feel that his mouth is dry? P: No (Dehydration)

D: Has it ever happened before? (CF) P: No, this is the first time
D: Has he been diagnosed with any medical condition in the past? P: No
D: Is he taking any medications other than PCM including OTC or herbal medications? P: No
D: Any allergies from any food or medications? P: No
D: Is there any other child around him with a similar condition? P: No (siblings)

D: How was the birth of your baby? P: It was normal vaginal delivery.
D: Was he born at term? P: No, he was born at 35 weeks.
D: How much was the birth weight? P: Normal
D: Are you happy with the red book? P: Yes.
D: Is he up to date with all his jabs? P: Yes.
D: Has he received any recent jab? P: No
D: Is he feeding well? P: No.
D: Since when? P: Since his nose became stuffy
D: Does he have any problems with his wee? P: No.
D: Have you noticed any tummy pain or change in his poo? (Gastroenteritis). P: No
D: Any diarrhea? P: No (CF)
D: How is the urine output? P: It is fine.
D: Who looks after her? P: It’s me

I would like to examine little Abdullah to assess him better. I would do a general physical
examination & would examine his chest.

Examiner:
Bilateral wheeze & crackles on auscultation

www.aspire2plab.com 36
No real PLAB2 cases discussed. All discussed scenarios are fictitious and for educational purposes only.
We would also like to do a few investigations. I would like to do pulse oximetry. We would
also take some swab/specimens from the nose & send for viral cultures.

We will keep him in the hospital as he was born 35 weeks of gestation. If needed we will
give him oxygen. If the child has the virus (RSV) then we need to keep him away from other
children.

This kind of infection usually goes away within 2 weeks on its own.

There's no medication to kill the virus that causes bronchiolitis. It’ll become better without
any treatment.

We usually give paracetamol or ibuprofen to bring the fever down, but you are already
giving him that, just continue it till his fever goes down. Also because of this infection, the
fluids in the child’s body can decrease from the normal levels, so take care that you give him
enough fluids otherwise he can become dehydrated.

If at any time you see that little Abdullah has difficulty in breathing (becomes blue), or is not
feeding well (dehydration), or becomes less active (drowsy), please bring him to the hospital
immediately.

Prevention:
1. wash your hands and your child's hands frequently
2. wash or wipe toys and surfaces regularly
3. keep infected children at home until their symptoms have improved
4. keep new-born babies away from people with colds or flu
5. prevent your child being exposed to tobacco smoke.

There are also a number of factors that can increase the risk of a child developing more
severe bronchiolitis like:
1. being under two months of age
2. having congenital heart disease
3. being born prematurely (before week 37 of pregnancy)
4. having chronic lung disease of prematurity (when injury to the lungs causes long-term
respiratory problems in premature babies)
Some signs of severe disease include:

1. poor feeding (less than half of usual fluid intake in preceding 24 hours)
2. significantly decreased activity
3. history of stopping breathing (Apnoea)
4. respiratory rate >70/min
5. presence of nasal flaring and/or grunting
6. severe chest wall recession (Hoover’s sign)
7. bluish skin.

www.aspire2plab.com 37
No real PLAB2 cases discussed. All discussed scenarios are fictitious and for educational purposes only.
Other investigations done in Atypical bronchiolitis:
1. CXR (When diagnosis is not certain)
2. FBC
3. U&Es (If child is dehydrated)
4. Blood and urine culture (If Fever >38.5)
5. ABG (Those who need mechanical ventilation)

Differentials:
1. Bronchiolitis
2. Cystic fibrosis
3. Asthma
4. Bronchitis
5. Foreign body
6. Pneumonia

www.aspire2plab.com 38
No real PLAB2 cases discussed. All discussed scenarios are fictitious and for educational purposes only.
Paediatrics – Telephone Conversation

You are an FY2 in Paediatrics. Mr John David, aged 10 months, has been sick for 2 days and
is on triage care call. His mother, Mrs. Maria David is concerned her sons health. Talk to the
mother, discuss the initial plan and address her concerns.

(Mother is very worried and concerned)

D: Hello. Confirm the mother’s name, child’ name and age.


M: Dr, can you please come and see my baby immediately, he is ill.

D: I am so sorry to hear that. We are here to help your baby. Could you please tell me what
exactly happened?
M: My baby has fever for 2 days and he is not feeling well.

D: Could you please tell me more about the temperature?


M: He has got very high temperature.

D: Have you measured the temperature?


M: Yes doctor. I measured it with my home thermometer, and it was 39 C.

D: Have you done anything for it?


M: I gave him paracetamol, but it has not improved.
D: How much paracetamol did you give?
M: 1 Spoon

D: Have you noticed any other symptoms? M: Like what doctor?

D: Any cough? M: Yes, since yesterday.


D: Any phlegm along with cough? M: No doctor.
D: Any difficulty in breathing? M: Yes. He has had difficulty in breathing since yesterday.
D: Any rashes over the body? M: No
D: Any difficulty in moving his neck? M: No
D: Does he shy away from light? M: No
D: Any headache? M: No
D: Any ear pain or discharge? M: No
D: Any vomiting? M: No
D: Any diarrhoea or loose stools? M: No
D: Is he passing urine normally? M: No, I haven’t changed his diaper since yesterday.
D: Any tummy pain? M: No

D: Is your baby active? M: No doctor, he is lethargic and tired.

D: Has your baby been diagnosed with any medical conditions in the past? M: No
D: Is he on any regular medications including OTC or supplements? M: No
D: Any family member with any medical problems? M: No
D: Any family member with any similar complaint? M: No

www.aspire2plab.com 39
No real PLAB2 cases discussed. All discussed scenarios are fictitious and for educational purposes only.
D: How was the birth of your baby? M: It was a normal delivery
D: Are you happy with the red book? M: Yes
D: Is he up-to-date with all his jabs? M: Yes
D: Did he receive any recent jab? M: No

D: Is he feeding well? M: No doctor he hasn’t


been feeding well since yesterday.
D: Have you travelled anywhere with your child recently? M: No
D: Who looks after your baby? M: I do.

D: Thank you for answering all my questions. I suspect your baby might be having a chest
infection. He needs immediate admission.
M: What should I do now doctor?

D: Don’t worry, I am going to send an ambulance to get your baby to the hospital. We will
have to examine him, do some blood test and a chest x-ray. If we find it is an infection, we
will give him antibiotics.
M: Ok Dr. When will the ambulance arrive?

D: We will send the ambulance immediately. Hopefully, it should reach you soon.
M: Ok Dr.
D: Do you have any other concerns?
M: No

D: Thank you.

www.aspire2plab.com 40
No real PLAB2 cases discussed. All discussed scenarios are fictitious and for educational purposes only.
Child Development Milestone

You are FY2 in Paediatrics. Mrs Hannah Williams, mother of Jenny Williams, aged 15
months, comes to the hospital with a concern that her child is not able to walk. The mother
is very concerned. Ask relevant questions and address her concerns.

D: How can I help you? P: I’m worried my child can’t walk.


D: I can see that you are worried about your child. I am here to help. Let me ask you a few
questions.
D: How old is your child? P: 15 months.
D: Is your child able to stand independently? P: No
D: Is your child able to stand by holding on to something? P: No
D: Have you tried making your child stand up?
P: I tried but fell, so I didn’t try. I can’t see my child falling and getting hurt.
D: Does your child sit with/without support? P: Yes/No
D: Does your child crawl? P: Yes
D: Does your child roll over? P: Yes/No
D: Does your child hold a cup? P: Yes/No
D: For how long? (if above answer is yes) P:
D: How does your child hold the cup? P:
D: How do you play with your child? P:
D: Does she talk to you? P: Yes, says Mama.
D: Does your child respond to you? P: Yes, when I clap.
D: Does your child respond to their name? P: Yes/No
D: Does your child call out your name? P: Yes, says Mama.
D: Does your child show anxiety to a stranger? P: Yes/No
D: Does your child play peek-a-boo? P: Yes/No
D: Does your child wave ‘bye bye’? P: Yes/No
D: Is your child up to date with the jabs? P: Yes/No

D: How has your child been recently? P: Fine


D: Has your child been diagnosed with any medical condition? P: Yes/No
D: Any bone disorders? P: No
D: Is your child on any medication or is allergic to anything? P: Yes/No
D: Did you or your partner/husband face any development delay? P: Yes/No
D: What is your child’s diet like? P: Stopped taking breast milk at 12 months.
D: Have you started feeding your child with bottle milk? P: Yes/No
D: Does your child eat food? P: Yes, baby food in puree form.
D: How is the wee and poo of your child? P: Normal
D: Do you have any other children? P: No, it’s my first baby.

D: Who looks after the child? P: Me and my husband


D: What do you do for a living? P: Manager
D: Any stress at work/home? P: Yes/No

I would like to assess your child.


We might refer your child to the specialist for further assessment if need be.

www.aspire2plab.com 41
No real PLAB2 cases discussed. All discussed scenarios are fictitious and for educational purposes only.
From the history it seems like your child at this moment in time doesn’t not have any
development delay. However, we will keep monitoring the progress of your child.

I can see that you are worried about your child development but let me reassure you that
we will together take care of her.

P: How can I help my baby to walk?


D: As your baby learns to stand, he may need some help working out how to get back down
again. If he gets stuck and cries for you, don't just pick him up and plop him down. Instead,
show him how to bend his knees so he can sit down without toppling over, and let him give
it a try himself.

You can encourage your baby to walk by standing or kneeling in front of him, holding both
his hands as you help him walk towards you.

You could also buy him a toddle truck or a similar toy that he can hold on to and push. Look
for toddler toys that are stable and have a wide base of support. Baby walkers can cause
accidents by tipping over, so it's best not to use them.

As your baby learns to walk, it's a good idea to keep his feet as free as possible. Let him
toddle barefoot if you can. Going barefoot helps him to improve his balance and
coordination. If cramped by tight shoes or socks, your baby's feet can't straighten out and
grow properly.

As long as your baby is bearing weight on his legs and shows an interest in learning new
things, you don't have to be concerned. If your baby took a little longer than other babies to
learn to crawl, chances are he'll need a few more months for walking as well. Babies
develop skills differently, some more quickly than others.

Remember that if your baby started to move around by bottom shuffling before he learned
to crawl, then he may walk later.

The speed at which a baby learns a new skill is often inherited from his parents. If you or
your partner walked early or late, then there is a chance your baby will be the same.

www.aspire2plab.com 42
No real PLAB2 cases discussed. All discussed scenarios are fictitious and for educational purposes only.
Normal Development
Gross Motor Fine Motor & Vision Hearing & Language Social

Neonate Moves all limbs Looks, startles Startles to noise Cries


Smiles (6 Weeks)
3 Months Head control Reach for objects Cries, laughs, vocalizes Laughs
Fixes & follows (4months)

6 Months Sits Co-ordination Localises sound Alert and interested


Transfers Babbles Starts solids

9 Months Crawls Pincor grip Inappropriate sounds Stranger anxiety

12 Months Stands Babbles Socially responsive


Walks at 15m Understand simple Wave bye
commands
Says Mamma/Dadda

2 Years Runs Circular scribbles & lines 2 word phrases Uses fork and spoon
Stairs
3-4 Years Stand on one foot Builds bridge with bricks 3 word sentences Interactive play
Knows colours

5 Years Skips/Hops Full drawing Fluent speech Dresses self

If a certain milestone is not reached by the respective date you should refer for a
specialist community paediatric assessment:

• Unable to sit unsupported at 12 months of age


• Unable to walk by 18 months of age
• No speech at 18 months of age (at 15 months onwards, if unable to speak monosyllabic words
and difficulty understanding speech, consider arranging hearing test)
• Unable to run by 2.5 years of age
• Unable to hold objects placed in hand by 5 months of age
• Unable to reach for objects by 6 months of age

www.aspire2plab.com 43
No real PLAB2 cases discussed. All discussed scenarios are fictitious and for educational purposes only.
Gross Motor Development:
Limbs flexed, symmetrical pattern
Newborn
Marked head lag on pulling up

6 – 8 weeks Raises head to 45 degrees in prone (tummy-time)


Sits without support (initially with a curved back, then eventually with a straight
back by 8 months)
6 – 8 months
Limit age: 9 months

8 – 9 months Crawling

Stands independently
10 months
Cruises around furniture
Walks unsteadily – a broad gait, with hands apart
12 months
Limit age: 18 months

15 months Walks steadily


2.5 years Runs and jumps

Vision and Fine Motor Development

Follows moving object or face by turning the head (fixing and


following)
6 weeks
Limit age: 3 months

Reaches out for toys


4 months
Limit age: 6 months

4 – 6 months Palmar grasp

Transfers toys from one hand to another


7 months
Limit age: 9 months

Mature pincer grip


10 months
Limit age: 12 months
16 – 18 months Makes marks with crayons

www.aspire2plab.com 44
No real PLAB2 cases discussed. All discussed scenarios are fictitious and for educational purposes only.
Tower of three – 18 months

Tower of six – 2 years


14 months – 4 years
Tower of eight or a train with four bricks – 2.5 years
Brick building
Bridge (from a model) – 3 years

Steps (after demonstration) – 4 years

Line – 2 years
2 – 5 years Pencil skills (see
below): Circle – 3 years

Drawing without seeing how it is Cross – 3.5 years


done.
Square – 4 years
Can copy 6 months earlier.
Triangle – 5 years

Hearing, Speech and Language Development

Newborn Startles to loud noises

3 – 4 months Vocalises alone or when spoken to, coos and laughs “aa, aa”

Turns to soft sounds out of sight


7 months
Polysyllabic babble (“babababa, lalalalala”)

Sounds used indiscriminately at 7 months


7 – 10 months
Sounds used discriminately to parents at 10 months “Dada, Mama”

Two to three words other than ‘Dada’ or ‘Mama’


12 months
Understands name “Drink”

6-10 words
18 months
Is able to show two parts of the body “Where is your nose?” – Baby will point
20 – 24 months Joins two or more words to make simple phrases “Give me teddy”

Talks constantly in 3 – 4-word sentences


2.5 – 3 years
Understands 2 joined commands “Push me fast Daddy”

www.aspire2plab.com 45
No real PLAB2 cases discussed. All discussed scenarios are fictitious and for educational purposes only.
Social, Emotional and Behavioural Development

Smiles responsively
6 weeks
Limit age: 8 weeks

6 – 8 months Puts food in mouth

10 – 12 months Waves bye-bye, plays peek-a-boo

12 months Drinks from a cup with two hands


18 months Holds spoon and gets food safely to mouth

Symbolic play
18 – 24 months
Limit age: 2 – 2.5 years

Toilet training: dry by day


2 years
Pulls off some clothing

Parallel play

2.5 – 3 years Interactive play evolving

Takes turns

www.aspire2plab.com 46
No real PLAB2 cases discussed. All discussed scenarios are fictitious and for educational purposes only.
Twin Delay

You are an F2 in GP. Miss Diana Whales, mother of 15 months old boy and a girl came to
the clinic because of some concern. Please talk to her and discuss your plan of management
with her and address her concerns. Children are not in the cubicle.

Reassurance to the mother.


D: How can I help you? P: I’m worried my child can’t talk.
D: I can see that you care about your child. P: Let me ask you a few questions.
D: How old is your child? P: 15 months.
D: Is your child able to stand independently? P: Yes
D: Is your child able to stand by holding on to something? P: Yes
D: Does your child sit with/without support? P: Yes
D: Does your child crawl? P: Yes
D: Does your child roll over? P: Yes
D: Does your child hold a cup? P: Yes
D: For how long? (if above answer is yes) P:
D: How does your child hold the cup? P:
D: How do you play with your child? P:
D: Does she talk to you? P: Yes, but he only speaks 2 words Dada and mama
D: Does your child respond to you? P: Yes, when I clap.
D: Does your child respond to their name? P: Yes
D: Does your child call out your name? P: Yes, says Mama.
D: Does your child show anxiety to a stranger? P: Yes
D: Does your child play peek-a-boo? P: Yes
D: Does your child wave ‘bye bye’? P: Yes
D: Is your child up to date with the jabs? P: Yes
D: How has your child been recently? P: Fine

D: Has your child been diagnosed with any medical condition? P: Yes/No
D: Any bone disorders? P: No
D: Is your child on any medication or is allergic to anything? P: Yes/No
D: Did you or your partner/husband face any development delay? P: Yes/No

D: What is your child’s diet like? P: Stopped taking breast milk at 12 months.
D: Have you started feeding your child with bottled milk? P: Yes/No
D: Does your child eat food? P: Yes, baby food in puree form.
D: How is the wee and poo of your child? P: Normal
D: Do you have any other children? P: Yes, I have a twin daughter
D: How is she doing?
P: She is doing well. In fact, she is saying many words compared to her brother
D: Who looks after the children? P: Me and my husband

D: What do you do for a living? P:


D: Any stress at work/home? P: Yes/No

www.aspire2plab.com 47
No real PLAB2 cases discussed. All discussed scenarios are fictitious and for educational purposes only.
I would like to assess your child. We might refer your child to the specialist for further
assessment if need be.

From the history it seems like your child at this moment in time doesn’t not have any
development delay. However, we will keep monitoring the progress of your child.
I can see that you are worried about your child's development but let me reassure you that
we will together take care of her.

P: How can I help my baby to talk?


D:
12 to 18 months
● If your child is trying to say a word but gets it wrong, say the word properly. For
example, if they point to a cat and say "Ca!" you should respond with, "Yes, it's a
cat". Do not criticise or tell them off for getting the word wrong.
● Increase your child's vocabulary by giving them choices, such as, "Do you want a
apple or a banana?".
● Toys and books that make a noise will help your child's listening skills.
● Enjoy singing nursery rhymes and songs together as your baby grows, especially
those with actions, such as "Pat-a-cake", "Row, row, row your boat" and "Wind the
bobbin up". Doing the actions helps your child to remember the words.
The speed at which a baby learns a new skill is often inherited from his parents. If you or
your partner walked early or late, then there is a chance your baby will be the same.

www.aspire2plab.com 48
No real PLAB2 cases discussed. All discussed scenarios are fictitious and for educational purposes only.
Child Waking up at Night (Night Terrors)

You are FY2 in GP Surgery. Mrs Victoria Smith aged 33 has made an urgent appointment.
She has concerns about her daughter Jasmine Smith aged 5. Talk to her and address her
concerns.

D: How can I help you?


P: My daughter Jasmine is waking up at night and screaming. I can’t put her back to sleep.

D: Tell me more about it? P: What would you like to know?


D: Since when has this been happening? P: It started 4 months ago.
D: How often does this happen? P: Every other day
D: Is it regular since it started? P: Yes
D: What time of the night does she wake up? (Night Terrors-early part of night) P: Midnight
D: How many episodes in a single night? P: 1
D: How was she before that? P: She was fine
D: Any (tragic/traumatic) event occurred recently? P: She started school 5 months back.
D: How is she coping up with it? P: She is loving it and she is happy.
D: Do you think she is getting tired/distressed with the school? P: Yes/No
D: Anything else she does when she wakes up? P: No
D: Does she recognise you? P: Yes/No
D: Have you spoken to Jasmine about this? P: Yes/No
D: Tell me more about Jasmine. P: What do you want to know?
D: How has she been recently? P: She has been fine
D: Has she got fever or flu like symptoms? P: No
D: How is her sleep pattern? P: Ok
D: Does she go to the toilet before going to sleep? P: Yes/No
D: How much does she play on the smart devices? (phones/tablets) P: Limited
D: Parental control on? (Violent videos) P: Yes
D: What time does she go to sleep? P: Usual 8pm
D: How is her diet? P: Good
D: Is she up to date with her jabs? P: Yes
D: Are you happy with the Red Book? P: Yes
D: How is her mood? P: She is cheerful and playful
D: Is she lethargic? P: No

D: Has she been diagnosed with any medical conditions? P: No


D: Is she on any medications? P: No
D: Any allergy? P: No
D: Anyone in your family have a similar problem? P: No
D: How is the home condition? P: Good
D: Who looks after Jasmine? P: Me and my husband
D: Is your husband the biological father? P: Yes
D: How is your relationship with your husband? P: Good

D: Have you had any arguments in front of Jasmine? P: No, we are a loving family.

www.aspire2plab.com 49
No real PLAB2 cases discussed. All discussed scenarios are fictitious and for educational purposes only.
I will need to examine your child and order some investigations.

From my assessment, it seems that Jasmine may be having Night Terrors.


They are not dangerous, and they usually grow out of them by the age of 8.

Before Sleep:
Advice about sleep hygiene
During Episode:
Do not wake up your child
After Episode:
Ask the child if she is ok in the morning
Avoid discussion of episode to prevent anxiety

General Advice:
Sleep Diary:
● Time to bed
● Time to rise
● Any Episode 1) number 2) duration
● Any distress expressed
● Any routine change

Come back to us if episodes occur a few times during the night or most nights. We will try to
find out whether something that’s easily treatable.

Sometimes referral to specialists is done for those children who have frequent episodes.
If the night terror is frequent and occurs at specific times every night, you may find that
waking your child may break the cycle. Wake your child up 15 minutes before anticipated
time of the episodes every night for 7 days.

Nightmare Night Terror


Occurs during REM sleep Occurs during non-REM sleep and often
between sleep cycles Crying, thrashing, and
rapid breathing

No crying or moving during dream Crying, thrashing, and rapid breathing

Will likely remember the dream No recollection of event

Occurs in the early morning hours [2nd half Occurs within 90 minutes of bedtime [1st
of the night] third of the night]

More likely to occur at times of stress, Hereditary or overtiredness


illness, or when overtired

Peaks between 3-6 years of age Can begin at 18 months, peaks at 5-7 years
of age

www.aspire2plab.com 50
No real PLAB2 cases discussed. All discussed scenarios are fictitious and for educational purposes only.
Autism

You are a FY2 in GP. Mrs Vicky Becker, mother of a 3-year-old child Jason is concerned about
her son. Talk to her and address her concerns.

D: How can I help you? P: I am very concerned about my son.


D: May I know what your concern is?
P: His health assessor is worried that Jason is not interacting well with other children at the
nursery.
D: Could you please tell me what exactly you mean by not interacting?
P: He does not pay attention to me.

D: Did you notice that yourself? P: Yes


D: May I know, since when you started noticing this? P: Last few weeks
D: Does he respond to his name? P: Yes
D: Is he avoiding eye contact or not smiling when you smile at him? P: Yes
D: Does Jason have any kind of repetitive movements, such as flapping his hands, flicking his
fingers? P: Yes
D: Does he repeat the same phrases? P: Yes
D: Is he talking as much as other children? P: No
D: Is he walking and playing? P: Yes, he plays with blocks.

D: Is there any trouble/stress at school? P: I don’t know


D: Are you happy with his red book? P: Yes/No
D: Did any incident take place after which Jason is not interacting well? P: I Don’t know
D: How is his health recently? P: Good
D: Has your child been diagnosed with any medical condition? P: No
D: Is your child on any medication or is allergic to anything? P: No

D: Did you or your partner/husband face any development delay? P: Yes/No


D: What is your child’s diet like? P: Good
D: Does your child eat food? P: Yes
D: How is the wee and poo of your child? P: Normal
D: Is he up to date with all the jabs? P: Yes
D: Do you have any other children? P: No
D: Who looks after the child? P: Me
D: What do you do for a living? P: Manager

D: I would like to talk to Jason and examine him.

P: What’s going on?


D: From my assessment, I suspect your son might be having Autism. I need to involve my
seniors and ask them to review Jason and confirm the diagnosis.
D: The detailed assessment, management, and care and support for your child should
involve local specialist community-based multidisciplinary teams (sometimes called "local
autism teams") working together.

www.aspire2plab.com 51
No real PLAB2 cases discussed. All discussed scenarios are fictitious and for educational purposes only.
The team may include:
• a paediatrician
• mental health specialists, such as a psychologist and psychiatrist
• a learning disability specialist (if appropriate)
• a speech and language therapist
• an occupational therapist
• education and social services representatives from your local council

P: What are you going to do for him?


D: An autistic spectrum disorder (ASD) is a lifelong condition. And unfortunately, there's no
"cure" for autism, but with the right support, many autistic people live fulfilled and active
lives. As the severity can vary, it is difficult to predict the outcome for each child.
Some adults with ASD manage to work and get by with just a little support. In particular,
many people with Asperger's syndrome are able to manage well and live independently or
need little support when adult.

o The mainstay of treatment is special education support.


o Behavioural therapy.
o Medication may be considered to help with specific ASD-related symptoms. These
symptoms may be anxiety, depression or obsessive-compulsive disorder, outbursts of
excitement or aggression. Medication can also be used to help with sleep, epilepsy and
also with any repetitive behaviours.

P: Why did he get this?


D: The exact cause of autism is currently unknown. Autism is a complex
neurodevelopmental condition. The causes are still being investigated. Current evidence
suggests that autism may be caused by many factors that affect the way the brain develops.
These include:
- genetics
- environmental triggers like lifestyle factors, such as diet and exercise, being born
prematurely, being exposed to alcohol or certain medicines, such as sodium
valproate in the womb.

P: Is this because of the MMR vaccine?


D: No, this is not because of the MMR vaccine. (Explain about MMR vaccine and autism
study).

D: We have support services to support your child, family, carers and friends - National
Autistic Society.

D: How to communicate with your autistic child


Communication can be particularly difficult for autistic children and young people.
Helping them communicate can reduce anxiety and the risk of behavior that may be difficult
or challenging.
Try these tips when interacting with your child:
- use your child's name so they know you're addressing them
- keep background noise to a minimum

www.aspire2plab.com 52
No real PLAB2 cases discussed. All discussed scenarios are fictitious and for educational purposes only.
- for some autistic children, it can help if you keep language simple and literal
- speak slowly and clearly
- some parents find it useful to accompany what they say with simple gestures or
pictures
- allow extra time for your child to process what you have said

Help for behaviour that may be seen as challenging.


It's important to remember that behaviour is a way of communicating. If your child is
behaving in a challenging way and this is affecting family life, ask for help and support from
a GP or another healthcare professional.
PEADIATRICS
Dx AUTISM
S/S 1. Not responding to their name,
2. Avoiding eye contact,
3. Not smiling when you smiling at them,
4. Repeating same phrase and movements like flapping of hands,
5. Not talking as much as other child,
6, Developmental delay,
7. Not socializing with others.
RF Genetics, Lifestyle factors such as Diet, Premature birth, Exposed to alcohol and drugs like sodium Valproate.

Mx 1. Use your child name.


2. Simple Gestures, pics, language. Speak slowly
3. Keep background noise to minimum.
4. Medications.

Support:
1. Family, friends.
2. National Autistic Society, Local Support group.
3. Social media, Special Education, Behavior therapy.
4. Financial benefits if needed.

www.aspire2plab.com 53
No real PLAB2 cases discussed. All discussed scenarios are fictitious and for educational purposes only.
Temper Tantrums

You are an FY2 in GP. Mrs Daniella McLeod, mother of Mr David McLeod who 3 years old,
has come to you with concerns. Please talk to her and address her concerns.

D: How can I help you? P: My son is throwing things like his toys and food everywhere.
D: Can you please elaborate more?
P: He doesn’t listen to me and when I hold him, he bites and when playing with other kids,
he kicks and hits them.
D: Since when has this been happening? P: Last few months.
D: Anything in particular that happened? P: I don’t know.
D: How often does this happen? P:
D: Has it been regular since it started? P: Yes/No
D: How was he before that? P: Fine
D: Anything else? P: Like what?

D: Does he respond to sounds around him? (Hearing loss) P: Yes


D: Does he respond to his name? (Autism) P: Yes
D: Does he make eye contact while talking? (Autism) P: Yes
D: Does he engage in repetitive movements? (Autism) P: No
D: Is he unable to sit still? (ADHD) P: No
D: Does he do excessive talking? (ADHD) P: No
D: Is he unable to concentrate on tasks? (ADHD) P: No
D: Is he constantly changing activities or tasks? (ADHD) P: No

D: How has he been recently? P: He has been fine


D: Has he got fever or flu like symptoms? P: No
D: How is his sleep pattern? P: Ok

D: How much does he play on the smart devices? P: Very limited


D: Parental control on? (Violent videos) P: Yes
D: What time does he go to sleep? P: 8pm
D: How is his diet? P: Good
D: Is he up to date with his jabs? P: Yes
D: Are you happy with the Red Book? P: Yes
D: How is his mood? P: Fine
D: Is he lethargic? P: No

D: Has he been diagnosed with any medical conditions? P: No


D: Is he on any medications? P: No
D: Any allergies? P: No
D: Anyone in your family with a similar problem? P: No
D: How is the home condition? P: Good
D: Who looks after him?
P: Me and my husband.

D: Can you bring your child in to the hospital for assessment? P: Yes

www.aspire2plab.com 54
No real PLAB2 cases discussed. All discussed scenarios are fictitious and for educational purposes only.
From what you have told, it seems that your child is having temper tantrums. This is a battle
between freedom and frustration which can lead to tantrums.

As a parent, you should:


• Not panic
• Ignore the tantrum
• Be consistent with rules
• Pay attention to any good behaviour

Prevention of tantrums

Planning ahead.

Here are some examples:


• Manage boredom when in a waiting room by taking their favourite books and toys to
the doctor's surgery with you.
• Storing their favourite biscuits out of sight, rather than where they can see them.
• Manage a tired child by giving them an afternoon nap, instead of staying awake all day.
• Manage hunger by offering a snack after nursery at 3.30 pm, instead of having to wait
until 5.00 pm for tea.
• Distraction can help - you may be able to avoid a tantrum by diverting your child's
attention

Signs and Symptoms

Tantrums can also occur when a child is:


• Tired.
• Hungry.
• Feeling ignored.
• Worried or anxious - a younger child may be unable to tell you that they are anxious,
and they may cry, become clingy and have tantrums.

www.aspire2plab.com 55
No real PLAB2 cases discussed. All discussed scenarios are fictitious and for educational purposes only.
Temper Tantrums
What it is: One reason for a toddler to throw temper tantrums is that they want to express
themselves but find it difficult to do so. They feel frustrated, and the frustration comes out
as a tantrum.

When does it start: Temper tantrums usually start at around 18 months and are very
common in toddlers.

When does it reduce: Reduces by age 4 as the verbal skills would have developed by then so
they are able to express themselves more freely.

How to manage it?


- Try to understand the underlying reason: Hunger and tiredness are two major trigger factors for
temper tantrums. It’s easier to calm the toddlers when you know the reason.
- Distraction: Offer the toddler something else in place of what they can’t have or start a new
activity. You could also change the environment by taking them inside or outside the room.
Distract them by looking out of the window to see something they like, e.g. a dog. Make yourself
sound very surprised and interested to get to the child’s attention.
- Don’t give in
- Praising good behaviour
- Giving control over little things
- Understanding the child’s need
- Talk to them: show them you love them but not their behaviour
- Avoid smacking: Children learn by example so, if you hit your child, you're telling them that
hitting is OK. Children who are treated aggressively by their parents are more likely to be
aggressive themselves. It's better to set a good example instead.

www.aspire2plab.com 56
No real PLAB2 cases discussed. All discussed scenarios are fictitious and for educational purposes only.
Negativism

You are an FY2 in GP. Mrs Lisa James, aged 30, is concerned about her 7-year-old daughter.
Talk to her and address her concerns.

D: How can I help you?


P: My child doesn’t listen to me. She started her nursery 2 weeks ago.
D: Can you tell me more?
P: She is always playing with her food. She is taking roughly 30 mins to go to bed.
D: Does she sleep alone? P: Yes
D: Since when has this been happening? P: This has been going on for 3 months.
D: How often does this happen? P:
D: Has it been regular since it started? P:
D: How was she before that? P: She was fine
D: Anything else? P: Like what?

D: Does she respond to sounds around her? (Hearing loss) P: Yes


D: Does she respond to her name? (Autism) P: Yes
D: Does she make eye contact while talking? (Autism) P: Yes
D: Does she engage in repetitive movements? (Autism) P: No
D: Does she have tantrums often? (Autism) P: No
D: Is she unable to sit still? (ADHD) P: No
D: Does she do excessive talking? (ADHD) P: No
D: Is she unable to concentrate on tasks? (ADHD) P: No
D: Is she constantly changing activities or tasks? (ADHD) P: No

D: Any (tragic/traumatic) event occurred recently? P: She started school 2 weeks ago.
D: How is she coping up with it? P:
D: Do you think she is getting tired/distressed with the school? P: Yes/No

D: Have you spoken to your daughter about this? P: Yes/No


D: Tell me more about Jasmine. P: What do you want to know?
D: How has she been recently? P: She has been fine
D: Has she got fever or flu like symptoms? P: No
D: How is her sleep pattern? P: Ok
D: Does she go to the toilet before going to sleep? P: Yes/No

D: How much does she play on the smart devices? P: Limited


D: Parental control on? (Violent videos) P: Yes
D: What time does she go to sleep? P: Usual 8pm
D: How is her diet? P: Good
D: Is she up to date with her jabs? P: Yes
D: Are you happy with the Red Book? P: Yes
D: How is her mood? P:
D: Is she lethargic? P: No

D: Has she been diagnosed with any medical conditions? P: No

www.aspire2plab.com 57
No real PLAB2 cases discussed. All discussed scenarios are fictitious and for educational purposes only.
D: Is she on any medications? P: No
D: Any allergies? P: No
D: Anyone in your family with a similar problem? P: No
D: How is the home condition? P: Good
D: Who looks after Jasmine?
P: I’m her only carer, sometimes my friends look after her if I am busy

D: Can you bring your child in to the hospital for assessment?


P: Yes

Patient Concerns:
1. Can you prescribe any medications?
2. Will this stay forever?

Conduct disorders are the most common type of mental and behavioural problem in
children and young people. They are characterised by repeated and persistent patterns of
antisocial, aggressive or defiant behaviour, much worse than would normally be expected in
a child of that age. Types of behaviour include stealing, fighting, vandalism, and harming
people or animals.

Conduct disorder is different from the occasional tantrum or “naughtiness” in a child.

Younger children often have a type of conduct disorder called “oppositional defiant
disorder”. In these children, the antisocial behaviour is less severe and often involves
arguing (“opposing”) and disobeying (“defying”) the adults who look after them.

In teenagers with conduct disorders, the pattern of behaviour can become more extreme
and include:

• aggression towards people or animals


• destruction of property
• persistent lying and theft
• serious violation of rules

What causes conduct disorders?


Possible environmental factors include:

• a “harsh” parenting style


• parental mental health problems such as depression and substance misuse
• parental history, such as the breakup of a marriage
• poverty
• individual factors, such as low achievement
• the presence of other mental health problems

How can conduct disorder be treated?

www.aspire2plab.com 58
No real PLAB2 cases discussed. All discussed scenarios are fictitious and for educational purposes only.
Several approaches have been developed for children at risk of, or diagnosed with, conduct
disorders. In particular, parenting programmes are run by health and social care
professionals to help parents improve their children’s behaviour. Treatment for the children
themselves includes psychological therapies and sometimes, medication.

Warning signs and symptoms:


Younger children (aged under 11) may repeatedly argue with, disobey and defy those
looking after them.

Selective prevention

One of the key messages contained in the NICE guidelines is the importance and usefulness
of selective prevention. Selective prevention means identifying individual children with an
above average risk of developing a conduct disorder and then providing treatment to try
and prevent that from occurring. The rationale being that it is usually easier to prevent a
disease than to cure one.

NICE recommend that younger children aged three to seven years should be considered for
selective prevention if:

o they are growing up in a poor household


o they are underachieving at school
o there is a history of child abuse or parental conflict
o their parents are separated or divorced
o one or both of the parents has a history of mental health problems and
/or substance abuse problems
o one or both parents have come into contact with the criminal justice
system

Initial assessment should involve checking for the following complicating factors:

o a coexisting mental health problem (for example, depression or post-


traumatic stress disorder)
o a neurodevelopmental condition (in particular ADHD and autism)
o a learning disability or difficulty
o substance misuse (in older children)

In younger children aged between three and 11 years, a type of treatment programme
known as group parent training programme is recommended. In some cases, drug
treatments may also be recommended.

Parent/foster parent/guardian training programmes

NICE recommend that this treatment should be offered to children who:

o have been identified as being at high risk of developing oppositional


defiant disorder or conduct disorder
o have oppositional defiant disorder or conduct disorder

www.aspire2plab.com 59
No real PLAB2 cases discussed. All discussed scenarios are fictitious and for educational purposes only.
o are in contact with the criminal justice system because of antisocial
behaviour

Parent/foster parent/guardian training programmes are based on the premise of helping


parents make the most of their parenting skills so they can help improve their child's
behaviour. The programmes are run by specially trained health or social care professionals.
They cover communication skills, problem-solving techniques and how to encourage
positive behaviour in children.

It is best if both parents, foster carers or guardians attend the programme if this is possible
and in the best interests of the child or young person.

The programmes are usually run on a group basis involving 10 to 12 parents, over the course
of 10-16 meetings, with each meeting lasting around 1½ to 2 hours.

Medication

In a minority of cases, where a child or young person is finding it especially difficult to


control their anger, a medication called risperidone, which helps reduce aggressive
tendencies, may be recommended.

The most common side effects of risperidone include:

o Parkinson’s like symptoms such as muscle jerks and problems with body
movements
o headaches
o insomnia

Autism Symptoms:
1. Not responding to their name,
2. Avoiding eye contact,
3. Not smiling when you are smiling at them,
4. Repeating same phrase and movements like flapping of hands,
5. Not talking as much as other child,
6, Developmental delay,
7. Not socializing with others.

ADHD Symptoms:
The main signs of inattentiveness are:

● having a short attention span and being easily distracted


● making careless mistakes – for example, in schoolwork
● appearing forgetful or losing things
● being unable to stick to tasks that are tedious or time-consuming
● appearing to be unable to listen to or carry out instructions
● constantly changing activity or task
● having difficulty organising tasks

www.aspire2plab.com 60
No real PLAB2 cases discussed. All discussed scenarios are fictitious and for educational purposes only.
Hyperactivity and impulsiveness
The main signs of hyperactivity and impulsiveness are:

● being unable to sit still, especially in calm or quiet surroundings


● constantly fidgeting
● being unable to concentrate on tasks
● excessive physical movement
● excessive talking
● being unable to wait their turn
● acting without thinking
● interrupting conversations
● little or no sense of danger

www.aspire2plab.com 61
No real PLAB2 cases discussed. All discussed scenarios are fictitious and for educational purposes only.
Negativism

It is doing the opposite of what others want and is closely related to autonomy: the toddler
wants to do things by herself/himself.

What it is: A parent offers their 18-month-old her favourite story, her best-loved shirt, her
must-have and the response you get is exasperating: No. No. No.

Why it happens: Beneath this burst of negativity are the seeds of self-expression. "No" is
actually a toddler's declaration of independence, her way of laying down the law after
months of letting her parent call the shots, now that she can sort of articulate her feelings.
What you need to know: Frustrating though it is, negativity is part of a healthy
developmental phase that kids can't avoid.

How to handle it?


Negativism is doing the opposite of what others want and is closely related to
autonomy: the toddler wants to do things by herself/himself.

What it is: A parent offers their 18-month-old her favourite story, her best-loved shirt, her
must-have and the response you get is exasperating: No. No. No.

Why it happens: Beneath this burst of negativity are the seeds of self-expression. "No" is
actually a toddler's declaration of independence, her way of laying down the law after
months of letting her parent call the shots, now that she can sort of articulate her feelings.
What you need to know: Frustrating though it is, negativity is part of a healthy
developmental phase that kids can't avoid.

Be consistent and do not give up: Children need consistency. If you react to your child's
behaviour in one way one day and a different way the next, it's confusing for them. It's also
important that everyone close to your child deals with their behaviour in the same way.
Talk to your child: Children do not have to be able to talk to understand. It can help if they
understand why you want them to do something. For example, explain why you want them
to hold your hand while crossing the road. Once your child can talk, encourage them to
explain why they're angry or upset. This will help them feel less frustrated.

Offer rewards: You can help your child by rewarding them for good behaviour. If your child
behaves well, tell them how pleased you are. Be specific. Say something like, "Well done for
putting your toys back in the box when I asked you to.” Do not give your child a reward
before they've done what they were asked to do. That's a bribe, not a reward.

www.aspire2plab.com 62
No real PLAB2 cases discussed. All discussed scenarios are fictitious and for educational purposes only.
Primary Enuresis

You are an FY2 in GP Surgery. Mrs Rachel Williams, mother of 4-year-old David Williams,
has brought him in to the clinic because of Bed Wetting. Talk to her and address her
concerns.

D: What brought you to the hospital? P: My child is not dry at night.


D: Tell me more about it? P: Dr he is 4 years old but still he wets the bed in the night.
D: Did you child use to be dry at night before? P: My child has never been dry before.

D: Is it daily or off and on? P: It is daily, doctor.


D: Have you noticed any dry nights before or in between? P: No
D: Did anything significant happen before the onset that led to this condition? P: No
D: What is the bathroom routine of your child before going to bed?

D: Anything else? P: No

D: Any fever? P: Yes/No


D: Any daytime wetting? P: Yes/No

D: Excessive crying? P: Yes/No


D: Any burning while passing urine? P: Yes/No
D: Any cloudy/smelly urine? P: Yes/No
D: Any lethargy? P: Yes/No
D: Loss of appetite? P: Yes/No
D: Weight loss? P: Yes/No
D: Any abnormal swellings in the tummy? P: Yes/No

D: Has he been diagnosed with any medical condition in the past? P: No


D: Is she taking any medications including OTC or supplements? P: No
D: Any allergies from any food or medications? P: No
D: Has anyone in the family been diagnosed with any medical condition? P: No

D: How was the birth of your baby? P: It was normal vaginal delivery.
D: Are you happy with the red book? P: Yes.
D: Is she up to date with all her jabs? P: Yes.
D: Has she received any recent jab? P: No
D: Is she feeding well? P: Yes. She is feeding very well.
D: Does she have any problems with her wee and poo? P: No.

D: Who looks after her? P: It’s me.

D: Who else lives with the child? P: Me and my husband.


D: Any other child? P: No
D: Is he going to school? P: Yes.
D: Any problem at school? P: No

www.aspire2plab.com 63
No real PLAB2 cases discussed. All discussed scenarios are fictitious and for educational purposes only.
I would like to check the vitals, general physical examination and abdominal examination.

Bedwetting is common in young children and children usually grow out of it. We can devise
a plan to help your child with this.

Plan:
Plenty of water during the day. It's best to avoid drinks for an hour before bedtime.
Avoid drinks that contain caffeine, such as cola, tea, coffee or hot chocolate, because they
increase the urge to wee.

Encourage your child to go to the toilet regularly during the day.

• If the child wakes up at night, encourage them to go to the toilet.


• Make a habit of asking your child to go to the toilet before going to bed.

Reward your child for having plenty of drinks during the day and remembering to have a
wee before bed

We will be following your child up and if these measures do not help, or your child develops
daytime wetting, or your child does not outgrow this after 5 years of age, then we will further
test your child.

Further plan of action:


• Referral to enuresis clinic/ specialist.
• Urine test
• Alarm clock for encouraging the child to visit the loo

GP may suggest a medicine called desmopressin.

Please come for the follow up and come back to the hospital with your child if he develops
fever, lethargy, tummy swelling, daytime symptoms and your child has suddenly started
wetting the bed after they've been dry at night for a while.

We will give you a leaflet.

Note: if the child is above 5 years, then you have to make a referral, offer general advice same
as above and also include an alarm clock and positive reward system. If the child is of any age
and has daytime symptoms as well, make referral to enuresis clinic. If the child was dry before
and now started wetting the bed, make a referral.

www.aspire2plab.com 64
No real PLAB2 cases discussed. All discussed scenarios are fictitious and for educational purposes only.
Cystic Fibrosis

You are an FY2 in GP Surgery. Mrs Alisha Mansfield, 30-year-old, has come to you for pre-
natal counselling. Talk to her and address her concerns.

D: How can I help you?


P: I am thinking of becoming pregnant and I am concerned about CF.

D: May I know why?


P: My half-brother is suffering from Cystic Fibrosis.

D: What kind of symptoms is he experiencing?


P: I’m not sure because we don’t live together

D: Are you related to your half-brother by blood?


P: No, he is my stepfather’s son.

D: How is your general health? P: I’m fine


D: Any symptoms? P: No
D: Do you have any cough? P: No
D: Any recurrent fevers? P: No
D: Tummy pain? P: No
D: Recurrent diarrhoea? P: No
D: Recurrent chest infections? P: No
D: Anyone else in your family with similar symptoms? P: No

D: How is your partner? P: My partner is fine


D: Does he have any symptoms of cystic fibrosis? P: No
D: Anyone in his family have similar symptoms? P: No

D: Have you been diagnosed with any medical condition in the past? P: No
D: Are you currently taking any medications, over-the-counter drugs or supplements? P: No
D: Any allergies from any food or medications? P: No
D: Any previous surgeries or procedures done around your private parts? P: No

D: When was your last menstrual period? P: 2/3 weeks ago.


D: Are they regular? P: Yes.
D: Any bleeding or spotting between your periods? P: No.
D: Any painful or heavy periods? P: No
D: Have you been pregnant before? P: No
D: Are you currently sexually active? P: Yes

D: Do you smoke? P: No
D: Do you drink alcohol? P: No
D: Tell me about your diet? P: Good/Bad
D: Are you physically active? P: Yes/No

www.aspire2plab.com 65
No real PLAB2 cases discussed. All discussed scenarios are fictitious and for educational purposes only.
D: What do you do for a living? P:

I would like to do GPE, vitals and examine your chest, and your tummy.

From what you have told me, it appears that the chances of your child having cystic fibrosis
are very low. You mentioned that your half-brother is not related to you by blood which
reduces your chances considerably, and that you and your partner, along with your
extended families have no symptoms. This means there is no chance of cystic fibrosis being
inherited or passed down to your child.

Cystic fibrosis is an inherited condition that causes sticky mucus to build up in the lungs
and digestive system. This causes lung infections and problems with digesting food.

Symptoms of cystic fibrosis include:


• recurring chest infections
• wheezing, coughing, shortness of breath and damage to the airways (bronchiectasis)
• difficulty putting on weight and growing
• jaundice
• diarrhoea, constipation, or large, smelly poo
• a bowel obstruction in new-born babies (meconium ileus) – surgery may be needed

Cause of cystic fibrosis


Cystic fibrosis is a genetic condition. It's caused by a faulty gene that affects the movement
of salt and water in and out of cells.
This, along with recurrent infections, can result in a build-up of thick, sticky mucus in the
body's tubes and passageways – particularly the lungs and digestive system.
A person with cystic fibrosis is born with the condition. It's not possible to "catch" cystic
fibrosis from someone else who has it.

How cystic fibrosis is inherited


To be born with cystic fibrosis, a child has to inherit a copy of the faulty gene from both of
their parents. This can happen if the parents are "carriers" of the faulty gene, which means
they don't have cystic fibrosis themselves.

Diagnosing cystic fibrosis


In the UK, all newborn babies are screened for cystic fibrosis as part of the newborn blood
spot test (heel prick test) carried out shortly after they're born.
If the screening test suggests a child may have cystic fibrosis, they'll need these additional
tests to confirm they have the condition:

● a sweat test – to measure the amount of salt in sweat, which will be abnormally high
in someone with cystic fibrosis
● a genetic test – where a sample of blood or saliva is checked for the faulty gene that
causes cystic fibrosis
These tests can also be used to diagnose cystic fibrosis in older children and adults who
didn't have the newborn test.

www.aspire2plab.com 66
No real PLAB2 cases discussed. All discussed scenarios are fictitious and for educational purposes only.
Antenatal: Amniocentesis/chorionic villus sampling (CVS)

Treatments for cystic fibrosis:

Medicines for lung problems


Medicines for lung problems include: Antibiotics, Steroids, bronchodilators, medicine to
reduce the level of mucus and to make the mucus in the lungs thinner.

Exercise
Any kind of physical activity, like running, swimming or football, can help clear mucus from
the lungs and improve physical strength and overall health.
A physiotherapist can advise on the right exercises and activities for each individual.

Dietary and nutritional advice


Eat high-calorie diet, vitamin and mineral supplements, and taking digestive enzyme
capsules with food to help with digestion.

Lung transplants.

Complications of cystic fibrosis:


People with cystic fibrosis also have a higher risk of developing other conditions.
These include: Osteoporosis, DM, Nasal polyps, sinus infections, Liver problems, fertility
problems.

Prognosis:
Cystic fibrosis tends to get worse over time and can be fatal if it leads to a serious infection
or the lungs stop working properly.

Choices for having children:


There are options available to couples who are both CF carriers when planning a family. One
option involves prenatal testing to check if CF has been passed on in a pregnancy. If both
members of a couple are CF carriers and planning children, a referral can be made to the
local genetics service. An appointment will then be arranged to discuss reproductive
options.
!

www.aspire2plab.com 67
No real PLAB2 cases discussed. All discussed scenarios are fictitious and for educational purposes only.
Haemangioma (Birth Mark)

You are an FY2 in GP. Mr David Waile, aged 1, has been brought in by his mother to you.
Talk to her and address her concerns.

D: How can I help? P: My son has a red bump on his chest


D: That’s fine. We will assess you first and see what can be done. P: Ok.
D: When did you first notice it? P: It’s been there since his birth.
D: Can you tell me more about the lesion please? P: Like what
D: May I know the size of the lesion? P: It’s about 1x1cm.
D: What shape is it? P: Round
D: What is the colour of the lesion? P: Red
D: Any pain or itchiness? P: No.
D: Any bleeding or discharge from the lesion? P: No
D: Did you notice any ulcer on the lesion? P: No
D: Have you noticed any change in its size, shape or colour since it started? P: No
D: Do you have any other skin lesions anywhere else? P: No.
D: Any other problems? P: No

D: Does this lesion bleed on touch? P: No


D: Any discharge? P: No
D: Any fever? P: No

D: Has he been diagnosed with any medical conditions in the past? P: No


D: Is he taking any medications including OTC or supplements? P: No
D: Any allergy to medications or food? P: No
D: Has anyone in the family been diagnosed with any medical conditions? P: No

D: How was the birth of your baby? P: Normal Vaginal Delivery.


D: Are you happy with Red Book? P: Yes
D: Is he up to date with all his jabs? P: Yes
D: Is he feeding well? P: Yes
D: Does he have any problem with his wee and poo? P: No
D: Have you noticed a change in colour and smell of his wee? P: No
D: Have you noticed any tummy pain or changes in his poop? P: No
D: Who looks after him? P: Me and my husband.

I would like to examine your son. I would like to do vitals, GPE and examine the lesion.

We have examined your son, it seems that he has a skin lesion called haemangioma. It is a
type of birthmark.

Haemangiomas:

● are blood vessels that form a raised red lump on the skin
● appear soon after birth
● usually look red on light and dark skin

www.aspire2plab.com 68
No real PLAB2 cases discussed. All discussed scenarios are fictitious and for educational purposes only.
● are more common in girls, premature babies (born before 37 weeks), low birth weight
babies, and multiple births, such as twins
● get bigger for the first 6 to 12 months, and then shrink and disappear by the age of 7
● sometimes appear under the skin, making it look blue or purple
● may need treatment if they affect vision, breathing, or feeding

Treatment for birthmarks


Most birthmarks do not need treatment, but some do. This is why it's important to get a
birthmark checked if you're worried about it.

A birthmark can be removed on the NHS if it's affecting a person's health. If you want a
birthmark removed for cosmetic reasons, you'll have to pay to have it done privately.

Possible treatments for birthmarks include:


● medicines – to reduce blood flow to the birthmark, which can slow down its growth and
make it lighter in colour
● laser therapy – where heat and light are used to make the birthmark smaller and light-er
(it works best if started between 6 months and 1 year of age)
● surgery – to remove the birthmark (but it can leave scarring)

See your GP:


● you're worried about a birthmark
● a birthmark is close to the eye, nose, or mouth
● a birthmark has got bigger, darker or lumpier
● a birthmark is sore or painful
● your child has 6 or more cafe-au-lait spots
● you or your child has a large congenital mole

The GP may ask you to check the birthmark for changes, or they may refer you to a skin
specialist (dermatologist).

The compression test is useful, or the lesion can be examined with a dermatoscope (an
instrument which assists in close examination of the skin) and the blood-filled cavities
observed.

www.aspire2plab.com 69
No real PLAB2 cases discussed. All discussed scenarios are fictitious and for educational purposes only.
UTI in Female Child

You are an FY2 in GP. Mrs Lizzie Robbins, mother of Katie aged 3, has come to you as she
has noticed foul smell in her daughter’s urine. Talk to her and address her concerns.

Signs and Symptoms

Fever, Vomiting, Frequency, Dysuria, Abdominal Pain, Loin Pain, Poor Feeding
Offensive urine, Failure to thrive, Lethargy, Cloudy Urine.

Risk Factors

Female sex, <1 year old, recurrent UTI, Voiding dysfunction (Structural abnormalities),
Vesicoureteral Reflex, Constipation.

Examinations and Investigations

GPE, Vitals, Abdominal examination, Urine Dipstick, Urine Culture, US

Management
Antibiotics
Fluids
Symptomatic treatment
PCM
!

www.aspire2plab.com 70
No real PLAB2 cases discussed. All discussed scenarios are fictitious and for educational purposes only.
Meconium Ileus
Meconium Ileus (MI) is a condition where the content of the baby's bowel (meconium) is
extremely sticky and causes the bowel to be blocked at birth. In most cases the bowel itself is
complete and intact but it is just the inside that is blocked.
In some cases, there has been a twist of the bowel before birth, which has caused the bowel
to be blind ending (an atresia). Most babies with meconium ileus (90%) have Cystic
Fibrosis (CF) and it is this that has caused the sticky meconium. Meconium ileus is a rare
condition affecting only 1 in 25,000 babies.

Symptoms:
1. Delay in passing meconium (black sticky stool normally passed within 24 hours of delivery)
2. Reluctant to feed and may vomit a green fluid called bile which would normally pass
through the bowel.
Your baby may be uncomfortable because of constipation and trapped air in the bowel and
the abdomen (tummy) will become distended. Some babies present at delivery with a
distended abdomen and may be unwell due to infection around the bowel.

Dx: X-ray Abdomen. GPE, FBC


What is the treatment?

Milk feeds will be stopped


A tube will be passed through your baby’s nose into the stomach to drain away the bile
(green fluid) that collects here. This reduces the risk of your baby vomiting and reduces
discomfort.

Your baby will be given fluid through a drip sited in a vein

Antibiotics may be started to treat any infection

Can I feed my baby?


Milk feeds will be started slowly over the first few days. This may be your own breast milk or
a formula feed.

Fine granules of Creon, a medicine to help your baby absorb feeds, may need to be given by
mouth. This is because babies with CF usually lack the natural gut juices (enzymes) that are
needed to digest milk and food. If you baby has CF it is likely that your baby will need this
treatment for life.
!

www.aspire2plab.com 71
No real PLAB2 cases discussed. All discussed scenarios are fictitious and for educational purposes only.
Counselling
MMR

You are F2 in GP. 12 months old child is due to be given her MMR vaccine in the next two
weeks. Mrs. Gabriela Evans, Jenny’s mother, presented to the GP surgery to discuss her
daughter's MMR vaccination. You haven’t seen Jenny since birth. Please talk to the mother
and address her concerns. It is a non-emergency visit.

D: If I am not wrong, we are here to discuss MMR vaccination as I suppose Jenny is due for
that in the next two weeks? P: Yes doctor.

D: Where should we start from? Is there anything particular you want me to talk about?
P: Doctor, I read an article which says there is a link between MMR and autism. Is this true?

D: There is no evidence to suggest that there is a link between MMR and autism. The jab
was linked to autism in 1998 when a study of 12 children was published in a medical journal
(Lancet) which said MMR jab could cause autism. Since then, many other research studies
have been conducted and showed there is no link between MMR Jabs and Autism. There is
no evidence that supports a link between MMR and any problems with the gut
(enterocolitis, which is inflammation of the small bowel causing diarrhoea, vomiting and
weight loss).

MMR is a safe and effective combined vaccine that protects against three separate illnesses:
measles, mumps and rubella.

It is a single injection. We ideally give two doses of this vaccination.


The first dose of vaccine is usually given between 12 and 13 months.
A second dose is usually given at age 3 years and 4 months to 5 years. It is usually given at
the same time as the DTaP/IPV injection. (DTaP stands for diphtheria (D), tetanus (T) and
acellular pertussis (aP) which is whooping cough. IPV stands for inactivated polio vaccine.
Polio is short for poliomyelitis).

MMR are highly infectious conditions that can cause complications such as meningitis,
deafness and swelling of the brain (encephalopathies).

It is rare for children in the UK to develop these serious conditions. However, outbreaks may
happen and the chance increases when children are not vaccinated. There have been cases
of measles in recent years. Therefore, it is important to ensure that you and your children
are up to date with the jabs. Moreover, you may travel to countries where MMR is still
present and your child may catch these infections if not vaccinated.

Between 2001 and 2013 there was a sharp rise in the number of UK measles cases (and
three people died). Numbers of cases fell in 2014 and 2015, but have started to increase
again in 2016.

Between January and September 2016, 488 measles cases were confirmed. This is more

www.aspire2plab.com 72
No real PLAB2 cases discussed. All discussed scenarios are fictitious and for educational purposes only.
than five times as many as the total number of cases in 2015. The majority of cases of
measles have been in people who are unvaccinated.

As I mentioned earlier, MMR is made up of three different vaccines (measles, mumps and
rubella) and each of these cause reactions at different times after the injection.

1.After six to ten days, the measles vaccine starts to work and may cause a fever, a measles-
like rash, and loss of appetite. Individuals with vaccine-associated symptoms are not
infectious to others.
2. Two to three weeks after the injection, the mumps vaccine may cause mumps-like
symptoms in some children such as fever and swollen glands.
3. The rubella vaccine may cause a brief rash and possibly a slightly raised temperature,
most commonly around 12 to 14 days after the injection, but a rash may also rarely occur up
to six weeks later.

Call the doctor immediately if, at any time, your child has a temperature of 39-40°C or
above, or has a fit. If the GP is closed and you cannot contact your doctor, go to the nearest
hospital with an A&E.

There are no licensed alternatives to MMR in the UK (individual vaccines).

For those who have untreated malignant disease, MMR vaccine is contraindicated.
In immunocompromised patients such as those who are on immunosuppressant
medications, steroids, radiotherapy, cytotoxic drugs or for those who received such
treatment within six months, MMR vaccine is contraindicated

Having a previous history of infection with pertussis, measles, rubella or mumps or having
conditions such as asthma, eczema, hay fever or rhinitis is not a contraindication.
Neurological conditions such as epilepsy are not a contraindication although, if the
condition is poorly controlled, immunisation should be postponed.
Treatment with antibiotics or locally acting such as topical or inhaled steroids is not
contraindicated.

P: Doctor, what is MMR? / What is the use of this vaccine?


P: How many shots of this jab should be given, doctor?
P: Why do we give MMR vaccine?
P: Doctor, are these diseases still present in the UK? / Doctor, these diseases are not that
common in the UK, so why should my little one receive this vaccine?
P: Doctor, you said there were cases of measles in the UK in recent years. When was the last
time?
P: Doctor, is there an alternative to MMR?
P: Does MMR cause any problems with the gut?
P: Any contraindications to it?

D: Since I haven’t seen your daughter since birth, if you don’t mind let me ask you a few
questions about her health to make sure there is no problem in giving her MMR jabs.

www.aspire2plab.com 73
No real PLAB2 cases discussed. All discussed scenarios are fictitious and for educational purposes only.
D: How has your little one's health been recently? P: It has been fine,
doctor.
D: Has she had any fever recently? P: No.
D: Has she ever been diagnosed with any medical condition? P: No.
D: Does she take any medications? P: No.
D: Does she have any allergy? P: No doctor.
D: Has she received any blood products recently? P: No.

(MMR is contraindicated for those who have received any blood products such as
immunoglobulins, within three months.)

D: How was the birth of your baby? P: Normal Vaginal delivery


D: Is she up-to-date with all her jabs? P: Yes.
D: Has she received any jabs recently? P: No doctor
D: Any complications after previous jabs? P: No doctor
D: Are you happy with her red book? P: Yes.
D: Is she feeding well? P: Yes.
D: Does she have any problems with her wee? P: No.
D: How about poo? P: No.
D: From our discussion, there seems to be no reason that your daughter can’t be given the
vaccination.
D: Do you have any other concerns? P: No doctor

www.aspire2plab.com 74
No real PLAB2 cases discussed. All discussed scenarios are fictitious and for educational purposes only.
Vaccination (Flu Jab)

You are F2 in GP Surgery. 3-year-old child Luke Riley is due for Flu vaccination in one week.
Mother Samantha Riley wants to talk to a doctor. Talk to her and address her concerns.

D. How can I help you?


P: Doctor my son Luke is going to get a flu jab & I am very worried about him?

D: May I know why you worried?


P: Doctor I’ve heard that flu jab causes seizures; I don’t want that to happen to my son.

D: Is this his 1st flu jab or has he had it before? P: It’s his 1st flu jab.

D: OK. Do you know what the flu is exactly?


P: Yes, it’s the common cold that we sometimes get in the winters.

D: Well yes, it happens seasonally, but flu is not exactly the same as a common cold. In the
common cold, people usually get mild symptoms only like a stuffy or a runny nose. Whereas
flu is actually worse than the common cold & the symptoms are more intense. There are
different viruses that cause flu & there are usually body aches, fever, tiredness, cough,
tummy pain in flu.
P: Oh alright. Does Luke really need the flu jab?

D: Well, the flu virus is a very unpredictable virus & can cause unpleasant illness in children,
severe illness, older people, pregnant women or people who have some other health
problem/disease. Flu jab is actually a vaccine for flu, which is the best thing we have to
protect people from the serious illness caused by these viruses.
P: Oh okay. So, does this mean that my son will never have the flu after he gets the flu jab?

D: There are different viruses identified at the start of each year that cause flu, some might
even be new, & the vaccine might not protect the person from the effects of the new
viruses. However, in case young Luke does get the flu after having had the vaccine, the
illness will be much milder & shorter than the original one. Also, the protection given by the
injected flu vaccine decreases over time as the viruses causing flu change every year. To
counteract that, new vaccines are produced every year & people are advised to get the flu
jab every year too.
P: Oh, okay, so will he have to get it every year then?
D: Yes, to protect him against the new disease causing viral strains.

P: But what about the seizures, doctor? I don’t want my son to get seizures.
D: The flu illness can cause high fever which can cause seizures known as febrile
convulsions. Children usually don’t get these seizures as they get older.

P: Are there any other side effects of the jab?


D: Yes, there are. It’s very rare to get any serious side effects from the flu vaccine.
Occasionally, people may get a mild fever or aching muscles for a couple of days, and it

www.aspire2plab.com 75
No real PLAB2 cases discussed. All discussed scenarios are fictitious and for educational purposes only.
might happen that your arm might be a bit sore where you were given the injection but
these will subside in a few days.

P: Is it given as an injection?
D: Yes, it’s available as an injection. However, nasal spray is also available. Children are
usually given nasal spray. In fact, it’s more effective in the spray form as it gets absorbed
through the skin in the nose real quick.

D: Can I ask if young Luke has had any stuffy or runny nose in the past few days?
P: No, he hasn’t.

D: That’s good because we don’t give the nasal spray if the person is having a runny or stuffy
nose because that would prevent absorption of the vaccine in the nose.

D: Has he ever been diagnosed with any medical conditions in the past? P: No
D: Is he taking any medications? P: No
D: Does he have any allergies? P: No
D: Any egg allergy? P: No
D: That’s good, we also don’t give it if the person has had any egg allergies. P: Right

P: Doctor, does he have to get the jab now though? Can he not get it in spring maybe? It will
be a bit warmer too then.
D: I can see that you’re very concerned. But, the best time to get a flu jab is in the autumn,
from the beginning of October to early November. That’s when the flu affects people more.

P: Where can I get the flu jab for Luke?


D: These jabs are available at your GP Surgery or in some local pharmacy.

Best time to get the FLU JAB is beginning of October to early November

Who should be immunised? Where can I get the Flu Jab?


1) Elderly > 50 1) GP Surgery
2) Patient underlying condition 2) Local Pharmacy
3) Pregnant Women 3) Midwife (Pregnant Women)
4) Frontline health or social care workers 4) Hospital Appointment
5) Carer for elderly people
6) Children

Nasal spray is not licensed for children under 2 years.

6 months – 2 years (has long term health condition) à Injections


2 years – 17 years à Nasal spray

Side Effects & Contraindications:


1) Fever, Rash, Seizures & Encephalitis
2) Egg Allergy, Vaccine Allergy

www.aspire2plab.com 76
No real PLAB2 cases discussed. All discussed scenarios are fictitious and for educational purposes only.
8 Weeks Vaccination

You are an FY2 in GP. Evana, a 30-year-old female, presented to your clinic with her 8 weeks
old boy. She wants to know about the vaccines that can be given to her baby. Please talk to
her, explain to her about the vaccines that can be given at 8 weeks of birth and address her
concerns.

Please refer to the Immunisation Table in the Introduction Section.


!

8 Weeks Old Immunisation

1. 6 in 1 Vaccination (Thigh)
2. Rotavirus Vaccine (By mouth)
3. MenB (Left Thigh)

6 in 1 Vaccine
It is given to babies when they are 8, 12 and 16 weeks old.
It is injected into baby's thigh.

6 in 1 Vaccine is very safe. It is killed (inactivated) which means it does not contain any live organism.
So, there is no risk in your baby getting the conditions from the vaccine.
It produces very good immunity to:
• Diphtheria
• Tetanus
• Whooping Cough
• Hib
• Polio
• Hep B
!
!
Side Effects of Vaccine:
1. Pain/Redness/Swelling/Fever
2. Sickness/Irritability/Loss of Appetite
3. Fits/Anaphylaxis

Babies who should not have 6 in 1 vaccine:


1. Allergic to vaccine
2. Have high temperature at the time of vaccine
3. Neurological problem including poorly controlled epilepsy
(wait for specialist review)

Side effects of Rotavirus Vaccine:


1. Restlessness/Irritability
2. Diarrhoea
3. Intussusception

Side effects of MenB Vaccine:


1. Unusual crying/ Irritability
2. Diarrhoea/Vomiting
3. Loss of Appetite/ Sleepiness
!

www.aspire2plab.com 77
No real PLAB2 cases discussed. All discussed scenarios are fictitious and for educational purposes only.
Prescribing IV Fluid in a Child

You are FY2 in Surgery. Aisha Desai aged 6 has been diagnosed with Appendicitis and has
been scheduled for appendectomy. Her father is worried about his daughter. Everything has
been explained about the surgery to the father. Surgery is scheduled in the next 24 hours.
Your senior has asked you to start the IV Fluids. Child must be kept ‘nil by mouth’. Explain
to her Father Mustafa Abdul why she cannot have oral fluids and explain the necessity of
giving IV Fluids to the child. Calculate and write the prescription of fluids. Consent has been
taken from the father for the surgery. Child’s weight is 25kg. Consultant has asked you to
prescribe fluids. Sodium Chloride 0.9% with 5% Dextrose. Be aware the available solution is
500ml.

D: Hello, how can I help you?


P: Dr why are you going to give IV Fluids to my daughter?

D: Right! Let me ask you a few questions first to be able to answer you properly. How much
do you know about the condition of your daughter?
P: She has been having pain since yesterday and I brought her to the hospital and she was
diagnosed with Appendicitis. Then she was planned for surgery.
(If he doesn’t know about Appendicitis then explain).

Appendicitis is a painful swelling of the appendix. The appendix is a small, thin pouch about
5-10 cm (2-4 inches) long. It's connected to the large bowel, where stools (faeces) are
formed. The treatment for this is to remove this organ by performing a surgery.

D: How is she now? P: She’s fine.


D: Any pain? P: No.
D: By any chance any vomiting? P: No Doctor.
D: Has she been diagnosed with any medical conditions in the past? P: No
D: Is she taking any medications including OTC or supplements? P: No
D: Any allergy to medications or food? P: No
D: How was the birth of your baby? P: Normal Vaginal Delivery.
D: Are you happy with Red Book? P: Yes
D: Is she up to date with all her jabs? P: Yes
D: Was she feeding well? P: Yes
D: Did she have any problem with her wee and poo? P: No
D: Has anyone told you about the surgery? P: Yes, they have explained everything.

D: We have to give fluids because we can’t allow her to drink or eat at the moment until and
after 1 or 2 days of the surgery. We have to give her Anaesthesia for the surgery and if there
is any food in the tummy, it may come back and enter the windpipe and obstruct it, causing
severe infections of the lungs. Therefore, we need to keep her tummy empty before going
ahead with the operation. Her surgery is scheduled in the next 24 hours so to prevent the
dehydration, we need to give her fluids as an IV drip.

www.aspire2plab.com 78
No real PLAB2 cases discussed. All discussed scenarios are fictitious and for educational purposes only.
P: What type of fluid are you going to give?
D: We are going to give her a liquid solution which contains salt that resembles the normal
composition body fluid (NaCl) and glucose to give her energy.
Let me write up the fluid prescription for her: please write up the patient's name, DOB on
the prescription pad. (There is a wrist band on the table with patient details).

1st 10Kg = 100mls x 10kg = 1000mls


Next 10kg = 50mls x 10kg = 500mls
Next 5kg = 20ml x 5kg = 100mls
Total fluid in 24hrs = 1000+500+100/24 = 1600/24 = 67mls per hour (use calculator)

Per Day (Over 24 Hours) Rate Per Hour


First 10KG 100mL/kg/day 4mL/kg/hr
Next 10KG 50mL/kg/day 2mL/kg/hr
≥ 20.1KG 20mL/kg/day 1mL/kg/hr
Take Total /24 to get rate per hour

Prescribing Maintenance Fluid Example:

Date Infusion Fluid Volume Name of Dose of Route Infusion Signature


Drug to be Drug Rate
infused Added

0.9% NaCl with 500ml Nil - IV 67ml/hr


5% Dextrose

P: When can I take her home?


D: If everything goes fine you may be able to take her home the day after surgery.
!

www.aspire2plab.com 79
No real PLAB2 cases discussed. All discussed scenarios are fictitious and for educational purposes only.
www.aspire2plab.com 80
No real PLAB2 cases discussed. All discussed scenarios are fictitious and for educational purposes only.
www.aspire2plab.com 81
No real PLAB2 cases discussed. All discussed scenarios are fictitious and for educational purposes only.
Epilepsy

You are FY2 in Paediatrics. Molly Evra, aged 10, presented to hospital with fits and the
diagnosis of Epilepsy has been made. She has been medically managed and is fit for
discharge. Neurology consultant has prescribed her some medication. Mother Sarah Evra
has some concerns. Please talk to the mother and address her concerns.

D: We are glad Molly is in good health now and fit to go home. I am here to talk to you
regarding some of the important things that need to be done while she is at home, that are
important to all the patients of epilepsy. P: OK doctor.

D: I suppose you understand that she has been diagnosed with epilepsy, would you like me
to explain to you what exactly it is, so that we have a better understanding of how to deal
with it? P: Yes, sure

D: It is a condition that is associated with abnormality in electrical activity of the brain. This
causes seizures in patients like the one your daughter experienced. P: OK doctor

D: So now that we know what causes this, there are some medications that need to be given
to her on a regular basis to prevent this happening in future. Has anyone talked to you
regarding this? P: No doctor.

D: It is very important to take the medication regularly every day, as prescribed. This
medication helps prevent attacks. This medication is given every day to maintain the level of
medication in the body. P: Ok

D: Has she been diagnosed with any medical condition? P: No doctor


D: Does she take any medication including otc, herbal and supplements? P: No doctor
D: Does she have any allergy? P: No doctor

D: There are some triggers that I wish to discuss with you. It is very important to find out if
there are any, as this can help prevent any future attacks . We can work together on this, as
this is very important. Spending a lot of time in front of TV or Computers and listening to
loud music can trigger fits. Does she watch a lot of TV or play with computers?
P: Yes she does

D: I can understand that considering her age, but we can always do that in small sessions
instead of being exposed to screens continuously for a longer period of time.
D: Lack of sleep can also trigger her condition. Please make sure that she has enough sleep.
P: Ok doctor

D: There are a few more things like dehydration and skipping meals that can cause a fit. So
please make sure that she drinks plenty of water and does not skip meals. Excessive exercise
can also be a trigger. However, most sports and leisure activities are possible for people
with epilepsy, as long as necessary precautions are taken.

P: She goes for swimming and dance classes, doctor.

www.aspire2plab.com 82
No real PLAB2 cases discussed. All discussed scenarios are fictitious and for educational purposes only.
D: Sports on or near water, or at heights, may need extra safety measures or supervision.
If you have seizures, it’s a good idea to swim with someone who knows about the condition
and type of seizures you have and how to help you if you have a seizure in the water. So, it
would be great if you could accompany your child when she goes swimming.

Please make sure that the swimming pool has a lifeguard who is aware of your daughter’s
condition. You can also tell the lifeguard how to help your daughter if she has an attack.

Some people swim during quieter swimming sessions, so it is easier for the lifeguard to spot
them.

Some swimming pools have got a shallow area, so it is advisable to swim in shallow water.

As long as her dance classes are not so intensive and there is no loud music, you can take
her to dance class. It is important to inform her instructor about her condition.

D: Epilepsy medications usually have interaction with some medication. Please don’t give
your daughter any medication including OTC. Please always seek advice from her GP.
It is very important for her to be seen by GP regularly. Her GP will review your child’s
condition and re-prescribe her medications. P: Sure Doctor

D: Some health problems such as vomiting and diarrhoea especially after having medication
will decrease the efficiency of the medication. So, it is very important to seek medical advice
in such a case. P: Ok doctor thank you for letting me know.

D: Like any other medication, epilepsy medication has some side effects. Some side effects
are temporary and subside after some time. If she has any persistent side effect, please seek
advice from her GP. It is very important to not stop the medication. We’ll try to tackle the
SE, if not possible, we will review the medications. P: Sure doctor

D: Anything else I can help you with?


P: What should I do if she has another attack?

It is important to put her in a safe position and stay with her while the convulsion is
happening. Lay her on her side with her head slightly tilted backwards (recovery position).

This will ensure that she will not swallow any sickness (vomit) and the tongue does not
cause any obstruction, which helps to keep her airway clear.

Remove any sharp objects if there are any around.

Loosen any tight objects around her neck like a tie if any.

Don’t put anything in her mouth. Your child should be seen by a doctor after having a fit. If
your child has a seizure that lasts more than 5 minutes, or doesn’t regain consciousness
after a seizure, you need to call an ambulance.

www.aspire2plab.com 83
No real PLAB2 cases discussed. All discussed scenarios are fictitious and for educational purposes only.
P: Is it bad for the brain? Does it cause damage to the brain?
D: Fortunately, in your child, the duration of seizure was not that long so hopefully it won’t
cause any damage to the brain.

P: Will she be able to have a normal life?


D: Like I said it is very important to take the medication regularly and avoid the triggering
factors. Most patients can have an almost normal life. Some patients after a while will be
symptom free and may not even need any medication.

P: What precautions do I need to take?


D: Since a seizure can happen at any time, it is important that your daughter is not in any
situation where she is in imminent danger or harm if she has a fit. For example, a shower is
considered to be safer than a bath. Instead of locks on the door you can have “engaged”
signs. These are just some examples of how she can be kept safe.

www.aspire2plab.com 84
No real PLAB2 cases discussed. All discussed scenarios are fictitious and for educational purposes only.
Obstetrics & Gynaecology
!
History (4P’s)
P - Pregnancy
P - Periods
P - Pills
P - Pap smear

www.aspire2plab.com 85
No real PLAB2 cases discussed. All discussed scenarios are fictitious and for educational purposes only.
Pre-eclampsia

You are FY2 in OBG. Sara Robbins, 29 years old, who is 36 weeks pregnant, has been
referred to the hospital. The midwife has seen the lady and made a note.
Vitals: BP 160/110, Urine protein – (+++). This must be reviewed by a doctor.
The patient’s BP in her first booking was 110/70. Please talk to the patient, assess her
condition, discuss your further management with the patient and address her concerns.

D: What brought you to the hospital?


P: I went for my check up and I was sent here by the midwife because of my blood pressure
and protein in my urine.

D: Could you confirm the age of your pregnancy? P: 36/38 weeks


D: Is this your first pregnancy? P: Yes
D: How was the pregnancy confirmed? P: Pregnancy Test at home
D: Were you using contraception? P: No
D: Estimated date of delivery (EDD)? P:

D: Could you feel the movements of your baby? P: Yes Dr.


D: Planned method of delivery? P: Normal
D: Medical illness during pregnancy? P: No
D: Any medications during pregnancy? Iron or folic acid? P: No

D: Have you attended all your antenatal checkups? P: Yes


D: Do you know your blood pressure on your first visit? P: 110/70
D: Have you got any scans done? P: Yes
D: Do you have twins in your pregnancy? P: No
D: How has your pregnancy been so far? P: Fine
D: Did you develop any complications? P: No
D: Have you got any symptoms now? P: No

D: Do you feel sick? Any vomiting? (hyperemesis gravidarum) P: No


D: Any tummy pain? P: No
D: Vaginal bleeding? P: No
D: Any pain or burning sensation during urination? urinary frequency? (UTI) P: No
D: Do you feel tired? (anaemia) P: No
D: Any Headache/visual changes/swelling of feet, ankles or face? (Pre-eclampsia) P: No
D: Fever or flu like symptoms? P: No

D: Have you been diagnosed with any medical condition in the past? P: No
D: Any high blood pressure, diabetes or kidney problems? P: No
D: Are you currently taking any regular medications, OTC drugs or supplements? P: No
D: Any allergies from any food or medications? P: No
D: Any previous hospital stay or surgeries? P: No

D: Has anyone in the family been diagnosed with any medical condition? P: No
D: Did your mother or sister have any complications during their pregnancies? P: No

www.aspire2plab.com 86
No real PLAB2 cases discussed. All discussed scenarios are fictitious and for educational purposes only.
D: Do you smoke? P: Yes/ No
D: Do you drink alcohol? P: Yes/ No
D: Have you been taking any recreational drugs? P: No

D: What do you do for a living? P: Office job


D: May I know whom do you live with? P: My partner

I would like to check your vitals including blood pressure and do an Antenatal Examination.
I would like to send for some initial investigations including routine blood tests and urine
tests.

If the patient is 36weeks pregnant:


Examiner:
BP - 150/100 mmHg
Urine Protein ++

From my assessment, you seem to have developed a condition called Preeclampsia.


Pre-eclampsia is a condition that affects some pregnant women, usually during the second
half of pregnancy (from around 20 weeks) or soon after their baby is delivered.

Early signs of preeclampsia include having high blood pressure and protein in your urine.
We checked your blood pressure and its high and your urine test shows there is protein in
your urine.

Pre-eclampsia is thought to be caused by the placenta not developing properly due to a


problem with the blood vessels supplying it. The exact cause isn't fully understood.
Some factors have been identified that could increase your chances of developing pre-
eclampsia.

These include:
● having an existing medical problem – such as diabetes, kidney disease, high blood
pressure, lupus or antiphospholipid syndrome
● previously having pre-eclampsia
● it's your first pregnancy
● it's been at least 10 years since your last pregnancy
● you have a family history of the condition
● you're over the age of 40
● you were obese at the start of your pregnancy
● you're expecting multiple babies, such as twins or triplets

In your case it is most probably due to your first pregnancy.

D: We will keep you in the hospital and monitor you and your baby closely.
● Your blood pressure will be checked regularly for any abnormal increases.
● Urine samples may be taken regularly to measure protein levels.
● Various blood tests – to check your kidney and liver health, for example.

www.aspire2plab.com 87
No real PLAB2 cases discussed. All discussed scenarios are fictitious and for educational purposes only.
● You may have ultrasound scans to check blood flow through the placenta, measure
the growth of the baby, and observe the baby's breathing and movements.
● The baby's heart rate may be monitored electronically in a process called
cardiotocography, which can detect any stress or distress in the baby.

We will give you medication to reduce your blood pressure (Labetalol).


We may have to give medication through your blood vessel (vein) (Magnesium Sulphate) to
prevent fits from happening.

These medications are generally safe for you and your baby. We have to monitor you as this
condition can cause some problems to you and your baby. You can have a normal delivery if
your blood pressure and the amount of protein in your urine reduces. But we may have to
induce your labour by giving you some medication if needed. At any point of time if
complications develop, we have to do a caesarian section to deliver your baby.

It is not advisable for you to have a pool delivery as we have to monitor you and your baby
continuously both during and after the delivery as some complications can happen even
after the delivery.

This condition has some complications like you may develop fits(eclampsia), your liver,
kidney and lungs may get damaged, you may have clotting problems, your baby’s growth
may be slow or you may even have a stillbirth. Please inform us if you have any tummy pain
or you feel drowsy or confused.

If the patient is 38weeks pregnant:


Examiner:
BP - 150/100 mmHg
Urine protein ++
Fetal movement is normal.
Fetal heart rate is normal.
Head engaged.

We will have to deliver the baby within 24-48 hrs. Your labour needs to be started artificially
by giving you some medication (known as induced labour) or you may need to have
a caesarean section, if things get complicated.

P: What’s going on doctor?


P: Why did I get this condition?
P: What are you going to do for me?
P: Can I go home with some medication?
D: May I know why you want to go home?
P: I have work to do/ my maternity leave starts in a few days.
P: How long should I stay in the hospital?
P: Are these medications going to affect my baby? (Labetalol/Nifedipine)
P: Can I give a natural birth?
P: Can I have pool delivery?

www.aspire2plab.com 88
No real PLAB2 cases discussed. All discussed scenarios are fictitious and for educational purposes only.
PID (RIF Pain)

You are FY2 in A&E. Mrs. Sara Johnson, aged 28, presented to the hospital complaining of
right lower abdominal pain (RIF). Please talk to the patient, take history, do relevant
examination, discuss initial management with the patient and address her concerns. Ask
your examiner about the specific findings you are looking for. Your examiner will tell you
the results of those findings.

D: What brought you to the hospital? P: I have tummy pain.

Dr: May I know where the pain is exactly? P: It’s here doctor. In my right lower tummy.

D: When did it start? P: It started 2-3 days ago.


D: What were you doing when pain started? P: I was sitting at home when it started.
D: Does the pain come and go? P: The pain is always there since it started.
D: Has it changed P: It is getting worse.
D: What kind of pain is it? P: It is a dull pain.
D: Does it go anywhere? P: No.
D: Anywhere else you have the pain? P: It started at the same place.
D: Did you do anything for the pain? P: I took paracetamol but still I have pain.
D: How many tablets did you take? P: Just two tablets whenever I had pain, doctor.
D: Could you score the pain? P: Score is around 5.

D: Is there anything else that’s bothering you? P: I have discharge from my front passage.
D: When did the discharge start? P: 2-3 days ago.
D: What is the colour of discharge? P: Green.
D: Is there any blood in it? P: No.
D: Is there any smell? P: Yes doctor, it smells bad.
D: How much was the discharge? P: Not too much.

D: Is there anything else that’s bothering you? P: No.

D: Did you have any fever or flu like symptoms? P: No/Yes I had some temperature
D: Were you feeling sick? P: Yes, from the last 2-3 days.
D: Did you vomit? P: No
D: Any pain or burning sensation while passing urine? P: No
D: Have you had any of these symptoms before? P: No

D: When was your last menstrual period? P: 2/3 weeks ago.


D: Are they regular? P: Yes.
D: Any bleeding or spotting between your periods? P: No.
D: Any painful or heavy periods? P: No
D: Have you been pregnant before? P: No
D: Are you currently sexually active? P: Yes
D: When did you last have sexual activity? P: Yesterday
D: Do you have any partner? P: Yes, I met my partner 2weeks ago
D: Have you had any other partners previously? P: Yes, I had two other partners previously

www.aspire2plab.com 89
No real PLAB2 cases discussed. All discussed scenarios are fictitious and for educational purposes only.
D: What kind of sexual contact do you have? Genital? Anal? Oral? P: Genital/Oral
D: Do you and your partner(s) use any contraception or protection against STIs?
P: Yes, I have a coil inserted.
D: Since when? P: Since 3 years.
D: Is it in place? P: Yes
D: Was there any issues with the contraception used? P: No
D: Do you use condoms? P: No, we enjoy our sex.
D: When was the last time you had unprotected sex? P: 2 weeks back
D: Any pain during or after sex? P: No

D: What other concerns or questions regarding your sexual health or sexual practices would
you like to discuss? P: Nothing.

D: Have you been diagnosed with any medical condition in the past? P: No
D: Did you have any cyst in your ovaries or any sexually transmitted infections before? P: No
D: Are you currently taking any medications, over-the-counter drugs or supplements? P: No
D: Any allergies from any food or medications? P: No
D: Any previous surgeries or procedures done around your private parts? P: No

I would like to check your vitals and examine your abdomen, do bimanual and per speculum
examination.

I would like to send for some initial investigations including routine blood test and urine
test.

Examiner:
Right adnexal tenderness and cervical excitation

From our assessment, I suspect you have a condition called Pelvic Inflammatory Disease, in
short, PID. This is an infection around your womb and surrounding structures including the
ovaries and the tubes connecting your ovaries to your womb. This is caused by some bugs
spreading from your front passage to your womb.

P: What are you going to do, doctor?


D: We are going to run some tests to confirm the diagnosis. We are going to do some blood
and urine tests to see if there is any bug and also take swabs from your vagina and send it to
the lab to test for the bug that is causing these symptoms. We will do an ultrasound scan
through your vagina to see off there is any abnormality in your womb and surrounding
structures.

Examiner:
USG is normal.

We will prescribe you some pain killers for your pain and give you antibiotics for your
condition. A course of antibiotics for 14 days. You'll be given a mixture of antibiotics to cover
the most likely infections, and often an injection as well as tablets.

www.aspire2plab.com 90
No real PLAB2 cases discussed. All discussed scenarios are fictitious and for educational purposes only.
We may change the antibiotics later depending upon your improvement and your test
results.

It's important to complete the whole course and avoid having sexual intercourse during this
time to help ensure the infection clears.

We may need to remove your coil if your symptoms doesn’t subside within a few days
It is very important to complete the treatment by bringing in your partner and treating him
as well, if he has got the infection.

If you are not able to bring your partner, we can contact him through the Partner Initiation
Programme.

Please make sure you come back for follow-up after 3 days to see if there is any
improvement. We will test you after 14 days of treatment to see whether the infection has
cleared or not.

It is advisable to practice safe sex all the time.

It has many causes, the coil can be one of the causes, one of the commonest causes of PID is
sexually transmitted infections.

It does not mean that your partner is cheating on you, these bacteria can stay in your body
for many months before causing any symptoms. You/He might have got this infection from
your/his previous relationships.

This condition can lead to some complications in your pregnancy like pregnancy outside
your womb. This can also lead to infertility.

D: We usually offer HIV test for those who have sexually transmitted infections. Do you wish
to have one? P: Yes/No

D: If you develop any fever or redness, hotness, swelling around your private parts or groin
area, any burning sensation while passing urine, any cloudy or smelly urine, please come
back to us.

P: What’s happening doctor?


P: What are you going to do now?
P: Why do I have it?
P: Is my partner cheating on me?

www.aspire2plab.com 91
No real PLAB2 cases discussed. All discussed scenarios are fictitious and for educational purposes only.
PID (Lower Abdominal Pain)

You are FY2 in A&E. Mrs. Katie Johnson, aged 40, presented to the hospital complaining of
lower abdominal pain. CS was done 15 years back She is taking desogestrel 150 mcg daily.
She was diagnosed with depression and is on sertraline for the same. Please talk to the
patient, take history, do relevant examination, discuss about initial management with the
patient and address her concerns.

D: What brought you to the hospital? P: I have tummy pain.


D: May I know where the pain is exactly? P: In my right lower tummy.
D: When did it start? P: It started 2-3 weeks ago.
D: What were you doing? P: I was sitting at home when it started.
D: Does the pain come and go? P: Doctor, the pain is always there since it started.
D: Has it got worse? P: It is getting worse.
D: What kind of pain is it? P: It is a dull pain.
D: Does it go anywhere? P: No
D: Has the pain moved? P: It started at the same place.
D: Did you do anything to make it better? P: I took paracetamol but still I have pain.
D: How many tablets did you take? P: Just two tablets whenever I had pain, doctor.
D: Could you score the pain between 1-10? P: Score is around 5.

D: Is there anything else that is bothering you? P: No.


D: Any pain or burning sensation while passing urine? P: No
D: Did you have any fever or flu like symptoms? P: No.
D: Were you feeling sick? P: No
D: Did you vomit? P: No

D: When was your last menstrual period? P: 5 years back


D: Any bleeding or spotting? P: No.

D: Any discharge? P: I have discharge from my front passage.


D: When did the discharge start? P: 2 weeks
D: What is the colour of discharge? P: yellow.
D: Is there any blood in it? P: No.
D: Is there any smell? P: Yes doctor, it smells bad.
D: How much was the discharge? P: Not too much.

D: Have you been pregnant before? P: Yes

D: Are you currently sexually active? P: Yes


D: When did you last have sexual activity? P: 4 days ago
D: Do you have any other partner? P: No
D: What kind of sexual contact do you have? (Genital? Anal? Oral) P: Normal.
D: Do you and your partner(s) use any contraception? P: Pills
D: Do you use condoms? P: No, we enjoy our sex.
D: Any pain during or after sex? P: No

www.aspire2plab.com 92
No real PLAB2 cases discussed. All discussed scenarios are fictitious and for educational purposes only.
D: What other concerns or questions regarding your sexual health or sexual practices would
you like to discuss? P: Nothing

D: Have you been diagnosed with any medical condition in the past?
P: Depression, I am taking Sertraline for that.
D: Any other medical conditions? P: No
D: Did you have any cyst in your ovaries or any sexually transmitted infections before? P: No
D: Are you currently taking any medications, over-the-counter drugs or supplements? P: No
D: Any allergies from any food or medications? P: No
D: Any previous surgeries or procedures done apart from CS? P: No

I would like to check your vitals and examine your abdomen, do bimanual and per speculum
examination.
I would like to send for some initial investigations including routine blood test and urine
test.

P: What is per speculum?


D: It is a pelvic examination. I have to examine your front passage.
P: Can I come another time? I am not prepared for this right now.
D: What is the problem?
P: Nothing, actually I am not ready for it.
P: What is going on with me?

D: From our assessment, I suspect you have a condition called Pelvic Inflammatory Disease
in short PID. This is an infection around your womb and surrounding structures including the
ovaries and the tubes connecting your ovaries to your womb. This is caused by some bugs
spreading from your front passage to your womb.

P: What are you going to do, doctor?


D: We are going to run some tests to confirm the diagnosis. We are going to do some blood
and urine tests to see if there is any bug and also take swabs from your vagina and send it to
the lab to test for which bug is causing you these symptoms.
We will do an ultrasound scan through your vagina to see off there is any abnormality in
your womb and surrounding structures.

We will prescribe you some pain killers for your pain and give you antibiotics for your
condition. A course of antibiotics for 14 days. You'll be given a mixture of antibiotics to cover
the most likely infections, and often an injection as well as tablets.

We may change the antibiotics later depending upon your improvement and your test
results.

It's important to complete the whole course and avoid having sexual intercourse during this
time to help ensure the infection clears.

www.aspire2plab.com 93
No real PLAB2 cases discussed. All discussed scenarios are fictitious and for educational purposes only.
It is very important to complete the treatment by bringing in your partner and treating him
as well, if he has got the infection.

If you are not able to bring your partner, we can contact him through the Partner Initiation
Programme.

Please make sure you come back for follow-up after 3 days to see if there is any
improvement. We will test you after 14 days of treatment to see whether the infection has
cleared or not.

It is advisable to practice safe sex all the time.

This condition can lead to some complications in your pregnancy like pregnancy outside
your womb. This can also lead to infertility.

D: We usually offer HIV test for those who have sexually transmitted infections. Do you wish
to have one? P: Yes/No

D: If you develop any fever or redness, hotness, swelling around your private parts or groin
area, any burning sensation while passing urine, any cloudy or smelly urine please come
back to us.

www.aspire2plab.com 94
No real PLAB2 cases discussed. All discussed scenarios are fictitious and for educational purposes only.
Gonorrhoea

You are FY2 in a GUM clinic. Mrs. Juliet Romeo, aged 24, has come to the clinic for her
investigation results. Swab has been taken and the result shows gonorrhoea. Please talk to
the patient, take sexual history, inform the patient about the results, discuss further
management and advise the patient.

D: What brought you to the hospital?


P: I got a sexual checkup done as I got to know about sexually transmitted infections, and I
am here for my results.

D: Yes I have your results with me but before I disclose them to you I would like to ask you a
few questions in order to get a better overview. P: Ok Dr.

D: Did you have any symptoms that made you have the checkup done?
P: No Dr. It’s just that I read somewhere about STIs and thought I should get it done.
D: That was really thoughtful.

D: Have you got any fever or flu-like symptoms? P: No


D: Do you have any bleeding or Discharge from your vagina? P: No
D: Any pain or discomfort in your lower tummy or your private parts? P: No
D: Any pain or burning sensation while passing urine? P: No
D: Cloudy or smelly urine, Frequency, Haematuria, Incontinence? P: No
D: Any redness, hotness or swelling around your private parts or groin area? P: No
D: Any eye problem or joint problem? P: No

D: Have you got any of these symptoms any time before? P: No

D: Are you currently sexually active? P: Yes


D: When did you last have sexual activity? P: Yesterday
D: Do you have any partner? P: Yes
D: Have you had any other partners previously? P: Yes, I had two other partners previously
D: What kind of sexual contact do you have? (Genital? Anal? Oral) P: Genital/Oral
D: Do you and your partner(s) use any contraception or protection against STIs?
P: Yes Dr. We use condom
D: How often do you use this protection? P: Sometimes Dr.
D: Was there any issues with the contraception used? P: No
D: Any pain during or after sex? P: No

D: What other concerns or questions regarding your sexual health or sexual practices would
you like to discuss? P: Nothing Dr.

D: Have you been diagnosed with any medical condition in the past? P: No
D: Did you have any sexually transmitted infections before? P: No
D: Are you currently taking any medications, otc drugs or supplements? P: No
D: Any allergies from any food or medications? P: No
D: Any previous surgeries or procedures done around your private parts? P: No

www.aspire2plab.com 95
No real PLAB2 cases discussed. All discussed scenarios are fictitious and for educational purposes only.
From our assessment and from your test results, it shows that you have a condition called
Sexually Transmitted Infection in short STI.

STIs are caused by different types of organisms but in your case it is caused by a bacteria
called Gonorrhoea.

This is a condition which is usually transmitted by having unprotected sex. And these
bacteria can stay in our body for months without causing any symptoms, that is the reason
why you are not having any symptoms. You might have got this condition from your partner
or from your previous relationships.

This condition is not serious as we can treat this condition with antibiotics.

Sexually Transmitted Infection

If chlamydia infection is suspected or confirmed, strongly recommend referral to a Genito-


Urinary Medicine (GUM) Clinic for management.

Doxycycline 100 mg twice daily for 7 days


• If doxycycline is contraindicated or not tolerated prescribe
• azithromycin 1 g orally for one day, then 500mg orally once daily for two days or
• erythromycin 500 mg twice daily for 10–14 days.
In the unusual event that confirmed or suspected uncomplicated anogenital
gonorrhoea needs to be treated in primary care, prescribe:
• Ceftriaxone 1g Intramuscular (IM) injection as a single dose.

You should attend a follow-up appointment a week or two after treatment, so another test
can be carried out to see if you're clear of infection.

You should avoid having sex until you've been given the all-clear.

It is very important to complete the treatment by bringing in your partner and treating him
as well, if he has got the infection.

If you are not able to bring your partner, we can contact him through the Partner Initiation
Programme.

It is advisable to practice safe sex all the time. Pills cannot protect you from sexually
transmitted infections. Using condoms is the only way to protect yourself from getting these
infections.

It does not mean that your partner is cheating on you, as I told you earlier these bacteria
can stay in your body for many months before causing any symptoms. You/He might have
got this infection from your/his previous relationships.

www.aspire2plab.com 96
No real PLAB2 cases discussed. All discussed scenarios are fictitious and for educational purposes only.
The complications of this condition are Pelvic Inflammatory Disease (infection around your
womb and surrounding structures including the ovaries and the tubes connecting your
ovaries to your womb). This can also lead to some complications in your pregnancy like
pregnancy outside your womb. This can also lead to infertility.

D: We usually offer HIV test to those who have sexually transmitted infections. Do you wish
to have one? P: Yes/No

D: If you develop any fever or redness, hotness, swelling around your private parts or groin
area, any burning sensation while passing urine, any cloudy or smelly urine please come
back to us.

P: Is Gonorrhoea a serious condition?


P: How are you going to treat me Dr.?
P: How did I get this condition?
P: Did I get it from my partner?
P: Is he cheating on me?
P: What are the complications of Gonorrhoea?
!

www.aspire2plab.com 97
No real PLAB2 cases discussed. All discussed scenarios are fictitious and for educational purposes only.
Bacterial Vaginosis

You are an FY2 in Gynaecology. Miss Hannah Mills aged 25 is calling you for the swab results
which she did 2 days ago. The swab test was positive for Bacterial Vaginosis.
Explain to her the results on the phone and discuss the management.

D: How can I help you today? P: I am here to ask about my swab test results
D: I have got your swab results I will explain you shortly but before that let me ask some
questions first. P: Ok.
D: Could you please tell me why you had these swab tests?
P: I have discharge coming from my front passage.
D: When did the discharge start? P: 2 weeks
D: What is the colour of discharge? P: Greyish White.
D: Is the discharge thick? P: No, it’s thin/watery
D: Is there any blood in it? P: No.
D: Is there any smell? P: Yes doctor, it smells fishy.
D: How much was the discharge? P: Not too much.
D: Does it happen at any particular time? P: It happens after sex
D: Does your partner have any symptoms? P: No

D: Is there anything else that is bothering you? P: No.


D: Any pain or burning sensation while passing urine? P: No
D: Any soreness or itching? P: No
D: Any tummy (pelvic) pain P: No
D: Any blisters or sores P: No
D: Did you have any fever or flu like symptoms? P: No.

D: Have you been diagnosed with any medical condition in the past? P: No
D: Are you currently taking any medications, OTC drugs or supplements? P: No
D: Are you taking any birth control pills? P: No
D: Any allergies from any food or medication? P: No
D: Any previous surgeries or procedures done? P: No

D: When was your last menstrual period? P: 3 week ago.


D: Are they regular? P: Yes.
D: How long does your period last? P: 5 days
D: Any bleeding or spotting between your periods? P: No.
D: Any painful or heavy periods? P: No
D: Have you been pregnant before? P: No
D: Are you currently sexually active? P: Yes
D: Do you have a stable partner? P: Yes
D: Do you and your partner(s) use any contraception or protection against STIs?
P: We use condoms
D: What kind of sexual contact do you have? (Genital? Anal? Oral) P: Normal
D: Any pain during or after sex? P: No
D: When was the last time you had unprotected sex? P: 4 Days ago

www.aspire2plab.com 98
No real PLAB2 cases discussed. All discussed scenarios are fictitious and for educational purposes only.
D: Were there any issues with the contraception used? P: No

D: Do you smoke? P: No
D: Do you drink alcohol? P: No
D: Tell me about your diet? P: Good/Bad
D: Are you physically active? P: Yes/No
D: What do you do for a living? P:
D: Whom do you live with? P: With my partner

From the assessment and the swab result, you have Bacterial Vaginosis.

Bacterial vaginosis (BV) is a common cause of unusual vaginal discharge. BV is not a sexually
transmitted infection (STI), but it can increase your risk of getting an STI such as chlamydia.

The most common symptoms of bacterial vaginosis are vaginal discharge that has a strong
fishy smell, particularly after sex, sometimes it might be greyish-white and thin and watery.
But half of women with bacterial vaginosis do not have any symptoms.

Causes:
Bacterial vaginosis is caused by a change in the natural balance of bacteria in your vagina.
You're more likely to get it if:
● you're sexually active (but women who have not had sex can also get BV)
● you have had a change of partner
● you have an IUD (contraception device)
● you use perfumed products in or around your vagina.

BV is not an STI, even though it can be triggered by sex.


A woman can pass it to another woman during sex.
You're more likely to get an STI if you have BV. This may be because BV makes your vagina
less acidic and reduces your natural defences against infection.

Treatment for bacterial vaginosis


Bacterial vaginosis is usually treated with antibiotic tablets or gels or creams If the woman is
asymptomatic, treatment is not usually required, unless she is undergoing termination of
pregnancy.

If the woman is symptomatic:


Advise that, where possible, she should reduce exposure to contributing factors, such as
vaginal douching and the use of antiseptics, bubble baths, or shampoos in the bath.
Prescribe oral metronidazole 400 mg twice a day for 5 to 7 days.
If adherence to treatment is an issue, a single oral dose of 2 g may be used, if appropriate.

If the woman prefers topical treatment or cannot tolerate oral metronidazole:


Prescribe intravaginal metronidazole gel 0.75% once a day for 5 days (off-label for women
aged younger than 18 years) or intravaginal clindamycin cream 2% once a day for 7 days.

Oral clindamycin and oral tinidazole are alternatives but are less preferred.

www.aspire2plab.com 99
No real PLAB2 cases discussed. All discussed scenarios are fictitious and for educational purposes only.
If you have a same-sex partner, they may also need treatment. Male partners usually do not
need any treatment.

Recurring bacterial vaginosis


It's common for BV to come back, usually within 3 months.
You'll need to take treatment for longer up to 6 months if you keep getting BV (you get it
more than twice in 6 months).

A GP or sexual health clinic will help identify if something is triggering your BV, such as sex
or your period.

Things you can do yourself to help relieve symptoms and prevent bacterial vaginosis
returning:
Do’s
● use water and plain soap to wash your genital area
● have showers instead of baths

Don’t
● do not use perfumed soaps, bubble bath, shampoo or shower gel in the bath
● do not use vaginal deodorants, washes or douches
● do not put antiseptic liquids in the bath
● do not use strong detergents to wash your underwear
● do not smoke
!

www.aspire2plab.com 100
No real PLAB2 cases discussed. All discussed scenarios are fictitious and for educational purposes only.
Ectopic Pregnancy (LIF Pain & PV Bleeding)

You are FY2 in the OBG Department. Miss Jenny Smithers, aged 18, presented to the hospital
complaining of left iliac fossa pain. Patient has come with six weeks of amenorrhea.
Pregnancy test has been done and is positive. Please talk to the patient, take relevant
history and discuss different steps of management with the patient.

D: What brought you to the hospital? P: I have got pain here (Patient points at LIF).
D: Are you comfortable to talk? P: Yes, doctor.

D: When did it happen? P: It started last night.


D: How did it start? P: It started suddenly.
D: What were you doing? P: I was not doing anything.
D: Does it come and go? P: It is always there.
D: Has it got better? P: It is getting worse with time.
D: What kind of pain is it? P: It is a dull pain doctor.
D: Does it go anywhere? P: No doctor.
D: Does anything make it worse? P: No doctor, it is becoming worse on its own.
D: Did you do anything to make it better? P: I tried paracetamol, but it didn’t work.
D: When and how many tabs did you take? P: 2 tabs few hrs back.
D: Could you score the pain for me? P: Around 6-8.

D: Is there anything else that’s bothering you? P: I have got bleeding from my front passage.
D: May I know since when? P: Started around four hours ago.
D: What is the colour of bleeding? P: I didn’t notice it..
D: Was there any clots in it? P: No.
D: How many pads have you changed since the bleeding started?
P: Only one pad. It is more like spotting, doctor.
D: Was there any discharge present? P: No

D: Is there anything else that’s bothering you? P: I have been feeling sick since last night
D: Did you vomit? P: No doctor.

D: Anything else? P: No.

D: When was your last menstrual period? P: 6 weeks ago.


D: Are they regular? P: Yes.
D: Any bleeding or spotting between your periods? P: No.
D: Any painful or heavy periods? P: No

D: Are you pregnant by any chance? P: Yes


D: Did you test it and confirm?
P: I did an over the counter pregnancy test and it was positive.
D: Is this your first pregnancy? P: Yes

D: Did you use any type of contraception before? P: Yes/No


D: Have you ever used IUCD or coil? P: No

www.aspire2plab.com 101
No real PLAB2 cases discussed. All discussed scenarios are fictitious and for educational purposes only.
D: Have you had any fever or flu like symptoms? P: No
D: Any breast tenderness? P: No
D: Any pain around the tip of your shoulder? P: No
D: Any bowel problems? P: No
D: Any problem with your urination? P: No

D: Any dizziness or heart racing? P: No


D: Do you feel tired these days? P: No
D: Are you currently sexually active?
P: No, I broke up with my partner 2 weeks back.
D: When did you last have sexual activity? P: 2 weeks back
D: Have you had any other partners previously? P: Yes, I had many partners.
D: What kind of sexual contact do you have? (Genital? Anal? Oral) P: Genital/Oral
D: Do you and your partners use any contraception or protection against STIs? P: No
D: Do you use condoms? P: No, we enjoy our sex.
D: When was the last time you had unprotected sex? P: 2 weeks back

D: Have you been diagnosed with any medical condition in the past? P: No.
D: Any sexually transmitted infections or pelvic inflammatory disease?
P: Yes, I was diagnosed with Chlamydia.
D: When was that? P: When I was 15.
D: May I know how it was treated? P: My GP gave me antibiotics.
D: Are you currently taking any medications, over-the-counter drugs or supplements? P: No
D: Any blood thinner? P: No
D: Do you have any allergy to any food or drug? P: No
D: Any procedure or instrumentation through your front passage? P: No.
D: Any previous hospital stay? P: No

D: Has any member of your family ever been diagnosed with any medical condition? P: No

D: Do you smoke? P: Yes/ No


D: Do you drink alcohol? P: Yes/ No

I would like to check your vitals and examine your tummy, do bimanual and per speculum
examination.

I would like to send for some initial investigations including routine blood test and urine
test.

Examiner:
Tenderness in LIF.

From my assessment, I suspect you have a condition called Ectopic Pregnancy. This means
the pregnancy is outside the womb.

www.aspire2plab.com 102
No real PLAB2 cases discussed. All discussed scenarios are fictitious and for educational purposes only.
We need to do further investigations to confirm the diagnosis like we need to do a special
blood test to check the amount of substance called Beta HCG produced by your pregnancy.
We will do an US scan tomorrow morning to confirm the diagnosis and also to find out the
location and size of the pregnancy.

For this we need to keep you in the hospital. We will give you painkillers for your pain.

P: I don’t want to stay in the hospital.


D: May I know if you have any concern?

P: I don’t want my parents to know about this.


D: I see but it is very important for you to stay in the hospital and undergo all the
investigations. And if the pregnancy is confirmed to be outside the womb, then we have to
put you under observation.

P: I can come back for the tests tomorrow?


D: As I told you earlier, we need to observe you, as this condition may cause some severe
complications. The pregnancy may rupture and you may go into shock and collapse, which is
a serious condition.

D: If this happens we need to treat you immediately, by giving you fluids through your blood
vessels and taking you to the operation theatre for emergency surgery to remove the
ruptured pregnancy.

P: What are you going to do for me?


D: If it is confirmed that you have ectopic pregnancy, we have few options to treat you
depending upon your blood results and the age, size and location of the pregnancy.
● We may have to wait and watch for the pregnancy to terminate by itself.
● We may have to give you some medication called Methotrexate to terminate the
pregnancy.
● We have an option of surgery also to remove the pregnancy.

D: In the meanwhile if you get short of breath, dizzy or if you develop severe pain or
bleeding, please inform us immediately.

www.aspire2plab.com 103
No real PLAB2 cases discussed. All discussed scenarios are fictitious and for educational purposes only.
Miscarriage

You are F2 in Antenatal Clinic. Mrs Emma Worthing, aged 28, is referred by her GP for her
first ANC check-up. She is 6 weeks pregnant. Nurse has examined the patient. Urine test has
been done. Vitals have been checked. BP – 130/80, Pulse-70, Urine Test Negative (infection
& protein). Talk to the patient, take relevant history, do the initial assessment and address
her concerns.

D: What brought you to the hospital? P: My GP has sent me for my antenatal check-up
D: May I know why? P: I am pregnant.
D: Could you confirm the age of pregnancy? P: 6 weeks.
D: How has the pregnancy been so far? P: It’s Okay, Doctor.
D: Any complications so far? P: No.
D: That’s great. Any symptoms? P: Like what doctor?
D: Any bleeding, spotting or discharge from your front passage? P: No.
D: Feeling any tummy pain? (ectopic pregnancy) P: No.
D: Any pain in the breast? P: No.
D: Have you been feeling sick? P: No.
D: Any pregnancies before? P: Yes.
D: How many pregnancies? P: 2
D: When was that? P: 1st pregnancy was 3 years back and the 2nd was 1 years ago.
D: What was the outcome? P: Ended in miscarriage.
D: I’m sorry (Sympathy). May I know at what age did the pregnancy terminate?
P: One at 6 weeks and the other at 8 weeks.

D: How were they managed?


P: First time, I went to the hospital as I had some bleeding. They checked and confirmed
miscarriage. I was discharged and advised rest. Second time, I had bleeding again and went
to the hospital. They asked me to wait and watch. I came back some time later and they
confirmed the miscarriage.

D: Did you have any other symptoms apart from the bleeding? P: No.
D: Any sort of pain anywhere? P: No
D: Did they tell you why you had the miscarriage?
P: No, they did some test but they didn’t give any reason for the miscarriage.
D: Any fever? P: No.

D: Have you been diagnosed with any medical condition? P: No.


D: DM? HTN? Kidney problem? Thyroid problem? PCOS? Fibroids? STD/HIV? P: No.
D: Are you on any Medications? Allergies? Folic Acid? P: No.
D: Any surgical procedures around your private part or womb? P: No.
D: Any family members diagnosed with any medical conditions? P: No.

D: How are your periods? Regular? P: Yes/No


D: Are they painful? P: Yes/No
D: How long does the bleeding last? P: Yes/No
D: Any bleeding between the periods? P: Yes/No

www.aspire2plab.com 104
No real PLAB2 cases discussed. All discussed scenarios are fictitious and for educational purposes only.
D: Any usage of contraception? P: No.

D: Do you smoke? P: Yes/No


D: Do you drink alcohol? P: Yes/No
D: How is your diet? P: Good.
D: Do you drink coffee or tea? P: Yes/ No
D: How is your physical activity? P: I’m quite active.
D: Any usage of recreational drugs? P: No.

D: Are you in a stable relationship? P: Yes Doctor, I’m married.


D: What do you do for a living? P: Office job.
D: Whom do you live with? P: With my husband.

I would like to do some examination:


Blood Pressure, Pulse, Temperature and Breathing Rate;
Measure height and weight;
GPE (general physical examination);
Tummy exam (Abdominal/Antenatal)

Examination:
Everything normal
Investigation: Bloods for blood group/ sugar/ infections (rubella/syphilis/hepatitis/HIV).
US.

P: Why do I have these miscarriages?


D: There are many reasons why miscarriages may happen, although the exact cause is not
identified. If a miscarriage happens during the first trimester of pregnancy (the first three
months), it's usually caused by problems with the unborn baby (foetus).

If a miscarriage happens during the second trimester of pregnancy (between weeks 14 and
26), it may be the result of an underlying health condition in the mother.
We will keep monitoring you and your baby. Hopefully everything will be fine.

P: Is there anything I should be doing?


D: In many cases, cause is not known. However, you can lower the risk of miscarriage by:
Not smoking during pregnancy

Not drinking alcohol or using illegal drugs during pregnancy eating a healthy, balanced diet
with at least five portions of fruit and vegetables a day. Making attempts to avoid certain
infections during pregnancy, such as rubella. Avoiding certain foods during pregnancy,
which could make you ill or harm your baby being a healthy weight before getting pregnant.

P: Is it possible to have another miscarriage?


For most women, it’s a one-off event and they go on to have a successful pregnancy in the
future. Most women are able to have healthy pregnancy after a miscarriage, even in cases
of recurrent miscarriages.

www.aspire2plab.com 105
No real PLAB2 cases discussed. All discussed scenarios are fictitious and for educational purposes only.
First trimester miscarriages
Most first trimester miscarriages are caused by problems with the chromosomes of the
foetus.
Chromosome problems:
Sometimes, something can go wrong at the point of conception and the foetus receives too
many or not enough chromosomes. The reasons for this are often unclear, but it means the
foetus won't be able to develop normally, resulting in a miscarriage.
Placental problems:
The placenta is the organ linking the mother's blood supply to her baby's. If there's a
problem with the development of the placenta, it can also lead to a miscarriage.
Things that increase your risk
An early miscarriage may happen by chance. But there are several things known
to increase your risk of problems happening.
The age of the mother has an influence:
in women under 30, 1 in 10 pregnancies will end in miscarriage
in women aged 35-39, up to 2 in 10 pregnancies will end in miscarriage
in women over 45, more than half of all pregnancies will end in miscarriage
Other risk factors include: Obesity, Smoking, Drug misuse during pregnancy, Drinking more
than 200mg Caffeine and drinking 2 units of alcohol a week.
Second trimester miscarriages
Long-term health conditions like DM, HTN, Lupus, Kidney disease, Thyroid disease.
Infections like Rubella, CMV, HIV, STIs, Malaria, Syphilis.
Food poisoning: Listeriosis, Toxoplasmosis, Salmonella
Medicines: Misoprostol, Retinoids and NSAIDs
Womb structure
Weakened cervix
Polycystic ovary syndrome

P: What are you going to do for me?


D: It would be good if you incorporate certain lifestyle modifications.

We may offer US. We will follow up regularly. If you have 3 miscarriages, then we shall do
further tests to find out the underlying causes. However, no cause is found in half of the
cases.

P: Which tests?
D: An ultrasound scan to check the structure of your womb for any abnormalities. Second
procedure may be done with 3D US to study lower tummy and pelvis to get accurate
diagnosis. Scan also checks for weakened cervix. This can only be carried out if you are
pregnant again. In this case, it is done between 10 and 12 weeks pregnant.
Karyotyping for any genetic abnormality (chromosomes).
Blood tests to check for APS antibody levels and lupus anticoagulant. APS Antibodies are
known to increase blood clots and alter the way afterbirth attaches (aspirin/ heparin).

Please contact your GP, maternity team or early pregnancy unit at your local hospital in case
of vaginal bleeding immediately.

www.aspire2plab.com 106
No real PLAB2 cases discussed. All discussed scenarios are fictitious and for educational purposes only.
Missed Abortion/Miscarriage

You are an FY2 in Antenatal Clinic. Mrs Victoria Blair, aged 37, has come for her ultrasound
result. Transvaginal ultrasound scan was done which shows 6 weeks of gestation. Foetal
pole was found but no foetal heart was detected. Talk to her and address her concerns.

D: How can I help you today? P: I am here to ask about my ultrasound test results
D: I have got your results. I will explain to you shortly but before that let me ask some
questions first. P: Ok.
D: Why did you have this ultrasound done?
P: I was having discharge from my front passage
D: When did the discharge start? P: Yesterday
D: What is the colour of discharge? P: Brown colour
D: Is there any fresh blood in it? P: No.
D: Is there any smell? P: No.
D: How much was the discharge? P: Not too much.
D: Does your partner have any symptoms? P: No
D: Is there anything else that is bothering you? P: No.
D: Any pain or burning sensation while passing urine? P: No
D: Any soreness or itching? P: No
D: Any tummy (pelvic) pain P: Yes/No
D: Any blisters or sores P: No
D: Did you have any fever or flu like symptoms? P: No.
D: Any nausea or vomiting? (lack of pregnancy symptoms) P: No
D: Any breast tenderness? P: No

D: Have you been diagnosed with any medical condition in the past? P: No
D: Are you currently taking any medications, OTC drugs or supplements? P: No
D: Are you taking any birth control pills? P: No
D: Any allergies from any food or medication? P: No
D: Any previous surgeries or procedures done? P: No

D: When was your last menstrual period? P: 7 weeks ago.


D: Have you been pregnant before? P: No, this is our first pregnancy
after 6 years of trying

D: Are you currently sexually active? P: Yes


D: Do you have a stable partner? P: Yes
D: Any pain during or after sex? P: No
D: Were there any issues with the contraception used? P: No

D: Do you smoke? P: No
D: Do you drink alcohol? P: No
D: Tell me about your diet? P: Good/Bad
D: Are you physically active? P: Yes/No

www.aspire2plab.com 107
No real PLAB2 cases discussed. All discussed scenarios are fictitious and for educational purposes only.
D: What do you do for a living? P: Accountant
D: Whom do you live with? P: With my partner

I’d like to do vitals, GPE, per vaginal and abdominal examination. We will do initial
investigations and routine bloods.
I have seen your ultrasound and it showed your pregnancy at 6 weeks of gestation. However,
it did not detect your baby’s heartbeat. You also mentioned that you have been having brown
discharge from your front passage. This appears to be a missed miscarriage. We will check the
hormones which are associated with the pregnancy (Beta HCG, Progesterone) and we will be
do 2 blood test 48 hours apart to see if the levels are going or down.

We will repeat abdominal and transvaginal ultrasound in couple of day’s time.

A foetal heartbeat may first be detected by a vaginal ultrasound as early as 5 1/2 to 6 weeks
after gestation. That’s when a foetal pole, the first visible sign of a developing embryo, can
sometimes be seen.

If there's no pregnancy tissue left in your womb, no treatment is required.


However, if there's still some pregnancy tissue in your womb, your options are:
● expectant management – wait for the tissue to pass out of your womb naturally
● medical management – take medicine that causes the tissue to pass out of your womb
● surgical management – have the tissue surgically removed

A miscarriage can be very upsetting, and you and your partner may need counselling or
support.

If you're worried that you or your partner are having problems coping with grief, you may
need further treatment and counselling. There are support groups that can provide or
arrange counselling for people who have been affected by miscarriage.

www.aspire2plab.com 108
No real PLAB2 cases discussed. All discussed scenarios are fictitious and for educational purposes only.
Polycystic Ovarian Syndrome

You are FY2 in a GP clinic. Ms. Kim Jonas, aged 29, has come to GP Surgery with complaints
of Acne and Amenorrhea. On her first visit, a pregnancy test was done and found to be
negative. Her hormonal profile was done. Discuss these test results with her, take
appropriate history, discuss management and address her concerns.

Hormonal Results:
LH – 18 mlU/ml (1-12 mlU/ml)
FSH – 6 mlU/ml (1-9 mlU/ml)
BMI - 32

D: How can I help you today?


P: I came here for my results today.

D: Yes, I have your results with me but please tell me why you had these tests done?
P: I did not get my periods and also my ACNE was troubling me.

D: You did a very good thing by having these tests. Let me ask you a few questions to be
able to explain these better.

D: Since when have you not been getting your periods? P: 3 months
D: Were they regular before 3 months? P: Yes
D: Did anything happen before your periods stopped? P: Yes/No
D: Do you have any pain around your pelvis? P: Yes/No
D: Any pain in your breasts or discharge from your nipples? P: Yes/No

D: Tell me about your ACNE?


P: What do you want to know?
D: Since when have you had ACNE? P: 3 months
D: Have you done anything about it? P: I am using emollients.
D: Has it improved? P: Yes

D: Any other symptoms? P: No

D: Any fever/flu like symptoms? P: No


D: Do you feel tired? P: Yes/No
D: Any changes in your weight? P: Yes, I gained weight
D: How much in how much time? P: In last 3 months I gained 2 stones
D: Any bowel problem? P: Yes/No
D: Do you feel cold when others are feeling normal? P: Yes/No
D: Any excess hair anywhere? P: Yes/No

D: How is your mood? P: Good/Bad

D: Have you ever been diagnosed with any medical conditions in the past? P: No
D: Any Diabetes or Hypertension? P: No

www.aspire2plab.com 109
No real PLAB2 cases discussed. All discussed scenarios are fictitious and for educational purposes only.
D: Are you taking any other medications including OTC or herbal medications? P: No
D: Do you have any allergies from food or medicines? P: No
D: Any previous surgery or hospitalizations? P: No
D: Any surgeries around your womb or ovaries? P: No

D: Has anyone in your family suffered from a similar condition in the past?
P: Yes, my sister had a similar problem. She is coping with it.

D: Do you smoke? P: No
D: Do you take alcohol? P: Wine on weekend
D: Are you physically active? P: Yes/No
D: Tell me about your diet? P: I love Burgers and Chips
D: Any Stress? P: Yes, at work sometimes.

D: Who do you live with? P: I live with my partner


D: Do you have any children? P: No, I am not keen on having
children. I will think about it after I get married in a year’s time.

D: Are you sexually active? P: Yes


D: Do you use any contraception? P: Yes/No

D: I would like to check your vitals and do GPE. Ex: All normal.

From my assessment, I am suspecting you are having a condition called PCOS (Elaborate and
explain PCOS). You mentioned you have not had periods for the last 3 months and also have
acne which is getting better. Your BMI is also on the higher side which suggests PCOS.

We will be doing further investigation to confirm.


We will do some more blood tests including cholesterol levels and thyroid function test.
We will do an US of your ovaries to see follicles (fluid filled sacs) in which the egg develops.

The mainstay of PCOS treatment is lifestyle modification and reducing your weight.
We will refer you to a dietician to help with a diet plan as the diet is not so good and BMI is
also on the higher side. (Counsel about lifestyle accordingly).
We may also give you some medication to help with weight.

We will refer you to a specialist for further management.

We may have to give you medications to regularize your periods. These are usually
combined contraceptive pills. Sometimes, you may develop excess hair growth on your face
or chest. These OCPs also help in that case.

Usually prognosis is good with treatment.


Complications include infertility and also developing DM.
We will follow you up regularly with tests for hormone levels, blood pressure and diabetes.
!

www.aspire2plab.com 110
No real PLAB2 cases discussed. All discussed scenarios are fictitious and for educational purposes only.
Contraception

You are FY2 in GP. Miss Margaret Lewis, 29 years old, presents to the hospital to get some
advice about contraception. Talk to the patient, take relevant history and discuss different
methods of contraception.

D: What brought you to the hospital? P: I want to discuss contraception.


D: I am here to talk to you about everything you want to discuss. May I know, what do you
want to talk about?
P: I just changed my partner and my new partner doesn’t like condoms.
D: Okay, we have many types of contraception which we can offer you. There are 15
different methods of contraception currently available in the UK. The type that works best
for you will depend on your health and circumstances. But before that, do you have any
particular type of contraception in mind that you want me to talk about.

P: I want to know about contraceptive pills.


D: No problem, before offering you any type of contraception I would like to ask you a few
questions to see which type of contraception is better for you. P: Ok Doctor

D: Did you use any kind of contraception before?


P: Yes I used diaphragm before but I got pregnant.

D: Do you have any children? P: 2 kids aged 2yrs and 5yrs.


D: Have you completed your family? P: I am not sure. I just met my new partner.

D: Have you been diagnosed with any medical condition in the past?
P: I had a blood clot in my legs after a long flight 2yrs ago.

D: May I know what was done for that?


P: I went to the hospital and I was given warfarin for 6 months.
D: Any other medical condition? P: No

D: Any Stroke, Heart or liver disease or any ovarian cyst? P: No


D: Any STI or PID or pregnancy outside the womb? P: No
D: Are you currently taking any medications, otc drugs or supplements? P: No
D: Any blood thinner? P: No
D: Do you have any allergies? P: No
D: Any procedure or instrumentation through your front passage? P: No
D: Any previous hospital stay? P: No

D: Has any member of your family ever been diagnosed with any medical condition? P: No
D: Any problem with womb or pregnancy in the family? P: No

D: When was your last menstrual period? P: I am on my periods now.


D: Are they regular? P: Yes.
D: Any bleeding or spotting between your periods? P: No.
D: Any painful or heavy periods? P: No

www.aspire2plab.com 111
No real PLAB2 cases discussed. All discussed scenarios are fictitious and for educational purposes only.
D: Any pain or bleeding during or after sex? P: No

D: Do you smoke? P: Yes/ No


D: Do you drink alcohol? P: Yes/ No

D: From my assessment, you cannot have contraceptive pills as you have got blood clot in
your legs before. And taking these pills can cause you to have a blood clot again. But don’t
worry, we have many other types of contraception which we can offer you.

There are temporary, short term, long term and permanent methods of contraception.
There is tablet form, an injection, as a patch on your skin, as an implant under your skin, as a
device inserted into your womb and sterilisation which is permanent and irreversible.

Progestogen-only pill (mini pill):


Advantages:
• it's useful if you cannot take the hormone oestrogen, which is in the combined
pill, contraceptive patch and vaginal ring
• you can use it at any age – even if you smoke and are over 35
• it can reduce the symptoms of premenstrual syndrome (PMS) and painful periods.
Disadvantages:
● Your periods may be lighter, more frequent, or may stop altogether, and you may
get spotting between periods
● It does not protect you against STIs
● You need to remember to take it at or around the same time every day
● Medications, like certain types of antibiotics, can make it less effective.
Side effects:
● acne
● breast tenderness and breast enlargement
● an increased or decreased sex drive
● mood changes
● headache and migraine
● nausea or vomiting
● small fluid-filled sacs called cysts on your ovaries – these are usually harmless and
disappear without treatment
● weight gain.

These side effects are most likely to occur during the first few months of taking the
progestogen-only pill, but they generally improve over time and should stop within a few
months.

Contractive Injection (Depo-Provera):


There are three types of contraceptive injections in the UK: Depo-Provera, which lasts for 12
weeks, Sayana Press, which lasts for 13 weeks, and Noristerat, which lasts for eight weeks.
The most popular is Depo-Provera.

www.aspire2plab.com 112
No real PLAB2 cases discussed. All discussed scenarios are fictitious and for educational purposes only.
Advantages:
The main advantages of the contraceptive injection are:
• each injection lasts for either eight, 12 or 13 weeks
• the injection does not interrupt sex
• the injection is an option if you cannot use oestrogen-based contraception, such as the
combined pill, contraceptive patch or vaginal ring
• you do not have to remember to take a pill every day
• the injection is safe to use while you are breastfeeding
• the injection is not affected by other medicines
• the injection may reduce heavy, painful periods and help with premenstrual symptoms
for some women
• the injection offers some protection from pelvic inflammatory disease (the mucus from
the cervix may stop bacteria entering the womb) and may also give some protection
against cancer of the womb.

Disadvantages:
• Disrupted periods
• Weight gain
• Headaches
• Acne
• Tender breasts
• Changes in mood
• Loss of sex drive

Risks:
There is a small risk of infection at the site of the injection. In very rare cases, some people
may have an allergic reaction to the injection.

Contraceptive Implant:
Advantages:
• it works for three years
• the implant does not interrupt sex
• it is an option if you cannot use oestrogen-based contraception, such as the
combined contraceptive pill, contraceptive patch or vaginal ring
• you do not have to remember to take a pill every day
• the implant is safe to use while you are breastfeeding
• your fertility should return to normal as soon as the implant is removed
• implants offer some protection against pelvic inflammatory disease (the
mucus from the cervix may stop bacteria entering the womb) and may also give some
protection against cancer of the womb
• the implant may reduce heavy periods or painful periods after the first year
of use
• after the contraceptive implant has been inserted, you should be able to
carry out normal activities.

Disadvantages: Disrupted Periods.


Side effects:

www.aspire2plab.com 113
No real PLAB2 cases discussed. All discussed scenarios are fictitious and for educational purposes only.
• headaches
• acne
• nausea
• breast tenderness
• changes in mood
• loss of sex drive.

Intrauterine Device:
Advantages of the IUD:
• Most women can use an IUD, including women who have never been
pregnant.
• Once an IUD is fitted, it works straight away and lasts for up to 10 years or
until it's removed.
• It doesn't interrupt sex.
• It can be used if you're breastfeeding.
• Your normal fertility returns as soon as the IUD is taken out
• It's not affected by other medicines.
There's no evidence that having an IUD fitted will increase the risk of cancer of the
cervix, endometrial cancer (cancer of the lining of the womb) or ovarian cancer. Some
women experience changes in mood and libido, but these changes are very small. There is
no evidence that the IUD affects weight.
Disadvantages of the IUD:
• Your periods may become heavier, longer or more painful, though this may
improve after a few months.
• An IUD doesn't protect against STIs, so you may have to use condoms as well.
If you get an STI while you have an IUD, it could lead to a pelvic infection if not treated.
• The most common reasons that women stop using an IUD are vaginal
bleeding and pain.
Risks of the IUD:
● Damage to the womb
● Pelvic infections
● Rejection
● Ectopic Pregnancy.

Intrauterine system(Mirena):
Advantages of the IUS:
• It works for five years (Mirena) or three years (Jaydess).
• It's one of the most effective forms of contraception available in the UK.
• It doesn't interrupt sex.
• An IUS may be useful if you have heavy or painful periods because your
periods usually become much lighter and shorter, and sometimes less painful – they may
stop completely after the first year of use.
• It can be used safely if you're breastfeeding.
• It's not affected by other medicines.
• It may be a good option if you can't take the hormone oestrogen, which is
used in the combined contraceptive pill.
• Your fertility will return to normal when the IUS is removed.

www.aspire2plab.com 114
No real PLAB2 cases discussed. All discussed scenarios are fictitious and for educational purposes only.
There's no evidence that an IUS will affect your weight or that having an IUS fitted will
increase the risk of cervical cancer, uterus cancer or ovarian cancer. Some women
experience changes in mood and libido, but these changes are very small.

Disadvantages of the IUS:


• Some women won't be happy with the way that their periods may
change. For example, periods may become lighter and more irregular or, in some cases, stop
completely. Your periods are more likely to stop completely with Mirena than with Jaydess.
• Irregular bleeding and spotting are common in the first six months after
having an IUS fitted. This is not harmful and usually decreases with time.
• Some women experience headaches, acne and breast tenderness after
having the IUS fitted.
• An uncommon side effect of the IUS is the appearance of small fluid-filled
cysts on the ovaries – these usually disappear without treatment.
• An IUS doesn't protect you against STIs, so you may also have to use
condoms when having sex. If you get an STI while you have an IUS fitted, it could lead to
pelvic infection if it's not treated.
• Most women who stop using an IUS do so because of vaginal bleeding and
pain, although this is uncommon. Hormonal problems can also occur, but these are
even less common.

Risks of the IUS:


● Damage to the womb
● Pelvic infections
● Rejection
● Ectopic Pregnancy.

Female sterilisation:
Female sterilisation is usually carried out under general anaesthetic, but can be carried out
under local anaesthetic, depending on the method used. The surgery involves blocking or
sealing the fallopian tubes, which link the ovaries to the womb (uterus).
This prevents the woman’s eggs from reaching sperm and becoming fertilised. Eggs will still
be released from the ovaries as normal, but they will be absorbed naturally into the
woman's body.
There are two main types of female sterilisation:
• when your fallopian tubes are blocked – for example, with clips or rings (tubal
occlusion)
• when implants are used to block your fallopian tubes (hysteroscopic
sterilisation, or HS).
Removing the tubes (salpingectomy):
If blocking the fallopian tubes has been unsuccessful, the tubes may be completely
removed. Removal of the tubes is called salpingectomy.
Advantages:
• female sterilisation can be more than 99% effective at preventing pregnancy
• tubal occlusion (blocking the fallopian tubes) and removal of the tubes
(salpingectomy) should be effective immediately – however, doctors strongly recommend
that you continue to use contraception until your next period

www.aspire2plab.com 115
No real PLAB2 cases discussed. All discussed scenarios are fictitious and for educational purposes only.
• hysteroscopic sterilisation is usually effective after around three months –
research collected by NICE found that the fallopian tubes were blocked after three months
in 96% of sterilised women
Other advantages of female sterilisation are that:
• there are rarely any long-term effects on your sexual health
• it will not affect your sex drive
• it will not affect the spontaneity of sexual intercourse or interfere with sex
(as other forms of contraception can)
• it will not affect your hormone levels.
Disadvantages:
• female sterilisation does not protect you against STIs, so you should still use a
condom if you are unsure about your partner's sexual health
• it is very difficult to reverse a tubal occlusion – this involves removing the
blocked part of the fallopian tube and rejoining the ends, and reversal operations are rarely
funded by the NHS
• a 2015 US study found that around 1 in 50 women who had a hysteroscopic
sterilisation required further surgery due to complications such as persistent pain.
Risks:
• there is a very small risk of complications, including internal bleeding and
infection or damage to other organs
• it is possible for sterilisation to fail – the fallopian tubes can rejoin and make
you fertile again, although this is rare (about one in 200 women become pregnant in their
lifetime after being sterilised)
• if you do get pregnant after the operation, there is an increased risk that it
will be an ectopic pregnancy (when the fertilised egg grows outside the womb, usually in the
fallopian tubes)
If you miss a period, take a pregnancy test immediately. If the pregnancy test is positive, you
must see your GP so that you can be referred for a scan to check if the pregnancy is inside or
outside your womb.

Contraceptives that are more than 99% effective if used correctly:


• contraceptive implant (lasts up to three years)
• intrauterine system, or IUS (up to five years)
• intrauterine device, or IUD, also called the coil (up to five to 10 years)
• female sterilisation (permanent)
• male sterilisation or vasectomy (permanent)
Contraceptives that are more than 99% effective if always used correctly, but generally less
than 95% effective with typical use:
• contraceptive injection (renewed every eight weeks or every 12 weeks,
depending on the type)
• combined pill (taken every day for three weeks out of every month)
• progestogen-only pill (taken every day)
• contraceptive patch (renewed each week for three weeks in every month)
• vaginal ring (renewed once a month)
Contraceptives that are 99% effective if used according to teaching instructions:

www.aspire2plab.com 116
No real PLAB2 cases discussed. All discussed scenarios are fictitious and for educational purposes only.
• symptothermal method of natural family planning (daily monitoring of body
temperature and cervical mucus)
Contraceptives that are 98% effective if used correctly:
• male condom (every time you have sex)
Contraceptives that are 95% effective if used correctly:
• female condom (every time you have sex)
Contraceptives that are 92-96% effective if used correctly:
• diaphragm or cap with spermicide (every time you have sex)

D: Can you make contraception part of your daily routine?


Methods that are to be used each time you have sex:
• male condoms and female condoms
• diaphragm or cap
Methods that are to be taken every day:
• the pill (the combined pill or the progestogen-only pill)
Methods that are to be replaced every week:
• contraceptive patch
Methods that are to be replaced every month:
• vaginal ring
Methods that are to be renewed every two to three months:
• contraceptive injection
Methods that are to be renewed every three years:
• contraceptive implant
Methods that are to be renewed every five to 10 years:
• intrauterine device (IUD)
• intrauterine system (IUS)

D: Would you prefer contraception that you don't have to remember every day?
Not all contraceptives have to be taken every day or each time you have sex.
You don't have to think about some contraceptives for months or years. These methods
need to be inserted by a health professional into your uterus (IUD or IUS) or arm (the
implant):
• intrauterine device (IUD) (lasts up to five to 10 years, depending on the type)
• intrauterine system (IUS) (lasts up to three to five years, depending on the
type)
• contraceptive implant (lasts three years)
The contraceptive injection can be given one of two ways: either by an intramuscular
injection into the buttock, or as a subcutaneous injection into the thigh or abdomen. This is
given every eight weeks or every 12 weeks, depending on the type.
The subcutaneous injection can be given by a health professional, or you can be shown how
to inject it yourself.
Other contraceptives that need to be changed or replaced every month or week are:
• vaginal ring (worn for three weeks out of every four)
• contraceptive patch (a new patch is used each week for three weeks out of
every four)
Other contraceptives used or inserted just before sex are:
• diaphragm or cap

www.aspire2plab.com 117
No real PLAB2 cases discussed. All discussed scenarios are fictitious and for educational purposes only.
• male condom or female condom
P: May I know why I cannot use contraceptive pills, doctor?
P: What are the advantages, disadvantages and failure rates of Mini pill, Injection, Implant,
Coil and IUCD?
D: Can you make contraception part of your daily routine?
D: Would you prefer contraception that you don't have to remember every day?
D: Remember, the only way to protect yourself against sexually transmitted infections (STIs)
is to use a condom every time you have sex. Other methods of contraception prevent
pregnancy, but they don't protect against STIs.
D: If you miss a period, take a pregnancy test immediately and see your GP.
P: Thank you Doctor. I will consult my partner and get back to you.

15 Contraceptive Methods:

Barrier Method: (Prevents Fertilisation)


1. Male condom
2. Female Condom
3. Diaphragm
4. Cervical Cap

5. Natural Family Planning (Prevents Fertilisation)

Combined (Inhibits Ovulation)


6. Combined Pills
7. Patch
8. Vaginal Ring

Progesterone Only: (Inhibit Ovulation)


9. POP
10. Injections
11. Implant

Coil (Prevents Fertilisation and Implantation)


12. Hormonal Coil
13. Copper T

Permanent Sterilisation (Prevents Fertilisation)


14. Male Sterilisation
15. Female Sterilisation
!

www.aspire2plab.com 118
No real PLAB2 cases discussed. All discussed scenarios are fictitious and for educational purposes only.
Combined Pill Prescription

You are an FY2 in OBG. Miss Avery Smith aged 22 has come to you asking for 6 months
prescription of COCP. Talk to her and address her concerns.

D: How can I help you? P: I would like to be prescribed COCP for 6 months
D: May I know why? P: I am going on a holiday.
D: How much do you know about COCP? P:

D: Have you been diagnosed with any medical condition in the past? P: No
D: Have you ever had a stroke, blood clot in your legs or lungs, or heart attack? P: No
D: D: Do you have gallbladder disease or serious liver disease? P: No
Do you have repeated severe headaches, often on one side? (Migraine) P: No
D: Have you felt a breast lump? (breast cancer) P: No
D: Are you currently taking any medications, OTC drugs or supplements? P: No
D: Do you regularly take any pills for tuberculosis (TB), seizures (fits), or HIV? P: No
D: Any allergies from any food or medication? P: No
D: Any previous surgeries or procedures done? P: No
D: Do you have any family history of stroke, blood clots, heart disease, or diabetes? P: No

D: When was your last menstrual period? P: 1 week ago.


D: Are they regular? P: Yes.
D: How long does your period last? P: 5 days
D: Any bleeding or spotting between your periods? P: No.
D: Any painful or heavy periods? P: No
D: Are you pregnant by any chance right now? P: No
D: Have you been pregnant before? P: No
D: Are you currently sexually active? P: Yes
D: Do you have a partner? P: Yes
D: Do you and your partner(s) use any contraception or protection against STIs?
P: We used to use condoms, but we haven’t used one in 5 months
D: When was the last time you had unprotected sex? P: A week ago
D: Were there any issues with the contraception used? P: No

D: Do you smoke? P: No
D: Do you drink alcohol? P: No
D: Tell me about your diet? P: Good/Bad
D: Are you physically active? P: Yes/No
D: What do you do for a living? P: MUA (Make-up artist)
D: Whom do you live with? P: With my partner

I would like to do vitals, GPE, an examination of the breasts, chest, tummy, legs, and
measure the weight and height. I would also like to do baseline investigations, and a
pregnancy test.

Patient Concerns:
I am on holiday, so how can I avoid having periods?

www.aspire2plab.com 119
No real PLAB2 cases discussed. All discussed scenarios are fictitious and for educational purposes only.
Starting the combined pill:

It is important to remember that you cannot put in repeat prescription requests for
the Pill or for HRT as you would for other medications. Once the doctor is satisfied that
the contraceptive pill or HRT you are on is the most suitable one for you they will usually
issue a prescription for a six-month supply.

Pill checks
If you are taking the contraceptive pill then, for your safety, you will need check-ups every
6-12 months with the nurse, before your repeat prescriptions can be continued. The GP or
nurse will tell you how often you need a check. Please book your check-up well before you
will run out of your tablets.
If you do need a repeat prescription at the same time as your check-up we can arrange this
for you, but you will need to call back to collect your prescription once the GP has signed it.
This may be after 5.30pm the same day or another day. Alternatively, some pharmacies can
collect a prescription for you, if you arrange this with them.

Taking 2 packets of the combined pill back-to-back


If you take a combined contraceptive pill, you can delay your period by taking 2 packets
back-to-back.
How you do this will depend on which pill you take.
Examples are:

● monophasic 21-day pills, such as Microgynon and Cilest – you take a combined pill
for 21 days, followed by 7 days without pills, when you have a bleed (period). To
delay your period, start a new packet of pills straight after you finish the last pill and
miss out the 7-day break.
● everyday (ED) pills, such as Microgynon ED and Lorynon ED – you take a combined
pill every day. The first 21 pills are active pills and the next 7 pills are inactive or
dummy pills, when you have your period. To delay your period, miss out and throw
away the dummy pills, and start the active pills in a new packet straight away.
● phasic 21-day pills, such as Binovium, Qlaira and Logynon – the mix of hormones in
each pill is different, depending on which phase you're in. You need to take these
pills in the correct order to have effective contraception. Ask your pharmacist,
community contraception clinic or GP for more information.

You can get contraception free of charge, even if you're under 16, from:
• contraception clinics
• sexual health or GUM (genitourinary medicine) clinics
• some GP surgeries
• some young people's services

www.aspire2plab.com 120
No real PLAB2 cases discussed. All discussed scenarios are fictitious and for educational purposes only.
COCP (Combined Oral Contraceptive Pills)

The COCP may not be right for you if you:


- are pregnant
- smoke and are 35 or older
- stopped smoking less than a year ago and are 35 or older
- are very overweight
- take certain medicines (Antibiotics, Antiepileptics, HIV Drug)

The pill may also not be right for you if you have (or have had):
- blood clots in a vein, for example in your leg or lungs
- stroke or any other disease that narrows the arteries
- anyone in your close family having a blood clot under the age of 45
- a heart abnormality or heart disease, including high blood pressure
- severe migraines, especially with aura (warning symptoms)
- breast cancer disease of the gallbladder or liver
- diabetes with complications or diabetes for the past 20 years
!
Starting on the 1st day of your period
If you start the combined pill on the 1st day of your period (day 1 of your menstrual cycle)
you will be protected from pregnancy straight away. You will not need additional
contraception.

Starting on the 5th day of your cycle or before


If you start the pill on the 5th day of your period or before, you will still be protected from
pregnancy straight away, unless you have a short menstrual cycle (your period is every 23
days or less). If you have a short menstrual cycle, you will need additional contraception,
such as condoms, until you have taken the pill for 7 days.

Starting after the 5th day of your cycle


You will not be protected from pregnancy straight away and will need additional
contraception until you have taken the pill for 7 days.

What should I do if I miss a pill (combined pill)?

What if I have missed 1 pill?


You should take the last pill you missed now, even if this means taking 2 pills in 1 day

What if I have missed 2 or more pills?


You should:
• take the last pill you missed now, even if this means taking 2 pills in 1 day
• leave any earlier missed pills
• carry on taking the rest of the pack as normal
• use extra contraception, such as condoms for the next 7 days

www.aspire2plab.com 121
No real PLAB2 cases discussed. All discussed scenarios are fictitious and for educational purposes only.
Progesterone Only Pill

You may not be able to use the progesterone only pill if you:
• think you might be pregnant
• do not want your periods to change
• take other medicines that may affect the pill
• have unexplained bleeding in between periods or after sex
• have developed arterial disease or heart disease or have had a stroke
• have liver disease
• have breast cancer or have had it in the past
• have severe cirrhosis or liver tumours

If you're healthy and there are no medical reasons why you should not take the
progestogen-only pill, you can take it until your menopause or until you're 55.
!
Starting the first pack of pills
• choose a convenient time in the day to take your first pill
• continue to take a pill at the same time each day until the pack is finished
• start your next pack of pills the following day – there's no break between packs of pills

You can start the progestogen-only pill at any time in your menstrual cycle.
• If you start it on day 1 to 5 of your menstrual cycle (the first 5 days of your period), it'll work
straight away, and you'll be protected against pregnancy. You will not need additional
contraception.
• If you have a short menstrual cycle, you'll need additional contraception, such as condoms, until
you've taken the pill for 2 days.
• If you start the progestogen-only pill on any other day of your cycle, you will not be protected
from pregnancy straight away and will need additional contraception until you've taken the pill
for 2 days.

What should I do if I miss a pill (Progesterone Only Pill)?


There are 2 different types of :
• 3-hour progestogen-only pill (traditional progestogen-only pill) – must be taken within 3
hours of the same time each day
• 12-hour progestogen-only pill (desogestrel progestogen-only pill) – must be taken
within 12 hours of the same time each day

If you're less than 3 or less than 12 hours late taking the pill
If you're taking a 3-hour progestogen-only pill and are less than 3 hours late taking it, or if
you're taking the 12-hour progestogen-only pill and are less than 12 hours late:
• take the late pill as soon as you remember, and
• take the remaining pills as normal, even if that means taking 2 pills on the same day.

If you're more than 3 or more than 12 hours late taking the pill
What you should do:
• take a pill as soon as you remember – only take 1, even if you've missed more than 1
pill

www.aspire2plab.com 122
No real PLAB2 cases discussed. All discussed scenarios are fictitious and for educational purposes only.
• take the next pill at the usual time – this may mean taking 2 pills on the same day (1
when you remember and 1 at the usual time); this is not harmful
• carry on taking your remaining pills each day at the usual time
• use extra contraception such as condoms for the next 2 days (48 hours) after you
remember to take your missed pill, or do not have sex
• if you have unprotected sex from the time that you miss your pill until 2 days after
you start taking it reliably again, you may need emergency contraception – get
advice from your contraception clinic or GP

Emergency contraception
Emergency contraception can prevent pregnancy after unprotected sex or if your
contraceptive method has failed.

For example, a condom has split or you've missed a pill.


There are 2 types:
• emergency contraceptive pill (sometimes called the morning after pill)
• IUD (intrauterine device, or coil)

There are 2 kinds of emergency contraceptive pill:


Levonelle has to be taken within 72 hours (3 days) of sex.
ellaOne has to be taken within 120 hours (5 days) of sex.

Both pills work by preventing or delaying ovulation (release of an egg).


Emergency contraception is best taken as soon as possible to be effective.

The IUD can be inserted into your uterus up to 5 days after unprotected sex, or up to 5 days
after the earliest time you could have ovulated. It may stop an egg from being fertilised or
implanting in your womb.

Emergency contraception does not protect against sexually transmitted infections (STIs).

www.aspire2plab.com 123
No real PLAB2 cases discussed. All discussed scenarios are fictitious and for educational purposes only.
Premenstrual Syndrome (PMS)

You are an FY2 in GP. Miss Angela Di Caprio, aged 17, has come to you with complaint of
headache. Talk to her and address her concerns.

D: How can I help you?


P: I am having headaches on the right side of my head for the last one year.

D: What brought you to the hospital? P: I have been having headaches


D: Tell me more about your pain? P: Like what?
D: Where exactly do you have the pain? P: Right side of the head
D: When did it start? P: For the past 1 year
D: How did the pain start? P: It starts close to my periods
D: Is it continuous or comes and goes? P: Continuous during my periods
D: Is it sudden or gradual? P: Gradual
D: What type of pain is it? P: Dull
D: Does the pain go anywhere? P: No
D: Is there anything that makes the pain better or worse?
P: I had paracetamol, but it didn’t help
D: Could you please score the pain on a scale of 1 to 10, where 1 being no pain and 10 being
the most severe pain you have ever experienced. P: 7/8

D: Anything else? P: No
D: Any visual problems? P: No
D: Any blurry vision? P: No
D: Any weight loss? P: No
D: Do you have the worst ever headache in the back of your head? (SAH) P: No
D: Do you have an early morning headache? (SOL) P: No
D: Do you have projectile vomiting? (SOL) P: No
D: Do you have any numbness in your arm or leg? (SOL) P: No
D: Do you have eye pain or red/watery eyes? (Glaucoma, cluster headache) P: No

D: Have you been diagnosed with any medical condition in the past? P: No
D: Are you currently taking any medications, OTC drugs or supplements? P: No
D: Are you taking any birth control pills? P: No
D: Any allergies from any food or medication? P: No
D: Any previous surgeries or procedures done? P: No

D: When was your last menstrual period? P: 2/3 weeks ago.


D: Are they regular? P: Yes.
D: How long does your period last? P: 5 days
D: Any bleeding or spotting between your periods? P: No.
D: Any painful or heavy periods? P: No
D: Have you been pregnant before? P: No
D: Are you currently sexually active? P: No

D: Do you smoke? P: No

www.aspire2plab.com 124
No real PLAB2 cases discussed. All discussed scenarios are fictitious and for educational purposes only.
D: Do you drink alcohol? P: No
D: Tell me about your diet? P: Good

D: Are you physically active? P: Yes/No


D: What do you do for a living? P: Student

I would like to do vitals, GPE, an examination of the tummy and neurological examination.
I would also like to do baseline investigations.

Patience concerns: What could this be? How can you help me with this?

From the history you have told me, it appears you have PMS. PMS (premenstrual syndrome)
is the name for the symptoms women can experience in the weeks before their period.
Most women have PMS at some point. Each woman's symptoms are different and can vary
from month to month.

The most common symptoms of PMS include:


• mood swings
• feeling upset, anxious or irritable
• tiredness or trouble sleeping
• bloating or tummy pain
• breast tenderness
• headaches
• spotty skin or greasy hair
• changes in appetite and sex drive

Treating PMS (premenstrual syndrome)


As well as changes to your lifestyle, a GP can recommend treatments including:
• hormonal medicine – such as the combined contraceptive pill
• cognitive behavioural therapy – a talking therapy
• antidepressants

If you still get symptoms after trying these treatments, you may be referred to a specialist.
This could be a gynaecologist, psychiatrist or counsellor.

Do
regular exercise
eat a healthy, balanced diet
get plenty of sleep – 7 to 8 hours is recommended
try reducing your stress by doing yoga or meditation
take painkillers such as ibuprofen or paracetamol to ease the pain
keep a diary of your symptoms for at least 2 to 3 menstrual cycles – you can take this to a
GP appointment

Don't
do not smoke
do not drink too much alcohol

www.aspire2plab.com 125
No real PLAB2 cases discussed. All discussed scenarios are fictitious and for educational purposes only.
Mood Swings (Depo-Provera)

You are an FY2 in GP. Miss Katie Yale, aged 29, has come to you with some concerns. One
week ago, investigations were done in the GP and came back normal. Talk to her and
address her concerns.

D: How can I help you? P: My husband thinks I am moody on a monthly basis


D: Can you tell me more? P: Like what?
D: Can you tell me since when? P: Since the last 8 months
D: Is it continuous or comes and goes? P: Happens monthly during my periods

D: Anything else? P: No
D: Do you feel sad, hopeless or irritable most of the time? P: No
D: Do you have a loss of interest in everyday activities? P: No
D: Do you have feelings of emptiness or worthlessness? P: No
D: Do you have episodes of feeling very happy, elated or overjoyed? P: No
D: Do you sometimes feel full of great new ideas and important plans? P: No
D: Do you make decisions or say things that are out of character and that others see as
being risky or harmful? P: No

Please ask PMH and lifestyle questions.

D: When was your last menstrual period? P: 2/3 weeks ago.


D: Are they regular? P: Yes.
D: How long does your period last? P: 5 days
D: Any bleeding or spotting between your periods? P: No.
D: Any painful or heavy periods? P: No
D: Have you been pregnant before? P: No
D: Are you currently sexually active? P: Yes
D: Do you have a partner? P: Yes, my husband
D: Do you and your partner(s) use any contraception or protection against STI’s?
P: I was on Depo-Provera 1 year ago, now we are using the natural family planning method
for contraception.
D: Were there any issues with the contraception used? P: No
D: Do you use condoms? P: No

D: How has your mood been recently? P: It has been fine doctor.
D: Could you please score your mood for me? P: It’s 7/8
D: What do you do for a living? P: Lecturer
D: Whom do you live with? P: With my husband and 2 children

I would like to do vitals, GPE, an examination of the tummy and neurological examination.
I would also like to do baseline investigations. From the history you have told me, it appears
you have PMS.

www.aspire2plab.com 126
No real PLAB2 cases discussed. All discussed scenarios are fictitious and for educational purposes only.
Pregnancy (16-Year-Old Vomiting)

You are an FY2 in GP. Miss Leanne Bailey, aged 16, has booked an emergency appointment.
Talk to her, assess and manage her concerns.

D: How can I help you?


P: I've been feeling sick, and I have vomited a few times.
D: Can you tell me more? P: Like what?
D: Since when did this start? P: Since the last couple of days
D: What were you doing when it started? P: I wasn’t doing anything special.
D: Is it there all the time or does it come and go? P: I have nausea from time to time.
D: How often do you have nausea and vomiting? P: I have had 4 episodes so far.
D: Is there anything which triggers your symptoms? P: No, I can’t think of anything.
D: Is there anything which makes it better? P: Not really.
D: Is there anything which makes it better? P: No

D: Anything else? P: No
D: Do you feel thirsty? (dehydration) P: No
D: Is your urine dark yellow? (dehydration) P: No
D: Do your mouth and lips feel dry? (dehydration) P: No

D: Do you have any Tummy pain? P: No


D: Do you have Fever? P: No
D: Do you have Loose stools? P: No
D: Did you hurt yourself recently? (Trauma) P: No
D: Do you have a Headache? P: No

D: Have you been diagnosed with any medical condition in the past? P: No
D: Are you currently taking any medications, OTC drugs or supplements? P: No
D: Are you taking any birth control pills? P: No
D: Any allergies from any food or medication? P: No
D: Any previous surgeries or procedures done? P: No

D: When was your last menstrual period? P: It’s late by 1 week, I haven’t gotten it.
D: Are they regular otherwise? P: Yes.
D: How long does your period last? P: 5 days
D: Any bleeding or spotting between your periods? P: No.
D: Any painful or heavy periods? P: No
D: Have you been pregnant before? P: No

D: Are you currently sexually active? P: Yes


D: Do you have a partner? P: Yes, my boyfriend
D: How old is your partner? P: He is 17 years old
D: Do you and your partner(s) use any contraception or protection against STIs?
P: We practice pulling out (coitus interruptus)
D: Do you use condoms? P: No, we enjoy our sex.

www.aspire2plab.com 127
No real PLAB2 cases discussed. All discussed scenarios are fictitious and for educational purposes only.
D: Do you smoke? P: No
D: Do you drink alcohol? P: No
D: Tell me about your diet? P: Good/Bad
D: Are you physically active? P: Yes/No

D: Who do you live with? P: I live with my mother


D: How is your relationship with her? P: We have a good relationship

I would like to check your vitals, GPE and urine pregnancy test.
Ex: Vitals normal and UPT is Positive

D: From the history you have given me, and the tests we have done, it appears that you are
currently pregnant. This is why you have not been feeling well and have vomiting and
nausea.
P: I don’t know, there is a lot to take in.

D: Finding out you're pregnant when you're a teenager can be daunting, especially if the
pregnancy wasn't planned, but help and support is available.

As your pregnancy test is positive, it's understandable to feel mixed emotions: excitement
about having a child, worry about telling your parents, and anxiety about pregnancy and
childbirth.

You may also be feeling worried or frightened if you're not sure that you want to be
pregnant.

Make sure to talk through your options and think carefully before you make any decisions.
Try talking to a family member, friend or someone you trust. I would like to suggest you
discuss your pregnancy with your mother for support and understanding.

P: Don’t tell my mom I am pregnant.


D: Of course, as this conversation is confidential, we will not be discussing this with your
mother without your consent.
P: What are my options?
D: Your options are:
● continuing with the pregnancy and keeping the baby
● having an abortion
● continuing with the pregnancy and having the baby adopted

If you decide to continue your pregnancy, the next step is to start your antenatal care.
If you decide not to continue with your pregnancy, you can talk to a GP or visit a sexual
health clinic to discuss your options.
They can refer you for an assessment at a clinic or hospital if you choose to have an
abortion.

www.aspire2plab.com 128
No real PLAB2 cases discussed. All discussed scenarios are fictitious and for educational purposes only.
Antenatal Check-up (Rubella/RH Negative)

You are an FY2 in Antenatal Clinic. Mrs Chelsea Stokes, aged 30, is 14 weeks pregnant and
has come to the hospital for the reports. She came to antenatal clinic when she was 12
weeks pregnant for routine antenatal clinic. Talk to her and address her concerns.

Report:
Rubella; nonimmune
Rh: -ve or O Rhesus antibodies were negative
Blood: Normal.
Urine: Normal

D: How can I help you?


P: I’m here for my results

D: Could you confirm the age of pregnancy? P: 14 weeks doctor.


D: How has the pregnancy been so far? P: It’s Okay Doctor.
D: Any complications so far? P: No.
D: Any symptoms? P: Like what doctor?
D: Any bleeding, spotting or discharge from your front passage? P: No.
D: Feeling any tummy pain? (ectopic pregnancy) P: No.
D: Any pain in the breast? P: No.
D: Have you been feeling sick? P: No.
D: Was this a planned pregnancy? P: No
D: Any pregnancies before? P: No

D: Have you been diagnosed with any medical condition in the past? P: No
D: Are you currently taking any medications, OTC drugs or supplements? P: No
D: Are you taking any birth control pills? P: No
D: Any allergies from any food or medication? P: No
D: Any previous surgeries or procedures done? P: No

D: Do you smoke? P: Yes/No


D: Do you drink alcohol? P: Yes/No
D: Do you take recreational drugs? P: Yes, I smoke cannabis
D: Tell me about your diet? P: Good/Bad
D: Are you physically active? P: Yes/No
D: What do you do for a living? P: Manager
D: Whom do you live with? P: I live alone

I would like to do vitals, GPE, and an examination of the tummy.

Measles:
I have your blood results with me, and I will discuss them with you. Your blood test shows
that you do not have immunisation for rubella. It is an infection that is very similar to
measles, and people are vaccinated for it in their childhood.

www.aspire2plab.com 129
No real PLAB2 cases discussed. All discussed scenarios are fictitious and for educational purposes only.
If the patient is pregnant and the blood test showed that the patient is not immune to
rubella, it means that children who missed out on their MMR vaccinations could spread
rubella to you, so you need to be aware of this risk. If any of your friends or their children
have a rash, it’s better to stay away from them until the rash has gone. After you’ve had
your baby, you should have the two vaccinations, so you’re protected next time you get
pregnant.

A baby born damaged by rubella is said to have Congenital Rubella Syndrome (CRS). Many
will have hearing loss, cataracts, other eye conditions, and heart problems that require
significant hospital treatment and affect the child throughout their life. A baby’s brain can
also be affected It can cause loss of the baby (miscarriage)

There’s no evidence that the vaccine causes harm to unborn babies, but if you need the
vaccine you should have it after your baby is born. MMR immunisation during pregnancy is
not recommended.

RhD
Secondly, your blood test shows that you are RhD negative. Red blood cells sometimes have
another antigen, a protein known as the RhD antigen. If this is present, your blood group is
RhD positive. If it's absent, your blood group is RhD negative.

As you are RhD negative, you will be offered injections of anti-D immunoglobulin at certain
points in your pregnancy when you may be exposed to the baby's red blood cells. This anti-D
immunoglobulin helps to remove the RhD foetal blood cells before they can cause
sensitisation.

Rhesus disease can only occur in cases where all of the following happen:

● the mother has a rhesus negative (RhD negative) blood type


● the baby has a rhesus positive (RhD positive) blood type
● the mother has previously been exposed to RhD positive blood and has developed
an immune response to it (known as sensitisation)

Rhesus disease is a condition where antibodies in a pregnant woman's blood destroy her
baby's blood cells. It's also known as haemolytic disease of the foetus and newborn (HDFN).
Rhesus disease can largely be prevented by having an injection of a medication called anti-D
immunoglobulin.

This can help to avoid a process known as sensitisation, which is when a woman with RhD
negative blood is exposed to RhD positive blood and develops an immune response to it.

Anti-D immunoglobulin

www.aspire2plab.com 130
No real PLAB2 cases discussed. All discussed scenarios are fictitious and for educational purposes only.
The anti-D immunoglobulin neutralises any RhD positive antigens that may have entered the
mother's blood during pregnancy. If the antigens have been neutralised, the mother's blood
won't produce antibodies.

Alcohol:
Drinking in pregnancy can lead to long-term harm to the baby, with the more you drink, the
greater the risk. The safest approach is not to drink alcohol at all to keep risks to your baby
to a minimum.
When you drink, alcohol passes from your blood through the placenta and to your baby.
A baby's liver is one of the last organs to develop and does not mature until the later stages
of pregnancy.
Your baby cannot process alcohol as well as you can, and too much exposure to alcohol can
seriously affect their development.
Drinking alcohol, especially in the first 3 months of pregnancy, increases the risk of
miscarriage, premature birth and your baby having a low birthweight.
Drinking after the first 3 months of your pregnancy could affect your baby after they're
born.
The risks are greater the more you drink. The effects include learning difficulties and
behavioural problems.
Drinking heavily throughout pregnancy can cause your baby to develop a serious condition
called foetal alcohol syndrome (FAS).

Smoking cigarettes and cannabis:


Protecting your baby from smoke is one of the best things you can do to give your child a
healthy start in life. It can be difficult to stop smoking, but it's never too late to quit.
Using cannabis while pregnant may harm the unborn baby. Cannabis smoke contains many
of the same harmful chemicals found in cigarette smoke. Regularly smoking cannabis with
tobacco increases the risk of a baby being born small or premature.

Smoke can reduce your baby's birth weight and increase the risk of sudden infant death
syndrome (SIDS), also known as "cot death". Babies whose parents smoke are more likely to
be admitted to hospital for bronchitis and pneumonia during their first year.

The main reason that people smoke is because they are addicted to nicotine. We can offer
you nicotine replacement therapy. Nicotine replacement therapy is a medication that
provides you with a low level of nicotine poisonous chemicals present in tobacco smoke.
These can be given in the form of patch, spray or chewing gum.

It can help reduce unpleasant withdrawal effects such as bad mood and craving which may
happen when you stop smoking. You could also consider trying E cigarettes. Although they
are not risk free, they are very much safer than cigarettes and can help people stop
smoking.

The NHS Smoke free helpline offers free help, support and advice on stopping smoking and
can give you details of local support services.
You can also sign up to receive ongoing advice and support at a time that suits you.

www.aspire2plab.com 131
No real PLAB2 cases discussed. All discussed scenarios are fictitious and for educational purposes only.
Pre – Conception Counselling

You are an FY2 in GP. Miss Lucy Jenkins, 36 year old female, presented to your clinic. She
wants to become pregnant and is here for advice regarding that. Please talk to her and
address her concerns.

Previous Obstetrics History


4 P’s Questions
PMH:
- Diabetes
- Thyroid
- Hypertension
- Epilepsy & Mental Health Issues
- Kidney
- Cardiac
- Asthma
- Previous Clots or Blood Disorders
- IBD
- Rheumatological Conditions
- Genetic Disorders
o Usage of Prescription Medications including OTC & Herbal

Personal:
- Smoking
- Alcohol
- Recreational Drugs
- Dietary Habits and BMI
- Any plans to travel anywhere? (Epidemic Area)

The association between increasing maternal age and subfertility found an increased risk of
miscarriage, chromosomal abnormalities, congenital abnormalities, gestational diabetes,
placenta praevia, and caesarian delivery in women aged over 35 years, with women over 40
years experiencing an increased risk of abruption, preterm delivery, low birth weight and
perinatal mortality.

- Immunity to Infection: Chicken Pox and Rubella


- Use of Folic Acid to reduce the risk of Neural Tube Defect

www.aspire2plab.com 132
No real PLAB2 cases discussed. All discussed scenarios are fictitious and for educational purposes only.
Menopause/Premature Ovarian Insufficiency

Menopause is when menstruation stops permanently due to the loss of ovarian follicular
activity. It occurs with the final menstrual period and is usually diagnosed clinically after 12
months of amenorrhoea.

In the UK, the mean age of natural menopause is 51 years.

Perimenopause is the period before the menopause characterized by irregular cycles of


ovulation and menstruation and ends 12 months after the last menstrual period.
Postmenopause is the time after a woman has not had a menstrual period for 12
consecutive months.

Early menopause is the cessation of ovarian function between 40 and 45 years, in the
absence of other causes of secondary amenorrhoea.

Premature ovarian insufficiency (POI) is a clinical syndrome defined as the transient or


permanent loss of ovarian function before the age of 40 years.

A diagnosis of perimenopause or menopause should be suspected if there:


Is a change to the menstrual pattern.

Are symptoms including hot flushes/night sweats (vasomotor symptoms), mood disorders,
urogenital symptoms, altered sexual function, sleep disturbance, and fatigue.

Serum follicle-stimulating hormone (FSH) should not be used to diagnose perimenopause


or menopause in otherwise healthy women (who are not using hormonal contraception)
aged over 45 years, with typical symptoms.

Measurement of serum FSH may be considered in women aged over 45 years with
atypical symptoms; aged between 40–45 years with symptoms; and younger than 40 years
with suspected POI.

Assessment of a woman with suspected menopause should include:


Asking about symptoms and impact on quality of life, lifestyle, contraception, smear status,
family history, previous treatment(s) and wishes, current medication, and co-morbid
conditions.

www.aspire2plab.com 133
No real PLAB2 cases discussed. All discussed scenarios are fictitious and for educational purposes only.
Premature Ovarian Insufficiency

You are an FY2 in OBG. Miss Sarah Plunkett, aged 26, presented with history of
amenorrhoea. She had blood test done. Results were as follows:
Oestrogen is low
FSH and LH high
Diagnosis of Premature Ovarian Insufficiency was made.
Talk to her, explain the results and address her concerns.

D: How can I help you? P: Amenorrhoea.


D: Since when? P: For 2 years.
D: How were your periods before? P: Regular
D: How long did your period last? P: 5 days
D: Any bleeding or spotting between your periods? P: No.
D: Any painful or heavy periods? P: No

D: Any hot flashes? (early menopause) P: No


D: Any vaginal dryness? (early menopause) P: No
D: Do you have an uncomfortable sleep? (early menopause) P: No
D: Do you have problems with low libido? (early menopause) P: No
D: Any weight gain? (PCOS) P: No
D: Increased facial hair growth? (PCOS) P: No
D: Any weight loss? (Addison’s Disease) P: No
D: Increased thirst? (Addison’s Disease) P: No

Please ask about PMH and Lifestyle.

D: Have you been pregnant before? P: No


D: Are you currently sexually active? P: Yes
D: Do you have a partner? P: Yes
D: Do you and your partner(s) use any contraception or protection against STIs? P: No
D: Were there any issues with the contraception used? P: No
D: Do you use condoms? P: No, we enjoy our sex.

D: What do you do for a living? P: Manager


D: Whom do you live with? P: With my partner

I would like to do vitals, GPE, and tummy examination.


I would also like to send for baseline investigations including FBC, U&E, Creatinine, along
with specialised tests such as FSH, LH, Oestrogen and Testosterone.

Concerns:
She wants to become pregnant:

Causes of early menopause:


The ovaries stop working; Early menopause can happen naturally if a woman's ovaries stop

www.aspire2plab.com 134
No real PLAB2 cases discussed. All discussed scenarios are fictitious and for educational purposes only.
making normal levels of certain hormones, particularly the hormone oestrogen. This is
sometimes called premature ovarian failure, or primary ovarian insufficiency.

The cause of premature ovarian failure:


1. often unknown
2. chromosome abnormalities – such as in women with Turner syndrome
3. an autoimmune disease – where the immune system starts attacking body tissues
4. certain infections, such as tuberculosis, malaria and mumps.
5. Family history.
6. Certain Cancer treatments
7. Surgical removal of the ovaries.

Symptoms of early menopause:


1. periods becoming infrequent or stopping altogether.
2. hot flushes, night sweats
3. vaginal dryness and discomfort during sex
4. difficulty sleeping
5. low mood or anxiety
6. reduced sex drive (libido)
7. problems with memory and concentration

Women who go through early menopause also have an increased risk of osteoporosis and
cardiovascular disease because of their lowered oestrogen hormone levels.

Diagnosing early menopause:


based on your symptoms
your family history,
blood tests to check your hormone levels.

Treatments for early menopause:


Combined contraceptive pill or HRT unless contraindicated (C/I in breast cancer)
Lifestyle changes.
specialist menopause centre.

P: I want to have children?


D: Permanent early menopause will affect your ability to have children naturally. This can be
very distressing to women of all ages. You may still be able to have children by using IVF and
donating eggs from another woman or using your own eggs if you had some stored.
Surrogacy and adoption may also be options for you.

www.aspire2plab.com 135
No real PLAB2 cases discussed. All discussed scenarios are fictitious and for educational purposes only.
Menopause

You are an FY2 in GP. Mrs Hannah Jakes, aged 50, has come to you with some concerns.
Please talk to the patient, take history, assess, and discuss the initial plan of management
with the patient.

Common symptoms include:


- hot flushes – short, sudden feelings of heat, usually in the face, neck and chest, which
can make your skin red and sweaty
- night sweats – hot flushes that occur at night
- difficulty sleeping – this may make you feel tired and irritable during the day
- a reduced sex drive (libido)
- problems with memory and concentration
- vaginal dryness and pain, itching or discomfort during sex
- headaches
- mood changes, such as low mood or anxiety
- palpitations – heartbeats that suddenly become more noticeable
- joint stiffness, aches and pains
- reduced muscle mass
- recurrent urinary tract infections (UTIs)
- osteoporosis
- It happens when your ovaries stop producing as much of the hormone oestrogen and no
longer release an egg each month.

Premature or early menopause:


It can occur at any age, and in many cases there's no clear cause.

Sometimes it's caused by a treatment such as surgery to remove the ovaries


(oophorectomy), some breast cancer treatments, chemotherapy or radiotherapy, or it can
be brought on by an underlying condition, such as Down's syndrome or Addison's disease.

Investigations:
FBC, Urine (Pregnancy), FSH (40-45 years), TFT, Blood glucose, Cholesterol, Pelvic scan,
Cervical screening and mammograms.

Treatment:

- HRT
- Tibolone – suitable for women who had last period more than 1 year ago.
- Clonidine – non hormonal medication effective for hot flushes.
- Lifestyle Modification
- Mood changes – CBT, Anti-depressants
- Loss of libido – Testosterone gel on legs or tummy
- Vaginal dryness – Oestrogen pessary, cream, or ring

If you experience hot flushes and night sweats as a result of the menopause, simple
measures may sometimes help, such as:

www.aspire2plab.com 136
No real PLAB2 cases discussed. All discussed scenarios are fictitious and for educational purposes only.
- wearing light clothing
- keeping your bedroom cool at night
- taking a cool shower, using a fan or having a cold drink
- trying to reduce your stress levels
- avoiding potential triggers, such as spicy food, caffeine, smoking and alcohol
- taking regular exercise and losing weight if you're overweight

Side effects of HRT:


As with any medicine, HRT can cause side effects. But these will usually pass within 3
months of starting treatment.

Common side effects include:


- breast tenderness
- headaches
- feeling sick
- indigestion
- abdominal (tummy) pain
- vaginal bleeding

HRT may not be suitable if you:


- have a history of breast cancer, ovarian cancer or womb cancer
- have a history of blood clots
- have untreated high blood pressure – your blood pressure will need to be controlled
before you can start HRT
- have liver disease
- are pregnant – it's still possible to get pregnant while taking HRT, so you should use
contraception until 2 years after your last period if you're under 50, or for 1 year after
the age of 50
- In these circumstances, alternatives to HRT may be recommended instead.

Alternatives to HRT include:


- lifestyle measures. such as exercising regularly, eating a healthy diet, cutting down on
coffee, alcohol and spicy foods, and stopping smoking
- tibolone – a medicine that's similar to combined HRT (oestrogen and progestogen), but
may not be as effective and is only suitable for women who had their last period more
than 1 year ago
- antidepressants – some antidepressants can help with hot flushes and night sweats,
although they can also cause unpleasant side effects such as agitation and dizziness
- clonidine – a non-hormonal medicine that may help reduce hot flushes and night sweats
in some women, although any benefits are likely to be small
- Several remedies (such as bioidentical hormones) are claimed to help with menopausal
symptoms, but these are not recommended because it's not clear how safe and
effective they are.
- Bioidentical hormones are not the same as body identical hormones. Body identical
hormones, or micronised progesterone, can be prescribed to treat menopausal
symptoms.

www.aspire2plab.com 137
No real PLAB2 cases discussed. All discussed scenarios are fictitious and for educational purposes only.
Postmenopausal bleeding

You are an FY2 in GP. Mrs Katie Robbins, aged 50, has come to you with complaint of
vaginal bleeding. Please talk to her and address her concerns.

See your GP if you have postmenopausal bleeding, even if:


• it's only happened once
• there's only a small amount of blood, spotting, or pink or brown discharge
• you do not have any other symptoms
• you're not sure if it's blood

Postmenopausal bleeding is not usually serious but can be a sign of cancer. Cancer is easier
to treat if it's found early.

Cause Treatment
Cervical polyps the polyps may need to be removed by a specialist
Endometrial atrophy you may not need treatment, but may be offered oestrogen
cream or pessaries
Endometrial depending on the type of hyperplasia, you may be offered no
hyperplasia treatment, hormone medicine (tablets or an intrauterine
system, IUS) or a total hysterectomy (surgery to remove your
uterus, cervix and ovaries)
Side effect of HRT changing or stopping HRT treatment
Womb cancer total hysterectomy will often be recommended
Ovarian cancer surgery to remove your ovaries and your womb (total
hysterectomy)

www.aspire2plab.com 138
No real PLAB2 cases discussed. All discussed scenarios are fictitious and for educational purposes only.
Incontinence

You are an FY2 in GP Surgery. Mrs Katie Robbins, aged 55, has come to you with some
concerns. Please talk to her and address her concerns.

Other activities that may cause urine to leak include:

• sneezing
• laughing
• heavy lifting
• exercise

Causes

Stress incontinence:

1. Vaginal Delivery
2. Obesity
3. Hysterectomy
4. Multiple sclerosis

Urge Incontinence:
1. Tea, Coffee, Alcohol
2. Not drinking enough fluids
3. Constipation
4. UTI

Overflow Incontinence:
1. Constipation
2. Stones
3. Enlarged Prostate.

Certain medications can cause incontinence:


Medicines such as: ACE inhibitors, Antidepressants, HRT, Diuretics, Sedatives.

Risk factor:
1. Age, Family history,

Examination: GPE, Vitals, Vaginal, prostate in men,


Investigation: Routine blood, Urine dipstick, USG, Cystoscopy, urodynamic study.

Examiner: BMI 32

www.aspire2plab.com 139
No real PLAB2 cases discussed. All discussed scenarios are fictitious and for educational purposes only.
Bladder Diary for 3 days:
This should include:
• how much fluid you drink
• the types of fluid you drink
• how often you need to pass urine
• the amount of urine you pass
• how many episodes of incontinence you have
• how many times you experience an urgent need to go to the toilet

Conservative treatments, which do not involve medicines or surgery, are tried first. These
include:
• lifestyle changes
• pelvic floor muscle training (Kegel exercises)
• bladder training
Stress Incontinence:
1. Duloxetine

Urge Incontinence:
1. Oxybutynin
2. tolterodine

Surgery:
1. Colposuspension
2. Sling surgery.

www.aspire2plab.com 140
No real PLAB2 cases discussed. All discussed scenarios are fictitious and for educational purposes only.
Cervical Screening (Dyskaryosis)

You are an FY2 in GP. Miss Diane Jackson, aged 26, has come to you with some concerns.
She has had regular cervical screening. The results showed mild dyskaryosis and HPV was
found negative. She was advised to have regular follow up after 3 years. Talk to her,
assess her and address her concerns.

D: How can I help you? P: I am worried about having cancer.


D: May I know why?
P: I had my routine cervical screening, and I was told I have mild dyskaryosis. My
grandmother died of cervical cancer.
D: Let me ask you few questions. P: Ok
D: Do you have any symptoms? P: No
D: When was your LMP? P: 2 weeks ago.
D: Are your periods regular? P: Yes.
D: Any discharge from your vagina? P: No
D: Any bleeding between your periods? P: No
D: Any problem with the urine or bowel? P: No
D: How is your appetite these days? P: It’s alright
D: Have you noticed any decrease in weight? P: No
D: Any SOB or heart racing? P: No
D: Any pain in your lower back or pelvis? P: No
D: Any lumps and bumps in any part of the body? P: No

D: Are you sexually active? P: Yes


D: Do you practice safe sex? P: Yes, my partner uses condoms.
D: Any bleeding during or after sex? P: No
D: Do you have any children? P: No
D: Have you had a vaccine for HPV? P: Yes/No

D: Have you been diagnosed with any medical condition in the past? P: No
D: Are you taking any medications including OTC or supplements? P: No
D: Any allergies from any food or medications? P: No
D: Any previous hospital stay or surgeries? P: No
D: Has anyone else in the family been diagnosed with any medical condition? P: No

D: Do you smoke? P: Yes/No


D: Do you drink alcohol? P: Yes/No
D: Tell me about your diet? P: I try to eat healthy
D: Do you do physical exercise? P: Yes/No.

I would like to check your vitals and GPE.


I would like to send for some initial investigations including routine blood tests.

From my assessment, your report says you have mild dyskaryosis, which means mild
changes in your cervical cells. These are not cancerous. During your cervical screening, a
small sample of cells is taken from the cervix and tested for HPV. Fortunately, HPV testing

www.aspire2plab.com 141
No real PLAB2 cases discussed. All discussed scenarios are fictitious and for educational purposes only.
came back negative. It is advisable to come for next cervical screening after 3 years as
advised. If you develop any weight loss, lumps and bumps, pain during sex or any other
unusual symptoms, please come to the hospital.

Condoms can help you protect from HPV. Please practice safe sex.

The HPV vaccine protects against the types of HPV that causes the most cases of genital
warts and cervical cancer.

Concerns:
Is it cancer?
What are you going to do?

HPV Vaccination

In England, girls and boys aged 12 to 13 years are routinely offered the 1st HPV vaccination
when they're in school Year 8. The 2nd dose is offered 6 to 24 months after the 1st dose.

It's important to have both doses of the vaccine to be properly protected.

If you’re eligible and miss the HPV vaccine offered in Year 8 at school, it’s available for free
on the NHS up until your 25th birthday for:
• girls born after 1 September 1991
• boys born after 1 September 2006

www.aspire2plab.com 142
No real PLAB2 cases discussed. All discussed scenarios are fictitious and for educational purposes only.
Psychiatry
Mini Mental State Examination

You are FY2 A&E. Mr Albert Peterson, aged 77, has been brought to the hospital by the
police. He was wandering in the park and he was confused. He doesn’t know why he is in
the hospital. Please talk to the patient and assess the cognitive function. Explain your
findings and your further plan of management to your examiner. Do not take psychiatric
history. After 6 minutes, stop talking to your patient and talk to your examiner.

D: How can I help you?


P: Where am I?
D: You are in the hospital.

P: Why am I here?
D: You were wandering in the park. Police found you and brought you to the hospital.

P: Did I do something wrong?


D: No at all. You were just a bit confused.

P: Who are you?


D: My name is XYZ, I’m one of the junior doctors in this hospital. May I ask your name?
P: My name is Albert Peterson.

D: Pleasure to meet you.


P: Doctor, what am I doing here?

D: I am here to ask you some questions to assess how good your memory is?
P: Okay, no problem.

Time Orientation
You need to ask five questions of time from broadest to narrowest (year, season, month,
date and day) and for each correct answer you should give one score.

D: What year are we in?


P: It is 1956.

D: What season is it?


P: Doctor, it should be summer because the weather is so cold.

D: What month is it?


P: It is June.

D: What day is it?


P: It’s Monday, doctor.

D: What would be today’s date?


P: It's the 25th doctor.

www.aspire2plab.com 143
No real PLAB2 cases discussed. All discussed scenarios are fictitious and for educational purposes only.
Place Orientation
You need to ask five questions of place from broadest to narrowest (Country, county,
town/city, street and building) and for each correct answer you should give one score.

D: May I know what country are we in?


P: England

D: What county are we in?


P: Greater Manchester

D: What town/city are we in?


P: London.

D: What street are we in?


P: I don’t know.

D: Which building are we in?


P: I don’t know.

Registration
You should name three unrelated objects clearly and slowly and then ask the patient to
repeat them after you. You may remind him to remember them since you will be asking him
to recall them later.

D: I’m going to give you three words and I would like you to repeat them after me. Try to
remember them because I’m going to ask you to recall them later. The words are: apple,
table, penny.
P: Apple, table, penny.

Attention
In order to assess attention, give your patient a 5-letter word and ask him to spell it
backwards. You may use the word “WORLD”. The correct answer is: D-L-R-O-W.
For each correct answer, give him one score.

D: I would like you to spell the word WORLD backwards for me?
P: D… (Patient will take a pause and start thinking.)….It is difficult.
Who are you?
What am I doing here?
Why am I doing this thing? / Why are you asking me these questions?

Dr: My name is XYZ, I’m one of the junior doctors in this hospital. I am assessing your recent
memory.

P: Who brought me here?


D: You were wandering in the park. The Police were worried about you and so they brought
you to the hospital.

www.aspire2plab.com 144
No real PLAB2 cases discussed. All discussed scenarios are fictitious and for educational purposes only.
P: Oh okay doctor.

Note: An alternative way of assessing attention, mainly used if English is not their language,
is: “I would like you to count backward from 100 by sevens.”
If the patient makes a mistake, do not stop them. Let the patient carry on and check his
answers.
Stop after five answers.
The correct answers will be: 93, 86, 79, 72, 65.
Please give one score for each correct answer.

Recall
You should ask your patient to recall the three words you asked him to remember earlier.
For each word that he could remember, give him one score.

D: Earlier I had asked you to remember the three words, could you repeat that for me?

P: Which 3 words?
D: It’s okay, moving on

Language
You should show your patient simple objects, such as pen and pencil and ask him to name
them. For each correct answer, please give him one score.

D: Could you please name this object (pen) for me?


P: It’s a pen.

D: How about this one (Paper)?


P: (Patient is pushing himself.) It’s on the tip of my tongue but I cannot remember.

Repetition
Ask the patient to speak back a phrase. You may use this phrase “No ifs, ands, or buts”. If he
could repeat it after you correctly, give him one score.

D: Could you please repeat this sentence for me, No ifs, ands, or buts?
P: No ifs, ands, or buts.

Complex Command (3 Stage Command)


You need to give your patient 3 commands. Give one score for each correct task.

D: Take the paper in your right hand, fold it in half, and put it on the floor.

P: Okay.
(Sometimes the patient does as you say. Sometimes he keeps folding the paper and
sometimes he puts the paper back on the table instead of giving it back to you.)

Complex Command (Reading)

www.aspire2plab.com 145
No real PLAB2 cases discussed. All discussed scenarios are fictitious and for educational purposes only.
You should give your patient a written instruction and ask him to read it and do what it says.
If he follows your instruction correctly, please give one score.

D: Could you please follow the task written on this paper?


(You may write: “Close your eyes” on a piece of paper.)
P: (He will close his eyes.)

Complex Command (Writing)


You should give your patient a pen and a piece of paper and ask him to make up and write a
sentence about anything. If he writes a meaningful sentence that contains a noun and a
verb without any spelling or grammar mistake, please give him one score. (Usually he writes
a meaningful sentence, however, sometimes he may make spelling mistakes.)

D: Could you please write a meaningful sentence about anything for me?
P: (Patient writes different sentences every time.)
- The sun is shining today.
- It is a nice day.
- Sky is blue today.

Complex Command (Drawing)


You should draw the following picture on a piece of paper. Give your patient a pen and a
blank piece of paper and ask him to copy your picture. If he draws it correctly, please give
him one score. (All ten angles must be present and two angles must intersect.)

D: Could you please copy this picture for me?


P: (Usually he is not able to copy the picture correctly.)

Management with the Examiner


Note: You may not be able to complete your examination. In this case tell your examiner
that: “I would like to complete my mini-mental state examination.”

From my assessment, the MMSE suggests mild cognitive impairment.

Note:
According to the NICE guidelines,

25-30 - Normal.
21-24 - Mild Cognitive Impairment
10-20 - Moderate Cognitive Impairment
< 10 - Severe Cognitive Impairment.

www.aspire2plab.com 146
No real PLAB2 cases discussed. All discussed scenarios are fictitious and for educational purposes only.
My management plan would include

● Take complete medical history including social history from the patient.
● Perform necessary physical examinations.
● Order the routine blood tests.
● I would make a referral to the Psychiatric team once the patient is medically settled.
● There is a possibility of referral to Neuropsychiatry as well.
● They may involve the social services team and Homeless team if need be.

Full medical history including history of any medical illness, drug history and family history
should be taken.

Full examination looking for possible cardiac or neurological abnormalities should be


performed.

Some advanced cognitive assessment should also be done.

Some further investigations may be done in order to rule out physical causes.
These include some laboratory tests such as FBC, U&Es, LFT, calcium, vitamin B12, thyroid
function tests and random or fasting blood sugar

!
! !

www.aspire2plab.com 147
No real PLAB2 cases discussed. All discussed scenarios are fictitious and for educational purposes only.
!

www.aspire2plab.com 148
No real PLAB2 cases discussed. All discussed scenarios are fictitious and for educational purposes only.
Psychotic Patient

You are FY2 in A&E. Mr Gordon Brown, aged 29, has been brought to the hospital by the
police. He thinks he has been followed by the police, so he went to the police to give himself
up. Police investigated him and confirmed that he has not done anything wrong and there
is no evidence of his claim. He works in the post office. Please talk to the patient, take
history, explain the diagnosis to the patient and discuss your initial plan of management
with him.

Note:
Sometimes he looks anxious.
Sometimes he shows his anxiousness and agitation by moving his hands and legs while
talking to you.
Sometimes this patient doesn’t give good eye contact.
Sometimes he keeps looking at the door or on the floor.

A - Appearance (Clothing, posture, grooming)


B - Behaviour (Eye contact, Body language)
S - Speech ( Monotonous, loud, slurred)

M – Mood
C – Cognitive Function

S – Suicide
H –Hallucination (Visual, Auditory, Tactile, Olfactory)
I – Insight
T – Thought disorder (Insertion, withdrawal, broadcasting)

F – Family/Friends/Forensics/Finances
A – Addictions (Alcohol, drugs)
M – Medical Condition/Medications

D: How can I help you? P: Doctor, I don’t know…the police brought me to the hospital.
D: May I know why?
P: I don’t know, doctor. I noticed that the police have been following me. So I went to the
police to give myself up and they brought me here but they didn’t tell me why.

D: That’s a very good thing that you did, police were worried about you so they brought you
here. We are going to take good care of you. The police have told us that you have done
nothing wrong and there is nothing to be worried about.
P: No, Doctor, I must have done something wrong, which is why they have been following
me.

D: I see, why do you think they are following you?


P: I don’t know! I must have done something wrong! That’s why I went to the police today!
(Or may be some other delusion).

www.aspire2plab.com 149
No real PLAB2 cases discussed. All discussed scenarios are fictitious and for educational purposes only.
D: Alright, may I know since when do you have this feeling?
P: Doctor, I noticed it three months ago but it might have been longer than this, I’m not
sure.

D: Has it ever happened before? P: No, doctor.

D: Could you please elaborate as to what do you mean by “the police follow you”?
P: Doctor, they are everywhere. I can feel them…they are following me…they are watching
me! They have targeted me!

D: Have you ever confronted them? P: No. It’s just today that I gave myself up.

D: That’s alright, I will ask you some questions, it may sound strange however kindly bear
with me, is that okay? P: Sure Doctor

D: Do you hear the voice of these policemen?


P: No, I don’t but I’m sure they are following me.

D: Do you get a strange sense of smell that you get but nobody else around seems to smell
it? P: No
D: Do you feel that you can hear voices talking to you or talking about you, when there is
nobody else in the room? P: No Doctor.
D: Do you feel someone is putting thoughts in your brain? P: No.
D: Do you feel people can read your mind? P: No.
D: Do you think people can steal your thoughts? P: No.
D: Do you feel you have any special talents? P: No Doctor.
D: Could you please tell me today’s date? P: It’s 22rd of January.
D: Do you know which city we are in? P: Manchester.
D: Could you please spell the word WORLD backwards for me? OR “I would like you to count
backward from 100 by sevens.” P: D-L-R-O-W & 100,93,86,79,72
D: How has your mood and energy been recently? P: I feel fine doctor.
D: Could you please score your mood for me, on a scale of 1-10, one being the worst you
have ever felt and ten being the best you have ever felt? P: It’s around 6-7.
D: Have you ever tried to harm yourself? P: No.
D: Whom do you live with? P: I live alone.
D: Do you feel safe at home? P: Yes.
D: Do you feel safe here? P: Yes.
D: Do you have any partner or children? P: No.
D: Did you have any partners in the past? P: No, I love to be alone.
D: Do you have any family living nearby? P: My mom lives far away.
D: How is your relationship with her? P: It's fine doctor.
D: Do you have any friends? P: No, I prefer/want to be alone.
D: What do you do for a living? P: I used to work at a post-office
but I am not working there anymore.
D: Are you on any benefits? P: No doctor, I have my savings.
D: Have you ever had any problems with the law? P: No, I haven’t had any problems so far.

www.aspire2plab.com 150
No real PLAB2 cases discussed. All discussed scenarios are fictitious and for educational purposes only.
D: Do you drink alcohol? P: No.
D: Any recreational drugs? P: No.
D: Have you ever been diagnosed with any medical condition? P: No.
D: Are you taking any medications? P: No.
Dr: Has any member of your family ever been diagnosed with any mental health problems?
P: No doctor.
D: Thank you for being patient and answering my questions, it seems like you may have a
condition called delusional disorder. P: What is it?
D: It’s a condition in which a person has a firm false belief.
P: Okay, but I don’t think this is my case doctor!
D: How about let’s just agree to disagree and let me tell you what I can do for you.
As we are worried about you we would like to keep you in the hospital for sometime

To run some blood test (to rule out medical cause)


One of my colleagues from the mental health team will come and talk to you and if need be
we will get you some medications.

P: Will the police come and catch me here, doctor?


D: No my dear, do not worry you will be safe here. We are going to help you.
P: Okay, I’ll stay in the hospital if the police don't come.
Dr: Okay, let me tell you what we can do for you.

Causes:
1. History of delusional disorder or schizophrenia in the family.
2. Biochemical factors such as imbalance in neurotransmitters (which can interfere with the
transmission of messages, leading to symptoms.)
3. Environmental/Psychological factors such as:
- Excessive stress
- alcohol
- drug abuse
- Social isolation
- Being unmarried
- Being unemployed
- Low socioeconomic status

Differential Diagnosis:
1. Neurodegenerative disorders for example Multiple sclerosis.
2. Other CNS disorders for example Brain tumours, Epilepsy and Trauma.
3. Infectious diseases such as HIV.
4. Metabolic disorders such as Hypercalcemia, Hyponatremia.
5. Endocrine disorders such as Addison disease, Cushing syndrome and thyroid problems.
6. Vitamin deficiencies such as vitamin B12, folate, thiamine, niacin deficiency.
7. Medications such as anabolic steroids, corticosteroids, cimetidine and some antibiotics
such as penicillin.
8. Substance abuse such as Amphetamines, Cocaine, Alcohol, Cannabis.
9. Related psychiatric disorders such as Schizophrenia, delirium, Mood disorders with
delusional symptoms (manic or depressive type), Obsessive-compulsive disorder.

www.aspire2plab.com 151
No real PLAB2 cases discussed. All discussed scenarios are fictitious and for educational purposes only.
Plan:
Psych referral.
Complete medical history should be taken.
Complete physical examination {neurological and cardiovascular examinations}.
Blood tests
-infectious disease
-metabolic and endocrine disorders
-vitamin deficiencies.
Imaging
CT scan or MRI of the brain {tumours}.

Treatment:
Individual Psychotherapy.
Cognitive - Behavioural Therapy (CBT).
Support Therapy.
Insight-Oriented Therapy.
Family Therapy.
Address social stressors (Living alone, social isolation).
Possible Medication such as Antipsychotics, atypical antipsychotics

www.aspire2plab.com 152
No real PLAB2 cases discussed. All discussed scenarios are fictitious and for educational purposes only.
Psychosis

You are an FY2 in GP. Mother is concerned for her son Michael Smith, aged 40, as he has
been behaving strange for 3 weeks. The other day she made an appointment for face-to-
face consultation for her son. Talk to him and address his concerns.

D: How can I help you today?


P: I don’t have any clue why I am here. I am absolutely fine.

D: That’s alright, I will ask you some questions, it may sound strange however kindly bear
with me, is that okay?
P: Sure Doctor

D: How has your mood and energy been recently?


P: I feel fine doctor.

D: Could you please score your mood for me, on a scale of 1-10, one being the worst you
have ever felt and ten being the best you have ever felt? P: It’s around 6-7 doctor.

D: Do you get a strange sense of smell that you get but nobody else around seems to smell
it?
P: No Doctor?

D: Do you feel that you can hear voices talking to you or talking about you, when there is
nobody else in the room?
P: Yes, they are telling me to kill someone.

D: Do you feel someone is putting thoughts in your brain? P: No.

D: Do you feel people can read your mind? P: No.

D: Do you think people can steal your thoughts?


P: Yes, the radio is stealing my thoughts.

D: Do you feel you have any special talents?


P: No Doctor.

D: Could you please tell me today’s date?


P: It’s 22nd of January doctor.

D: Do you know which city we are in?


P: Manchester.

D: Could you please spell the word WORLD backwards for me? OR “I would like you to count
backward from 100 by sevens.”
P: D-L-R-O-W & 100,93,86,79,72

www.aspire2plab.com 153
No real PLAB2 cases discussed. All discussed scenarios are fictitious and for educational purposes only.
D: Have you ever tried to harm yourself?
P: No.

D: Whom do you live with?


P: Mother.

D: Do you feel safe at home?


P: Yes, Doctor.

D: Do you feel safe here?


P: Yes, Doctor.

D: Do you have any partner or children?


P: No.

D: Did you have any partners in the past?


P: No, I love to be alone.

D: Do you have any friends?


P: No, I prefer/want to be alone.

D: What do you do for living?


P: I used to work at a post-office, but I am not working there anymore.

D: Are you on any benefits?


P: No doctor, I have my savings.

D: Have you ever had any problems with the law?


P: No, I haven’t had any problems so far.

D: Have you ever been diagnosed with any medical condition?


P: No.

D: Are you taking any medications?


P: No.

D: Has any member of your family ever been diagnosed with any mental health problems?
P: No doctor.

D: Do you drink alcohol?


P: No.

D: Any recreational drugs?


P: No.

D: Thank you for being patient and answering my questions, it seems like that you may a
condition called delusional disorder.

www.aspire2plab.com 154
No real PLAB2 cases discussed. All discussed scenarios are fictitious and for educational purposes only.
P: What is it?

D: It’s a condition in which a person has a firm false belief.


P: Okay, but I don’t think this is my case doctor!

D: How about let’s just agree to disagree and let me tell you what I can do for you.

As we are worried about you, we would like to keep you in the hospital for some time to run
some blood test (to rule out medical cause).

One of my colleagues from the mental health team will come and talk to you and if need be
we will get you some medications.

www.aspire2plab.com 155
No real PLAB2 cases discussed. All discussed scenarios are fictitious and for educational purposes only.
Alcohol Dependency

You are FY2 Surgery. Mrs. Samantha Gray, aged 50, has undergone Hysteroscopy. The result
of the test is normal. She is about to be discharged. Please talk about her alcohol
consumption and discuss the management with the patient. Nurse has some concerns about
her alcohol intake and asked you to talk to her regarding her alcohol habits.

C - Cut Down, Craving


A – Annoyed, Anhedonia
G – Guilty
E – Eye Opener

D - Dependency
W – Withdrawal
T – Tolerance

M – Mood
I – Insight
S – Suicide
H – Hallucination (Visual, Auditory, Tactile, Olfactory)

F – Family/Friends/Forensics/Finances
A – Addictions (Alcohol, Drugs)
M – Medical Condition/Medications

D: How can I help you?


P: Doctor, I am going home. I was told that someone is going to talk to me.

D: May I ask why did you come into the hospital in the first place?
P: I came to the hospital for a test. What is the result of my test?

D: That’s right. We did an investigation to have a closer look at your womb and fortunately,
the result came back normal and you are good to go home.
P: Oh that’s wonderful, I may go home?

D: Yes, we will let you go home, however, if you do not mind, may I ask a few questions?
P: Sure Doctor.

D: So what has been your recent health apart from the reason why you came into the
hospital?
P: It’s been fine.

D: That’s great. Any chance that you suffer from any medical conditions? P: No.
D: Any family history of medical condition? P: No.
D: Do you take any regular medications? P: No.
D: Are you allergic to any medications/OTC/Herbal Remedies? P: No.

www.aspire2plab.com 156
No real PLAB2 cases discussed. All discussed scenarios are fictitious and for educational purposes only.
D: Do you smoke? P: No.
D: Do you do any recreational Drugs? P: No.
D: How about Alcohol? P: Oh Yes Doctor, I mainly drink wine.
D: How much do you drink? P: Not that much.
D: Could you please tell me how much you drink in a day or in a week approximately?
P: Doctor, as much as my friend does.

D: How about could you give me a rough estimate of how much you drank in the last week,
before you came into the hospital?
P: I drink around 6-7 bottles of wine per week

D: May I know since have you been drinking? P: Since I was a 16 year old.

D: Has it always been this much?


P: No doctor, earlier it was less, with the passage of time, it has increased.

D: I understand that, and honestly alcohol is not harmful to the body, if drunk in
moderation, however, having a bottle of wine a day will have some effect on your body.

D: Have you ever considered cutting down on your drinking?


P: Yes, about 7 months ago, I went to alcohol anonymous (AA) but I could not stop.

D: May I know what happened that you could not stop?


P: Doctor, I got many symptoms. I had diarrhoea, tummy pain and my hands started to
shake. I was so agitated.

D: Are there any symptoms if you don’t drink alcohol?


P: I start to sweat a lot when I don’t drink.

D: I am glad that you made some efforts. Let me ask you another question? Do you feel
annoyed when people talk about your alcohol habit? P: No.

D: Have you ever felt guilty about your drinking habit?


P: Yes, sometimes I feel guilty.

D: Have there been days where you had a drink first thing in the morning?
P: Yes. Sometimes I need a drink in the morning else my hands start to shake.

D: Does not having alcohol affect your daily activity?


P: I have to drink doctor; otherwise, I cannot do my job.

D: Do you have to increase the amount of your alcohol intake to get the same feeling?
P: Doctor, a long time ago I had to increase the amount to get the same feeling, but now I
stick to the same amount.

D: Thank you for answering these difficult questions, very quickly let me ask you a few more
general questions that we happen to ask anyone.

www.aspire2plab.com 157
No real PLAB2 cases discussed. All discussed scenarios are fictitious and for educational purposes only.
D: How has your mood been recently?
P: It has been fine.

D: Could you please score your mood for me, with 1 being the lowest and 10 being the
highest?
P: I would say 7-8.

D: Whom do you live with? P: I live with my husband.


D: How do you get on with him? P: We are fine but sometimes we fight.

D: Do you have any kids?


P: Yes, doctor. I have one daughter. She got married and she lives with her husband.

D: That’s wonderful, do they live nearby? P: Oh yes …we see each other from time to time.
D: Wonderful, how about any other relatives who live nearby?
P: Yes. My mom and my brother live in the town and we meet each other regularly.
D: I am glad you have a loving family, how about your friends? P: I have many friends.
D: That’s good, how do you spend your time with them?
P: We go out a lot. We gather to drink.
D: What do you do for a living? P: I work in a wine bar.
D: Oh I am sure you must be loving your job then, so how are things at work?
P: I am happy to be there. It is not stressful and I have many friends there.
D: Right, so are your finances sorted? P: Oh yes. I get paid enough.
D: Do you drink after work at the workplace? P: Yes, me and friends drink after work.
D: Have you ever had any trouble with the law? P: No Doctor.
D: By any chance, have you ever thought of hurting yourself? P: No.
D: Thank you for being so patient with me, by the looks of it, your report is normal so that is
sorted, however your alcohol consumption has been a bit off the recommended alcohol
allowance limit. Which is 14 units for females (14 for males), spread over the week.
P: Doctor, I know…but it is not easy for me to stop.
D: Well we are here to help you. P: Yes, please help me.
D: As per our conversation, it does NOT seem that there is

1. History of severe withdrawal symptoms.


2. Risk of self-harm.
3. Or family or social support.

So essentially we will not admit you to the hospital and we will refer to one of the services
and if need be use some medical interventions as well.

So there are Medical and non-Medical options.


What would you like me to talk about?

P: Whichever you want, doctor.

D: Let me start with the non-medical options.

www.aspire2plab.com 158
No real PLAB2 cases discussed. All discussed scenarios are fictitious and for educational purposes only.
1. Self-help groups (Alcohol Anonymous).
2. One to One counselling.
3. CBT
4. Family Therapy
5. Alcohol Diary.

Medical Options
1. Acamprosate
2. Naltrexone
3. Disulfiram
4. Chlordiazepoxide

In the Alcohol anonymous group, you will meet many people who used to drink but have
stopped now. They will share their experiences with you, which will motivate you. Such
groups help you to realize the problem and assist you to find a solution for it.
You can also have one to one counselling sessions.

You will have a talking therapy (CBT) with a solving approach towards alcohol dependence.
It involves identifying the beliefs that may possibly cause the obstacles in quitting alcohol.
For example, if you think that you are not able to work without taking alcohol.

The therapists will help you set a motivational goal. You may not be asked to stop
immediately instead you will be asked to reduce gradually and then eventually stop.
They try to identify and help to avoid and eliminate the triggers. For example, you told me
that you work in a wine bar; this may make it difficult for you to quit drinking alcohol, so
they will help to address this issue.

I am sure you agree that family support will have a positive impact on you and will improve
the efficiency of your treatment. Our colleagues can talk to your family about the problem
and the approach we are taking towards solving it if you wish to (Family therapy).

You may be asked to complete a diary in which you will be recording your alcohol intake
regularly. This will help to show you and your therapist how well you are doing and how
soon you reach your target (Alcohol diary).

Medical Options

Now I am going to talk about medical options you have:


There are some medications that help to reduce your craving for alcohol (Acamprosate).

There are some other medications that block the receptors in our body that are sensitive to
alcohol. This will help to reduce the amount of alcohol intake and prevent relapse. These
medications are usually prescribed along with counselling and other therapies
(Naltrexone/Nalmefene).

www.aspire2plab.com 159
No real PLAB2 cases discussed. All discussed scenarios are fictitious and for educational purposes only.
There is another type of medication that reacts with alcohol and leads to an unpleasant
feeling. So you might not want to drink again since you may have some symptoms like
feeling sick, vomiting, dizziness or chest discomfort when you drink alcohol (Disulfiram).

You may be prescribed some medications to reduce the withdrawal symptoms such as
agitation and hand shaking (Diazepam/Chlordiazepoxide).

P: Thank you Doctor.

D: You’re Welcome. The major credit goes to one of our nurses as she picked up that you
may have an alcohol issue.

D: Before I leave, any questions for me?


P: No Doctor.

www.aspire2plab.com 160
No real PLAB2 cases discussed. All discussed scenarios are fictitious and for educational purposes only.
Drug Dependency

You are FY2 in Psychiatry. Mr. Jack Rawlins, aged 35, has been referred to you. He wants to
quit taking drugs. Please talk to the patient, take history, talk about his opioid abuse and
discuss management options with him.

D: How can I help you?


P: Doctor, I am taking drugs and I am addicted to them. I want to give them up! I went to my
GP and he sent me to the hospital.

D: That’s a great decision that you have made. We are going to help you, however, in order
to provide you with the best treatment, how about I ask a few of questions first?
P: Sure!

D: So firstly what drug are you referring to?


P: I take heroin.

D: How do you take it?


P: I used to sniff it, but now I am injecting myself.

D: Do you share the needle?


P: No doctor. I am on a needle exchange program.

D: That’s good. May I know, how much do you take?


P: I take 1 gm per day. It costs me “X” every day.

D: How many times per day do you use it?


P: I take it 2-3 times per day.

D: Could you please tell me since when have you been doing it?
P: Since I was 17 doctor. / Since 4-5 years ago.

D: Any other drugs apart from heroin?


P: Yes, I have tried Cannabis, Amphetamine and Cocaine.

D: I will address these issues later, could we talk about the heroin problem at the moment.

D: So, when did you start taking Heroin?


P: It was just a mistake doctor.

D: However, it's great you want to stop. I want to know why do you want to stop?
P: Because of my family doctor.

D: I understand. Have you ever tried to cut it down or stop?


P: Yes, but I couldn’t maintain it.

D: May I know why?

www.aspire2plab.com 161
No real PLAB2 cases discussed. All discussed scenarios are fictitious and for educational purposes only.
P: I developed many symptoms, like tummy pain/discomfort. I would feel sick. My hands
began to shake and I also felt my heart racing. I would become nervous as well.

D: Do you crave to take the substance?


P: Yes/No

D: Do you enjoy activities which you use to enjoy before?


P: Yes/No

D: Do you feel annoyed when people talk about your addiction?


P: Yes, I feel uncomfortable.

D: Have you ever felt guilty about your addiction?


P: Yes, I do and that’s why I want to stop.

D: Do you need it first thing in the morning?


P: I always need it. It doesn’t really matter what time of the day it is.

D: That must be very difficult for you, so tell me are you able to manage your daily activities
without taking it?
P: No doctor.

D: Do you have to increase the amount to get the same feeling?


P: No.

D: Thank you for answering those difficult questions, let me ask you a few general questions
that we tend to ask everyone, for starters, how has been your mood recently?
P: It’s been alright.

D: how about you help me by scoring it for me, one being the lowest and ten being the
highest?
P: It’s been around 3-4.

D: May I know, whom do you live with?


P: I live with my partner and kids.

D: How is the relationship with your partner?


P: It’s alright but sometimes we fight.

D: May I know why?


P: She also does drugs.

D: How is the relationship with your kids?


P: Great! We all love each other.

D: That’s wonderful, having a motivation will help you quit, and one of the best supports is
the support from family. Talking about support, tell me about your friends?

www.aspire2plab.com 162
No real PLAB2 cases discussed. All discussed scenarios are fictitious and for educational purposes only.
P: I have a few.

D: How do you spend time with them?


P: Doctor, we do drugs together.

D: What do you do for a living?


P: I am not working at the moment doctor. I am on benefit. I was self-employed earlier.

D: How do you buy heroin?


P: My friends help me.

D: Have you ever had any trouble with the law?


P: I was arrested once for shoplifting and stealing.

D: Are you taking any medications?


P: No.

D: Any allergies?
P: No

D: Any Medical Condition?


P: No

D: Okay, so we will examine you and order routine investigations. Thereafter, we will admit
you and start you on your treatment.

Management

Non-medical

1. Narcotic Anonymous
2. One to One Counselling
3. Talking Therapy (CBT)

Medical

1. Methadone (along with non-medical treatment).

There are some things that you can do and some things that we can do to help and support
you.

You can try the self-help groups. Narcotics anonymous is one of them. You will meet many
people who had similar problems and have stopped. They will share their experiences which
will motivate you as well.

You can also try one to one counselling sessions.

www.aspire2plab.com 163
No real PLAB2 cases discussed. All discussed scenarios are fictitious and for educational purposes only.
You can go for a talking therapy with a solving approach towards drug dependence (CBT).

Medical
D: We can also give you a medication, called Methadone. It will reduce both craving as well
as your withdrawal symptoms.

D: Any questions for me?


P: No doctor?

D: Could you please bring in your partner so we can help her as well.
P: OK

Note: Sometimes the patient will tell you that Social service is taking care of my child.

Note:
If both the parents are taking drugs and they didn’t mention that social service is taking care
of the children. Then you need to mention the social services for the safety of the children.

www.aspire2plab.com 164
No real PLAB2 cases discussed. All discussed scenarios are fictitious and for educational purposes only.
Suicide

You are FY2 in Psychiatry. Miss Anastasia Hamilton, aged 16, took some tablets last night
and cut her wrist this morning. Medical management has been done and the patient is
medically stable. Please talk to the patient, take history and discuss management with the
patient.

Before
Any trigger factor (Bullying, Argument)
Was it planned, or impulsive?
Did the patient write a suicide note.
Have they tried to do it before?

During
What method of self-harm was involved?
Was the patient alone?
Where were they when they self-harmed?
What was going through their mind at the time?
What did they do straight after the incident?
How did they feel about it?

After
How did they get to the hospital?
How does the patient feel about the attempt now?
Any plans to do again in future?
What might prevent you from doing this again in the future?

Specific questions for overdose:


What medication or medications did you take?
Where did you get the medication from?
How much of the medication did you take?
What did you take the medication with?

Specific questions for cutting the hand:


Where are the cuts?
Number of cuts?
How deep are the cuts?
M – Mood
I – Insight
S – Suicide
H – Hallucination (Visual, Auditory, Tactile, Olfactory)
F – Family/Friends/Forensics/Finances
A – Addictions (Alcohol, Drugs)
M – Medical Condition/Medications

www.aspire2plab.com 165
No real PLAB2 cases discussed. All discussed scenarios are fictitious and for educational purposes only.
D: Hello my dear, what brings you to the hospital? P: Doctor, I want to go home.

D: I understand that you want to go home but let me first ask you a few questions and then
if everything is fine you can go home. Is that alright? P: Okay.

D: I see there is a bandage on your wrist, may I ask what happened?


P: I took some tablets and cut my wrist.

D: I’m sure it must have been a stressful situation that you did this. I am very sorry to hear
that. We are here to help you, you can trust me on this. Could you please tell me why you
did that?
P: Doctor, I missed my period and I realised that I’m pregnant. I called my boyfriend to let
him know and then we had a fight over the phone. He broke up with me…he left me.

D: Relationships can be difficult. I’m really sorry to hear that. You mentioned that you took
some tablets, may I know what did you take?
P: I took some OCP pills.

D: How many? P: 21.


D: Where did you get them from? P: I took my mom’s pills.

D: When did you take them?


P: I took them last night before going to bed.

D: Okay. You told me that you cut your wrist. How deep did you cut? P: It wasn’t that deep.

D: May I know when you did that?


P: I woke up this morning and realised that nothing has happened then I cut my wrist.

D: Where did you do that? P: I went to the bathroom and cut my wrist.
D: Who was there at the time? P: I was there by myself. My mom was at work.
D: May I know what you did after that?
P: I was so scared! I held my wrist and tried to press it to stop the bleeding and then I called
a cab and came here.

D: That’s a wonderful thing that you did. Did you inform any member of your family?
P: No doctor, I just rushed to the hospital.

D: Would you like us to inform your parents?


P: No. Please don’t let them know.

D: That’s okay, if you don’t want me to call them. Did you take any alcohol when you took
the pills or you cut your wrist? P: No doctor.

D: How about any recreational drugs? P: No.

P: Doctor, I am fine. I don’t want to stay in the hospital. Can I please go home?

www.aspire2plab.com 166
No real PLAB2 cases discussed. All discussed scenarios are fictitious and for educational purposes only.
D: I understand that you want to go home just a few more questions to make sure that
everything is fine. P: Okay fine.

D: Has this happened before? P: No doctor.


D: Do you think you are going to do it again?
P: It was so stupid of me. I’m embarrassed about what I did.
D: How do you feel about what you did? P: Bad
D: Did you plan for it? P: No doctor.
D: Did you tell anyone before doing it? P: No doctor.
D: By any chance, did you leave any note? P: No.

D: Doctor…please let me go home. I’m fine.


P: I can imagine how frustrated you are. Just bear with me for a couple of minutes.
D: How has your mood been recently? P: Doctor not too bad.
D: Could you please score your mood for me, with 1 being the lowest and 10 being the
highest? P: Doctor, I would say 5-6.
D: Whom do you live with? P: I live with my mom.
D: Does anyone else live with you and your mom? P: My little brother.
D: How is your relationship with your mother?
P: I’m very close to her but I haven’t told her about my boyfriend.
D: Do you get on well with your brother?
P: He is just a little kid so we don’t have much to talk about.
D: How about your dad? Where does he live?
P: Doctor, my parents got divorced a few years ago. He doesn’t live with us anymore.
D: I’m so sorry to hear that. Are you in touch with him? P: I see him once in a while.
D: What do you do? Are you going to school or you work? P: Doctor, I’m going to school.
D: How are things at school? Are you catching up well? P: I’m a bit lagging behind.
D: Do you have any friends in school? Are you getting on well with your friends?
P: Yes, doctor, I have many friends at school.
D: How about any friends outside school? P: Yes, I’ve got a few.
D: Do you mind if I ask you a few questions about your boyfriend? P: That’s alright.
D: May I know for how long you have been together? P: It’s been a few months now.
D: How did you guys meet? P: We go to the same school doctor.
D: May I know how old he is? P: He is 16 years old.
D: Have you ever had any trouble with the law? P: No.

D: Have you ever been diagnosed with any medical condition? P: No doctor.
D: Have you ever taken advice from mental health experts? P: No.
D: Has any member of your family ever been diagnosed with any mental health illness?
P: No.

D: Do you drink alcohol? P: No.


D: How about any recreational drugs? P: No.

P: Doctor, Can I please go home?

www.aspire2plab.com 167
No real PLAB2 cases discussed. All discussed scenarios are fictitious and for educational purposes only.
D: I totally get it that the hospital is not the place you want to be in, and clearly you have
been through a lot, and all seems like situational. It is indeed a difficult situation and I think
bringing in the family in the picture would help, what do you have to say about that?

P: Doctor, if I ask my mom to come, can I go home?


D: How about we call your mother, talk to your mom, address the safety issues, and
meanwhile my colleagues will come and talk to you regarding the help they can provide
from their side. Once everything is fine, we will send you ASAP. P: Okay.

D: My colleagues will come and repeat a pregnancy test, just to make sure everything is fine
and if need be what are the possible interventions needed. P: Thank you doctor.
D: Any questions for me? P: No.
D: Let me call your mother, would you mind waiting here till then? P: Okay.
D. Great. I will see you in sometime. P: Thank You

Treatment:
• Family and Friends support
• Inform senior doctors
• Recognise stressors and avoid alcohol use when stressed
• Support of the mental health team

Offer Support
• See a Doctor
• Phone local support group
• provide them with the number

www.aspire2plab.com 168
No real PLAB2 cases discussed. All discussed scenarios are fictitious and for educational purposes only.
Paracetamol Overdose (Homosexual Patient)

You are FY2 in A&E. Mr James Cook aged, 19 took 16 paracetamol tablets. Please talk to
him and make an initial plan of management.

D: What brought you to the hospital? P: I took some tablets.


D: May I know which tablets? P: PCM
D: May I know how many tablets did you take? P: 16
D: When did you take them? P: 2 hrs ago.
D: Did you take them in one go or at different times? P: I took them in one go.
D: Did you take anything else with it? P: I took it with a glass of water.
D: Did you take alcohol with it? P: No
D: Did you take any other medications with it? P: No.
D: By any chance did you take any recreational drugs? P: No.
D: Did you try to throw them up? P: No
D: How do you feel now? P: I am fine
D: Do you have any symptoms? P: I don’t have any symptoms.
D: Do you have any tummy pain? P: No
D: Do you feel sick? P: No
D: Any vomiting? P: No

D: Have you been diagnosed with any medical condition? P: No


D: Do you have any liver, kidney diseases? P: No
D: Any blood disorders? P: No
D: Do you take any regular medications, OTC or herbal remedies? P: No
D: Do you have any allergies? P: No
D: May I know why you took PCMtablets?
P: I had an argument with my mother after she found that I am gay and I have a boyfriend.
She is not accepting this.
D: I am so sorry to hear that. Where were you when you took the tablets?
P: I was in the hostel accommodation.
D: Who was there with you? P: I was alone
D: Who brought you to the hospital? P: I called the ambulance.
D: What did you do after that? P: I called my boyfriend and he told me to go to the hospital.
D: Have you ever tried to harm yourself in the past? Has it happened before? P: No.
D: Do you think you are going to do it again? P: No. I feel bad for what I have done.
D: How do you feel about what you did? P: I feel bad/stupid / guilty for what I have done.
D: Did you plan for it? P: No
D: Did you tell anyone before doing it? P: No
D: Did you leave any note? P: No
D: How has your mood been recently? P: Not too bad. It has been fine.
D: Could you please score your mood for me, with 1 being the lowest and 10 being the
highest? P: 6 – 7
D: Who do you live with? P: I live with my boyfriend in the hostel.
D: Do you have any other family members apart from your mother? P: No
D: How about your dad? P: My parents got divorced.

www.aspire2plab.com 169
No real PLAB2 cases discussed. All discussed scenarios are fictitious and for educational purposes only.
D: How is your relationship with your mother? P: She hates me being gay.
D: What do you do? P: I just started studying in university.
D: Do you work? P: I am a student.
D: Are you financially stable? P: Yes
D: Do you have any friends? P: Yes. I have many friends.
D: Tell me about your boyfriend? P: What would you like to know?
D: How long have you been together? P: 12 months
D: How is your relationship with your boyfriend? P: Good
D: Have you ever had any mental health problems? P: No

D: Do you smoke? P: Yes/No


D: Do you drink alcohol? P: No
D: Do you use recreational drugs? P: No

I would like to check your vitals and examine your tummy.

As you told me, you took some PCM tablets. We are going to keep you in the observation
unit to keep monitoring you. We also need to do some necessary investigations.

We will do some blood tests:


LFT, KFT, Bleeding and clotting profile and the level of PCM.

Depending on the level of paracetamol, you may need to receive some treatments.

I totally understand your concern. Like I said, it is very important for you to stay in the
hospital since we need to check the level of paracetamol in your blood in the next couple of
hours and then treat you accordingly (after 4 hours of paracetamol ingestion). If this is left
untreated, it can cause many complications.

We are going to give you a medication called N-Acetyl cysteine also known as NAC. If the
level of PCM is found high in your blood, we need to give you this medication through your
blood vessel as a drip. So, you need to be in the hospital while we are giving you the
medication.

If you need any treatment, the course of medication usually takes around 21 hours. Once
the course of treatment has been completed, we need to reassess you. We may do some
blood tests to make sure everything is fine.

When we have made sure that you are medically fine, you need to be referred to one of our
colleagues. I will arrange for you to be seen by our psychiatric colleague. Our colleague will
talk to you to find out if you have any stress in your life that may have caused this event.
Their aim is to support you. They will help you out in relieving your stress and improving
your mood.

Don’t you think you need someone to be with you? Do you want to inform your mother or
your boyfriend to be with you? You may need their help and support?
We can also have a talk with your mother if you wish to.!

www.aspire2plab.com 170
No real PLAB2 cases discussed. All discussed scenarios are fictitious and for educational purposes only.
Anorexia Nervosa

You are FY2 in Psychiatry. Miss Emily Morgan, aged 18, is sent to the hospital by the GP
because she has been losing weight for the past 6 months. She has not been diagnosed
with any medical condition. She is not taking any medication. Her BMI is 17. Talk to the
patient and discuss the initial plan of management with her.

Causes of Anorexia
You may be more likely to get an eating disorder if:
• you or a member of your family has a history of eating disorders, depression, or
alcohol or drug addiction.
• you have been criticised for your eating habits, body shape or weight
• you are overly concerned with being slim and feel pressure from society or your job.
• you have been sexually abused

B – Body Language
M – Mirror
W – Weight Loss

C – Clothes
L – Laxative
E – Excessive Exercise
V – Vomiting
E – Eating Less
R – Role Model

Complications:
Anaemia
Osteoporosis
Irregular periods (Amenorrhoea)
hair and skin problems
Infections
Heart problems (Bradycardia, abnormal heart rhythms)
Depression
Bowel problem (pain, constipation)

D: What brought you to the Hospital?


P: My parents have some concern about my weight loss. They took me to the GP and he
sent me to the hospital.

D: Are you losing weight? P: Yes


D: How much weight have you lost? P: 2 stones
D: In how much time have you lost this weight? P: In the last few months.
D: Has it happened before? P: No, doctor.
D: Is it intentional? P: Yes, I want to lose weight.
D: How are you losing weight? Any diet or exercise plan? P: Yes, I have a diet plan.
D: Tell me about your diet plan, what do you eat?

www.aspire2plab.com 171
No real PLAB2 cases discussed. All discussed scenarios are fictitious and for educational purposes only.
P: Every morning I eat an apple, I don’t eat lunch. I only eat biscuit or salad for dinner.
D: Do you do any exercise? P: Yes, I go to gym twice every day, morning and evening.
D: How long is each session? P: Each session lasts for around one and half hour.
D: Do you check your weight? P: Yes, I check my weight few times a day
D: How has your health been recently? P: Fine but I am tired all the time
D: Do you have frequent episodes of fever? P: Yes/No
D: Do you feel hungry? P: Yes
D: Do you feel lightheaded? P: Yes
D: Do you feel dizzy? P: Yes, but I want to lose weight
D: How are your periods? P: I've not had periods since last few months.
D: Are you on any contraceptives? P: No
D: Could it be possible that you’re pregnant? P: No
D: May I know why you want to lose weight?
P: I want to look thin and attractive. I want to wear nice clothes. I have a friend who looks
attractive and is slim and she also found a new boyfriend. I want to be just like her. She is
not hanging around with me.

Empathy, Try and find out if there is insight, and if she would be willing for treatment

D: Do you have a role model? P: Victoria Beckham.


D: How is your mood these days? P: I don’t have a good mood Doctor
D: Score your mood? P: 1-3/10, I have been upset since it is affecting my studies.
D: How are things at school? P: I’m behind my classmates as I can’t study because of this.
D: Do you get along well with your parents? P: Yes
D: Have you ever thought of harming yourself? P: No

Considering your BMI is 17 and from the conversation we had it seems like you have a
condition called Anorexia Nervosa and it is important to start treatment right away. Is that
okay? P: Yes

D: We are going to admit you in the hospital and run a series of initial blood investigations
that includes FBC, BM, U&Es, TFT, LFT along with that one of my colleagues will help you
with your diet plan as well. In addition to that, we will arrange for some talking therapy as
well so that we can have a holistic approach towards the situation.

Once we feel you are well enough to be discharged from the hospital we will refer you to
suitable services such as:

1. Diet Advice.
2. CBT.
3. Family Therapy.

Diet advice:

During your treatment you will probably be given advice on healthy eating and your diet.
However, this advice alone will not help you recover from anorexia, so you will need to have
talking therapy as well as dietary advice.

www.aspire2plab.com 172
No real PLAB2 cases discussed. All discussed scenarios are fictitious and for educational purposes only.
Your doctors will probably also advise you to take vitamin and mineral supplements so you
get all the nutrients you need to be healthy.

Anorexia can make your bones weaker, which can make you more likely to develop a
condition called osteoporosis. This is more likely if your weight has been low for a year or
more in children and young people, or 2 years or more in adults.

Because of this, your doctor may suggest you have a special type of X-ray called a bone-
density scan to check the health of your bones.

Girls and women are more at risk of getting weak bones than men, so your doctor may
prescribe you medicine to help protect your bones against osteoporosis.

Cognitive behavioural therapy (CBT):

If you are offered CBT, it will usually involve weekly sessions for up to 40 weeks (9 to 10
months), and 2 sessions a week in the first 2 to 3 weeks.

CBT involves talking to a therapist who will work with you to create a personalised
treatment plan.

They will help you to:


1. cope with your feelings
2. understand nutrition and the effects of starvation
3. make healthy food choices

They will ask you to practice these techniques on your own, measure your progress, and
show you ways to manage difficult feelings and situations so you stick with your new eating
habits
Treatment for children and young people:
Children and young people will usually be offered family therapy. You may also be offered
CBT or adolescent-focused psychotherapy. CBT will be very similar to the CBT offered to
adults.

Family therapy
Family therapy involves you and your family talking to a therapist, exploring how anorexia
has affected you and how your family can support you to get better.

Your therapist will also help you find ways to manage difficult feelings and situations to stop
you from relapsing into unhealthy eating habits once your therapy ends.

You can have the sessions together with your family or on your own with the therapist.
Family therapy is sometimes offered in a group with other families.

You will usually be offered 18 to 20 sessions over a year, and your therapist will regularly
check that the schedule is still working for you.

www.aspire2plab.com 173
No real PLAB2 cases discussed. All discussed scenarios are fictitious and for educational purposes only.
Where treatment will happen:
Most people with anorexia will be able to stay at home during their treatment. You will
usually have appointments at your clinic and then be able to go home.
However, you may be admitted to hospital if you have serious health complications. For
example, if:
● you are very underweight and still losing weight
● you are very ill and your life is at risk
● you are under 18 and your doctors believe you don't have enough support at home
● doctors are worried that you might harm yourself or are at risk of suicide
Your doctors will keep a careful eye on your weight and health if you're being cared for in
hospital. They will help you to reach a healthy weight gradually, and either start or continue
any therapy.

Treatment for adults:


A number of different talking therapies are available to treat anorexia. The aim of these
treatments is to help you understand the causes of your eating problems and feel more
comfortable with food so you can begin to eat better and reach a healthy weight.

You may be offered any of the following types of talking therapy. If you feel one isn't right
for you or isn't helping, you can talk to your doctors about trying a different kind of therapy.

Maudsley Anorexia Nervosa Treatment for Adults (MANTRA):


MANTRA involves talking to a therapist in order to understand what is causing your eating
disorder. It focuses on what's important to you and helps you to change your behaviour
when you are ready.

You can involve your family or carers if you think it would be helpful.
You should be offered 20 sessions. The first 10 should be weekly, with the next 10 scheduled
to suit you.

Specialist supportive clinical management (SSCM):


SSCM involves talking to a therapist who will help you understand what is causing your
eating disorder. You will learn about nutrition and how your eating habits cause your
symptoms.

You should be offered 20 or more weekly sessions. Your therapist will set you a target
weight and, over the course of the 20 weeks, will help you reach it.

Focal psychodynamic therapy:


Focal psychodynamic therapy is usually offered if you don't feel any of the above therapies
are right for you or if they don't work.

Focal psychodynamic therapy should include trying to understand how your eating habits
are related to what you think, and to how you feel about yourself and other people in your
life.

www.aspire2plab.com 174
No real PLAB2 cases discussed. All discussed scenarios are fictitious and for educational purposes only.
You will be offered weekly sessions for up to 40 weeks (9 to 10 months).

Once they are happy with your weight, as well as your physical and mental health, you
should be able to return home.

Compulsory treatment:
Occasionally, someone with anorexia may refuse treatment even though they're seriously ill
and their life is at risk.

In these cases, doctors may decide, as a last resort, to admit the person to hospital for
compulsory treatment under the Mental Health Act. This is sometimes known as
"sectioning" or "being sectioned".

Treatment for Anorexia Nervosa


The aim of treatment is to:
• Reduce risk of harm (and death) which can be caused by anorexia.
• Encourage weight gain and healthy eating.
• Reduce other related symptoms and problems.
• Help people become both physically and mentally stronger.
Management:
1. CBT
2. Diet Advice.
3. Family Therapy
4. Symptomatic treatment.

www.aspire2plab.com 175
No real PLAB2 cases discussed. All discussed scenarios are fictitious and for educational purposes only.
Concerned Daughter MMSE

You are an FY2 in GP. Mrs Mariah Smith, aged 55, came to the clinic with some concern.
Talk to her and address her concerns.

D: How can I help you? P: My daughter wanted me to see you.


D: May I know why? P: She said I keep forgetting things.
D: Do you think there have been any changes in you? P: No
D: Are you able to remember things? P: Yes, I remember most of the things.

D: Do you have to ask multiple times for information? P: No


D: Do you need notes to remember things? P: No
D: Do you have trouble following instructions? P: No
D: Do you find yourself confused mid-conversation? P: No
D: Do you lose things or feel like someone has stolen them? P: No
D: Have you noticed yourself becoming more irritable or anxious? P: No
D: Have you noticed difficulty walking and keeping balance? P: No (Vascular dementia)
D: Have you had any trouble with your vision? P: No (Vascular dementia)

D: Have you been diagnosed with any medical condition in the past? P: No
D: Any DM, HTN or stroke? P: No
D: Are you currently taking any medications, OTC drugs or supplements? P: No
D: Are you taking any birth control pills? P: No
D: Any allergies from any food or medication? P: No
D: Any previous surgeries or procedures done? P: No

D: Do you smoke? P: No
D: Do you drink alcohol? P: No
D: Tell me about your diet? P: Good/Bad
D: Are you physically active? P: Yes/No
D: What do you do for a living? P: Work from home.
D: Whom do you live with? P: With my daughter

I would like check your vitals, do GPE, MMSE and Neurological examination.
I would like to send for routine blood investigations.

Examiner: MMSE 26

From my assessment, the MMSE score is normal

I will do some routine and special blood tests. These include some laboratory tests such as
FBC, U&Es, LFT, calcium, vitamin B12, thyroid function tests and random or fasting blood
sugar, CT scan or MRI of the brain.
Routine Test – Normal

Note:
According to the NICE guidelines,

www.aspire2plab.com 176
No real PLAB2 cases discussed. All discussed scenarios are fictitious and for educational purposes only.
25-30 - Normal.
21-24 - Mild Cognitive Impairment
10-20 - Moderate Cognitive Impairment
< 10 - Severe Cognitive Impairment.

My management plan would include:


• Take complete medical history including social history from the patient.
• Perform necessary physical examinations.
• Order the routine blood tests.
• I would make a referral to the Psychiatric team once the patient is medically settled.
• There is a possibility of referral to Neuropsychiatry as well.
• They may involve the social services team and Homeless team if need be.
Full medical history including history of any medical illness, drug history and family history
should be taken.
Full examination looking for possible cardiac or neurological abnormalities should be
performed.
Some advanced cognitive assessment should also be done.
Some further investigations may be done in order to rule out physical causes.

Differential Diagnosis:
1. Neurodegenerative disorders for example Multiple sclerosis.
2. Other CNS disorders for example Brain tumours, Epilepsy and Trauma.
3. Infectious diseases such as HIV.
4. Metabolic disorders such as Hypercalcemia, Hyponatremia.
5. Endocrine disorders such as Addison disease, Cushing syndrome and thyroid problems.
6. Vitamin deficiencies such as vitamin B12, folate, thiamine, niacin deficiency.
7. Medications such as anabolic steroids, corticosteroids, cimetidine and some antibiotics
such as penicillin.
8. Substance abuse such as Amphetamines, Cocaine, Alcohol, Cannabis.
9. Related psychiatric disorders such as Schizophrenia, delirium, Mood disorders with
delusional symptoms (manic or depressive type), Obsessive-compulsive disorder.

www.aspire2plab.com 177
No real PLAB2 cases discussed. All discussed scenarios are fictitious and for educational purposes only.
Low Mood (Lesbian Miscarriage)

You are an FY2 in GP. Miss Joanna Mary, aged 30, has come to you with low mood. Talk to
her and address her concerns.

Core symptoms:

Are you feeling depressed or feeling low these days? (Low mood)
Do you still enjoy activities that you used to enjoy before? (Anhedonia)

SIGECAPS Mnemonic for Diagnosing Depression


Sleep changes:
Interest (loss of): Also known as “anhedonia.”
Guilt: blaming oneself
Energy: Usually a lack of energy.
Concentration: Loss of some cognitive abilities, like memory and focus.
Appetite: Can swing in either direction, with binge eating or anorexia.
Psychomotor changes: Again, either direction: a general slowing or anxious increase in
speed.
Suicidal ideation: A spectrum from self-harm to suicide.

D: How can I help you? P: I’m not feeling myself lately.


D: Tell me more about it. P: Like what doctor?
D: What do you mean by ‘not feeling myself’? P: I feel low.
D: Since when have you been feeling this way? P: For the past 2 weeks
D: Did something happen 2 weeks ago?
P: I had a miscarriage 2 weeks ago. (sometimes says partner had miscarriage)
D: I’m sorry to hear that.
D: Can you score your mood? P: 3 or 4
D: Do you feel sad, hopeless or irritable most of the time? P: Yes
D: Do you have a loss of interest in everyday activities? P: Yes
D: Do you have feelings of emptiness or worthlessness? P: I don’t know.
D: Do you have a disturbed sleep pattern? P: Yes
D: Do you have recurrent thoughts about dying? P: Yes/No
D: Do you feel suicidal/like hurting yourself? P: Yes/No
D: Are these feelings constant? P: Yes

D: Anything else? P: No
D: Do you have episodes of feeling very happy, elated or overjoyed? P: No
D: Do you sometimes feel full of great new ideas and important plans? P: No
D: Do you make decisions or say things that are out of character and that others see as
being risky or harmful? (Bipolar mania) P: No
D: Do you feel cold even in a warm environment? (Hypothyroidism) P: No
D: Do you have similar symptoms of feeling low near your periods? (PMS) P: No
D: Have you ever heard voices speaking when there is no-one around? P: No (Psychosis)
D: Have you been diagnosed with any medical condition in the past? P: No

www.aspire2plab.com 178
No real PLAB2 cases discussed. All discussed scenarios are fictitious and for educational purposes only.
D: Any history of depression and comorbid mental health or chronic physical disorder? P: No
D: Are you currently taking any medications, OTC drugs or supplements? P: No
D: Are you taking any birth control pills? P: No
D: Any allergies from any food or medication? P: No
D: Any previous surgeries or procedures done? P: No
D: Any family history of mental illness? P: No

D: Do you smoke? P: No
D: Do you drink alcohol? P: No
D: Do you do any recreational drugs? P: No
D: Tell me about your diet? P: Good/Bad
D: Are you physically active? P: Yes/No

D: Whom do you live with? P: With my partner


D: How is your relationship with your partner? P: It is good
D: How long have you been together? P: 2 years
D: Is she supportive? P: Yes
D: Do you have any other family members around? P: Yes
D: Are they supportive? P: Yes
D: How about friends? P: I have many supportive friends
D: Do you work? P: I’m on leave.
D: Are you financially stable? P: Yes

I would like to do GPE and vitals

From the history you have given me, it appears you may be suffering from depression.
Depression is more than simply feeling unhappy or fed up for a few days.

Most people go through periods of feeling down, but when you're depressed you feel
persistently sad for weeks or months, rather than just a few days.

Risk assessment and psychosocial history:

Most people with depression will get better without treatment. However, this may take
several months or even longer. Relationships, employment, etc, may be seriously affected.
There is also a danger that some people turn to alcohol or illegal drugs. Some people think
of suicide. Therefore, many people with depression opt for treatment.

Cognitive behavioural therapy (CBT):


Cognitive therapy is based on the idea that certain ways of thinking can trigger, or fuel,
certain mental health problems such as depression.

Antidepressant medicines:
Antidepressant medicines are commonly used to treat moderate or severe depression.
Antidepressant medication is not usually recommended for the initial treatment of mild
depression. However, an antidepressant may be advised With mild depression that persists

www.aspire2plab.com 179
No real PLAB2 cases discussed. All discussed scenarios are fictitious and for educational purposes only.
after other treatments have not helped, associated with a physical illness and the patient
had an episode of moderate or severe depression in the past.

Interpersonal therapy (IPT):


This is sometimes offered instead of CBT. IPT is based on the idea that your personal
relationships may play a large role in affecting your mood and mental state. For example,
IPT may focus on issues such as bereavement or disputes with others that may be
contributing to the depression.

Electroconvulsive therapy (ECT): may sometimes be recommended if the person has severe
depression and other treatments, including antidepressants, have not worked.

Don’ts

1. Don't bottle things up and 'go it alone'. Try to tell people who are close to you how you
feel. It is not weak to cry or admit that you are struggling.
2. Don't despair - most people with depression recover. It is important to remember this.
3. Don't drink too much alcohol. Drinking alcohol is tempting to some people with
depression, as the immediate effect may seem to relieve the symptoms. However,
drinking heavily is likely to make your situation worse in the long run. Also, it is very
difficult either to assess or to treat depression if you are drinking a lot of alcohol.
4. Don't make any major decisions whilst you are depressed. It may be tempting to give up
a job or move away to solve the problem. If at all possible, you should delay any major
decisions about relationships, jobs, or money until you are well again.

Do’s

1. Do try to distract yourself by doing other things. Try doing things that do not need much
concentration but can be distracting, such as watching TV. Radio or TV is useful late at
night if sleeping is a problem.
2. Do eat regularly, even if you do not feel like eating. Try to eat a healthy diet.
3. Do tell your doctor if you feel that you are getting worse, particularly if suicidal thoughts
are troubling you. See the separate leaflet called Dealing with Suicidal Thoughts.

Sometimes a spell of work is needed. However, too long off work might not be so good, as
dwelling on problems and brooding at home may make things worse. Getting back into the
hurly-burly of normal life may help the healing process when things are improving. Each
person is different and the ability to work will vary.

Sometimes a specific psychological problem can cause depression but some people are
reluctant to mention it. One example is sexual abuse as a child, leading to depression or
psychological difficulties as an adult. Tell your doctor if you feel something like this is the
root cause of your depression. Counselling may be available for such problems.

www.aspire2plab.com 180
No real PLAB2 cases discussed. All discussed scenarios are fictitious and for educational purposes only.
How does cognitive behavioural therapy work?

Cognitive behavioural therapy (CBT) can help you make sense of difficult problems by
breaking them down into smaller parts. In CBT, problems are broken down into five main
areas:
• Situations
• Thoughts
• Emotions
• Physical feelings
• Actions

CBT is based on the idea that these five areas affect each other. For example, your thoughts
about a certain situation can often affect how you feel physically and emotionally, as well as
how you act in response. CBT helps you to identify specific problems so that you can try to
solve them.

CBT is based on the concept that your thoughts, feelings, physical sensations and actions are
interconnected, and that negative thoughts and feelings can trap you in a vicious cycle.
CBT aims to help you deal with overwhelming problems in a more positive way by breaking
them down into smaller parts.
You're shown how to change these negative patterns to improve the way you feel.

SSRIs and SNRIs side effects:

• Agitation, sickness, indigestion


• loss of appetite
• dizziness
• a sedating effect.
• headaches
• low sex drive (difficulties achieving orgasm during sex or masturbation)
• (Erectile dysfunction)

These side effects should improve within a few weeks, although some can occasionally
persist.

Tricyclic antidepressants (TCAs) side effects:


• dry mouth
• slight blurring of vision
• constipation
• problems passing urine
• drowsiness
• dizziness
• weight gain
• excessive sweating (especially at night)
• heart rhythm problems, such as noticeable palpitations or a fast heartbeat (tachycardia)

www.aspire2plab.com 181
No real PLAB2 cases discussed. All discussed scenarios are fictitious and for educational purposes only.
The side effects should ease after a couple of weeks as your body begins to get used to the
medicine.

Depression
Doctors describe depression by how serious it is:

• mild depression – has some impact on your daily life


• moderate depression – has a significant impact on your daily life
• severe depression – makes it almost impossible to get through daily life; a few people
with severe depression may have psychotic symptoms

Mild depression
• Wait and see
• Exercise
• Talking through your feelings can be helpful. You can ask a GP any self-help group in
your area.
• Self-help books or online cognitive behavioural therapy (CBT)

Mild to moderate depression

• Cognitive behavioural therapy (CBT)


• Counselling for people who are generally healthy but need help coping with a current
crisis, such as anger relationship issues, bereavement redundancy, infertility.

Moderate to severe depression:


• Antidepressants
• Combination therapy: Antidepressants and CBT

www.aspire2plab.com 182
No real PLAB2 cases discussed. All discussed scenarios are fictitious and for educational purposes only.
Depression (CBT Failed)

You are an FY2 in GP. Mr Steven Douglas, aged 35, has been divorced from his wife and is
in depression. He saw the psychiatrist and he was given CBT treatment for 8 weeks. He
came to talk to you after 6 weeks. Speak to the patient, take focussed history and address
the patient's concern.

D: How can I help you? P: I’m not happy.


D: Tell me more about it. P: Like what doctor?
D: What do you mean by ‘not happy’? P: I feel low.
D: Since when have you been feeling this way? P: Since my divorce
D: I’m sorry to hear that.
D: Can you score your mood? P: 3 or 4
D: Do you feel sad, hopeless or irritable most of the time? P: Yes
D: Do you have a loss of interest in everyday activities? P: Yes
D: Do you have feelings of emptiness or worthlessness? P: I don’t know.
D: Do you have a disturbed sleep pattern? P: Yes
D: Do you have recurrent thoughts about dying? P: Yes/No
D: Do you feel suicidal/like hurting yourself? P: Yes/No
D: Are these feelings constant? P: Yes

D: Anything else? P: No
D: Do you have episodes of feeling very happy, elated or overjoyed? P: No
D: Do you sometimes feel full of great new ideas and important plans? P: No
D: Do you make decisions or say things that are out of character and that others see as
being risky or harmful? (Bipolar mania) P: No
D: Do you feel cold even in a warm environment? (Hypothyroidism) P: No
D: Have you ever heard voices speaking when there is no-one around? P: No (Psychosis)
Please ask about PMH, Lifestyle and Psychosocial history.
D: Whom do you live with? P: Alone
D: How is your relationship with your partner? P: Divorced
D: Do you have any other family members around? P: Yes
D: Are they supportive? P: Yes
D: How about friends? P: I have many supportive friends
D: Do you work? P: I’m on leave.
D: Are you financially stable? P: Yes

Concern: Will I ever feel better again?

Depression (CBT failed 2)

You are FY2 in GP Surgery. Maria Douglas, aged 35, has been on CBT treatment for her
depression for 8 weeks. She has come in at 6 weeks and does not want to continue with it.
Please take history, assess the patient and discuss about your plan of management with
the patient.

www.aspire2plab.com 183
No real PLAB2 cases discussed. All discussed scenarios are fictitious and for educational purposes only.
Depression (Weight Loss)

You are an F2 in GP. Mrs Diana Whales, aged 30 came to the clinic because of concern of
weight loss. Please talk to her and discuss your plan of management with her and address
her concerns.

D: How can I help you? P: I am losing weight


D: Tell me more about it. P: Like what doctor?
D: Since when have you been losing weight? P: For the past 1 year
D: Is it intentional or unintentional? P: It is unintentional
D: Have other people noticed it? P: Yes, people around me telling about it
D: How much weight have you lost? P: I have lost half a stone.
D: Anything else? P: Like what?

D: Do you have any lumps or bumps? P: No


D: Are you eating well? P: Yes
D: Do you feel tired? P: Yes
D: Do you feel a swelling in your neck? P: No
D: Do you have an irregular and/or unusually fast heart rate? P: No
D: Do you feel hot when others around you feel cold? P: No
D: Any fever or flu like symptoms? P: No
D: Any diarrhoea or vomiting? P: No

D: Can you score your mood? P: 3 or 4


D: Since when has your mood been this way? P: For the past 1 year
D: Did something happen 1 year ago? P: Husband left one year ago
D: Do you have a loss of interest in everyday activities? P: Yes/No
D: Do you have feelings of emptiness or worthlessness? P: Yes/No
D: Do you have a disturbed sleep pattern? P: Yes/No
D: Do you have recurrent thoughts about dying? P: Yes/No
D: Do you feel suicidal/like hurting yourself? P: Yes/No
D: Are these feelings constant? P: Yes/No

Please ask about PMH, Lifestyle and Psychosocial history.

D: Do you have any other family members around? P: Yes/No


D: Are they supportive? P: Yes
D: How about friends? P: Yes
D: Do you work? P: No
D: Is there a reason? P: I have to take care of my child at home.
D: Are you financially stable? P: No, I am on benefits at the moment.

www.aspire2plab.com 184
No real PLAB2 cases discussed. All discussed scenarios are fictitious and for educational purposes only.
Post-Partum Depression (Telephonic Consultation)

You are an FY2 in GP. Miss Amanda Lowe, aged 31, came to you with a complaint of
insomnia. Talk to her and address her concerns.

D: How can I help you? P: I have trouble sleeping.


D: Please tell me more about it? P: What do you want to know?
D: When did this problem start? P: It started 5 months ago.
D: Do you have trouble getting into sleep or do you wake up in the middle of the night?
P: I have trouble staying asleep.

D: What time do you go to bed? P: I go to bed around 10.


D: What time do you usually go to sleep? P: I go to sleep around 10:30
D: What time do you usually wake up? P: I wake up around 7.
D: Do you wake up in between? P: Yes
D: How often? P: At Least 3 to 4 times
D: Are you able to fall asleep afterwards? P: Yes/No
D: How was your sleep before this problem started? P: It was fine.
D: Do you take any naps during the day? P: Yes/No (Elaborate)
D: Anything else? P: I also feel tired
D: Did this start at the same time? P: Yes
D: Can you think of anything which might be the cause of your problem?
P: I gave birth 5 months ago

D: Tell me, what do you do before you go to bed? P: I finish my chores


D: How do you spend your time every day? P: I look after my baby

D: Have you been diagnosed with any medical condition in the past? P: No
D: Are you currently taking any medications, OTC drugs or supplements? P: No
D: Are you taking any birth control pills? P: No
D: Any allergies from any food or medication? P: No
D: Any previous surgeries or procedures done? P: No

D: How is your mood? P: It is low.


D: Could you please score the mood on a scale of 1 to 10, 1 is low and 10 is highest.
P: It is 2/3
D: Have you had any thoughts of harming yourself? P: No
D: Any thoughts of harming your baby? P: No
D: Do you sometimes feel like you can hear voices? P: No
D: Have you had difficulty bonding with your baby? P: Yes/No
D: Do you feel sad, hopeless or irritable most of the time? P: No
D: Do you have a loss of interest in everyday activities? P: No
D: Do you have feelings of emptiness or worthlessness? P: No

D: Do you smoke? P: No
D: Do you drink alcohol? P: No
D: Tell me about your diet? P: Good/Bad

www.aspire2plab.com 185
No real PLAB2 cases discussed. All discussed scenarios are fictitious and for educational purposes only.
D: Are you physically active? P: Yes/ No
D: What do you do for a living? P: I stay at home
D: Whom do you live with? P: With my husband
D: How is your relationship with him? P: Good, he is supportive
D: Do you have any relatives close by? P: Yes, my mother
D: Have you talked to her about how you feel?
P: Yes. My mom thinks that it’s normal to be low.
D: How about any friends? P: I don’t have any.

I would like to do vitals, GP examination. I would also like to do baseline investigations.

From the history you have given me, it seems that you may be having Postnatal depression.
It is a type of depression that many parents experience after having a baby.

Postnatal depression can be lonely, distressing and frightening, but support and effective
treatments are available.

These include:
● self-help – things you can try yourself include talking to your family and friends
about your feelings and what they can do to help, making time for yourself to do
things you enjoy, resting whenever you get the chance, getting as much sleep as you
can at night, exercising regularly, and eating a healthy diet
● psychological therapy – a GP may be able to recommend a self-help course or may
refer you for a course of therapy, such as cognitive behavioural therapy (CBT)

Cognitive behavioural therapy (CBT) is a type of therapy based on the idea that unhelpful
and unrealistic thinking leads to negative behaviour.

CBT aims to break this cycle and find new ways of thinking that can help you behave in a
more positive way.

For example, some women have unrealistic expectations about what being a mum is like
and feel they should never make mistakes.

As part of CBT, you'll be encouraged to see that these thoughts are unhelpful and discuss
ways to think more positively.
● antidepressants – these may be recommended if your depression is more severe or
other treatments have not helped; your doctor can prescribe a medicine that's safe
to take while breastfeeding

Local and national organisations, such as the Association for Post Natal Illness (APNI) and
Pre and Postnatal Depression Advice and Support (PANDAS), can also be useful sources of
help and advice.

www.aspire2plab.com 186
No real PLAB2 cases discussed. All discussed scenarios are fictitious and for educational purposes only.
Baby Blues, Post Natal Depression, Post Natal Pyschosis

Many people who have given birth will experience mild mood changes after having a
baby, known as the "baby blues". This is normal and usually only lasts for a few days.
But postpartum psychosis is very different from the "baby blues". It's a serious mental
illness and should be treated as a medical emergency.

Symptoms usually start suddenly within 1st 2 weeks after giving birth in post partum
psychosis.

Post natal depression can start anytime in the first year after giving birth.

Dx Post Natal Depression

Symptoms 1. a persistent feeling of sadness and low mood


2. lack of enjoyment and loss of interest in the wider world
3. lack of energy and feeling tired all the time
4. trouble sleeping at night and feeling sleepy during the day
5. difficulty bonding with your baby
6. withdrawing from contact with other people
7. problems concentrating and making decisions
8. frightening thoughts – for example, about hurting your baby
Causes • a history of mental health problems, particularly depression, earlier in life
• a history of mental health problems during pregnancy
• having no close family or friends to support you
• a difficult relationship with your partner
• recent stressful life events, such as a bereavement
• physical or psychological trauma, such as domestic violence
• having the "baby blues"
Mx - Self help (Exercise regularly, Eat healthy diet)
- CBT
- Antidepressants

Dx Post Natal Psychosis

Symptoms 1. hallucinations - hearing, seeing, smelling or feeling things that are not there
2. delusions – thoughts or beliefs that are unlikely to be true
3. a manic mood – talking and thinking too much or too quickly, feeling "high" or "on top of the world"
4. a low mood – showing signs of depression, being withdrawn or tearful, lacking energy, having a loss of appetite,
anxiety, agitation or trouble sleeping
5. sometimes a mixture of both a manic mood and a low mood - or rapidly changing moods
6. loss of inhibitions
7. feeling suspicious or fearful
8. restlessness
9. feeling very confused
10. behaving in a way that's out of character
Causes • already have a diagnosis of bipolar disorder or schizophrenia
• have a family history of mental health illness, particularly postpartum psychosis (even if you have no history of
mental illness)
• developed postpartum psychosis after a previous pregnancy

Mx Admit in specialist paediatric unit called as Mother and Baby Unit (MBU)

Medication:
Antidepressants
Antipsychotics
Mood stabilisers (for example, lithium) Psychological therapy

CBT
ECT

www.aspire2plab.com 187
No real PLAB2 cases discussed. All discussed scenarios are fictitious and for educational purposes only.
Colleagues Consultation

S = Seek information
P = Patient safety
I = Initiate
E = Escalate
S = Support

Patients Safety (Review)


Escalate to Senior
Educational Supervisor
Arrange Transport

www.aspire2plab.com 188
No real PLAB2 cases discussed. All discussed scenarios are fictitious and for educational purposes only.
Alcohol Colleague

You are an FY2 and have started your Surgery Rotation. Your colleague Mr Peter Roocroft
aged 23 is F2. He has also started his surgery rotation. You saw him on the ward round last
week and he smelt of alcohol. You have noticed it again today this morning that he smells
of alcohol. A few other people have also noticed this. Talk to your colleague about his
problem and assess his situation. You have asked your colleague to meet with you. You have
arranged a meeting in a quiet room with your colleague after you have finished your work.

D: Hello Peter. Thank you for meeting with me on such short notice. P: That's okay
D: How's your day going? P: Great. Thank you
D: Lovely. Peter, do you know why am I here for? P: No
D: No worries. So how has your life been in general?
P: Actually I broke up with my girlfriend recently. I'm drinking to help drown my feelings.
D: I’m very sorry that you are going through such a difficult period. If you don’t mind, could
you please tell me how much you are drinking these days?
P: One bottle of wine and/or a few pints of beer.
D: At what time of the day do you usually drink? P: I drink at night only.
D: Peter, I am here to help you, please be honest with me, it is just that I could smell alcohol
when I walked past you this morning. Have you been drinking in the morning before coming
to work? P: Last night I had some beer left so I drank it in the morning.

D: Thank you for having trust in me. Let’s talk about how things were in the past. I mean
how much did you use to drink before? P: Occasionally

D: So as I understand things so far, this drinking habit has been a recent event and not a
chronic problem. However, Peter, would you agree with me that working under the
influence of alcohol would hinder your decision making skills, especially as a doctor, when
patient’s lives are at stake? (INSIGHT)
P: Yes. I'm sorry I was just very stressed because of the break up.

D: I completely understand what you are going through. However, have you been seeking
any other substances to help yourself? (DRUGS) P: No
D: With whom do you live? P: I live alone.
D: Any close relatives live nearby? (FAMILY) P: No. My parents live a few hours away.

D: How would you describe your relationship with them?


P: It's fine. I plan to meet them next week.

D: How about your friends? (FRIENDS)


P: I have many.

D: That’s great Peter, I am sure this is just a phase and it will pass by with the help of your
friends and family.
I am going to ask a question, it may sound silly however, have you thought of harming
yourself? (SUICIDE)
P: Absolutely not.

www.aspire2plab.com 189
No real PLAB2 cases discussed. All discussed scenarios are fictitious and for educational purposes only.
D: Good, are you aware of any incident that has happened so far as a result of your
drinking?
P: No.

D: Have you ordered any investigations, prescribed medication or discharged any patient
today?
P: I did prescribe some medication to a few patients.

D: Okay. Nothing personal however, in the interest of patient safety one of our team
members will need to review those patients. Could you please provide me with a list of the
activities you have undertaken today?
P: Okay.

D: Peter, I'm afraid, we need to inform one of the seniors about this situation. Although I
can appreciate you are going through a hard time, it's important to remember that we value
patient safety more than anything else and that's why we must handle this in a professional
way.

P: What will happen next?


D: You may be required to have a test and depending on your test result, you may have an
interview with one of the senior managers to discuss the matter further.
P: Okay

D: I'm sorry that you've had to go through such a stressful period. It would be great to seek
help from your GP for confidential advice. You can also consult with your educational
supervisor for some help. Our seniors will also help you overcome this difficult period. At
the same time, if you need somebody to talk to I would be glad to hear you out.
P: Thank you.

D: For the time being, it's best not to remain in the clinical area or drive for a while. May I
ask how you plan to get home?
P: I will take a taxi.

D: Thank you Peter for your time, and as I said before, it’s a phase and it will certainly pass,
take it easy and I’ll see you around.

P: Thank you.

www.aspire2plab.com 190
No real PLAB2 cases discussed. All discussed scenarios are fictitious and for educational purposes only.
Cocaine colleague

You are FY2 in General Medicine. Mr John Bond, 25 years old, is a final year medical student.
You saw him at a birthday party last night. He was drinking heavily and taking cocaine. The
following morning you overheard the nurse colleagues talking about his behaviour. During
the ward round they noticed that he was agitated and hyper-excited. Please talk to your
colleague, assess his condition and manage the situation appropriately.

D: Hello John. Thank you for meeting with me on such short notice.
P: That's okay

D: How's your day going?


P: Great. Thank you

D: Lovely. John, do you know what I am going to talk to you about?


P: No

D: Never mind, so I saw you at last night’s party?


P: Oh, you saw me there?

D: I did and it is just that I am a bit concerned as you were probably drinking a bit too much.
P: It was just a party.

D: Fair Enough. However, by any chance did you take anything else besides alcohol last
night?
P: No

D: How about any recreational drugs?


P: No. Why do you think so?

D: It is just that this morning our colleagues noticed that you were a bit agitated and hyper
excited.
P: No I wasn’t. I was perfectly normal. What do you mean?

D: Actually, last night at the party I saw you taking something.


P: Like what?

D: I may be wrong however by the looks of it I think what you were taking looked like
cocaine. Is that true?

P: Is this conversation confidential?


D: Yes, Absolutely it will remain within the system. However, I do have to inform at least one
of the seniors in order to make the best decision for you and the care of our patients.

P: Okay. I took cocaine.


D: I appreciate your honesty. May I ask how long you've been doing cocaine for?

www.aspire2plab.com 191
No real PLAB2 cases discussed. All discussed scenarios are fictitious and for educational purposes only.
P: Not long. Just a few months.

D: How often do you take it?


P: Only on my night out with friends.

D: Is there any particular reason why you take it?


P: Just recreational use. Everyone does it. Even you've probably tried it at some point.

D: Well not really. Anyway, let’s get back to our discussion. So how much do you usually
take?
P: Not much, just a line or two.

D: Have you ever taken it during work hours?


P: Never. I make sure it doesn't affect my work.

D: Well John as I said, this morning our colleagues noticed that you were a bit agitated and
over excited. Don’t you think that it has affected your workplace already? (INSIGHT)
P: But I take it in my personal time.

D: Agreed, however I believe it may affect your decision making skills, and soon you may get
addicted to it as well. Thus, have you ever thought of cutting down or stopping completely?
(CUT DOWN)
P: I don't do it that often.

D: Do you feel guilty about it? (GUILTY)


P: No.

D: How's your mood recently? (MOOD)


P: It's fine.

D: Apart from cocaine, are you using any other recreational drugs? (DRUGS)
P: No

D: Could I ask briefly about your drinking? How often do you drink? How much do you
drink? (ALCOHOL)
P: I drink 1-2 glasses of wine every day. I drink with my friends after work. But it doesn’t
affect my work. I don’t take it in the workplace.

D: May I ask who you live with? (FAMILY)


P: I live with my parents.

D: How's your relationship with them?


P: It's fine.

D: How about friends? (FRIENDS)


P: I have many.

www.aspire2plab.com 192
No real PLAB2 cases discussed. All discussed scenarios are fictitious and for educational purposes only.
D: Thank you for having the conversation with me John. I'm sure you are aware that cocaine
is a very strong stimulant. It results in overconfidence that can lead to you taking risks. This
compromises patient safety which is our most important value. It will also affect your
behaviour and performance. Such behaviour is not consistent with the values of good
medical practice. This can have a very negative effect on your image as a doctor.
Aside from this, cocaine can cause some severe medical complications to you. It can lead to
heart attacks, seizures, strokes, coma and many other unexpected effects. I do not want this
for you and that’s why I’m here to help and support you in this matter.
P: I understand.

Are you aware of any incident that has happened so far as a result of taking cocaine?
P: No.

D: Have you ordered any investigations, prescribed medication or discharged any patient
today?
P: I did prescribe some medication to a few patients.

D: Okay. I hope you don't take this personally but in the interest of patient safety I need to
review those patients myself. Could you please provide me with a list of the activities you
have undertaken today?
P: Okay.

D: John, As I mentioned, I'm afraid I have to inform one of the seniors about this situation.
It's important to remember that we value patient safety more than anything else and that's
why we must handle this in a professional way.

P: What will happen next?


D: You may be required to have a test and depending on your test result, you may have an
interview with one of the senior managers to discuss the matter further.
P: Okay

D: This decision is what is best for not only our patients, but also for you. We are here to
support you. It would be great to seek help from your GP for confidential advice. You can
also consult with your educational supervisor for some help. Our seniors will also help you
overcome this difficult period.
P: Thank you.

D: For the time being, it's best not to remain in the clinical area or drive for a while. May I
ask how you plan to get home?
P: I will take a taxi.

www.aspire2plab.com 193
No real PLAB2 cases discussed. All discussed scenarios are fictitious and for educational purposes only.
Colleague Coming Late

You are FY2 in Medicine. Mr Darren Blake, a final year student who is a part of your medical
team, is coming late to the ward. Your colleague has also noticed this. Please talk to him.

D: Hello Darren. Thank you for meeting with me on such short notice.
P: That's okay

D: How's your day going?


P: Great. Thank you

D: How is your hospital posting going?


P: it is fine.

D: Are you enjoying talking to the patient in the hospital?


P: Yes.

D: Glad to hear that. Darren, do you have any idea what I'm going to talk to you about?
P: No

D: Actually I'm a little bit concerned about your time-keeping. I noticed you came late to the
ward today. This is not the first time I've noticed.
P: (Stays quiet)
(Pause briefly and allow Darren some time to open up. If he remains silent, encourage him
to speak about it)

D: There's no need to worry at the moment. Is there any particular reason why you are
coming late?
P: Actually I was stuck in traffic this morning

D: OK. How about the other days?


P: I'm usually on time. I was only late this morning.

D: Actually my colleague has also noticed that you are coming late quite often and brought
it to my attention.
P: Yes, I think I've been late a few times this week.

D: Thank you for your honesty. Is everything okay at home?


P: Yes. It won't happen again.

D: Is there anything bothering you?


P: No

NOTE - he doesn't have any personal problem which can be the reason. Rather, he
mentioned that he came late only today because he was stuck in traffic and usually he is on
time.

www.aspire2plab.com 194
No real PLAB2 cases discussed. All discussed scenarios are fictitious and for educational purposes only.
D: May I ask how far away you live from here?
P: a) It takes me one hour to reach by train and bus.
b) I live very far away. It takes me 2 hours to reach by train.

(In case of b) NOTE - an employee who is unable to get to their normal place of work should
discuss the possibility of working from an alternative place of work with their supervisor.
D: If you are finding it difficult to get to your normal place of work you should discuss the
possibility of working from an alternative place of work with your supervisor.

D: OK. I'm sure you know that it is very important for doctors to be punctual and maintain
good time-keeping skills. By coming late, we may not be present at a crucial time when a
patient needs us and this can compromise our care to patients.
P: I understand.

D: As a medical student it's very important that you demonstrate to all staff that you are
capable of coming on time and hence, we can rely on you. You will also find that you will be
able to learn a lot more if you come on time because the morning ward rounds offer a great
opportunity to learn.
P: OK. I'm sorry for coming late. I will not repeat it.

D: That's fine. It would be a good idea to attend a time-management training course. It will
enable you to manage priorities and schedule tasks more effectively. Anyone at any level,
including myself, will benefit from the strategies and techniques discussed in these courses.
You can find out more from your educational supervisor.
P: OK.

D: Do you keep alarms?


P: No.

D: I suggest you do that. It will help you in waking up early.


And also in the future, if you anticipate any travel problems or if you are unable to reach
your place of work, you must telephone your educational supervisor as soon as possible to
explain the circumstances. Does that sound okay?
P: Yes.

D: I think it would be better if you inform your supervisor because if he will come to know
from somewhere else then it won’t be good.

D: Regarding this matter, do you have any concerns or questions that I may be able to help
with?
P: No.

D: OK. Feel free to talk to me if I can help in any way. Have a good day.

www.aspire2plab.com 195
No real PLAB2 cases discussed. All discussed scenarios are fictitious and for educational purposes only.
Colleague Confidentiality Issue (Facebook Post)

You are FY2 working in A&E. Your colleague Peter Wilson, an FY1 doctor made a post on
Facebook about an elderly lady in the emergency department with confusion who
considered herself to be the queen of England. Talk to him and manage the situation.

D: Hello Peter. Thank you for meeting with me on such a short notice.
P: That’s okay.

D: How’s your day going so far?


P: Great. Thank you.

D: Peter do you know what I am going to talk about?


P: No

D: Did you recently make a post on social media regarding one of our patients?
P: Oh yes. You know I could not stop laughing while seeing the patient. Her name was Diana.
She was confused and she was thinking of herself as the Queen of England. Hilarious. She was
making funny comments like she lost her crown and she was looking for it. So, I made a video
of her and posted it.

D: Ok. Where did you put in on Social Media?


P: I posted it on my Facebook profile.

D: Did you post her name and other details as well on Facebook?
P: No, I just posted the video.

D: Peter I don’t think you did the right thing. Posting patient information on social media is a
breach of confidentiality. Being a medical professional, we must obey the rules and
regulations of the NHS. Before putting patient information online, think about why you are
doing it. You should definitely take the consent of your patient if you want to post something.
Don’t you think so?

P: Yes. But I didn’t mean to break the rules. It was just for fun
D: I do understand what you are trying to say. Many people are unaware of how easily this
information can spread on Facebook. Even if using the most stringent privacy setting,
information on social networking sites may still be widely available in search engines. Deleting
information is not sure-fire protection. It is stored in cyberspace and theoretically
permanently accessible.

P: To be honest I didn’t think that much before posting it.


D: Yes, I know. You wouldn’t have done it if you had thought about the consequence.
Breaching confidentiality can result in complaint to GMC and legal actions. Moreover, it can
erode public trust in the medical professionals, and it can hinder us from getting the
information from the patients to treat them better.

www.aspire2plab.com 196
No real PLAB2 cases discussed. All discussed scenarios are fictitious and for educational purposes only.
P: Yes, you are right. I am so sorry for what I have done. What should I do now?
D: I highly encourage you to delete the post immediately. I sincerely advise you not to do it
in future. How’s she doing now?

P: She is much better and fully conscious.


D: I am really happy to hear that. I would request you to talk to her and apologise to her for
the incident.

P: Ok. I will talk to her.


D: And It is very important to inform our seniors, especially our consultant about the incident.
It will be bad if he gets to know about it from others. To be honest, he is the best person to
help us if we are in trouble. If you want, I can be there with you while you talk with our
consultant and we can both explain to him a better way.

P: Thank you
!

www.aspire2plab.com 197
No real PLAB2 cases discussed. All discussed scenarios are fictitious and for educational purposes only.
Doctor & Patient Relationship (Nurse Concerned)

You are FY2 in Medicine ward. Miss Sandra Miles, your nurse colleague, wants to talk about
an incident related to a fellow doctor. Talk to her and address her concerns.

D: How can I help?


N: I’m concerned about a fellow doctor who works in the same department.

D: What is it that is concerning?


N: I am a friend on Facebook with the doctor, and I noticed that he has been adding patients
and tagging them on a post.

D: Thank you for bringing this up. What is the post about? N:
D: When did he post that? N:
D: Is the post still there? N: Yes/No
D: What is the post about? N:
D: How many people have seen the post? N:

D: How many patients were tagged in that post?


N: Multiple patients were added as friends and tagged.

D: Have you spoken to him about it?


N: No, I am not comfortable. I was wondering what can you do about it?

D: How has this affected you? N:


D: Have you spoken to the senior about it?
N: No, I didn’t want to because others will see me as weird and strange.

D: Would you like me to talk to him? N: Yes


D: Do you want to come with me to talk to him? N: Yes/No

We can involve the line manager or supervisor if required.


We can offer help to the doctor.
We can arrange training for social media use and confidentiality.
Training à Information governance

We must discourage any kind of post on social media.


Social media can blur the boundaries between doctors personal and professional life. It can
change the relationship between a doctor and patient.
!

www.aspire2plab.com 198
No real PLAB2 cases discussed. All discussed scenarios are fictitious and for educational purposes only.
Examination & Mannequin
Medicine
Diabetic Foot

You are FY2 working in diabetic clinic. Mr John Davies, aged 45, presents to the clinic for
his diabetic annual review. Patient was diagnosed with diabetes 5 years ago. He is not on
any medication. His condition has been controlled on diet. Please talk to the patient,
assess the patient, and discuss management with the patient.

D: I understand you have diabetes, and you came for your annual review. P: Yes
D: Since when you have diabetes? P: From the last 5 years.
D: Is it well controlled? P: Yes, it is well controlled.
D: How do you manage it? P: I am managing it with my diet.
D: Do you check your blood sugar regularly? P: Yes, I check it regularly.
D: When was the last time you checked it? P: I checked it yesterday.
D: How much was it? P: It was 6.
D: Was it before or after the meal? P: It was before the meal.
D: Are you going for your regular check-ups? P: Yes, I am going.
D: Do you have any complications of diabetes? P: No
D: Any heart, kidney, eye, foot problems? P: No

Ask about PMH, Lifestyle and Psychosocial history.

Examination:
I would like to do GPE, vitals including your blood pressure. I would like to check your eyes,
BMI and do foot examination.

www.aspire2plab.com 199
No real PLAB2 cases discussed. All discussed scenarios are fictitious and for educational purposes only.
Examination:
BMI - 30, BP-120/70
I will do a foot examination.

From our assessment, your diabetes is not well controlled as you have loss of sensation
below ankle/mid-shin, knee bilaterally.

I would like to do routine blood tests including your kidney function test, cholesterol levels,
glucose reading and your blood sugar reading in last 3 months (HbA1C). We will also do a
urine test.

We will also check the back of your eye. (Diabetic retinopathy)


We may consider giving you some medications.

Advise him about Lifestyle.


Advise him about foot care.
Follow up with your GP and Diabetic Clinic.

If you develop any eye symptoms or further problems with your foot, come back to us.

Concerns:
Am I going to lose my leg?

http://www.diabetes.co.uk/nhs/diabetes-annual-care-review.html

www.aspire2plab.com 200
No real PLAB2 cases discussed. All discussed scenarios are fictitious and for educational purposes only.
Alcoholic Foot

You are FY2 in Medicine. Mrs Juliet Davies, aged 50, presented to the hospital with a
burning sensation of both feet. She went to an alcohol cessation clinic 6 months ago.
Please talk to the patient, assess the patient, and discuss management with the patient.

D: What brought you to the hospital?


P: I have some burning sensation in my both legs for the last few months.
D: Any other symptom? P: No
D: Any Pain or weakness in your legs? (Sensory) P: No
D: Any tingling or numbness? P: No
D: Any Muscle spasm or cramps? P: No
D: Any wasting of muscles? P: No
D: Any balance problem or coordination? (Motor) P: No
D: Any problem with bowel or urine? (Autonomic) P: No

Ask about PMH, Lifestyle and Psychosocial history.


D: Have you been diagnosed with any medical condition in the past? P: No
D: Any DM, HTN Thyroid and kidney problems? P: No

Examination:
I would like to do GPE, check your vitals and examine your foot.

From our assessment, you have peripheral neuropathy because you have loss of sensation
below ankle/mid-shin, knee bilaterally.

There can be many reasons for it like DM, VitB12 deficiency or alcohol consumption. In your
case, it looks like you have alcoholic neuropathy.

I would like to send for some initial investigations including routine blood test to see kidney,
thyroid function, and VitB12 level.I would like to send a urine test and blood sugar level. We
may consider sending some nerve and muscle tests to see the functioning of your nerves
and muscles. (Nerve conduction tests or electro myelography).

We may need to take the samples from your nerves. (nerve biopsy) The cause of your
problem is alcohol consumption. To stop the progression of the disease you should stop
drinking.

Have a well-balanced diet, we will refer you to a dietician. We will prescribe you some
vitamins and minerals. We can give you some patches and creams for your problem.

Advise him about foot care.


Follow up with your GP.
If you develop foot ulcers, blood circulation problems, come back to us.

https://www.nhs.uk/conditions/peripheral-neuropathy/

www.aspire2plab.com 201
No real PLAB2 cases discussed. All discussed scenarios are fictitious and for educational purposes only.
Cerebellar Ataxia

You are F2 in the General medical ward. Mrs Janet Clarke aged, 65 has been referred by
her GP because of suspicion of Cerebellar ataxia. Patient is diagnosed with Diabetes. Her
diabetes is well controlled. Please talk to the patient, assess the patient, and discuss
management with the patient.

D: What brought you to the hospital? P: I feel clumsy these days.


D: What do you mean by clumsy?
P: I live to knit for my grandchildren, but because of this problem I cannot knit.
D: Since when? P: From the last 3 months.
D: Anything else? P: Whenever I walk, it feels like I will fall but I don’t fall.
D: Since when? P: From the last 3 months.
D: Anything else? P: Whenever I hold something in my hand, it falls from my hand.
D: Since when? P: From the last 3 months.

D: How is your general health? P: I am fine.


D: Any cough? P: No
D: Any fever or flu like symptoms? P: No

D: Any problem with urine or bowel? P: No


D: Any weight loss? P: No
D: Any loss of appetite? (Brain tumour) P: No
D: Any headache or vomiting? (SOL) P: No
D: Any eye problem? P: No
D: Any head trauma? P: No

Ask about PMH, Lifestyle and Psychosocial history.


Examination:
I would like to do GPE, check your vitals and do cerebellar examination.

Your GP was suspecting cerebellar ataxia that is why he referred you to us. Based on the
examination that I have done, everything seems to be normal, however, we will do some
investigations to make sure there is no problem.

We will do some urine test and blood test to see if there is any bug which is the cause of
your problem. We will do blood test to check the level of Vit B12 and also we will do some
genetic testing.

We will consider doing some scans like a CT scan or MRI of your head to make sure
everything is fine.

Concern:
P: Is it cancer?

www.aspire2plab.com 202
No real PLAB2 cases discussed. All discussed scenarios are fictitious and for educational purposes only.
Meningitis

You are FY2 in A&E. Mr David Mills, aged 27, has presented with headaches. Please talk to
the patient, assess the patient, and discuss management.

Clinical Features:
1. Photophobia
2. Headache
3. Neck stiffness (crouch position)
4. Brudzinski and Kernig sign
5. Fever
6. Rash

Examination:
I would like to do GPE, check your vitals and perform meningeal examination.
BP: 120/80, Pulse: 80, Temp: 38, RR: 12-20, Sats: >96%

Management:
I am suspecting my patient has Meningitis.
1. Admit
2. Analgesia
3. Inform a senior
4. Isolate the patient.
5. Investigations: Bloods Routine, consider CT rule out increased ICP.
6. LP (CSF analysis) to confirm diagnosis and know the cause.
7. Antibiotics (discuss with my seniors / follow hospital protocols):
a. Hospital: Ceftriaxone
b. GP: Benzyl-Penicillin
8. IV fluids
9. O2
10. Antipyretic (PCM)
11. Prophylaxis / Trace (I would discuss with the microbiology team; I would trace close
contacts of patients and give them prophylactic antibiotics usually Ciprofloxacin).

www.aspire2plab.com 203
No real PLAB2 cases discussed. All discussed scenarios are fictitious and for educational purposes only.
Cranial Nerve Examination

You are an FY2 in neurology ward. A 55-year-old male patient was admitted for TIA & has
been treated. Assess the cranial nerve function of the patient.

Peripheral Field of Vision

You are FY2 in GP. Mr. Andy, aged 43, has presented to the GP with visual problems. This
is the patient’s first visit. You don’t know about the patient’s past medical history. Please
talk to the patient, assess the patient, and discuss management.

D: What brought you to the hospital?


P: I am fine, my wife thinks I have got some problem with my vision.
D: Why does your wife think like this? P: Because I scratched my car.
D: Which side? P: Both sides of my car.
D: When did it happen? P: It happened yesterday.
D: How did it happen? P: I was parking my car and I scratched it.

D: Has it happened before? P: Yes/No

D: Do you usually bump into people or things? P: No


D: Any car accidents? P: No
D: Any headache? P: No
D: Any blurry or double vision? P: No
D: Do you see any colour halos? P: No
D: Any nausea or vomiting? P: No

Ask about PMH, Lifestyle and Psychosocial history.


I would like to do GPE, check your vitals and do eye examination.

We will refer you to a specialist for further examination like fundoscopy and investigations
to find out the cause. They may consider doing some scans like CT or MRI.
Please do not drive and inform DVLA.
D: How did you come to the hospital? P:

Concerns:
How long will it take to get better?
Is it serious?

www.aspire2plab.com 204
No real PLAB2 cases discussed. All discussed scenarios are fictitious and for educational purposes only.
Fundoscopy (Giant Cell Arteritis)

You are FY2 in A&E. Maria, aged 66. presents to the hospital complaining of loss of vision
of one eye. Please talk to the patient, assess the patient, and discuss management.

D: What brought you to the hospital? P: I cannot see from left eye.
D: Tell me more about it.
D: Since when?
D: Was it sudden or gradual?
D: Is it complete or partial loss?
D: Is it painful or painless?
D: What about your other eye?
D: Anything else?
D: Any Headache? P: Yes
D: Jaw pain?
D: Blurry vision?
D: Fever flu like symptoms?
D: Nausea, vomiting?

Ask about PMH, Lifestyle and Psychosocial history.


I would like to check your vitals and examine the back of your eye.

From our assessment, we are suspecting you have a condition called giant cell arteritis.
It is a condition that causes swelling on the inside of some blood vessels that is why you are
having pain.
You lost your vision because of inflammation and blockage of the blood vessels that supply
the main nerve in your eye.
We will do some investigations but before that we will start you on some medications.

We are going to keep you in the hospital and give you a high dose of a medication called
steroids through your blood vessels.
We will give you painkillers and anti-sickness medications.

We will do some blood tests to check the level of inflammation in your body (CRP/ESR).
We will refer you to the specialist in the next couple of days; they will do some
investigations like temporal biopsy.

After a few days, we will shift you on to the steroid tablets. Initially high dose steroids but
we will taper them slowly. You may need to take it for 2-3 years or life-long depending on
your response. We will give you mini aspirin and PPI.

P: Doctor, will I go blind?


D: The high dose steroids are an attempt to bring down the swelling in the hope that re-
establishing the blood supply quickly can bring sight back. Usually, the steroids are used to
stop the Giant Cell Arteritis affecting the sight in the other eye.

www.aspire2plab.com 205
No real PLAB2 cases discussed. All discussed scenarios are fictitious and for educational purposes only.
Thyroid Annual Review

You are FY2 in the GP. Mrs Lucy Talbot, aged 40, has come for the annual review. She was
diagnosed with Thyrotoxicosis and is taking Carbimazole for the last one year. Please talk
to the patient, assess the patient, and discuss management.

D: How can I help you?


P: I came for my thyroid review. I had overactive thyroid. I am on medication for that.
D: I am glad that you came for the follow up. Let me ask you a few questions to see
everything is fine with your condition. P: Ok.

D: How are you now? P: I am fine.


D: Do you have any problems now? P: No
D: Any weight change? Weight loss or weight gain? P: No
D: Any change in bowel habits? Any diarrhoea or constipation? P: No
D: Do you feel that your heart is racing? P: No
D: Tell me about your periods. P: it is regular.
D: Any problem with the periods? P: No
D: When was your last menstrual period? P: 2 weeks back.
D: May I know what medications are you taking now? P: I am taking Carbimazole.
D: May I know how much? P: I take 5 mg once a day.
D: Are you taking it regularly? P: Yes
D: Since how long have you had this problem? P: From last one year.
D: How is your health in general? P: I am fine.
D: Any fever or flu like symptoms? P: No
D: Any rashes? P: No
D: Any itching or yellow discoloration of your eyes and skin? P: No
D: Any plans to become pregnant in near future? D: No, my husband had a vasectomy.

Ask about PMH, Lifestyle and Psychosocial history.

I would like to do GPE, check your vitals and thyroid examination.

From my assessment, everything seems to be fine. We will do routine blood test


(agranulocytosis) and thyroid function test (mainly we do TSH initially if that is deranged
then we consider doing T4)
I will talk to my seniors and based on your blood results we will decide about the dosage of
your medication.

Once your thyroid hormone level is under control, your dose may be gradually reduced
and then stopped. But some people need to continue taking medication for several years or
possibly for life.

Please mention warning signs for hyperthyroidism, hypothyroidism, agranulocytosis.

www.aspire2plab.com 206
No real PLAB2 cases discussed. All discussed scenarios are fictitious and for educational purposes only.
Acoustic Neuroma

You are F2 in GP. Mrs. Selina Richards, aged 50, presented with a problem in her ear. She
has come to you for the first time. Please talk to the patient, assess the patient, and
discuss management.

D: What brought you to the hospital? P: I can’t hear properly from my left ear.
D: Could you please tell me more? P: Like what.
D: Since when are you having this problem? P: From last few months.
D: Is it the same or getting worse? P: It is getting worse.
D: Did anything happen when it started? P: No
D: Any loud sound? P: No
D: What about your other ear? P: It is fine.

D: Anything else? P: Like what


D: Any dizziness? P: I feel unsteady when I’m standing on the ground. I feel wobbly.
D: Do you feel sick? P: No
D: Any motion sickness? P: No
D: Do you often have to ask people to repeat themselves what they are saying? P: No

D: How is your health in general? P: I am fine.


D: Do you have any sore throat, fever or flu like symptoms? P: No
D: Do you have any pain or discharge from your ear? P: No
(Infection-otitis media or other viral illness)
D: Any numbness or weakness on one side of your face? P: No
D: Any blurry vision or double vision? Vertigo? (Acoustic neuroma) P: No
D: Any ringing sounds in your ear? (Meniere’s disease)
D: Swimming? P: No
D: Recent Flight travel? (Barotrauma) P: No

Ask about PMH, Lifestyle and Psychosocial history.


I would like to do GPE, check your vitals and ear examination

From my assessment it seems like you have a condition called sensorineural hearing loss. It
is a problem of your inner ear and the nerves that supplies this part of the ear
There can be many reasons for it and one of them could be Acoustic Neuroma.
An acoustic neuroma is a type of non-cancerous (benign) brain tumour. It can cause
problems with hearing and balance.
We need to do some tests like MRI and CT scan of the brain to confirm. We will refer you to
a specialist. Small tumours often just need to be monitored with regular MRI scans, and the
treatment is generally only recommended if scans show it is getting bigger. This is because
these growths are very slow-growing and may not cause any problems for a long time. If it is
big, then we may do surgery or radiotherapy.
It is difficult for people with sensorineural hearing loss to regain their hearing. We can offer
you a large variety of hearing aids.
Patient needs to inform the DVLA if they drive

www.aspire2plab.com 207
No real PLAB2 cases discussed. All discussed scenarios are fictitious and for educational purposes only.
Otoscopy (Otitis Media)

You are FY2 in A&E. Mr. John Taylor, aged 22, presented to the hospital with pain in the
ear. Please talk to the patient, assess the patient, and discuss management.

D: What brought you to the hospital? P: I have pain in my right ear.


D: Anything else? P: No
D: Any discharge? P: No
D: Hearing problem? P: No
D: What about the other ear? P: No
D: Any cough, fever or flu like symptoms (URTI)? Nausea or vomiting? P: No
D: Any rash, Photophobia, Neck stiffness? P: No

D: Have you had a similar kind of problem in the past? P: No


D: Any allergies from any food or medications? P: I am allergic to Penicillin.
D: Swimming pool? P: No
D: Air Travel? P: No
Ask about PMH, Lifestyle and Psychosocial history.
I would like to do GPE, check your vitals and ear examination
Vitals: Temperature: 38/min, HR: 80/min, BP: 120/80 mmHg, RR: 12-20/min, O2 stats: 99%

From my assessment, you have a condition called Otitis Media. It is an infection of the
middle ear that causes inflammation (redness and swelling) and a build-up of fluid behind
the eardrum. We will give you some Painkillers to control the pain and fever. We might
consider giving you antibiotics. As you are allergic to penicillin, we might give you something
else. Please follow with your GP. If you develop any hearing loss, weakness of one side of
the face and If you feel dizzy please come back to us.

Note: Treatment of choice for Otitis Media:


Amoxicillin 500mg TDS x 5 days
If patient is penicillin allergic
Erythromycin - 500 mg QDS x 5 days
Clarithromycin - 500 BD x 5 days

Admit if the patient is systemically unwell.


Any complications like:Meningitis, Mastoiditis, Cholesteatoma and Facial nerve paralysis.
We have to admit the patient and we will consider doing some scans like CT/MRI

Otoscopy
Normal: I can see the tympanic membrane which pearly grey in colour is. I can see the light
reflex, umbo and handle of malleus. Pars tensa and flaccida are visible. My most probable
diagnosis is normal tympanic membrane).
Otitis media: I can see the tympanic membrane, which is red, oedematous, congested and
oedematous. I cannot see the light reflex, umbo and handle of malleus. Pars tensa and
flaccida are not visible. My most probable diagnosis is otitis media.

www.aspire2plab.com 208
No real PLAB2 cases discussed. All discussed scenarios are fictitious and for educational purposes only.
Otitis Externa

You are an FY2 in GP Surgery. Mr Rowan Smith, aged 28, has come to you with ear pain.
Please talk to the patient, take history, assess, and discuss the initial plan of management
with the patient.

Otitis externa describes diffuse inflammation of the skin and subdermis of the external ear
canal, which may also involve the pinna or tympanic membrane.

Acute otitis externa is inflammation of less than 6 weeks duration, typically caused by
bacterial infection with Pseudomonas aeruginosa or Staphylococcus aureus.

Risk Factors
Underlying skin conditions including contact dermatitis; acute otitis media; trauma to the
ear canal; foreign body or obstruction in the ear canal; and water exposure.

Symptoms:
Itch, pain, or discharge of the ear canal; hearing loss; tenderness of the tragus and/or pinna;
red and oedematous ear canal; tympanic membrane erythema.

Chronic otitis externa is inflammation which has lasted longer than 3 months, and may be
caused by fungal infection with Aspergillus species or Candida albicans.

Risk Factors:
Diabetes mellitus or other causes of immunocompromise, or fungal infection due to
prolonged topical antibiotic or corticosteroid use.

Symptoms:
Itch in the ear, dry scaly skin or red moist skin in the ear canal, possible signs of fungal
infection.

Malignant otitis externa is a potentially life-threatening progressive infection of the


external ear canal causing osteomyelitis of the temporal bone and adjacent structures.

Risk Factors:
Diabetes mellitus or other causes of immunocompromise; older age; radiotherapy to the
ear, head, or neck; and previous ear surgery or irrigation.

Symptoms:
Unremitting pain, purulent ear discharge, systemic illness, hearing loss, granulation tissue in
the ear canal, possible facial nerve palsy.

Examinations & Investigations:


- GPE, Vitals
- Examining the ear canal, pinna, and local lymph nodes for possible signs.

www.aspire2plab.com 209
No real PLAB2 cases discussed. All discussed scenarios are fictitious and for educational purposes only.
- Ear swab for bacterial and fungal microscopy, culture, and sensitivity if there is
treatment failure; severe, recurrent, or chronic infection; ear canal occlusion, or cellulitis
beyond the ear canal.

Treatment:
- Avoid ear trauma; keeping the ear clean and dry; considering the use of over-the-
counter acetic acid 2% ear drops or spray.
- Managing any underlying causes or risk factors.
- Analgesics
- Aural toilet if there is ear canal debris or exudate.
- Topical antibiotic or antifungal preparation, with or without corticosteroid,
depending on clinical judgement.

Follow Up:
- If there are persistent or severe symptoms.
- The person is immunocompromised.
- There is ear canal stenosis or obstruction which is causing difficulty using topical
treatment effectively.
- Seeking specialist advice or arranging specialist referral

www.aspire2plab.com 210
No real PLAB2 cases discussed. All discussed scenarios are fictitious and for educational purposes only.
Ear Wax

You are an FY2 in GP Surgery. Mr Rowan Smith, aged 28, has come to you with hearing
loss. Please talk to the patient, take history, assess, and discuss the initial plan of
management with the patient.

Symptoms:
Hearing loss, blocked ears, ear discomfort, earache, fullness in the ear.

Risk Factors:
Variation in the anatomy of the ear canal.
Age >50, Male, Atopic eczema and psoriasis, use of cotton buds, insertion of hearing aids or
hearing plugs.

There may be a history of exposure to water (this causes expansion of the earwax and may
cause complete blockage of the ear canal).

Examination & Investigations:


GPE, Vitals, Ear Examination

Management
Sodium bicarbonate 5% ear drops, olive or almond oil drops, and sodium chloride 0.9%
nasal drops (off-label use as ear drops)

Do not: Recommend ear drops if you suspect the person has a perforated tympanic
membrane, active dermatitis, or active infection of the ear canal.

If symptoms persist: Consider ear irrigation (flushing the wax out using water) using an
electronic irrigator provided the expertise is available

Do not use ear irrigation to remove wax in people with suspected to have perforated
tympanic membrane, active dermatitis, or active infection of the ear canal, Grommets in
place. Hearing in only one ear if it is the ear to be treated

www.aspire2plab.com 211
No real PLAB2 cases discussed. All discussed scenarios are fictitious and for educational purposes only.
Otitis Media (Child unwell)

You are an FY2 in GP. Mr David Trump, aged 2, has been brought to the GP because he
was feeling unwell. Talk to the father and address his concerns.

D: How can I help you? P: My child has ear pain


D: Can you tell me more? P: It’s on the right side
D: Since when? P: 10 days.
D: Does anything make it better? P: No
D: Does anything make it worse? P: No

D: Is there anything else? P: Like what?


D: Is there any redness? P: Redness is there.
D: Is there any discharge? P: My wife noticed discharge on the pillow.
D: What kind of discharge? P: Yellow
D: Was there any blood mixed in? P: No
D: Any fever? P: Yes
D: Since when? P: He had it 1 week ago now its gone
D: Did you know why he had a fever? P: Yes he had a chest infection then
D: Does he have a cough now? P: No
D: Hearing loss? P: No
D: Any shyness towards light? P: No
D: Any rash on the body? P: No
D: How is his wee? P: Fine
D: How is his poo? P: OK
D: Is he eating well? P: Not feeding well.

D: Any similar episodes before? P: No


D: Has he been diagnosed with any medical conditions in the past? P: No
D: Is he taking any medications including OTC or supplements? P: No
D: Any allergy to medications or food? P: No
D: Has anyone in the family been diagnosed with any medical conditions? P: No
D: How was the birth of your baby? P: Normal Vaginal Delivery.
D: Are you happy with Red Book? P: Yes
D: Is he up to date with all her jabs? P: Yes
D: Who looks after her? P: Me and my wife.
I would like to examine your child’s vitals, GPE and do an ear examination
Examination:
External ear

Inspection
1. Inspect the pinnae
2. Inspect the mastoid
3. Preauricular region

www.aspire2plab.com 212
No real PLAB2 cases discussed. All discussed scenarios are fictitious and for educational purposes only.
4. Conchal bowl

Palpation
1. Palpate the tragus for tenderness.
2. Palpate the regional lymph nodes: Preauricular lymph nodes and post-auricular
lymph nodes

Traction: backward and upward (Downward, outward for children under 12 months)
Insert the otoscope
1. Inspect the external auditory canal
2. Tympanic membrane assessment (colour, shape, light reflex, perforation, scarring)

Ear mannequin: it will show ear perforation with otitis media.

Tongue depressor and torch were there in the cubicle.


- Throat is normal.
- Nasal cavity is normal.
- The other ear is normal.

Acute Otitis Media with Perforation

We have examined your son, on examination of his ear, there is a perforation is his right ear
drum. From our assessment, we suspect your child has a condition called OTITIS MEDIA with
perforation.

As you have mentioned that he isn’t feeding well, I will be informing my senior, and they
may plan to offer your child an admission for the time being.

P: Are you going to give him antibiotics?


D: Yes, as your child has discharge from his ear and is not feeding well, he will need
antibiotics.

Drug of choice: Amoxicillin for 5 days.


(If Allergic to Penicillin, then give Erythromycin or Clarithromycin for 5 days)

Antibiotics are usually only considered if your child:


- has a serious health condition that makes them more vulnerable to complications, such
as cystic fibrosis or congenital heart disease
- is less than three months old
- is less than two years old with an infection in both ears
- has discharge coming from their ear
If the hole in your eardrum is big, or does not heal in a few weeks, the GP may refer you to
an ear specialist to talk about having surgery to repair a perforated eardrum.

www.aspire2plab.com 213
No real PLAB2 cases discussed. All discussed scenarios are fictitious and for educational purposes only.
Cholesteatoma

You are FY2 in GP. Mr Josh Andrews, aged 26, came to the clinic complaining of pain in the
ear. Talk to the patient and discuss the management with the patient.

D: How can I help you? P: I have pain in my ear for 1 month


D: Where is it exactly? P: Right side
D: What were you doing when it started? P: I was just sitting
D: What kind of pain? P: Dull
D: Does it go anywhere? P: No
D: Has it been the same? P: It is getting worse
D: Anything makes your condition better?
P: I have been having this pain for 1 month, I took paracetamol, but it is not improving.
D: Anything that makes it worse? P: No
D: Could you please score the pain on a scale of 1 to 10, where 1 being no pain and 10 being
the most severe pain you have ever experienced? P: 6
D: Have you experienced a similar pain before? P: No
D: How about the other ear? P: It is fine

D: Anything else? P: I can’t hear properly with my right ear.

D: Any Fever? (OM, Meningitis) P: Yes/No


D: Any discharge from the ear? P: Yes
D: Since when? P: For 1 month, it comes and goes
D: What kind of discharge? P: It’s watery and grey
D: How much is it? P: Scanty
D: Is it foul smelling? P: Yes
D: Vertigo, Tinnitus, Numbness or tingling in the face? (Cranial Nerve Involvement) P: No
D: Aural fullness? P: No
D: Any trouble with the vision? (Blurring) P: No
D: Have you been swimming recently? (OE) P: No

D: Have you been diagnosed with any medical condition in the past? P: No.
D: Any DM and HTN? P: No.
D: Are you taking any medications including OTC or supplements? P: PCM.
D: Any allergies from any food or medications? P: No.
D: Any previous hospital stay or surgeries? P: No.
D: Has anyone in the family been diagnosed with any medical condition? P: No.

D: Do you smoke? P: No
D: Do you take alcohol P: No
D: Do you smoke? P: No
D: Do you take alcohol P: No
D: Tell me about your diet? P: I eat everything, its fine.
D: Tell me about your physical activity? P: I am active

D: What do you do for a living? P: Office job.

www.aspire2plab.com 214
No real PLAB2 cases discussed. All discussed scenarios are fictitious and for educational purposes only.
D: Tell me about your home condition? P: I live in a house.
D: Any recent travel? (Flight) P: No

D: I would like to check your vitals, do GPE, and examine your ear. I will be using an
instrument called an otoscope to look inside the ear, and I would also like to do hearing
tests and balance tests. We will also do some initial investigations.

Examiner:
Conductive hearing loss present.
Otoscopy: Perforation in the Middle Ear usually Pars Flacida

D: From what you’ve told me & from my examination, I suspect that you have a condition
called Cholesteatoma.

D: A cholesteatoma is an abnormal collection of skin cells deep inside your ear.


They're rare but, if left untreated, they can damage the delicate structures inside your ear
that are essential for hearing and balance. We would however do some investigations to
confirm this. We’ll do some routine blood tests, and we might plan special radiological tests
called a CT scan or an MRI.

P: Why did I get it?


D: Well, it can happen because of trauma to the middle ear, or a chronic ear infection. Some
people are born with it. Since you have had an earache since 1 month, it might be because
of an infection.

D: Can it happen again?


P: It can recur again in 5-30% cases. Around 10% can get it in another ear as well.
If you develop discharge or significant bleeding from your ear or wound, fever, and severe
pain come to the hospital immediately.

Treatment:
As it is an abnormal collection of skin cells, we will need surgery to remove the
cholesteatoma under general anesthesia.

After the cholesteatoma has been taken out, your ear may be packed with a dressing. This
will need to be removed a few weeks later.

The surgeon may be able to improve your hearing by a tiny artificial hearing bone
(prosthesis). In some cases, it may not be possible to reconstruct the hearing, or a further
operation may be needed.

The benefits of removing a cholesteatoma usually far outweigh the complications. However,
as with any type of surgery, there's a small risk of facial nerve damage resulting in weakness
of the side of the face.

Medical treatment where surgery is not possible will be with antibiotics and regular ear
cleaning.

www.aspire2plab.com 215
No real PLAB2 cases discussed. All discussed scenarios are fictitious and for educational purposes only.
Alcohol Related Liver Disease

You are F2 in GP. Olivia, aged 40, has come to hospital because of abdominal distension.
Please talk to the patient, assess the patient, and discuss management.

D: What brought you to the hospital? P: I have abdomen distension


D: Tell me more about it? P: What you want to know.
D: Since when do you have this problem? P: From the last 2-3 months.
D: Was it sudden or gradual? P: It was gradual.

D: Anything else? P: No
D: Any tummy pain? P: No
D: Any bloating? P: No
D: Do you feel sick? P: No
D: Any vomiting? P: No
D: Any yellow discoloration of the skin? P: No
D: Any loss of weight? P: No
D: Any loss of appetite? P: No
D: Any change in bowel habits? P: No
D: Any constipation? P: No
D: Any blood in stool or dark stool? P: No
D: Do you feel tired or dizzy? P: No

D: Do you have any fever or flu like symptoms? P: No

Ask about PMH, Lifestyle, Menstrual and Psychosocial history.

D: Do you drink alcohol? P: Yes, I drink daily. 4-5 pints of beer for 15 years.

D: Are you sexually active? P: Yes.


D: Are you in a stable relationship? P: Yes.
D: Do you practice safe sex? P: Yes, we use condoms.
D: Any chance you are pregnant? P: No

I would like to check your Vitals and examine your tummy.

Examination
Left Iliac Fossa pain on both superficial and deep palpation.
Examine for: Hepatomegaly, Liver span, shifting dullness, fluid thrill,

From my assessment, you have distension in your tummy. We will do some blood
investigations including LFT and bleeding and clotting profile.
We will do some scans of your liver like USG, CT or MRI

We may consider some further investigations like liver biopsy and endoscopy.
The main cause of your problem is your alcohol intake. It would be great if you can stop
drinking alcohol. I know it is not easy but we are here to help you.

www.aspire2plab.com 216
No real PLAB2 cases discussed. All discussed scenarios are fictitious and for educational purposes only.
Let me start with the non-medical options.
1. Self-help groups (Alcohol Anonymous).
2. One to One counselling.
3. CBT
4. Family Therapy
5. Alcohol Diary.

Medical Options
1. Acamprosate
2. Naltrexone
3. Disulfiram
4. Chlordiazepoxide

Take a balanced diet as you may have malnutrition. Avoid salty foods. Your GP can advise
you on a suitable diet or, in some cases, refer you to a dietitian. In the most serious cases of
malnutrition, nutrients may need to be provided through a feeding tube inserted through
the nose and into the stomach.

5F’s of Abdominal Distention


Flatus, Fluid, Faeces, Foetus and Fat

www.aspire2plab.com 217
No real PLAB2 cases discussed. All discussed scenarios are fictitious and for educational purposes only.
Surgery

www.aspire2plab.com 218
No real PLAB2 cases discussed. All discussed scenarios are fictitious and for educational purposes only.
Whiplash Injury

You are F2 in A&E. Mrs. Samaira Adams, aged 23, presented to the hospital after having a
road traffic accident. She has come to the hospital complaining of pain in her neck. Please
talk to the patient, assess the patient, and discuss management.

D: What brought you to the hospital?


P: I had a road traffic accident and now I have pain in my neck.
D: Please don’t move your head and neck. Could you please tell me more about the
accident?
P: I was driving and suddenly a car hit me from behind. I had sudden front and back
movements in my neck.
D: When did this accident happen? P: This morning.
D: Tell me more about the pain? P: Like what.
D: When did this pain start? P: Pain started 2 hours ago.
D: Where is the pain? P: It is at the back my neck.
D: Do you have anything else? P: Yes, my neck is stiff, whenever I move it.
D: Since when? P: Since this pain started.
D: Do you have anything else? P: No
D: Any headache? P: No
D: Any muscle spasm? P: No
D: Any pain in shoulder and arms? P: No
D: Any pins and needle sensations in your arm? P: No
D: Any injury in your head or neck in the past? P: No

Ask about PMH, Lifestyle and Psychosocial history.


I would like to do GPE, check your vitals, neck and upper limb examination.

From our assessment, we are suspecting you have a condition called whiplash injury.
Whiplash injury is a type of neck injury caused by sudden movement of the head forwards,
backwards or sideways. It occurs when the soft tissues in the neck become stretched and
damaged. As you mentioned you had some front and backward movements in your head
and neck, that is the reason for your problem.

Whiplash will often get better on its own or after some simple treatment in a few weeks
or months.

Keep your neck mobile.


It's important not to rest your neck for prolonged periods if you have whiplash.
Your neck may be painful at first, but keeping it mobile will improve its movement and
speed up your recovery. Any pain you experience when moving your neck is normal and
won't cause further damage.

The following measures can also help reduce your pain and aid your recovery:
Ice packs – for the first few days, holding an ice pack (a bag of frozen peas wrapped in a
towel will also work) to your neck for up to 10 minutes several times a day may help reduce
pain and swelling.

www.aspire2plab.com 219
No real PLAB2 cases discussed. All discussed scenarios are fictitious and for educational purposes only.
Warm compress – After a few days, holding a warm-water bottle to your neck for up to 15
minutes several times a day may be better at soothing your pain.
Good posture – always maintain a good, upright posture by keeping your back straight while
sitting, standing and walking.

Supportive pillow – some people find a firm; supportive pillow helpful when sleeping. Avoid
using more than one pillow and don't sleep on your front.

Painkillers can help relieve the pain of a whiplash injury.


Over-the-counter painkillers are usually recommended first, such as paracetamol and
ibuprofen. These should be used regularly rather than only when the pain is most severe.

If your neck pain is more severe, your GP can recommend a stronger painkiller, such as
codeine. This can be used on its own or in combination with other painkillers.

Physiotherapy
It may be recommended if your symptoms continue for several weeks.
A physiotherapist may use a range of physical techniques to help improve your symptoms,
such as neck exercises, massage and gentle manipulation of your neck.
It's best to try to carry on with your normal activities and not use a neck brace or collar. Try
to avoid staying in the same position, such as sitting or lying down, for long periods.
Follow up.

www.aspire2plab.com 220
No real PLAB2 cases discussed. All discussed scenarios are fictitious and for educational purposes only.
Brachial Plexus Injury

You are F2 in GP. Mrs Daisy Flower aged, 30 had RTA one year back and sustained injury in
the right brachial plexus and fractured her right leg. She was not able to move the right
forearm and now she has come for a review. Please talk to the patient, assess the patient,
and address her concerns.

D: How can I help you?


P: I met with an accident one year back and I had brachial plexus injury. I couldn’t move my
arm and now I have come for my review.

D: I am sorry. Could you please tell me how it was managed?


P: They applied cast.

D: May I know how you are feeling now? P: Now I am fine.


D: Any pain? P: No
D: Any tingling and numbness in your arm? P: No
D: Any swelling? P: No
D: Are you able to move your arm properly? P: Yes
D: Do you have any problem with writing? P: No
D: Are you able to hold objects? P: Yes
D: Are you able to lift weights? P: Yes
D: Are you able to stand properly? P: Yes
D: Are you able to walk properly? P: Yes

D: Are you taking any medications for it?


P: Yes, I am taking some painkillers for the pain and I am also getting physiotherapy. But I
don’t want to continue with it.

D: May I know why? P: Because I am fine and I have


been doing it for a long period of time.
D: I see, how is your general health? P: I am fine
D: Any fever, flu like symptoms? P: No

Ask about PMH, Lifestyle and Psychosocial history.


D: May I know what you do for a living?
P: I have not done anything since I have this problem. Before that, I used to work in an
office.

I would like to do GPE, check your vitals, neck and upper limb examination.
Upper Limb Examination – Normal
On examination, my patient doesn’t have any pain and all the examinations including
sensory and motor are normal.

Concerns:
Can I resume my work?

www.aspire2plab.com 221
No real PLAB2 cases discussed. All discussed scenarios are fictitious and for educational purposes only.
We will refer the patient to the physiotherapist and then we will see if she can resume her
work (if she is lifting heavy weight).

Many injuries to the brachial plexus will recover spontaneously without surgery over a
period of weeks to months, especially if they are mild. Nerve injuries that heal on their own
tend to have better functional outcomes. If your doctor believes that the injury has a good
potential for recovery without surgery, he or she may delay procedures and simply monitor
your injury. The process of nerve healing itself takes time and your doctor may recommend
physical therapy to prevent joint and muscle stiffness.

Electromyography (EMG) : During an EMG, your doctor inserts a needle electrode through
your skin into various muscles. The test evaluates the electrical activity of your muscles
when they contract and when they are at rest. You may feel a little pain when the
electrodes are inserted, but most people can complete the test without much discomfort.

Nerve Conduction studies: These tests are performed as part of the EMG and measure the
speed of conduction in your nerve when some small current passes through the nerve. This
provides information about how well the nerve is functioning.

Magnetic resonance imaging (MRI): This test uses powerful magnets and radio waves to
produce detailed views of your body in multiple planes. It often can show the extent of the
damage caused by a brachial plexus injury and can help assess the status of arteries that are
important for the limb or reconstruction of it. New methods of high-resolution MRI, known
as MR neurography may be used.

Computerized tomography (CT) Myelography: CT uses a series of x-rays to obtain cross


sectional images of your body. CT Myelography adds a contrast material, injected during a
spinal tap, to produce a detailed picture of your spinal cord and nerve roots during a CT
scan, This test is sometimes performed when MRIs don’t provide adequate information.

Angiogram: an imaging test where contrast material is injected into an artery or vein to
check the condition of your blood vessels.

www.aspire2plab.com 222
No real PLAB2 cases discussed. All discussed scenarios are fictitious and for educational purposes only.
Primary Survey (Conscious)

You are FY2 in A&E. Mr. Andy Charles, aged 45, years fell from 2 meters height on
concrete slab. Patient is on resus trauma. Assess the patient and talk about initial
management to the examiner. Please talk to the patient, assess the patient, and discuss
management.

I assume I have taken my universal precautions and the trauma team is with me.
Universal precautions + Trauma team
D: How are you? P: I am in pain (patient points towards pelvis).
D: What happened? P: I fell down from the ladder on a concrete slab.
D: When did this happen? P: It happened 2 hours ago.
D: Now I will examine you quickly. P: OK

Airway:
1. Conscious
2. Airway patent
3. Oxygen
4. Triple immobilization
5. Monitor
6. Primary Series of X-Rays (Cervical, Chest and Pelvis)

Breathing:
1. Shortness of Breath
Open Neck Collar (Instruct the patient not to move their head and neck)
2. Engorged neck veins.
3. Trachea, Exposure, Chaperone
4. Chest: Inspection, Palpation, Percussion & Auscultation

Circulation:
1. Blood on Floor / External Haemorrhage
2. Conjunctival Pallor
3. Cold Peripheries
4. Capillary Refill Time
5. Peripheral Cyanosis
6. NEWS Chart vitals Management:
I would like to put 2 Large Bore Cannulas. I would like to take blood for routine
investigations: grouping and cross matching for 4 units of blood. I would like to give warm
crystalloid solution 2 litres in 1 hour.

Abdomen: Inspection, Palpation, Percussion & Auscultation

Pelvis:
Inspection:
(Looking for deformity, Scrotal Haematoma and Bleeding from External Urethral Meatus)
Compression Distraction Manoeuvre: Positive

www.aspire2plab.com 223
No real PLAB2 cases discussed. All discussed scenarios are fictitious and for educational purposes only.
Legs:
Inspection: Looking for Redness, Swelling, Deformity, Scar, Sinuses & Bleeding.
Wriggle the Toes
Check Distal Pulses

Management:
C.D.M / Spring Test (old name) - Grab Pelvis, Thumb on ASIS and just move. Don’t Palpate,
just go for it. Compression (Close) then Distraction (Open).
When you do it in the exam, the patient will shout as soon as you grab, consider this as
positive.
CDM is positive, so I am suspecting pelvic fracture.

I have already resuscitated my patient. I am going to admit my patient.


I am going to give my patient painkillers:
Morphine in titration
Metoclopramide - (10mg I/V)
Naloxone 100 – 200ug PRN

I would like to put Pelvic binder / girdle (for pelvic fracture)


I would transfer the patient to the trauma centre for definite management.
I would like to order some investigations. (USG of Abdomen and CT Chest, Abdomen &
Pelvis)
I would like to cover my patient to protect from hypothermia.
I would arrange for a Suprapubic Catheter and NG tube for my patient.

Ideally, I would do a secondary survey and full neurological examination.

www.aspire2plab.com 224
No real PLAB2 cases discussed. All discussed scenarios are fictitious and for educational purposes only.
Primary Survey (Unconscious)

You are FY2 in A&E. Mr John Phil, a young man, has been found unconscious lying down on
the pavement. He has been brought to the hospital by ambulance to the A and E
department. Patient is now attached to the monitor. Do assessment and management of
the patient. Trauma team is busy in another cubicle with a more serious patient. A nurse is
there to assist you. You are seeing the patient as the first attending doctor.

I assume I have taken my universal precautions.


Talk to nurse:
D: What happened?
N: We have this patient who was brought in by the ambulance and he is unconscious.
D: Do you have any ambulance notes? N: No
D: Do you know the AVPU score? N: No
D: What have you done? N: I have attached him to the monitor and inserted a cannula.
D: Have you checked the blood sugar? N: Yes, it is normal.

Read vitals on monitor and tell the nurse to keep an eye on vitals.
Monitor:

BP - 130 / 80 RR - 18. O2 sat - 96 % temp - 37 PR - 83


ECG - normal (on paper / monitor).
Now talk to the patient:
D: Are you alright?
D: Tap on shoulders?
D: Ask ID from nurse
Airway:
1. Look / Feel/ Listen: No stridor
2. Airway patent
3. Oxygen
4. Triple immobilization
5. Primary Series of X-Rays (Cervical, Chest and Pelvis)

Breathing:
1. Shortness of Breath
Open Neck Collar (Ask the nurse for inline immobilisation)
2. Engorged neck veins.
3. Trachea, Exposure, Chaperone
4. Chest: Inspection, Palpation, Percussion & Auscultation

Circulation:
1. Blood on Floor / External Haemorrhage (BRUISE CT HEAD)
2. Conjunctival Pallor
3. Cold Peripheries
4. Capillary Refill Time

www.aspire2plab.com 225
No real PLAB2 cases discussed. All discussed scenarios are fictitious and for educational purposes only.
5. Peripheral cyanosis
6. NEWS Chart Vitals Management:

I would like to put another large bore cannula. I would like to take blood for routine
investigations: grouping and cross matching for 4 units of blood. I would like to arrange for
fluids.
Abdomen: Inspection, Palpation, Percussion & Auscultation

Pelvis:
Inspection:
(Looking for deformity, Scrotal Haematoma and Bleeding from External Urethral Meatus)
Compression Distraction Manoeuvre

Legs:
Inspection: Looking for redness, swelling, deformity, scar, sinuses & bleeding.
Wriggle the toes
Check Distal Pulses

4. Disability
Glasgow Coma Scale: EYE, VERBAL, MOTOR

GCS
Announce you are going to start GCS so that if the patient is sleeping he gets ready. “I am
going to assess your Glasgow coma scale now”

Eyes – open your eyes and keep them open for me.
For 2 – pain on sternum / supraorbital region
For 1 – No response.

Verbal
Do you know where you are?
5 - Hospital (Oriented)
4 - In a Park (a place but not) (Confused)
3 - I have Cats (Inappropriate speech)
2 - No words / crying / moaning / mumble / incomprehensible words.
1 - No response

Motor:
6 - Raise your right arm, Raise your left arm, Raise your right leg, Raise your left leg, Stick
your tongue out.
I’m going to squeeze you Thumb. (Give score according to response)

www.aspire2plab.com 226
No real PLAB2 cases discussed. All discussed scenarios are fictitious and for educational purposes only.
Neurological Examination, if you have time.

Management - to nurse:
We need to admit our patient.
Could you please inform the neuro / surgery department?
Could you please inform the Anaesthetists, in case his GCS drops to 8 or below?
Pain killer : we should consider giving him some pain killers.
Investigations : bloods, X-rays , CT, USG abdomen, Pelvis.
Ideally I would do pupillary reflexes
Could you please help me by covering the patient to prevent hypothermia?
Could you please pass catheter and an NG tube?
Ideally I would do a secondary survey and neurological examination.

www.aspire2plab.com 227
No real PLAB2 cases discussed. All discussed scenarios are fictitious and for educational purposes only.
Testicular Examination

You are an FY2 in GP. Mr Mark Jacob, 22 years old, has made an urgent appointment to
see you. Talk to him, do relevant examinations and address his concerns. Assume you are
gloved.

D: How can I help you?


P: I was watching TV and saw an advertisement on examining oneself. Then I examined my
testis at home and noticed a lump in my testes.

D: Tell me more about it? P: What do you want to know?


D: May I know when did you notice it? P: I noticed it yesterday.
D: May I know which side? P: Left side.
D: How many lumps did you notice? P: I noticed one lump.
D: Is it there all the time or comes and goes? P: It is always there.
D: Any change when you cough or in lying down position? (Hernia) P: No
D: Any fever, redness or discomfort? (Epididymitis) P: No
D: Any pain? (Torsion) P: No

D: Do you have anything else? P: No


D: Do you have any lumps and bumps anywhere in your body? P: No (Cancer)
D: Have you noticed any weight loss? P: No
D: Any Loss of Appetite? P: No.
D: Do you have Shortness of Breath or Heart Racing? P: No.
D: Have you hurt yourself? (Trauma) P: No

Ask about PMH, Lifestyle and Psychosocial history.


D: Are you sexually active? P: Yes/No

I would like to do GPE, Vitals, and Genital Examination.


I would like to examine your private area and for the purpose of examination, I would like
you to be undressed below the waist. I will be as gentle and as quick as possible. At any
point of time you feel uncomfortable and want me to stop please let me know I will stop my
examination. I have a chaperone throughout my examination to ensure your privacy. May I
proceed?

From our assessment, it seems that you have a small lump which is attached to the left
testicle. We will do some routine blood tests and some special blood test to detect certain
hormones in your blood known as markers {AFP & HCG & LDH (not specific for testicular
cancer) We will refer you to a specialist within 2 weeks’ time. We will do an US scan, you
may require a CXR, CT & MRI scan.

Concerns:
Is it cancer?

www.aspire2plab.com 228
No real PLAB2 cases discussed. All discussed scenarios are fictitious and for educational purposes only.
Epididymal Cyst

An epididymal cyst is a small, smooth fluid-filled swelling that slowly develops in the
epididymis (a coil-like structure behind the testicles that helps to store and transport sperm).
They are often painless, but the affected testicle may sometimes ache or feel heavy.

A spermatocele is an abnormal sac (cyst) that develops in the epididymis — the small, coiled
tube located on the upper testicle that collects and transports sperm. Noncancerous and
generally painless, a spermatocele usually is filled with milky or clear fluid that might contain
sperm.

- Often epididymal cysts are multiple and are frequently bilateral


- Small cysts may remain undetected and asymptomatic. Small cysts are tolerated by
patients. However, once epididymal cysts get large (with size equivalent to the size of a
testicle) they are, unsurprisingly, more likely to present for removal.
- As they are cystic and fluid-filled they are well defined, fluctuant and do not usually
transilluminate.
- As they arise in the epididymis, the testis is palpable quite separately from the cyst (unlike
a hydrocele where the testis is palpable within the fluid-filled swelling).
- Extra testicular, fluctuant, and cystic swellings which are readily palpable separate from
the body of the testis, are epididymal cysts and do not usually need further investigation.

Investigations:
Scrotal ultrasound will assist diagnosis if there is uncertainty. Aspiration of fluid is rarely useful
or necessary from either a diagnostic or a treatment perspective.

What is the treatment for epididymal cysts?


- If the cyst is small and causing no problems, then all you need to do is keep an eye on it
and see a doctor if it gets bigger or painful.
- Children do not usually need treatment because most cysts disappear by themselves.
However, it may take up to four years to resolve. Surgical removal may be necessary if
they become painful or don't start to shrink.
- Large or painful cysts can be surgically removed or treated by aspiration and injection of
a substance to shrink and seal the cyst.

1. Inguinal hernia is a protrusion of abdominal cavity contents through the inguinal canal.
2. Hydrocele is a collection of fluid in the scrotum.
3. Epididymal cyst is a fluid-filled sac which grows at the top end of the testicle.
4. Spermatocele is a cyst which feels like an Epididymal cyst but it is filled with sperm.
5. Varicocele is a mass of varicose veins in the spermatic cord.

www.aspire2plab.com 229
No real PLAB2 cases discussed. All discussed scenarios are fictitious and for educational purposes only.
Varicocele

Scrotal masses are the most difficult lesions to differentiate by inspection and palpation.
Varicoceles usually occur on the left in post pubertal men and have a "bag of worms" feel.
They should disappear or become less apparent with the patient in the recumbent position.

Symptoms:
Asymptomatic or Scrotal heaviness

Examination:
Careful examination, with the patient standing, is the most important method of detection:
- Dilation and tortuosity of the veins increase with standing and usually decrease on lying
down. The varicocele cannot usually be palpated with the patient lying down.
- Performing the Valsalva manoeuvre whilst standing increases the dilation.
- There may be a cough impulse.
- Most are in the left testicle (80-90%), some bilateral (as many as 35-40% radiologically)
and very few just on the right side.

Investigations:
- CBC
- Colour doppler
- Sperm count

Management:

Surgical repair of subclinical varicoceles is not usually recommended, although opinions


differ. Not all varicoceles require surgery. Surgery has the potential to cause testicular
damage.

However, the primary treatment of varicoceles is surgery and indications include:


- Pain.
- Infertility possibly (controversy surrounds this recommendation).
- To prevent testicular atrophy.

When should I refer a man with a varicocele?


- Refer urgently to a urologist if:
o A varicocele appears suddenly and is painful.
o The varicocele does not drain when lying down.
o Refer routinely to a urologist if there is pain or discomfort.
- Refer adolescents with a varicocele to a urologist:
o If there are concerns about reduced ipsilateral testicular volume.
o If the boy or parents/guardians are concerned by the appearance, or
symptoms, and cannot be fully reassured in primary care.
- Do not refer the male partner of an infertile couple for varicocele surgery as a form of
fertility treatment, as it does not improve pregnancy rates.
- Refer to a urologist if there is uncertainty about the nature of a scrotal swelling.

www.aspire2plab.com 230
No real PLAB2 cases discussed. All discussed scenarios are fictitious and for educational purposes only.
Hydrocele

You are an FY2 in GP. Dr Daniel Finnish, aged 23, came to the GP surgery with testicular
discomfort. Please talk to the patient, assess the patient, and discuss management.

D: How can I help? P: I have discomfort in my testis.


D: Can you tell me more? P: It’s on my right side
D: What do you mean by discomfort? P: I feel like there’s a heaviness present
D: Since when? P: 1 week
D: Does it come and go? P: It’s always there now
D: Does anything make it better? P: No
D: Does anything make it worse? P: No
D: Is there anything else? P: Like what?
D: Any pain P: No
D: Any discharge? P: No
D: Any fever? P: No
D: Any mass in the testis? P: No
D: Any lumps or bumps? P: No
D: Any weight loss? P: No

Ask about PMH, Lifestyle and Psychosocial history.


D: What do you do for a living? P: Student
D: Have you travelled recently? P: Yes
D: Are you sexually active? P: No

I would like to do GPE, Vitals, and Genital Examination.

What causes hydroceles?


Causes of hydrocele:
1. Idiopathic
2. Infection
3. Inflammation
4. Trauma
5. Tumours

Tests:
1. Transillumination test.
2. Fluid may be drained with a needle and syringe to examine testes more easily
3. Ultrasound scan.

Treatment:
1. Leaving it alone is an option

2. Surgery: Surgery may be recommended if your hydrocele is large or uncomfortable. The


operation for a hydrocele involves making a very small cut in the scrotum or lower tummy
(abdominal) wall. The fluid is then drained from around the testicle (testis).
This is a minor operation and is performed as a day case, so does not usually involve an

www.aspire2plab.com 231
No real PLAB2 cases discussed. All discussed scenarios are fictitious and for educational purposes only.
overnight stay in the hospital. A hydrocele may return after surgery, but this is very
uncommon.

3. Drainage The fluid can be drained easily with a needle and syringe. However, following
this procedure, it is common for the sac of the hydrocele to refill with fluid within a few
months. Draining every now and then may be suitable though, if you are not fit for surgery
or if you do not want an operation.

4. Sclerotherapy: Sclerotherapy is the injection of a solution to stop the hydrocele recurring


after having it drained. This is not commonly undertaken but may be offered to some
people who are not suitable to have an operation.

www.aspire2plab.com 232
No real PLAB2 cases discussed. All discussed scenarios are fictitious and for educational purposes only.
Epididymo-Orchitis

You are an FY2 in GP. Mr Daniel White, aged 23, came to the GP surgery with testicular
pain. Talk to him, assess him and address his concerns.

D: How can I help? P: I have pain in my testis.


D: Can you tell me more? P: It’s on my right side
D: Since when? P: 2 days
D: What kind of pain is it? P: Dull
D: Does it radiate anywhere? P: No
D: Does anything make it better? P: No
D: Does anything make it worse? P: Its painful.
D: Can you rate it on a scale of 1 to 10? P: 6

D: Is there anything else? P: Like what?


D: Any fever? P: Yes
D: Since when? P: 2 days
D: Did you take anything to relieve it? P: PCM but didn’t help.
D: Any pain P: No
D: Any discharge? P: No
D: Any mass in the testis? P: No
D: Any lumps or bumps? P: No
D: Any weight loss? P: No
D: Any recent infection or swelling of facial glands? P: No

Ask about PMH, Lifestyle and Psychosocial history.

D: Are you sexually active? P: Yes


D: Since when? P: 1 year
D: Do you practice safe sex? P: No
D: Do you have a stable partner? P: No
D: Any recent partner? P: Yes
D: What is your preferred route of sex? P: Normal

I would like to do GPE, Vitals, and Genital Examination.

Aetiology of epididymo-orchitis:
● sexually transmitted pathogen - eg, Chlamydia tracho-matis and Neisseria gonorrhoeae.
● Specific risk factors include recent instrumentation or cathe-terisation.
● However, there is an overlap between these groups and a thorough sexual history is
imperative for all age groups.
● Mumps should be considered as an aetiology since the epi-demic in 2005.
● Extra-pulmonary TB.

Aetiology of acute orchitis


● Viral: mumps orchitis is most common. Coxsackievirus A, varicella and echoviral
infections are rare.

www.aspire2plab.com 233
No real PLAB2 cases discussed. All discussed scenarios are fictitious and for educational purposes only.
● Bacterial and pyogenic infections: E. coli, Klebsiella, Pseudomonas, Staphylococcus and
Streptococcus species are unusual.
● Granulomatous: syphilis, TB, leprosy, Actinomyces spp. and fungal diseases are rare
● Trauma.
● Idiopathic.

Symptoms and sign:


● Unilateral scrotal pain
● Fever
● Erythema and Swelling in the scrotum
● Urethritis or a history of UTI
● Mumps usually presents with headache, fever and unilateral or bilateral parotid swelling
but may present with epididymitis. Scrotal involvement can occur without systemic
symptoms.

Examination: Groin and rectal examination.


Investigations:
1. Urine dip
2. STI screening
3. Consider HIV testing
4. colour Doppler ultrasound
5. R/o TB (Sputum AFB, CXR)
6. mumps IgM/IgG serology

Treatments for epididymitis


If you have an infection, you'll usually be given antibiotics (2 weeks)

Do
● take painkillers
● hold a cold pack (or a bag of frozen peas wrapped in a tea towel) on your groin
● wear underwear that supports your scrotum

Don’t
● do not have sex if you have gonorrhoea or chlamydia until you have finished the full
course of treatment

www.aspire2plab.com 234
No real PLAB2 cases discussed. All discussed scenarios are fictitious and for educational purposes only.
Mumps Orchitis

You are an FY2 in GP. Mr David Miles, aged 22, came to you with pain in testicles. Talk to
him and address his concerns.

D: How can I help? P: I have swelling in my testis.


D: Can you tell me more? P: It’s on my right side
D: Since when? P: 2 Days
D: Does it come and go? P: No
D: Does anything make it better? P: No
D: Does anything make it worse? P: No
D: Is there anything else? P: Like what?
D: Any fever? P: No
D: Any pain P: No
D: Any discharge? P: No
D: Any mass in the testis? P: No
D: Any lumps or bumps? P: No
D: Any weight loss? P: No
D: Any recent infection or swelling of facial glands? P: Yes
D: When was this? P: It was on both sides 1 weeks ago.

Ask about PMH, Lifestyle and Psychosocial history.


D: Any blood transfusion in the past? P: No
D: Has anyone in the family been diagnosed with any medical condition?
P: Yes, my sister has measles

D: Have you been vaccinated for MMR? P: Yes/No


D: Are you sexually active? P: No

I would like to do GPE, Vitals, and Genital Examination.

I would like to check your vitals including blood pressure and do genital examination.
I would like to send for some initial investigations including routine blood tests such as FBC,
liver and kidney function, blood sugar and cholesterol, and urine tests.

From my assessment, I suspect you have mumps leading to mumps-orchitis. It is when the
testicles and the tube (the epididymis) at the back of the testicles become swollen and
painful.

Management:
Any testicle pain can be eased using painkillers such as paracetamol or ibuprofen you buy
from the pharmacy or supermarket. If the pain is particularly severe, contact your GP, who
may prescribe you a stronger painkiller. Applying cold or warm compresses to your testicle
and wearing supportive underwear may also reduce any pain

www.aspire2plab.com 235
No real PLAB2 cases discussed. All discussed scenarios are fictitious and for educational purposes only.
OBG

www.aspire2plab.com 236
No real PLAB2 cases discussed. All discussed scenarios are fictitious and for educational purposes only.
Breast Lump

You are FY2 in GP. Mrs. Selina Gomez, aged 44 years, presented to the GP. She is concerned
because she found a lump in her breast. Please talk to the patient, take focused history, do
relevant examination, discuss about management with the patient and address her
concerns

D: What brought you to the hospital?


P: While I was taking a shower this morning and I noticed a lump in my left breast.
D: Tell me more about it. Where exactly is the lump? Any pain? Any change in shape and
size of the breast? Any discharge from the nipples? Any lumps and bumps in the body?
Any weight loss? Loss of appetite? Any SOB or tiredness? Any trauma?

D: Have you had a similar kind of problem in the past? P: No

Ask about PMH, Lifestyle and Psychosocial history.

D: Have you been diagnosed with any medical condition in the past? P: No
D: Has anyone in the family been diagnosed with any medical condition? P: No
D: Are you using any method of contraception? P: No
D: Tell me about your periods? Are they regular? Any pregnancy? If yes, how many kids?

I would like to do GPE, Vitals, and breast examination.

I can feel the mass in the right upper quadrant of the right breast. It is around 2x2 cm. It is
round, smooth, regular margin and firm in consistency. It is attached to the underlying
structures, and it is not attached to overlying skin. It is non-tender.

Management
From our assessment, you have a lump in …….area.
Treatment for a lump depends on the cause. Most are harmless and may go away on their
own without treatment. We will refer you to the specialist for further investigation to make
sure everything is fine in your case.

They will do some tests like a mammogram. (If >35, USG if <35)
If the cause of breast lump cannot be diagnosed on Mammogram or USG, we will do a
procedure to remove a tissue sample from the lump in your breast. (FNAC)
They can also consider doing a biopsy (where a needle is inserted into the lump to remove
some cells for testing).

These tests are often done during the same visit. You'll usually be told the results on the
same day, although biopsy results take longer – you should get them within a week.
Follow up with your GP and if you feel worried or frightened about anything or if you want
to talk things through or have a question about breast health, please come back.

www.aspire2plab.com 237
No real PLAB2 cases discussed. All discussed scenarios are fictitious and for educational purposes only.
Cyclical Breast Pain

You are F2 in GP. Jenny, aged 40, booked an urgent appointment to discuss her problem.
Talk to the patient, take history, assess her and discuss the plan of management and
address her concerns.

D: What brought you to the hospital? P: My breasts are lumpy and sore.
D: Could you please tell me more about the soreness?
P: I am having this soreness in both the breasts.
D: For how long have you been having this soreness? P: It has been there for the last
few months.
D: Is it all the time or comes and goes? P: It comes and goes.
D: How many episodes have you had till now? P: I have this soreness whenever
I am about to have my periods.
D: How often do you get this soreness? P: I have this soreness a few days
before my cycle. When my period comes the pain goes away.
D: Is it the same or getting worse? P: It is the same.
D: Is there anything that makes it better? P: I tried paracetamol, it didn’t help much.
D: Is there anything that makes it worse?
P: When I touch my breast, it becomes more sore.
D: Tell me more about the lumpiness in your breast?
P: It is also the same as soreness, I feel it before the start of my periods. When my periods
start, it gets resolved.

D: Do you have anything else? P: No


D: Any change in shape and size of the breast? P: No
D: Any change in the skin of the breast? P: No
D: Any discharge from the nipples? P: No
D: Any lumps and bumps anywhere else in the body? P: No
D: Any weight loss? P: No
D: Any loss of appetite? P: No
D: Any SOB or tiredness? P: No
D: Any fever or flu like symptoms? P: No

D: Tell me about your periods, when was your last menstrual period? P: 2 weeks ago
D: Are they regular? P: yes
D: Any heavy periods or bleeding in between the periods? P: No
D: Any pregnancy? If yes, how many kids? P:
D: Are you sexually active now? P: Yes
D: Are you using any method of contraception? P: Yes/No

Ask about PMH, Lifestyle and Psychosocial history.


I would like to do GPE, Vitals, and breast examination.

From my assessment, you seem to have a condition called Cyclical breast pain This pain is
related to periods. Typically, it occurs in the second half of the monthly cycle, becoming
worse in the days just before a period.

www.aspire2plab.com 238
No real PLAB2 cases discussed. All discussed scenarios are fictitious and for educational purposes only.
It is thought that women with cyclical breast pain have breast tissue which is more sensitive
than usual to the normal hormonal changes that occur each month. It is not due to any
hormone disease or to any problem in the breast itself. It is not related to any other breast
conditions. Although it is not serious, it can be a nuisance.

No treatment may be needed if the symptoms are mild. Many women are reassured by
knowing that cyclical breast pain is not a symptom of cancer or serious breast disease. The
problem may settle by itself within a few months. However, sometimes this pain may come
and go over the years.

Treatment options include the following:


1. Support your breasts with a well-supporting bra when you have pain.
2. Painkillers and anti-inflammatory medications (Oral and topical) like paracetamol or
ibuprofen can be taken on the days when the breasts are painful.
3. Medicines such as danazol, tamoxifen and goserelin injections can ease pain in most
cases. They are prescribed by specialists only in severe condition.
4. Evening primrose oil might be helpful.

Follow up with your GP and if you feel worried or frightened about anything or if you want
to talk things through or have a question about breast health please come back. If you have
breast pain with a lump in your breast or under your arms, discharge from a lump or nipple,
swelling and redness in your breast any symptoms of pregnancy, such as a missed period
please see a doctor.

Medicines may worsen cyclical breast pain:


1. The contraceptive pill or hormone replacement therapy (HRT).
2. Some antidepressants and some blood pressure medicine.

www.aspire2plab.com 239
No real PLAB2 cases discussed. All discussed scenarios are fictitious and for educational purposes only.
Nipple Discharge

You are an FY2 in GP. Miss Yara White, aged 29, has come to you because of Nipple
Discharge. Talk to her and address her concerns.

D: How can I help? P: I have some discharge coming from my nipples. (Single or Both)
D: Elaborate: When? How long? Colour? Consistency? Blood stained?

D: Do you have anything else? P: No


D: Any change in shape and size of the breast? P: No
D: Any change in the skin of the breast? P: No
D: Any swelling or mass? P: No
D: Any lumps and bumps in the body? P: No
D: Any weight loss? P: No
D: Any loss of appetite? P: No
D: Any SOB or tiredness? P: No
D: Any fever or flu like symptoms? P: No

D: Tell me about your periods, when was your last menstrual period? P: 2 weeks ago
D: Are they regular? P: Yes
D: Any heavy periods or bleeding in between the periods? P: No
D: Any pregnancy? If yes, how many kids? P:
D: Are you sexually active now? P: Yes
D: Are you using any method of contraception? P: Yes/No

Ask about PMH, Lifestyle and Psychosocial history.


I would like to do GPE, Vitals, and breast examination.

We may refer you to a hospital or breast clinic for further tests. At the hospital or breast
clinic, you may have a:
• breast examination
• scan – usually a breast X-ray (mammogram) or ultrasound
• biopsy – where a needle is inserted into your breast to remove some cells for testing

The tests are often done during the same visit.


You'll usually be told the results on the same day, although biopsy results can take longer –
you should get them in a week or two.

See a GP if you have nipple discharge and any of these:


• it happens regularly and isn't just a one-off
• it only comes from 1 breast
• it's bloodstained or smelly
• you're not breastfeeding, and it leaks without any pressure on your breast
• you're over 50
• you have other symptoms – such as a lump, pain, redness or swelling in your breast
• you're a man

www.aspire2plab.com 240
No real PLAB2 cases discussed. All discussed scenarios are fictitious and for educational purposes only.
Causes:
• breastfeeding or pregnancy – see leaking nipples in pregnancy
• a blocked or enlarged milk duct
• a small, non-cancerous lump in the breast
• a breast infection (mastitis)
• a side effect of a medicine – including the contraceptive pill

Nipple Discharge
● Nipple discharge isn't usually a sign of anything serious, but sometimes it's a good idea
to get it checked, just in case.
● Nipple discharge is often normal
● Lots of women have nipple discharge from time to time. It may just be normal for you.
● It's also not unusual for babies (boys and girls) to have milky nipple discharge soon after
they're born. This should stop in a few weeks.
● Nipple discharge in men isn't normal.
● The colour of your discharge isn't a good way of telling if it's anything serious. Normal
discharge can be lots of colours.

www.aspire2plab.com 241
No real PLAB2 cases discussed. All discussed scenarios are fictitious and for educational purposes only.
Antenatal

You are FY2 in Antenatal Clinic. 36-week pregnant lady, Mrs Jane Green, G3 P2+0 came to
the hospital for her antenatal check-up. She had 2 normal vaginal deliveries. Do an
antenatal examination and discuss management with the patient. Nurse did an
examination and found it was a breech presentation.

D: What brought you to the hospital? P: I came for my antenatal check-up.


D: Tell me how many weeks pregnant are you? P: 36 weeks doctor.
D: How is your pregnancy so far? P: It is fine.
D: Have you attended all your antenatal clinics so far? P: Yes
D: Did you have any complications? P: No
D: Have you been pregnant before? P: Yes doctor, 2 times.
D: What was the outcome? P: I have two kids and both were
born with normal vaginal delivery.

Ask about PMH, Lifestyle and Psychosocial history.


I would like to check your vitals and examine your tummy.

Management:
From my assessment you have a breech presentation.
Usually, the baby lies in head first position, but sometimes the baby lies in bottom or feet
first position rather than the usual Head first position which is called a breech presentation.

You are 36 weeks pregnant. There are chances that your baby might turn into the head first
position till 37 week. Otherwise, we have many options to deliver your baby. You may feel
breathless as your baby’s head presses on your diaphragm or you may feel a bit discomfort
under your ribs. You may also feel some sharp kicks on your bladder.

Most breech babies are born healthy.


Sometimes it’s a matter of chance that the babies don’t turn into headfirst position.
Otherwise, there might be other factors which prevent the turn, such as the amount of fluid
in the womb, the position of the after birth.

Options of delivery:
ECV: The first option is the External cephalic version in short ECV which can be offered from
37 weeks onwards since you have had babies before. In this procedure, we put external
pressure on your tummy to turn the baby to the headfirst position.
It can be slightly uncomfortable, but you will be given medication to relax the muscles of
your womb. If it still hurts, we can stop the procedure and try it on another day.
It’s usually safe and doesn’t cause labour to begin. Your baby will be monitored throughout
the procedure and even after that.
If you notice any tummy pain, bleeding, changes in the baby’s movements or contractions
after the ECV, please call the hospital immediately.

Caesarean section: Most breech babies in the UK are born by Caesarean Section as it is the
safest way for the breech babies to be born.

www.aspire2plab.com 242
No real PLAB2 cases discussed. All discussed scenarios are fictitious and for educational purposes only.
Breech vaginal delivery: As you said you have given birth vaginally before and so there
should be a problem, but if your baby is in a certain position like feet first or hyperextended,
or your baby is too large or small or lying in a problem with your after birth, we will not
recommend breech vaginal delivery.

P: Is there anything to be worried about breech?


P: Is my baby going to be alright?
P: Why is my baby breech?
P: What are you going to do for me?
P: Can I expect a vaginal delivery?

www.aspire2plab.com 243
No real PLAB2 cases discussed. All discussed scenarios are fictitious and for educational purposes only.
Cervical Smear

You are FY2 in GP. Mrs Michelle Aylsbury, aged 40, presented to the clinic for her routine
cervical screening test. Talk to the patient; perform the pap smear and assess the patient's
concern.

D: What brings you to the hospital? P: I came here for my routine pap smear test.
D: Do you have any symptoms? P: No
D: When was your LMP? P: 2 weeks ago.
D: Are your periods regular? P: yes.
D: Any discharge from your vagina? P: No
D: Any bleeding between your periods? P: No
D: Any problem with the urine or bowel? P: No
D: Are you sexually active? P: Yes.
D: Any bleeding during or after sex? P: No
D: How is your appetite these days? P: It’s alright
D: Have you noticed any decrease in weight? P: No
D: Any SOB or heart racing? P: No
D: Any pain in your lower back or pelvis? P: No
D: Any lumps and bumps in any part of the body? P: No

D: Do you have any children? P: Yes, I have one daughter.


D: Have you used any contraception in the past? P: Yes, my partner uses condoms.

Ask the PMH, Lifestyle and Psychosocial history.

Why we do PS examination:
- Speculum is the instrument most used to inspect the vagina.
- The purpose of the examination is to look at the size and shape of external and internal
reproductive organs.

The external examination will involve:


- Examination of anatomy.
- Looking for any lesions, ulcers, discharge or other signs of disease.

The internal examination will involve:

- Palpation of the vulva and vaginal walls.


- Examination of the cervix.
- Assessing the size and position of the uterus.
- Palpation of any adnexal tenderness.
- Location of the cervix using the speculum.
- Performing any appropriate swabs or smears using the speculum.

Preparation and Introduction:


1. Introduce yourself to patient (Greet, Introduce, Identify and Explain Procedure)
2. Explain the purpose of the examination.

www.aspire2plab.com 244
No real PLAB2 cases discussed. All discussed scenarios are fictitious and for educational purposes only.
3. Explain that it will involve undressing fully from the lower half and the examination
may be a bit uncomfortable but should not be painful.
4. Gain consent and offer a chaperone.
5. Explain to the patient that the position they should be lying in is supine, with knees
bent, heels brought up towards bottom, and then letting legs fall to either side of the
bed. Let the patient undress in privacy.
6. Tell the patient to empty the bladder.
7. Prepare trolley and equipment: flexible light source, gloves, lubricating jelly and
speculum.
8. Allow the patient to become comfortable before starting

Contraindication of pap smear:

● Pregnancy
● Active menstruation
● Use of spermicidal gel
● Recent sexual intercourse
● Active vaginal bleeding

Contraindication for Per speculum:


Bartholin cyst (You may feel a soft painless lump, this does not usually cause any problem,
but if the cyst grows very large, it can become noticeable and uncomfortable. You may feel
pain in the skin, vulva, when you walk, sit down or have sex.

Inspection:
1. Inspect the external genitalia for hair distribution, swelling, scarring, signs of infection for
example warts or ulcers.
2. Ask the patient to cough looking for signs of prolapse.

Speculum examination and Pap smear:


1. Think about the size of the speculum needed and use lubrication.
2. Explain to the patient what you are going to do before proceeding.
3. Spread the labia from below using the thumb and index finger.
4. Gently insert the speculum and rotate the speculum to a horizontal position and gently
open the blades until the cervix is in view. Secure the speculum by turning the thumb nut.
5. Visualize the cervix and vaginal walls for any abnormalities, such as ectopy, cysts or
polyps.
6. Comment on whether the cervical os is open or closed (multiparous or nulliparous) and if
it is downward and backward.
7. Perform any necessary tests, obtaining samples for culture and cytology. A small soft
brush will be used to gently collect some cells from the surface of your cervix.
a. Insert the brush through the speculum avoiding touching the sides of the speculum with
the brush.
b. Rotate the brush 5 times 360 degrees in clockwise direction.
c. Remove the brush and avoiding the sides of the speculum.
d. Sample collection:

www.aspire2plab.com 245
No real PLAB2 cases discussed. All discussed scenarios are fictitious and for educational purposes only.
1. Dip the brush 10 times in a liquid-based cytology container (Thin Prep);
2. Deposit the tip of the brush into liquid-based cytology container (Sure Path).

8. Withdraw the speculum slightly to clear the cervix and gently loosen the speculum to
close the blades.
9. Continue to withdraw whilst rotating the speculum.
10. Tell the patient that you have finished, offer wipes to the patient and tell her to dress
up.
11. The cell sample is then sent off to a laboratory for analysis and you should receive the
result within 2 weeks.

All women who are registered with a GP are invited for cervical screening:
• aged 25 to 49 – every 3 years
• aged 50 to 64 – every 5 years
• over 65 – only women who have recently had abnormal tests

1.Some women find the procedure a bit uncomfortable or embarrassing, but for most
women, it isn't painful.
2.If you find the test painful, tell the doctor or nurse as they may be able to reduce your
discomfort.
3. Try to relax as much as possible as being tense makes the test more difficult to carry
out. Taking slow, deep breaths might help. You can also bring someone along to the
appointment with you if you want support.

www.aspire2plab.com 246
No real PLAB2 cases discussed. All discussed scenarios are fictitious and for educational purposes only.
Procedures

www.aspire2plab.com 247
No real PLAB2 cases discussed. All discussed scenarios are fictitious and for educational purposes only.
Blood Sampling (Paracetamol)

You are F2 in A&E. Mrs. Billie Brown, aged 25, has taken some paracetamol tablets. Do
venepuncture for necessary investigations, talk to the patient, take relevant history, discuss
further management with the patient and address her concerns.

D: What brought you to the hospital? P: I took some paracetamol tablets.


D: How many tablets? P: 40 tablets doctor.
D: May I know when you took them? P: 6 hours ago.

D: Did you take them in one go?


D: Did you take anything else with it?
D: Any alcohol or drugs?
D: Do you feel sick?
D: Did you vomit?
D: Any tummy pain?
D: What did you do after that?

Ask about PMH, Lifestyle and Psychosocial history.

Do the blood sampling.

After getting the blood, I would like to send the sample for investigations including FBC, LFT,
Bleeding and clotting profile and plasma paracetamol level concentration.
I would like to check your vitals.
Examination: Plasma Paracetamol Concentration is 81 mmol/lt

Explain the graph properly to the patient.


The level of PCM is 81 in your blood and you told you took the tablets 6 hours ago.
As you can see in the graph, the horizontal line tells us the time and vertical line shows us
the level of paracetamol in your blood. If you draw an imaginary line, it comes above the
treatment line.

It means you have to stay in the hospital, and we have to give you the medications through
your blood vessels. If it is left untreated, it can be dangerous for your liver and kidneys.

We will give you a medication called N-Acetylcysteine (NAC) through your blood vessels.
We must give you 3 drips of this medication and usually it takes 21 hours.

After that, we will reassess you and repeat your blood test and if everything is fine, we will
refer you to one of my colleagues (Psychiatrist). He will talk to you and will help to relieve
your stress.

S/E of NAC:
This is a safe medication, but it can cause nausea, vomiting, diarrhoea or constipation, rash,
fever, headache, drowsiness, low blood pressure, and liver problems. Don’t worry, we will
manage accordingly.

www.aspire2plab.com 248
No real PLAB2 cases discussed. All discussed scenarios are fictitious and for educational purposes only.
Paracetamol Overdose Treatment Graph

Blood Culture

You are FY2 in General Surgery. Mr. Peter Smith, 27, has been diagnosed with a ruptured
appendix. Patient underwent a surgery a few days ago for his burst appendix. Consultant
has asked you to do blood culture. Please talk to the patient, do the blood culture and
address his concerns.

www.aspire2plab.com 249
No real PLAB2 cases discussed. All discussed scenarios are fictitious and for educational purposes only.
IV Cannulation

You are FY2 in surgery. Mr. Peter Smith, 27, has been diagnosed with a ruptured appendix.
Patient underwent a surgery a few hours ago for his burst appendix. Patient IV cannula is
blocked now. Please talk to the patient, change the IV cannula and discuss the
management.

D: What brought you to the hospital?


P: Doctor I am in pain, I had an operation for my appendix. Give me some painkiller.
D: I am sorry. Is there anything else? P: Vomiting.
D: As you are vomiting, we can’t give you a painkiller through your mouth. We have to give
you medication through your blood vessels, but your cannula is blocked. So please let me
change your cannula first so that I can give you medication through blood vessels.

Ask about PMH, Lifestyle and Psychosocial history.

Change the IV Cannula

Management
Ask which pain killers has my patient already received
Also please tell me the vitals of my patient.

Examination:
Chart: 5 mg IV Morphine every 4 hours.
Last dose received 1 hour ago.
RR: 12-20/min, Temperature: 38, PR: 94, O2 sat: 92-93%, BP: 120/80.

I will give my patient oxygen and will take blood for some investigations like electrolytes.
I will give my patient 1 gm paracetamol and 10 mg metoclopramide.
As my patient is vomiting, I will give IV fluids as well.

If the patient is still in pain, I will tell him that I have given the pain killer and it will take
some time to show its effect.

If the patient remains in pain, I will check the vitals of my patient and after discussion with
the seniors, I will give another dose 5mg of morphine. I will consider writing naloxone prn.

If patient hasn’t received anything tell the examiner, I would like to give my patient
morphine in titration,
Paracetamol 1 gm,
Metoclopramide 10 mg.

www.aspire2plab.com 250
No real PLAB2 cases discussed. All discussed scenarios are fictitious and for educational purposes only.
Arterial Blood Gas

You are an FY2 in Respiratory Medicine. Jamie Preston, aged 50, has been admitted in the
hospital because of COPD. Please talk to the patient, assess him, do ABG sampling and
address his concerns.

Take history, explain the procedure and perform it.

An ABG is a blood test that measures the acidity, or pH, and the levels of oxygen (O2) and
carbon dioxide (CO2) from an artery. The test is used to check the function of the patient’s
lungs and how well they can move oxygen into the blood and remove carbon dioxide.

ABGs are drawn for a variety of reasons. These may include Lung Failure and various
respiratory conditions, Kidney Failure, Shock, Trauma, Uncontrolled diabetes, Drug
Overdose, Metabolic Disease, Chemical Poisoning

Procedure

Position the patient’s hand preferably on a pillow for comfort with the wrist extended by
approximately 20-30°. Assess the course of the radial artery to determine where you plan to
perform arterial sampling:

1. Palpate the radial artery over the wrist of the patient’s non-dominant hand to identify an
ideal puncture site. You should use the tips of your fingers to clearly map out the course of
the radial artery and then identify a distal site where the artery is most pulsatile. The radial
artery is typically most superficial over the lateral anterior aspect of the wrist.
2. Once you have identified your planned puncture site, clean it with an alcohol wipe for 30
seconds and allow it to dry before proceeding.

Arterial puncture
1. Remove the protective cover from the ABG needle and then flush through the heparin
from the syringe.
2. Hold the patient’s wrist extended by approximately 20-30°.
3. Palpate the radial artery with your non-dominant hand’s index finger around 1cm
proximal to the planned puncture site (avoiding contaminating the planned puncture site
that you previously cleaned).
4. Warn the patient you are going to insert the needle.
5. Holding the ABG syringe like a dart, insert the needle through the skin at the insertion
site.
6. Continue to advance the needle slowly towards the pulsation until you feel a sudden
reduction in resistance and see a rush of blood back into the ABG syringe (this is known as
“flashback”).
7. The ABG syringe should then begin to self-fill in a pulsatile manner. If this doesn’t happen,
it may indicate you have gone through or missed the artery and therefore need to re-adjust
your position based on your understanding of the course of the radial artery (e.g. change in
angulation or slight withdrawal of the needle).

www.aspire2plab.com 251
No real PLAB2 cases discussed. All discussed scenarios are fictitious and for educational purposes only.
8. Once the required amount of blood has been collected, remove the needle and apply
immediate firm pressure over the puncture site with some gauze. Secure the gauze with
some tape and continue to apply pressure.
9. Engage the needle safety device (often a clip that covers the needle or a bung that the
needle is inserted into).
10. Remove the ABG needle from the syringe and discard immediately into a sharps bin.
11. Place a cap onto the ABG syringe and carefully expel any air from the sample if present.
12. Label the ABG sample with the patient’s details.
13. Either the patient or a colleague should continue to apply firm pressure to the puncture
site for 3-5 minutes to reduce the risk of haematoma formation.

Take the ABG sample to be analysed as soon as possible after being taken as delays longer
than 10 minutes can affect the accuracy of results. Also, if it is taking time please put the
sample in the ice box.

Contraindication for ABG:

Absolute contraindications: peripheral vascular disease in the limb, cellulitis surrounding


the site or arteriovenous fistula.

Modified Allen’s test


Before taking a sample from the radial artery, a modified Allen’s test should be performed
to assess the collateral arterial supply of the hand from the ulnar artery.

To perform a modified Allen’s test:


1. Ask the patient to clench their fist and apply pressure over the radial and ulnar artery to
occlude both vessels.
3. Ask the patient to open their hand, which should now appear blanched. If the hand does
not appear it suggests you are not completely occluding the arteries with your fingers.
4. Remove the pressure from the ulnar artery whilst maintaining pressure over the radial
artery.
5. If there is adequate blood supply from the ulnar artery, the normal colour should return
to the entire hand within 5-15 seconds. If the return of colour takes longer, this suggests
poor collateral circulation Do not perform arterial blood gas sampling on a hand that does
not appear to have an adequate collateral blood supply.

www.aspire2plab.com 252
No real PLAB2 cases discussed. All discussed scenarios are fictitious and for educational purposes only.
Catheterisation

You are FY2 in A&E. George Jefferson, aged 45 years, comes to you complaining of lower
abdominal discomfort. No one has seen the patient before you. Please talk to the patient,
take relevant history, assess the patient, do emergency management, and discuss further
management with the patient. Do the relevant procedure, if needed.

D: What brought you to the hospital? P: I cannot pass urine.


D: Since when? P: From the last 24 hours.
D: Anything else? P: I have tummy pain (Point towards supra-pubic area)

I would like to check your vitals, GPE and tummy examination.

Examination: Abdomen is distended, and the bladder is palpable. There is dullness over
percussion.

Rule Out Contraindications:


Any Recent Instrumentation in your private area?
Any trauma in your private part?
Any blood at the tip of penis?

Do foleys catheterisation.

Rule out UTI, BPH and Prostate cancer.

Do you have anything else with the pain?


Any fever or flu like symptoms? Frequency? Nocturia? Urgency? Dribbling? Poor emptying?
Poor stream? Haematuria? Hesitancy?
Any weight loss? Loss of appetite? Tiredness or breathlessness?

Ask about PMH, Lifestyle and Psychosocial history.

I would like to check blood pressure, amount of urine output and examine your prostate.

Examination: Prostate is enlarged in both lobes and the surface is smooth.


From our assessment, we are suspecting you have a condition called benign prostatic
hyperplasia or big prostate.

Prostate is a walnut-sized gland that is located under your bladder. The tube connecting
your bladder to your penis goes through this gland. When it gets enlarged, it can cause
retention of urine.

We will do some blood tests to check anaemia and kidney function and urine test to see if
there is any bug in your urine. We may give you some antibiotics if we find any bug there.

www.aspire2plab.com 253
No real PLAB2 cases discussed. All discussed scenarios are fictitious and for educational purposes only.
We will do some special blood tests to check the amount of substance produced from your
prostate gland. (PSA)

We will give you medication to relax the neck of the bladder and the tube connecting your
bladder to your penis and we may give you some IV fluids if needed.

We will observe you in the observation unit and if everything goes smoothly, you are able to
go home.

We will provide you with some extra bags and we will tell you how to change them.
We will arrange an outpatient clinic appointment in the next two weeks and we will give a
trial without a catheter. We will do USG to check If you need a catheter or not.

If you are unable to pass urine with catheter or you develop some fever, shivering please
dial 999 and come back to us.

www.aspire2plab.com 254
No real PLAB2 cases discussed. All discussed scenarios are fictitious and for educational purposes only.
Joint Examinations

www.aspire2plab.com 255
No real PLAB2 cases discussed. All discussed scenarios are fictitious and for educational purposes only.
Hip Examination

You are an FY2 in Orthopaedics. A 50-year-old male patient to the hospital with hip pain.
Please talk to the patient, do relevant examination, give the management plan and
address his concerns.

D: What brought you to the hospital today? P: I have pain in my hip


D: May I know which side? P: Right side
D: SOCRATES
P: For 3 weeks. Pain just started. Getting worse with time. Continuous. Dull pain. Doing
some activities makes pain worse.

D: Did you hurt yourself? P: No


D: Any previous medical diagnosis? P: No
D: Any regular medications? P: No
D: Any allergies? P: No

Ex: I am going to examine your hip.

D: Any surgeries to the hip? P: No


D: Are you able to stand and walk? P: Yes

Joint examination: LOOK (Standing)/ FEEL (Lying flat)/ MOVE (Lying flat) & Special Tests

Look (Inspection):
Anatomical position: Joint symmetry
Front: Skin changes/ scars/ swelling/ level of ASIS/ Quadriceps muscle wasting
Side: Lumbar lordosis (Normal or Hyper lordosis)
Back: Scoliosis / gluteal wasting / pelvic tilt
Gait: Normal/ Antalgic/ Trendelenburg
Trendelenburg Test

Fell (Palpation):
Temperature
Tenderness: Around the joint/ Greater trochanter (Tenderness on palpation of greater
trochanter suggests greater trochanteric bursitis).
Leg length: True/ Apparent
True: Umbilicus to the tip of medial malleolus.
Apparent: ASIS to the tip of medial malleolus

Move (Movements):
Active: Flexion: Bring your right knee towards the chest and relax. Bring your left knee
towards the chest and relax.

www.aspire2plab.com 256
No real PLAB2 cases discussed. All discussed scenarios are fictitious and for educational purposes only.
Abduction/Adduction: Bring your right leg at the edge of the couch. Bring your right leg and
cross it over the left leg.
Bring your left leg at the edge of the couch. Bring your left leg and cross it over the left leg.

Internal and external rotation: Join your toes and keep your heel apart. Join your heel and
keep the toes apart.

Special Tests:

Thomas Test:
1. Place the hand on the patient’s spine.
2. Flex the unaffected leg as far as you are able to and your hand should detect the lumbar
lordosis now flattened.
3. The test is positive if the affected thigh rises off the bed indicating a loss of extension in
the hip. This would suggest a fixed flexion deformity in the affected hip.
(NOTE: Don’t do if hip replacement)

Trendelenburg Test:
1. Place your hands on the iliac crest on either side of the pelvis.
2. Ask the patient to stand on the affected side for 30 second and observe your hands to see
which one moves up and down.
3. Normally the iliac crest on the side with the foot off (Unaffected) the ground should rise
up.
4. The test would be positive if the pelvis falls on the side with the foot on the ground
(Unaffected). The abnormal result suggests weak hip abductors on the contralateral side.

I would like to finish my examination by examining a joint above (lumbar spine) and a joint
below (Knee).

I would also like to do a full neurological examination of lower limbs including checking for
distal pulses.

Management:
From my assessment, you seem to be having a condition called Greater Trochanteric
Bursitis. This means inflammation of bursa on the bony bump called greater trochanter, on
the outside of your hip. (Bursa is a fluid filled sac that provides cushion around bony
prominences).

Do PRICE avoid HARM

Rest – try not to move the joint too much and avoid activities that'll put pressure on it.
Ice – gently hold an ice pack (or a bag of frozen peas) wrapped in a tea towel on the area for
around 10 minutes at a time and repeat every few hours during the day.
Anti-inflammatory medication

www.aspire2plab.com 257
No real PLAB2 cases discussed. All discussed scenarios are fictitious and for educational purposes only.
Avoid aggravating positions
Stretches
Physiotherapy
In persistent cases steroid injection may be helpful.

P: What causes GTPS (Greater Trochanteric Pain Syndrome?


D: It is most common in middle aged females. Most cases of greater trochanteric pain
syndrome are due to minor tears or damage to the nearby muscles, tendons or fascia, so that
an inflamed bursa is an uncommon cause. So, rather than the term trochanteric bursitis, the
more general term, greater trochanteric pain syndrome, is now preferred.
The exact causes of GTPS are not fully understood, but there are many factors that can
contribute to it, including: direct fall on outside edge of hip
· excessive load e.g. prolonged walking or running. Poor running style can also lead to
increased load on this area of the hip.
· prolonged or excessive pressure to your hip area (for example, sitting in bucket car seats, or
sleeping on your affected side, may aggravate the problem)
· weakness of the muscles surrounding the hip, called the gluteus Medius and minimus.

P: How long does it take to get better?


D: Everybody will recover differently, but for most people it will take six to nine months of
focused rehabilitation to make a return to all normal activities without pain.

If you have a very high temperature, cannot move the affected joint and you have very
severe, sharp or shooting pains in the joint, come back to the GP.

www.aspire2plab.com 258
No real PLAB2 cases discussed. All discussed scenarios are fictitious and for educational purposes only.
Knee Examination

You are F2 in Orthopaedics. Tom came to the hospital with some knee problem. Your
colleague David has taken the history but he doesn’t know how to do knee examination.
Please teach him knee examination. Don’t let him do the examination.

D: Hello David. How are you? David: I am good

D: How can I help you?


David: We have a patient Tom. He came with a knee injury. I have taken history but I don’t
know how to do a knee examination.

D: Don’t worry, I will tell you how to do a knee examination. Could you please tell me the
history of the patient?
David: I just asked his name.

D: Let’s talk to our patient. Hello, Tom, how are you? P: I am good.

D: Today I will be doing a knee examination and I will be teaching my colleague David as
well. But don’t worry I will be the one who will do the examination.
P: Ok.

D: Tell me which knee is bothering you? P: It is my right knee.


D: Have you had any knee surgery in the past? P: No
D: Are you able to stand independently? P: Yes.

Exposure
Chaperone
Being gentle
Consent

D: Could you please stand for me? P: Ok

Look:
Anatomical position
Symmetry of joints (shoulder / hip / ankles)
Inspection of the knees: (Deformity, Redness, Scar, Swelling, Wasting)

Gait:
D: Are you able to walk? P: Yes Dr

Position patient on an examination couch at 45°

Feel:
Temperature
Tenderness
Bent:

www.aspire2plab.com 259
No real PLAB2 cases discussed. All discussed scenarios are fictitious and for educational purposes only.
Patella (2 Finger Palpation)
Joint Lines
Popliteal Fossa

Straight:
Tibial Tuberosity
Patellar Tendon

Sweep/Swipe test (Small Effusion): On the medial border of patella, from down to up,
several times use your hands to bring effusion to the upper border, hold your hand there.
Then with one hand, push the effusion down on the lateral aspect, you will be able to see
the bulge. (Positive)

Patella tap (Large Effusions): Use the hand's web space between the thumb and index finger
to empty the pouch and hold the hand there. With one hand, two fingers, gently tap over
the patella, if you feel the bounce, it’s positive.

Grind test (PatelloFemoral Arthritis): Use left hand web space between thumb and index
finger and put beneath the upper border of patella, then tell the patient to squeeze the
muscles of the thigh. Positive if painful.

Move:
Active Movements: - flexion and extension
Special Tests -
1. Stress Test: - For Collateral Ligament (Medial and Lateral)
MCL: Make a hook of your right hand, pick up the ankle, bend knee at 20 - 30, left hand on
lateral aspect and push it right towards yourself.

2.Mc Murray’s Test: - For Meniscus (Medial and Lateral)


LM: - bend the heel towards yourself and push it towards yourself then back to original
position

3. Drawers Test: - For Cruciate Ligament (Anterior and posterior):


ACL: - Ask the patient if he has pain, tell him you will sit on his foot to stabilize the knee and
press over it then put your thumbs on TT. Then pull it towards yourself.

Check Peripheral Pulses.


I would like to finish my examination by examining one joint above and below this joint. I
will do a full neurological examination.

www.aspire2plab.com 260
No real PLAB2 cases discussed. All discussed scenarios are fictitious and for educational purposes only.
Elbow Examination

You are an F2 in GP. Mrs Kate Brunt, aged 53, came to the clinic with some concerns.
Please talk to her, assess her and discuss your plan of management with her and address
her concerns.

D: How can I help? P: I have pain in my elbow.


D: Can you tell me more? P: It’s on my right side
D: Since when? P: 1 month
D: What kind of pain is it? P: Dull
D: Does it radiate anywhere? P: No
D: Does anything make it better? P: Paracetamol
D: Does anything make it worse? P: No
D: Can you rate it on a scale of 1 to 10? P: 5-6
D: Is there anything else? P: Like what?

D: Any stiffness? P: No
D: Any trauma? P: No
D: Any fever? P: No
D: Any difficulty with movement? P: Yes
D: Any swelling? P: No
D: Any bruising? P: No

D: Have you had similar kinds of problems in the past? P: No


D: Have you been diagnosed with any medical condition in the past? P: No
D: any DM, HTN, Heart disease or high cholesterol? P: No
D: Are you taking any medications including OTC or supplements? P: No
D: Any allergies from any food or medications? P: No
D: Any previous hospital stay or surgeries? P: No
D: Has anyone in the family been diagnosed with any medical condition? P: No

D: Do you smoke? P: Yes/No


D: Do you drink alcohol? P: Yes/No
D: Tell me about your diet? P: I try to eat healthy.
D: Do you do physical exercise? P: I don’t have much time.
D: Do you work? P: Yes, I am an assistant in the kitchen.
D: Do you have any kind of stress? P: No.

I would like to check your vitals and examine your elbow.


I would like to send for some initial investigations including Routine Blood Test, X-Ray.

Look
Inspect from the front:
There are no Scars, Swelling, erythema of the joint. Carrying angle is normal.

Inspect from the back:


There are no Scars, Swelling, erythema of the joint. There are no Rheumatoid nodules and

www.aspire2plab.com 261
No real PLAB2 cases discussed. All discussed scenarios are fictitious and for educational purposes only.
Psoriatic plaques.

Feel
Temperature: Compare the elbow joint with the other elbow and the forearm.
Palpate around the elbow to elicit any localised tenderness:
1. Lateral epicondyle
2. Olecranon
3. Medial epicondyle
4. Biceps tendon.

Move
1. Elbow flexion.
2. Elbow extension.
3. Pronation.
4. Supination.

Special tests
- Medial epicondylitis – “Golfer’s elbow”

Active wrist flexion against resistance


1. The patient should be seated for this assessment, with their elbow flexed at 90º
2. Stabilise the patient’s elbow by supporting the forearm with one hand and firmly
palpating the patient’s medial epicondyle
3. Hold the patient’s wrist with your other hand
4. Ask the patient to make a fist and actively flex the wrist
5. Ask the patient to hold the wrist in flexion while you try to passively extend it

- Lateral epicondylitis “Tennis elbow”

Active wrist extension against resistance


1. Stabilise the patient’s elbow by supporting the forearm with one hand and firmly
palpating the patient’s lateral epicondyle
2. Hold the patient’s wrist with your other hand
3. Ask the patient to make a fist and extend their wrist
4. Ask the patient to hold the wrist in extension while you try to passively flex it by pushing
down on the dorsum of the hand

Causes of elbow pain


• playing racquet sports – such as tennis, badminton or squash.
• throwing sports – such as the javelin or discus
• using shears while gardening
• using a paint brush or roller while decorating
• manual work – such as plumbing or bricklaying
• activities that involve fine, repetitive hand and wrist movements – such as using
scissors or typing
• other activities that involve repeatedly bending the elbow – such as playing the violin

www.aspire2plab.com 262
No real PLAB2 cases discussed. All discussed scenarios are fictitious and for educational purposes only.
Treatment:
Do PRICE (Pain Killers, Rest, Ice, Compression, Elevation)
Avoid HARM (Heat, Alcohol, Running, Massage)

Steroid injections
PRP injections.
1. Avoid the activities responsible for the condition.
2. wear a tennis elbow splint when you are using your arm
3. Increasing the strength of your forearm muscles can help prevent tennis elbow.

Patient concern:
I want a sick note.

www.aspire2plab.com 263
No real PLAB2 cases discussed. All discussed scenarios are fictitious and for educational purposes only.
Hand & Wrist Examination

You are an FY2 in Surgery. Michael Roster, a final year student, has missed his class on
examination of the hand. Please teach him hand examination. Please do not let him do the
examination.

Rapport
Assess his knowledge.
D: How can I help you today?
S: Doctor, I want to learn how to do a hand and wrist examination. Can you help me with
that?
D: OK. How much do you know about this examination? S: I don’t know much.
D: Don’t worry, I’ll explain it to you. S: Ok.

Preparation and Introduction:


1. Introduce yourself to patient (Greet, Introduce, Identify and Explain Procedure)
2. Explain the purpose of the examination.
3. Explain that it will involve examination of the hand and wrist joints.
4. Gain consent and offer a chaperone.

General Inspection:
1. Inspect of the hands
2. Inspect the palms for Pallor, Palmar erythema and Dupuytren’s contracture
3. Inspect the nails for the Koilonychia and Leukonychia.
4. To assess for finger clubbing check for Schamroth’s window.
5. To assess for asterixis or flapping tremor.
6. Assess the patient’s radial pulse for Rhythm and volume.

Look:
Inspect hands from Dorsum:
There are no skin, nail changes, scar marks, swelling, deformities or muscle wasting.

Palms up:
There are no scars and swelling, Dupuytren’s contracture or thenar and hypothenar muscle
wasting.

Elbows:
There are no evidence of psoriatic plaques or rheumatoid nodules

Feel:
Palms up

Temperature:
Assess and compare the temperature of the wrists and small joints of the hand.

Radial and ulnar pulse:


Palpate the radial and ulnar pulse to confirm there is adequate blood supply to the hand

www.aspire2plab.com 264
No real PLAB2 cases discussed. All discussed scenarios are fictitious and for educational purposes only.
Thenar/hypothenar eminence bulk:
The muscle bulk of the thenar and hypothenar eminences is normal. There is no
palmar thickening

Median, ulnar and radial nerve nerve sensation:


median nerve sensation over the thenar eminence and index finger
ulnar nerve sensation over the hypothenar eminence and little finger
Radial nerve sensation over the first dorsal web space

Dorsum:

Assess and compare temperature using the back of your hand:


Wrist and MCP joint.

Gently squeeze across the metacarpophalangeal (MCP) joints, bimanually palpate the joints
of the hand (MCPJ/PIPJ/DIPJ/CMCJ)

Assess and compare joints for tenderness, irregularities and warmth:


Metacarpophalangeal joint (MCPJ), Proximal interphalangeal joint (PIPJ), Distal
interphalangeal joint (DIPJ), Carpometacarpal joint (CMCJ) of the thumb (squaring of the
joint is associated with OA). Palpate the wrists for evidence of joint
line irregularities or tenderness

Palpate the anatomical snuffbox: Tenderness may suggest scaphoid fracture.

Palpate the elbow: Along the ulnar border to the elbow feel for any rheumatoid nodules or
psoriatic plaques (extensor surface).

Move:
Active movements:
Finger flexion – Make a fist.
Finger extension – Open your fist and splay your fingers
Wrist extension – Put the palms of your hands together and extend your wrists fully.
Wrist flexion – Put the backs of your hands together and flex your wrists fully

Passive movement:
Assess movements passively, feeling for crepitus and noting any pain.

Motor assessment
Wrist/finger extension – radial nerve
Finger ABduction of the index finger – ulnar nerve
Thumb ABduction – median nerve

Function
Assess the patient’s hand function using the following screening tests:
Power grip – “Squeeze my fingers with your hands”

www.aspire2plab.com 265
No real PLAB2 cases discussed. All discussed scenarios are fictitious and for educational purposes only.
Pincer grip – “Squeeze my finger between your thumb and index finger “
Pick up a small object or undo a shirt button – “Can you pick up this small coin out of my
hand?”

Special tests
Tinel’s test:
Tinel’s test is used to identify nerve irritation and can be useful in the diagnosis
of carpal tunnel syndrome.
Tap over the carpal tunnel with your finger. If the patient develops tingling in
the thumb and radial two and a half fingers this is suggestive
of median nerve irritation and compression.

Phalen’s test:
Ask the patient to hold their wrist in complete and forced flexion (pushing the dorsal
surfaces of both hands together) for 60 seconds.
patient’s symptoms of carpal tunnel syndrome are reproduced then the test
is positive (e.g burning, tingling or numb sensation in the thumb, index, middle and ring
fingers)

To finish the examination, I will do a full neurological examination, I will examine one joint
above.

www.aspire2plab.com 266
No real PLAB2 cases discussed. All discussed scenarios are fictitious and for educational purposes only.
Teaching

www.aspire2plab.com 267
No real PLAB2 cases discussed. All discussed scenarios are fictitious and for educational purposes only.
Basic Life Support (Adult)

You are F2 in the general medical ward. Alex is a FY1 doctor who has not attended his BLS
workshop. Please demonstrate and teach him. Please explain the steps of BLS, assess him
and give him feedback.

www.aspire2plab.com 268
No real PLAB2 cases discussed. All discussed scenarios are fictitious and for educational purposes only.
SEQUENCE Technical description
SAFETY Make sure you, the victim and any bystanders are safe

RESPONSE Check the victim for a response


• Gently shake his shoulders and ask loudly: “Are you all right?"
If he responds, leave him in the position in which you find him,
provided there is no further danger; try to find out what is wrong with
him and get help if needed; reassess him regularly.

AIRWAY Open the airway


• Turn the victim onto his back
• Place your hand on his forehead and gently tilt his head back;
with your fingertips under the point of the victim's chin, lift
the chin to open the airway.

BREATHING Look, listen and feel for normal breathing for no more than 10
seconds
In the first few minutes after cardiac arrest, a victim may be barely
breathing, or taking infrequent, slow and noisy gasps. Do not confuse
this with normal breathing. If you have any doubt whether breathing
is normal, act as if it is they are not breathing normally and prepare to
start CPR.

DIAL 999 Call an ambulance (999)


• Ask a helper to call if possible, otherwise call them yourself.
• Stay with the victim when making the call if possible.
• Activate the speaker function on the phone to aid
communication with the ambulance service.

SEND FOR AED Send someone to get an AED if available


If you are on your own, do not leave the victim, start CPR

www.aspire2plab.com 269
No real PLAB2 cases discussed. All discussed scenarios are fictitious and for educational purposes only.
CIRCULATION Start chest compressions
• Kneel by the side of the victim
• Place the heel of one hand in the centre of the victim’s chest;
(which is the lower half of the victim’s breastbone (sternum))
• Place the heel of your other hand on top of the first hand
• Interlock the fingers of your hands and ensure that pressure is
not applied over the victim's ribs
• Keep your arms straight
• Do not apply any pressure over the upper abdomen or the
bottom end of the bony sternum (breastbone)
• Position your shoulders vertically above the victim's chest and
press down on the sternum to a depth of 5–6 cm
• After each compression, release all the pressure on the chest
without losing contact between your hands and the sternum;
• Repeat at a rate of 100–120 min-1

www.aspire2plab.com 270
No real PLAB2 cases discussed. All discussed scenarios are fictitious and for educational purposes only.
GIVE RESCUE After 30 compressions open the airway again using head tilt and
BREATHS chin lift and give 2 rescue breaths
• Pinch the soft part of the nose closed, using the index finger
and thumb of your hand on the forehead
• Allow the mouth to open, but maintain chin lift
• Take a normal breath and place your lips around his mouth,
making sure that you have a good seal
• Blow steadily into the mouth while watching for the chest to
rise, taking about 1 second as in normal breathing; this is an
effective rescue breath
• Maintaining head tilt and chin lift, take your mouth away from
the victim and watch for the chest to fall as air comes out
• Take another normal breath and blow into the victim’s mouth
once more to achieve a total of two effective rescue breaths.
Do not interrupt compressions by more than 10 seconds to
deliver two breaths. Then return your hands without delay to
the correct position on the sternum and give a further 30
chest compressions
Continue with chest compressions and rescue breaths in a ratio of
30:2

If you are untrained or unable to do rescue breaths, give chest


compression only CPR (i.e. continuous compressions at a rate of at
least 100–120 min-1)

www.aspire2plab.com 271
No real PLAB2 cases discussed. All discussed scenarios are fictitious and for educational purposes only.
IF AN AED Switch on the AED
ARRIVES • Attach the electrode pads on the victim’s bare chest
• If more than one rescuer is present, CPR should be continued
while electrode pads are being attached to the chest
• Follow the spoken/visual directions
• Ensure that nobody is touching the victim while the AED is
analysing the rhythm
If a shock is indicated, deliver shock
• Ensure that nobody is touching the victim
• Push shock button as directed (fully automatic AEDs will
deliver the shock automatically)
• Immediately restart CPR at a ratio of 30:2
• Continue as directed by the voice/visual prompts
If no shock is indicated, continue CPR
• Immediately resume CPR
• Continue as directed by the voice/visual prompts

CONTINUE CPR Do not interrupt resuscitation until:


• A health professional tells you to stop
• You become exhausted
• The victim is definitely waking up, moving, opening eyes and
breathing normally
It is rare for CPR alone to restart the heart. Unless you are certain the
person has recovered continue CPR

www.aspire2plab.com 272
No real PLAB2 cases discussed. All discussed scenarios are fictitious and for educational purposes only.
RECOVERY If you are certain the victim is breathing normally but is still
POSITION unresponsive, place in the recovery position
• Remove the victim’s glasses, if worn
• Kneel beside the victim and make sure that both his legs are
straight
• Place the arm nearest to you out at right angles to his body,
elbow bent with the hand palm-up
• Bring the far arm across the chest, and hold the back of the
hand against the victim’s cheek nearest to you
• With your other hand, grasp the far leg just above the knee
and pull it up, keeping the foot on the ground
• Keeping his hand pressed against his cheek, pull on the far leg
to roll the victim towards you on to his side
• Adjust the upper leg so that both the hip and knee are bent at
right angles
• Tilt the head back to make sure that the airway remains open
• If necessary, adjust the hand under the cheek to keep the
head tilted and facing downwards to allow liquid material to
drain from the mouth
• Check breathing regularly
Be prepared to restart CPR immediately if the victim deteriorates or
stops breathing normally

Initial assessment:
For clarity, the algorithm is presented as a linear sequence of steps. It is recognised that the
early steps of ensuring the scene is safe, checking for a response, opening the airway,
checking for breathing and calling the ambulance may be accomplished simultaneously or in
rapid succession.

(Credits: Resus Council UK)

www.aspire2plab.com 273
No real PLAB2 cases discussed. All discussed scenarios are fictitious and for educational purposes only.
Basic Life Support (Paediatric)

You are FY2 in Paediatrics. Alex is a Final Year Medical Student. He wants to learn how to
do Paediatric BLS. Please demonstrate and teach him. Please teach Alex how to perform
BLS on a 6 Year Old Child.

Paediatric basic life support

Unresponsive

Call for help / activate 2222

Open airway

Breathing normally, YES


signs of life?

NO Assess ABCDE
or any doubt O2 monitoring,
vascular access

5 rescue breaths
• Competent providers should use Call further
bag-mask ventilation with oxygen assistance as
appropriate
• If unable/unsafe to ventilate perform
continuous chest compressions,
ventilate as soon as possible

If no signs of life observed


during rescue breaths,
immediately commence
chest compressions
• 15 chest compressions: 2 breaths
• Attach ECG monitoring/defibrillator
when available

Follow
PAEDIATRIC ADVANCED LIFE SUPPORT
ALGORITHM
on arrival of in-hospital clinical
emergency team

www.aspire2plab.com 274
No real PLAB2 cases discussed. All discussed scenarios are fictitious and for educational purposes only.
Paediatric out-of-hospital
basic life support
Unresponsive

Second rescuer or
Shout for help single rescuer suspecting
a primary cardiac arrest
• Call EMS on 999
• Collect and apply AED
Open airway if feasible

YES
Breathing normally?

Observe
NO and re-assess as
or any doubt necessary

5 rescue breaths Single rescuer


Infant: mouth to nose/mouth • Call EMS if phone
Child: mouth to mouth available, using speaker
function
If unable/unsafe to ventilate, perform • If no phone available
continuous chest compressions continue with CPR for 1
Add rescue breaths as soon as possible minute before calling EMS

If no signs of life observed


during rescue breaths

30 chest compressions

2 rescue breaths

Clear signs of life? YES

Keep child in safe position,


NO continue to assess and
await EMS

Those trained only in ‘adult’ BLS (may include healthcare providers and lay rescuers) who have no specific knowledge of paediatric
resuscitation, should use the adult sequence they are familiar with, including paediatric modifications.

www.aspire2plab.com 275
No real PLAB2 cases discussed. All discussed scenarios are fictitious and for educational purposes only.
Per Speculum

You are FY2 in Gynaecology. Dr. Ankit Kumar has missed his classes for Pelvic
examination. Please teach him how to do per speculum examination. Don’t let him do the
examination.

Introduce yourself to the medical student: build rapport with him. Ask how his studies are
going.

Assess his knowledge about per speculum examination. Remember to make sure that the
student is following what you are teaching and appreciate the student.

Explain why we do PS examination.

- Speculum is the instrument most commonly used to inspect the vagina.


- The purpose of the examination is to look at the size and shape of external and internal
reproductive organs.

The external examination will involve:

- Examination of anatomy.
- Looking for any lesions, ulcers, discharge or other signs of disease.

The internal examination will involve:

- Palpation of the vulva and vaginal walls.


- Examination of the cervix.
- Assessing the size and position of the uterus.
- Palpation of any adnexal tenderness.
- Location of the cervix using the speculum.
- Performing any appropriate swabs or smears using the speculum.

Preparation and Introduction:


1. Introduce yourself to patient (Greet, Introduce, Identify and Explain Procedure)
2. Explain the purpose of the examination.
3. Explain that it will involve undressing fully from the lower half and the examination may
be a bit uncomfortable but should not be painful.
4. Gain consent and offer a chaperone.
5. Explain to the patient that the position they should be lying in is supine, with knees bent,
heels brought up towards bottom, and then letting legs fall to either side of the bed. Let
the patient undress in privacy.
6. Tell the patient to empty the bladder.
7. Prepare trolley and equipment: flexible light source, gloves, lubricating jelly and
speculum.
8. Allow the patient to become comfortable before starting.

www.aspire2plab.com 276
No real PLAB2 cases discussed. All discussed scenarios are fictitious and for educational purposes only.
Contraindication of pap smear:

● Pregnancy
● Active menstruation
● Use of spermicidal gel
● Recent sexual intercourse
● Active vaginal bleeding

Inspection
1. Inspect the external genitalia for hair distribution, swelling, scarring, signs of infection
for example warts or ulcers.
2. Ask the patient to cough looking for signs of prolapse.

Examination

1. Think about the size of the speculum needed and use lubrication.
2. Explain to the patient what you are going to do before proceeding.
3. Spread the labia from below using the thumb and index finger.
4. Gently insert the speculum and rotate the speculum to a horizontal position and gently
open the blades until the cervix is in view. Secure the speculum by turning the thumb
nut.
5. Visualize the cervix and vaginal walls for any abnormalities, such as ectopy, cysts or
polyps.
6. Comment on whether the cervical os is open or closed (multiparous or nulliparous) and
if it is downward and backward.
7. Perform any necessary tests, obtaining samples for culture and cytology (below).
8. Withdraw the speculum slightly to clear the cervix and gently loosen the speculum to
close the blades.
9. Continue to withdraw whilst rotating the speculum.
10. Tell the patient that you have finished, offer wipes to the patient and tell her to dress
up.

Contraindication for Per speculum:


Bartholin cyst (You may feel a soft painless lump, this does not usually cause any problem,
but if the cyst grows very large, it can become noticeable and uncomfortable. You may feel
pain in the skin, vulva, when you walk, sit down or have sex.

www.aspire2plab.com 277
No real PLAB2 cases discussed. All discussed scenarios are fictitious and for educational purposes only.
Inguino - Scrotal Examination

You are an FY2 in Surgery. Mr Tom Jerry, a final year medical student, has come to you to
learn inguino scrotal examination. Please teach him and do not let him examine the
patient.

D: How can I help you today?


S: Doctor, I want to learn how to do an inguino-scrotal examination. Can you help me with
that?
D: OK. How much do you know about this examination? S: I don’t know much.
D: Don’t worry, I’ll explain it to you. S: Ok.

Preparation and Introduction:


1. Introduce yourself to patient (Greet, Introduce, Identify and Explain Procedure)
2. Explain the purpose of the examination.
3. Explain that it will involve undressing fully from the abdomen to the thighs.
4. Gain consent and offer a chaperone.

Examination:
a. Exposure
b. Chaperone
c. Being gentle
d. Consent

Assessment of a suspected hernia:

An inguinal hernia is a protrusion, or movement of abdominal contents, from within the


abdominal cavity. This tissue then protrudes, or emerges, at the exit point, the superficial
inguinal ring. Inguinal hernias are most commonly found superomedial to
the pubic tubercle.

Femoral hernias occur just below the inguinal ligament, when abdominal contents pass
through a naturally occurring weakness in the abdominal wall called the femoral canal.

It is important to note that the space a femoral hernia protrudes through is quite tight, and
it is bordered medially by the sharp edge of the lacunar ligament. Therefore, femoral
hernias are at high risk of strangulation and obstruction. Femoral hernias are
typically found infero-lateral to the pubic tubercle (and medial to the femoral pulse).

Position of Hernia

● Above and medial to the pubic tubercle: Inguinal hernia


● Below and lateral to the pubic tubercle: Femoral hernia

www.aspire2plab.com 278
No real PLAB2 cases discussed. All discussed scenarios are fictitious and for educational purposes only.
Direct vs Indirect Inguinal Hernia

● Locate the deep inguinal ring (midway between the anterior superior iliac spine and
pubic tubercle)
● Ask the patient to reduce their hernia (if able) or alternatively reduce it yourself by
starting inferiorly compressing the lump towards the deep inguinal ring
● Once reduced, apply pressure over the deep inguinal ring
● Ask the patient to cough
● If a hernia reappears it is more likely to be a direct inguinal hernia whereas if it does
not, it is more likely to be an indirect inguinal hernia
● In the latter case, release the pressure from the deep inguinal ring and observe for
the hernia reappearing (further supporting the diagnosis of an indirect inguinal
hernia)
● It should be noted that this clinical test is unreliable and further imaging (e.g.
ultrasound scan) would be required before any management decisions were made

1. The mid-inguinal point: It is halfway between the anterior superior iliac spine and the
pubic symphysis.

2. The midpoint of the inguinal ligament: It is halfway between the anterior superior
iliac spine and pubic tubercle.

3. The two openings to the inguinal canal are known as rings:

a) The deep (internal) ring is found above the midpoint of the inguinal ligament
which is lateral to the epigastric vessels.
b) The superficial (external) ring marks the end of the inguinal canal, and lies just
superior to the pubic tubercle.

Inguinal hernias can extend into the scrotum, so if you note swelling or suspect an inguinal
hernia, palpation of the scrotum can be performed (with consent)
Typically, an inguinal hernia will present as a testicular lump that you cannot get above.

www.aspire2plab.com 279
No real PLAB2 cases discussed. All discussed scenarios are fictitious and for educational purposes only.
www.aspire2plab.com 280
No real PLAB2 cases discussed. All discussed scenarios are fictitious and for educational purposes only.
Teaching ECG

You are an F2 in A&E. Alexa, your nurse colleague has come to you to learn ECG. Talk to
her about ECG.

D: How can I help you today?


N: Doctor, I want to learn how to read this ECG. Can you help me with that?

D: OK. How much do you know about ECG?


N: I don’t know much.

D: Don’t worry, I’ll explain it to you. Do you know what ECG means?
N: No

D: An ECG records the electrical activity of the heart on a graph. It gives information about
the rate & rhythm of your heart. So, we can assess if the heart is working normally or not.
N: OK

D: Do you know when it is done?


N: No

D: It is done when patients present with symptoms of heart disease such as chest pain,
palpitations and syncope.

The heart is actually composed of 4 chambers; 2 atria & 2 ventricles. Between these
chambers there are valves which control the pumping of blood through them.

The electrical activity of the heart is actually because of depolarization & repolarization that
takes place in the heart muscles.

Our heart has pacemakers. The SA(sinoatrial) node is located in the right atrium which
generates the electrical impulses which travel through the heart.

We can translate what is happening inside the heart onto a piece of paper known as the
graph.

www.aspire2plab.com 281
No real PLAB2 cases discussed. All discussed scenarios are fictitious and for educational purposes only.
D: The horizontal axis represents time & the vertical axis represents the voltage. Graph has
big squares with 5 small squares within 1 big square. Each large square (5 mm) represents
0.2 second (s).

ECG machines run at a standard rate of 25 mm/s. The squares on this graph paper are of

standard size.

A heart beat comprises different parts.

(OHCM)

P-waves:
P-waves represent atrial depolarisation.

PR interval:
It represents the time taken for electrical activity to move between the atria and ventricles.
• Location—from the beginning of the P wave to the beginning of the QRS complex
• Duration—0.12 to 0.20 second.

www.aspire2plab.com 282
No real PLAB2 cases discussed. All discussed scenarios are fictitious and for educational purposes only.
QRS complex:
The QRS-complex represents depolarisation of the ventricles.

ST segment:
→ The ST-segment starts at the end of the S-wave and finishes at the start of the T-wave.
→ The ST segment is an isoelectric line
→ A change in the ST segment may indicate myocardial damage.
→ An ST segment may become either elevated or depressed.

T-wave:
The T-wave represents ventricular repolarisation.

Heart Rate:
Heart rate can be calculated using the following method (if regular):

Count the number of large squares present within one R-R interval
Divide 300 by this number to calculate the heart rate
e.g. 4 large squares in an R-R interval: 300/4 = 75 beats per minute

www.aspire2plab.com 283
No real PLAB2 cases discussed. All discussed scenarios are fictitious and for educational purposes only.
Heart rate can be calculated using the following method (if irregular):

Count the number of the R Waves between 30 Large Squares. Multiply it by 10.

What’s a normal adult heart rate?


Normal = 60 – 100 bpm
Tachycardia > 100 bpm
Bradycardia < 60 bpm

Heart Rhythm

1) Each P Wave should be followed by QRS Complex


2) PR Interval should be < 0.2 Seconds
3) QRS should be < 0.12 Seconds
4) QRS Complex should be equidistant apart
5) T Wave upright and round
6) Rate should be 60 – 100 Beats per Minute

Electrodes

www.aspire2plab.com 284
No real PLAB2 cases discussed. All discussed scenarios are fictitious and for educational purposes only.
ECG Abnormalities

Ventricular Tachycardia

Myocardial Infarction

www.aspire2plab.com 285
No real PLAB2 cases discussed. All discussed scenarios are fictitious and for educational purposes only.
Atrial Fibrillation

Atrial Flutter

www.aspire2plab.com 286
No real PLAB2 cases discussed. All discussed scenarios are fictitious and for educational purposes only.
Epi Pen Teaching

You are an FY2 in A & E. Miss Amanda Holmes, mother of 3 year old child has come to you
because her child was brought in to the hospital because of anaphylaxis for the second
time. Talk to the mother and address her concern.

D: How can I help you?


M: My child has got peanut allergy and he got this anaphylaxis attack second time. I was
given an Epi-Pen and I was explained how to use it previously. However, I didn’t listen
properly to the doctors that time. This time I was very worried when he developed the
attack.

D: Can you tell me how this episode happened today?


P: I don’t know

D: What did you do when you saw your child have this episode?
P: I called the ambulance.

D: You mentioned a previous episode of anaphylaxis, can you tell me about it?
P:

D: Do you know what she is allergic to?


P: Peanuts

D: Is she taking any medications including OTC or supplements? P: No


D: Any allergies from any food or medications? P: No
D: Has anyone in the family been diagnosed with any medical condition? P: No

D: How was the birth of your baby? P: It was normal vaginal delivery.
D: Are you happy with the red book? P: Yes.
D: Is she up to date with all her jabs? P: Yes.
D: Has she received any recent jab? P: No
D: Is she feeding well? P: Yes. She is feeding very well.
D: Does she have any problems with her wee? P: No.
D: Have you noticed any tummy pain or change in her poo? P: No

D: Who looks after her? P: It’s me.


D: Does she go to school/part of a play group? P: Nursery

D: Do you know how to use the Epi-Pen?


P: No

Procedure:
1. Pull off the blue safety Cup. Grasp EpiPen in dominant hand, with thumb nearest
blue cap and form fist around EpiPen and pull of the blue safety cap. Remember:
‘Blue to the sky, orange to the thigh’.

www.aspire2plab.com 287
No real PLAB2 cases discussed. All discussed scenarios are fictitious and for educational purposes only.
2. Position Orange Tip. Hold the EpiPen at a distance of approximately 10cm away from
the outer thigh. The orange tip should point towards the outer thigh.
3. Jab Orange Tip. Jab the EpiPen firmly into the outer thigh at a right angle (90O angle).
Hold firmly against the thigh for 3 seconds. EpiPen should be removed and safely
discarded. The orange needle cover will extend to cover the needle.
4. Dial 999. Dial 999 and ask for an ambulance and state ‘anaphylaxis’.

Severe Symptoms:
● Difficulty/noisy breathing
● Swelling of throat and mouth
● Difficulty swallowing and/or speaking
● Wheeze or persistent cough
● Loss of consciousness and/or collapse
● Pale and floppy (young children)
● Feeling lightheaded/faint/confused
● Fast heartbeat
● Confusion and anxiety
● Blue skin or lips

How to use:
Lie down with your legs slightly elevated to keep your blood flowing or sit up if breathing is
difficult.
Pregnant women should lie down on the left side.
Place the patient in the recovery position if unconscious.

Remember:
1. Note down the time the Epi-pen was used, if there is no response within 5 mins use a
second Epi-pen if available.
2. Always carry 2 pens with you.
3. Discard the pen to pharmacy, ambulance or hospital staff.
4. If you are not sure that you are having anaphylaxis symptoms, then you can still take the
epi pen and follow the same procedure.

If your child goes to school, please inform the school nurse and teachers about her allergies
and provide them with epi-pens.

www.aspire2plab.com 288
No real PLAB2 cases discussed. All discussed scenarios are fictitious and for educational purposes only.
www.aspire2plab.com 289
No real PLAB2 cases discussed. All discussed scenarios are fictitious and for educational purposes only.
Urine Dipstick

You are an FY2 in Medicine. Nurse, Sarah Mills, has asked you to help with urine analysis.
Please teach her how to do urine analysis. Don’t let her do it.

Sarah: I want to know how to do the urine dipstick and how to interpret it.
D: OK

D: Name age sex of patient.


D: Frothy dark colour.

D: Open it and smell. If it has sweet odour, it is likely to be DM.

Wear gloves and apron.

1. What if this patient has red urine?


2. What if patient having protein?
3. If patient is having Sugar

You have to document everything.

2 tubes of dipstick (Confirm expiry)


Tissues
Urine sample.
Dustbin.
Alcohol wipes.

A urine dipstick test is a test of urine, using a special strip of paper that is dipped into a
sample of urine. The result is available almost immediately. It is sometimes called a rapid
urine test.

What is a urine dipstick test?


A urine dipstick test is the quickest way to test urine. It involves dipping a specially treated
paper strip into a sample of your urine. This can be done during your appointment with your
doctor, midwife or other health professional. The results are usually available within 60-120
seconds. Your sample doesn't need to be sent to a laboratory for a urine dipstick test,
although if the test is abnormal you might need to do another sample to be sent for further
testing in a laboratory.

What do you need to do for a urine dipstick test?


To have a urine dipstick test done you will need to give your medical professional a sample
of your urine. This needs to be in a sterile container. Sterile means it has no living germs
(bacteria) in it. Usually if you are asked to provide a sample, you will be given a sterile pot to
use. Your urine can still be tested for some things if it isn't in a sterile container, but the
results might not be as accurate and your urine can't be tested for infection.

www.aspire2plab.com 290
No real PLAB2 cases discussed. All discussed scenarios are fictitious and for educational purposes only.
To get your sample, you need to pee (urinate) straight into the pot. Ideally you shouldn't
pee into something else and then transfer it to the special pot, as then it won't be sterile
anymore. The best sample is one which is called a midstream specimen of urine (MSU). This
means that you start to pee then pass the open pot into the stream of urine and then take it
away again when you have caught enough urine. You don't stop peeing while doing this.
Tightly put the lid on the pot then wash your hands and the pot.

How is a urine dipstick test done?


Your health professional will use a urine testing strip to test your urine. There are several
different kinds of urine testing strips that test for different things in the urine. The strip has
squares on it that change colour in the presence of certain substances. The strip will then be
compared to a chart on the side of the urine testing strip package. Often the more intense
the colour change, the more of the substance there is in the urine.

Why is a urine dipstick test done?


A urine dipstick test is done for lots of different reasons, such as:
● A routine check when you're pregnant.
● To screen for diabetes.
● If you think you might have a urine infection.
● If you have tummy (abdominal) pain.
● If you have back pain.
● If you think you've seen blood in your urine (haematuria).
● Some people with diabetes will do their own urine dipstick testing.

The first thing that your health professional will do is look at your urine. Normal urine varies
in colour from colourless to dark yellow. Various things can affect the way your urine looks,
from how much you've been drinking to what you've eaten recently. The smell can also vary
widely and is not a good gauge of illness.

What are the results of a urine dipstick test?


A urine dipstick test can look for the following:
Blood
Not normally found in the urine.
May be present if there is an infection.
Can be due to significant underlying disease, such as cancer, in the bladder or kidney (and
the prostate gland in men).

In women, it may be contamination if you are on your period at the time of doing the
sample.

Will always need to be confirmed by sending the sample for a full laboratory test, as the
dipstick test can be positive when there's nothing wrong.

Protein
Protein is normally found in the urine in tiny amounts but these tiny amounts aren't usually
picked up on a urine dipstick test.

www.aspire2plab.com 291
No real PLAB2 cases discussed. All discussed scenarios are fictitious and for educational purposes only.
Protein may be found in larger amounts if there is an infection or if you're pregnant and are
developing pre-eclampsia.
Can be the first sign of kidney disease.

Glucose
Glucose, a type of sugar, is never normally found in the urine.
Glucose in the urine may be the first sign of diabetes.

Ketones
Ketones are chemicals made by the body when it's breaking down fat for energy, which is
what we do when we are starving and have used up all, or nearly all, of our stores of sugar
(glucose). This is normal.
Ketones are not usually found in the urine.
Ketones may be present if you have diabetes and it's out of control and the levels of sugar in
the blood are very high.
Ketones may also be present in women who are pregnant.

Bilirubin and urobilinogen


Bilirubin and urobilinogen are chemicals produced by the liver. They are not normally found
in the urine.
If bilirubin is found in the urine it usually means there's a problem with the liver.
Small amounts of urobilinogen may be found in the urine but large amounts suggest a
problem with the liver or with red blood cells being destroyed too quickly.

Leukocyte esterase and nitrite


These are both tests for the presence of infection.
Leukocyte esterase test looks for a reaction that only happens if there are lots of white cells
in the urine. White cells get into the urine when there is an infection.

Nitrite is not normally found in the urine but will appear there if there are lots of germs
(bacteria) in the urine.

Nitrite test can be negative even if there are lots of bacteria in the urine if the urine hasn't
been in the bladder for long. For example it will be negative if you do your urine sample
soon after having passed urine. Ideally you should wait at least four hours from the last time
you had a pee before doing your urine sample.
The urine dipstick test also measures how concentrated the urine is and how acidic it is.

www.aspire2plab.com 292
No real PLAB2 cases discussed. All discussed scenarios are fictitious and for educational purposes only.
Sub Cutaneous Injection

You are an FY2 in Medicine. Larry Jackson 3rd year medical student. Please teach him how
to put subcutaneous heparin injection. Don’t ask him to do the procedure.

D: How can I help you today?


S: Doctor, I want to learn how to give a subcutaneous heparin injection. Can you help me
with that?
D: OK. How much do you know about this procedure? S: I don’t know much.
D: Don’t worry, I’ll explain it to you. S: Ok.

Preparation and Introduction:


1. Introduce yourself to patient (Greet, Introduce, Identify and Explain Procedure)
2. Explain the purpose of the procedure.
3. Gain consent and offer a chaperone.

Subcutaneous (SC) injections pierce the epidermis and dermis of the skin to deliver
medication to the subcutaneous layer. It is a common route of delivery for medications such
as insulin and low molecular weight heparin (LMWH).

Things you may find inside:


Yellow Bin. (Labelled Needle Only)
A pen and paper
A kidney tray for other waste.
A kidney tray with the needle/alcohol swab/cotton/ labelled prefilled syringe.

1. Draw-up the appropriate medication into the syringe using a drawing-up needle. (You
may get prefilled syringe)

2. Remove the drawing-up needle and immediately dispose of it into a sharps bin, then
attach the needle to be used for performing the injection.

3. Choose an appropriate site for the injection:

● Abdomen: avoid injecting within a 2-inch radius around the umbilicus


● Upper outer aspect of the arm
● Outer aspect of the upper thigh
● Upper buttock
● Do NOT use a site that is scarred, inflamed, irritated or bruised.
● If multiple injections need to be administered, use different sites for each subsequent
injection. If frequent injections are administered, rotate injection sites.

4. Cleaning the site:

● Routine cleaning is not usually required prior to subcutaneous injection.


● In older patients and those who are immunocompromised, skin preparation using an
alcohol-impregnated swab (70% isopropyl alcohol) may be recommended.

www.aspire2plab.com 293
No real PLAB2 cases discussed. All discussed scenarios are fictitious and for educational purposes only.
5. Pinch a 5cm fold of skin between the thumb and index finger, using your non-dominant
hand (pinching the skin increases the depth of the subcutaneous tissue available).

6. Warn the patient of a sharp scratch.

7. Pierce the skin at a 45-90° angle, aiming to remain in the subcutaneous tissue layer. Insert
the needle quickly and firmly, with the bevel facing upwards.

8. Inject the contents of the syringe whilst holding the barrel firmly. Aspiration is not
recommended for subcutaneous injections, as there are no major blood vessels in the
subcutaneous tissue and the risk of inadvertent intravenous administration is minimal.5 You
should, however, always follow your local guidelines.

9. Remove the needle and immediately dispose of it into a sharps container.

10. Apply gentle pressure over the injection site with a cotton swab or gauze and avoid
rubbing the site.

11. Replace the gauze with a plaster.

12. Dispose of your gloves and equipment into an appropriate clinical waste bin.

Post-injection care:

● it is normal for the injection site to be sore for one or two days. Advise that if they
experience worsening pain after 48 hours they should seek medical review.
● Reiterate the potential complications of subcutaneous injections including haematoma
formation, persistent nodules, local irritation and rarely anaphylaxis.

Please document the details of the procedure and the medication administered.

www.aspire2plab.com 294
No real PLAB2 cases discussed. All discussed scenarios are fictitious and for educational purposes only.
8th Nerve Examination Teaching

You are an F2 working in neurology. David, your colleague, wants to learn 8th cranial
nerve examination. Talk to him and teach him and do not let him do the examination.

- Build rapport with colleague


- Assess his knowledge about the nerve and give him knowledge about the function of
8th nerve
- Greetings with the patient (Both mannequin and patient are there)
- Parts of 8th nerve examination:
● Inspection of ear
● Palpation of ear: temperature, tenderness, Tragus test
● Otoscopy on mannequin
● Hearing test: Rinne and weber
● Vertical and horizontal nystagmus
● Romberg test
● Marching test
● Gait and tandem gait

Arterial Blood Gas Teaching

You are an FY2 in Respiratory Medicine. Jamie Preston, final year medical student, has
missed his class for Arterial Blood Gas sampling. Please teach him and do not let him do
the examination.

Thyroid Teaching

You are an FY2 in Neurology department. Jamie Portsmouth, final year medical student,
has missed his class for thyroid examination. Please teach him and do not let him do the
examination.

Testicular Examination Teaching

You are an FY2 in Medicine department. Jamie Brighton, final year medical student, has
missed his class for testicular examination. Please teach him and do not let him do the
examination.

Breast Examination Teaching

You are an FY2 in Breast clinic. Jamie Sunderland, final year medical student, has missed
his class for Breast examination. Please teach him and do not let him do the examination.

www.aspire2plab.com 295
No real PLAB2 cases discussed. All discussed scenarios are fictitious and for educational purposes only.
Abdominal Examination

You are an FY2 in Medicine. Andy Russell is a 2nd year medical student, he has missed his
class for an abdominal examination. Teach him an abdominal examination. Do not let him
do the examination.

D: How can I help you today?


S: Doctor, I want to learn how to do an abdominal examination. Can you help me with that?
D: OK. How much do you know about this examination? S: I don’t know much.
D: Don’t worry, I’ll explain it to you. S: Ok.

Preparation and Introduction:


1. Introduce yourself to patient (Greet, Introduce, Identify and Explain Procedure)
2. Explain the purpose of the examination.
3. Explain that it will involve undressing fully from the chest to the thighs.
4. Gain consent and offer a chaperone.

General Inspection:
1. Inspect of the hands
2. Inspect the palms for Pallor, Palmar erythema and Dupuytren’s contracture
3. Inspect the nails for the Koilonychia and Leukonychia.
4. To assess for finger clubbing check for Schamroth’s window.
5. To assess for asterixis or flapping tremor.
6. Assess the patient’s radial pulse for Rhythm and volume.

www.aspire2plab.com 296
No real PLAB2 cases discussed. All discussed scenarios are fictitious and for educational purposes only.
Face

Eyes
• Conjunctival pallor: Ask the patient to gently pull down their lower eyelid.
• Jaundice: ask the patient to look downwards as you lift their upper eyelid.

Mouth:
Tell the patient to open his mouth and check for Angular stomatitis, Glossitis, Oral
candidiasis and Aphthous ulceration.

Neck
• Palpate the supraclavicular fossa on each side, paying particular attention to Virchow’s
node on the left for evidence of lymphadenopathy.

Chest
• Spider naevi:
• Gynaecomastia:
• Hair loss:

Abdominal inspection
Position the patient lying flat on the bed, with their arms by their sides and legs uncrossed
for abdominal inspection and subsequent palpation.

Inspect the patient’s abdomen for signs suggestive of gastrointestinal pathology


• Scars:
• Abdominal distension:
• Caput medusae:
• Striae (stretch marks):
• Hernias:
• Cullen’s sign:
• Grey-Turner’s sign:

Abdominal palpation
• Superficial Palpation:
Tenderness: note the abdominal region(s) involved and the severity of the pain.
• Deep Palpation:
Palpate the liver
Murphy’s sign
Palpate the spleen

Ballot the kidneys

Abdominal percussion
Percuss the liver
Percuss the spleen
Assess shifting dullness

www.aspire2plab.com 297
No real PLAB2 cases discussed. All discussed scenarios are fictitious and for educational purposes only.
Abdominal auscultation
Assess bowel sounds

Legs
Assess the patient’s lower legs for evidence of pitting oedema.

Further assessments and investigations


• Check hernial orifices
• Perform a digital rectal examination (PR)
• Perform an examination of the external genitalia

Students Concerns:

Peritonitis: It is an infection of the inner lining of the tummy.

Abdominal rigidity: It is stiffness of your stomach muscles that worsens when you touch, or
someone else touches, your abdomen. This is an involuntary response to prevent pain
caused by pressure on your abdomen.

Abdominal guarding: It is the tensing of the abdominal wall muscles to guard inflamed
organs within the abdomen from the pain of pressure upon them. The tensing is detected
when the abdominal wall is pressed.

www.aspire2plab.com 298
No real PLAB2 cases discussed. All discussed scenarios are fictitious and for educational purposes only.
Respiratory Examination

You are an FY2 in Medicine. Danial James is a 2nd year medical student. Teach him
Respiratory Examination. Patient came with ankle sprain surgery and is about to leave the
hospital. Patient has agreed to be the surrogate in this teaching.

D: How can I help you today?


S: Doctor, I want to learn how to do a respiratory examination. Can you help me with that?
D: OK. How much do you know about this examination? S: I don’t know much.
D: Don’t worry, I’ll explain it to you. S: Ok.

Preparation and Introduction:


1. Introduce yourself to patient (Greet, Introduce, Identify and Explain Procedure)
2. Explain the purpose of the examination.
3. Explain that it will involve undressing fully from the chest to the abdomen.
4. Gain consent and offer a chaperone.

General Inspection:
1. Inspect of the hands
2. Inspect the palms for Pallor, Palmar erythema, Nicotine tar staining, Peripheral cyanosis,
Skin changes or any rheumatological diseases.
3. Inspect the nails for the Koilonychia and Leukonychia.
4. To assess for finger clubbing check for Schamroth’s window.
5. To assess for asterixis or flapping tremor.
6. Assess the patient’s radial pulse for Rhythm and volume.
7. Check for fine or flapping tremor.

Head and neck:


There is no conjunctival Pallor.
Central cyanosis – bluish discolouration of the lips / inferior aspect of tongue

Jugular venous pressure (JVP): At 45°. Measure the JVP – number of centimetres measured
vertically from the sternal angle to the upper border of pulsation

Inspection of the chest


Look at the patient from foot end:
Patient is not coughing, no expiratory wheeze, no stridor.
Check for the respiratory rate as well.
Chest is moving bilateral symmetrical, no scars or skin changes and no deformities.

Palpation:
There are no Engorged neck veins.
Trachea is central in position.
Chest expansion is normal.
Apex beat: Normal position is 5th intercostal space – mid-clavicular line

www.aspire2plab.com 299
No real PLAB2 cases discussed. All discussed scenarios are fictitious and for educational purposes only.
Percussion
Percuss the following areas, comparing side to side:
1. Supraclavicular (lung apices)
2. Infraclavicular
3. Chest wall (3-4 locations bilaterally)
4. Axilla

Auscultation

Ask the patient to take deep breaths in and out through their mouth.

Assess quality:
Vesicular (normal)
Bronchial (harsh sounding – similar to auscultating over the trachea – inspiration and
expiration are equal and there is a pause between) – associated with consolidation

Added sounds:
Wheeze – asthma / COPD
Coarse crackles – pneumonia / bronchiectasis / fluid overload
Fine crackles – pulmonary fibrosis

Vocal resonance:
● Ask patient to say “99” repeatedly and auscultate the chest again
● Increased volume over an area suggests increased tissue density (especially if there is a
dull percussion note over the same area) – consolidation / tumour / lobar collapse
● Decreased volume over an area (especially if there is an associated dull percussion note)
suggests fluid outside of the lung (pleural effusion)

Ask patient to sit forwards

Lymph nodes:

Palpate the following areas:


Anterior and posterior triangles, Supraclavicular region and Axillary region

Assess the posterior chest:


Repeat inspection, chest expansion, percussion and auscultation on the posterior aspect of
the chest. We will Examine the sacrum for oedema and Examine the legs Pitting oedema.

Assess the calves for signs of deep vein thrombosis

Concerns:
What is abnormal breath sound?

www.aspire2plab.com 300
No real PLAB2 cases discussed. All discussed scenarios are fictitious and for educational purposes only.
News Chart

www.aspire2plab.com 301
No real PLAB2 cases discussed. All discussed scenarios are fictitious and for educational purposes only.
Glasgow Coma Scale

www.aspire2plab.com 302
No real PLAB2 cases discussed. All discussed scenarios are fictitious and for educational purposes only.
Examinations

1. Greeting
2. Introduction
3. Identification of the patient
4. Explain the examination
5. Exposure Adequate
6. Position
7. Pain – ask if patient is in pain
8. Chaperone
9. Consent
10. Contraindications

I will be as gentle and as quick as possible. At any point of time, you feel uncomfortable and
want me to stop please let me know, I will stop my examination.

www.aspire2plab.com 303
No real PLAB2 cases discussed. All discussed scenarios are fictitious and for educational purposes only.
Foot Examination

2. Tuning Fork
1. Neuro Pin

3. Orange Stick
4.. Reflex Hammer

www.aspire2plab.com 304
No real PLAB2 cases discussed. All discussed scenarios are fictitious and for educational purposes only.
Inspection:
a. Front of legs: redness, swelling, shiny skin, pigmentation, dryness, ulcers, hair loss.
b. Dorsum of the foot: hard corn, thick skin.
c. Toes: pallor, discolouration, amputation, in growing toenails.
d. Web-spaces: fungal infection, ulcers, and cracks.
e. Sole: callus, dry scaly skin and ulcers.
f. Heel: pressure sores
g. Back of the legs: redness, swelling, shiny skin, pigmentation, dryness, ulcers, hair loss.

Palpation:
a. Temperature:
1. We are looking for any local rise in the temperature.
2. Start from the dorsum of the foot and go towards the knee.
3. Use the dorsum of your hand on right and left, simultaneously.
4. Cool (PVD, Local ischemia), Hot (cellulitis)

Warm up your hands and ask the patient if he is in pain.


Please look at the patient’s face while doing it.

b. Tenderness:
1. We are looking for any kind of tenderness.
2. Check both the legs one by one.
3. If the patient feels pain, then stop and say sorry and take consent again.

NOTE: Please check tenderness one by one otherwise you won’t know in which leg the
patient is feeling tenderness.

c. Pulses:
1. We are looking for absent distal pulses.
2. Check for distal pulses.
Dorsalis pedis (lateral to extensor hallucis longus tendon)
Posterior tibial (posterior and inferior to medial malleolus)

d. Capillary refill time: (Normal CRT is <2 sec)


1. We are looking for prolongation of the capillary refill time.
2. Hold the big toe with two fingers and press the thumb for a few seconds and look for
capillary refill time. (If >2 sec- PVD)

Neurological Examination:
A. Sensory Examination.
B. Motor Examination.

A. Sensory Examination:
a. Fine touch:
1. We are looking for loss of fine touch sensation in the feet.
2. Give sternum as the reference point.

www.aspire2plab.com 305
No real PLAB2 cases discussed. All discussed scenarios are fictitious and for educational purposes only.
3. Tell the patient to close his eyes.
4. Start from the distal end and move proximally.

b. Pain touch:
1. We are looking for loss of pain touch sensation in the feet.
2. Give sternum as the reference point.
3. Tell the patient to close his eyes.
4. Start from the distal end and move proximally.

c. Vibration:
Different kinds of tuning Fork.
128 (vibrating) – Upper and Lower Limb Examination.
512 (buzzing) – Hearing test
256 (buzzing)

1. We are looking for loss of vibration sense in the feet.


2. Give sternum as the reference point.
3. Tell the patient to close his eyes.
4. Start from the ball of the big toe.
6. Next point will be medial malleolus, shin of tibia and tibial tuberosity.

d. Proprioception:
1. We are looking for joint position sense in the feet.
2. Hold the big toe and show the patient upward and downward movements of the big toe.
3. Then tell the patient to close his eyes.
4. Move the big toe in an upward and downward direction and stop and then ask the
patient if it is up or down.
5. Repeat it 2-3 times.
6. If the patient is unable to sense, then move upward to the next joint ankle, knee, and hip.

B. Motor Examination:
a. Bulk:
1. it can be checked with the measuring tape.

b. Tone:
1. Tell the patient to go floppy.
2. We are looking for hypotonia / hypertonia.
3. Rolling of the knees.
4. Raise the knee slightly with both the hand.
5. Check the ankle tone.

c. Power
1. Tell the patient to push against your force.
2. Check the power of Hip
- We are looking for Flexion – put your hand above the thigh.
- We are looking for Extension – put your hand below the thigh.
- We are looking for Adduction – put your hand on the outer side of the thigh.

www.aspire2plab.com 306
No real PLAB2 cases discussed. All discussed scenarios are fictitious and for educational purposes only.
- We are looking for Abduction – put your hand on the inner side of the thigh.

3. Check the power of Knee. (Tell the patient to bend the knee)
- We are looking for Extension – put your hand below the Shin.
- We are looking for Flexion – put your hand above the Calf.

4. Check the power of Ankle (Tell the patient to straighten the leg)
- We are looking for Dorsiflexion – put your hand on the dorsum of the foot.
- We are looking for Plantar flexion – put your hand at the sole of the foot.

Reflexes
1. Tell the patient to go floppy.
2. We are looking for Hyporeflexia & Hyperreflexia.
- Check the Knee reflex with the help of Hammer (L3 L4).
Hold the patient's knee with your left hand, feel for the tendon and tap the tendon with
your right hand.
Look at the quadriceps muscle (Do it on both legs).

- Check the Ankle reflex with the help of Hammer (L5 S1).
Bend the patient's knee, dorsiflex the ankle with one hand, feel for Achilles tendon and tap
the tendon. Hold the patient's knee with your left hand, feel for the tendon and tap the
tendon with your right hand.
Look at the calf muscle (Do it on both legs).

- Check the plantar reflex with orange stick.

Gait:
1. We are looking for Antalgic or high stepping gait.

To complete the lower limb examination, please check the footwear of the patient.
Further assessments including cranial nerves, upper limb and cerebellar examinations can
be done.

www.aspire2plab.com 307
No real PLAB2 cases discussed. All discussed scenarios are fictitious and for educational purposes only.
Upper Limb Examination

Neck Examination
Inspection: Redness, swelling, deformity, scar and wasting.

Palpation:
a. Look for Spinal or Right/Left Paraspinal tenderness.
b. Palpate spinal area with the thumb starting from C5 to T2.
c. Palpate paraspinal area with both the thumbs and go in Zig-Zag manner.

Upper limb examination:


Fine sensation
Pain Sensation
Tone
Power:
Reflexes:
Cervical X-ray: If not mentioned in the task.
Cervical Movements.

Fine Sensation
1. We are looking for loss of fine touch sensation in the arms.
2. Give sternum as the reference point.
3. Tell the patient to close the eyes.
4. Start from the distal end and move proximally.

www.aspire2plab.com 308
No real PLAB2 cases discussed. All discussed scenarios are fictitious and for educational purposes only.
Pain Sensation
1. We are looking for loss of pain touch sensation in the arms.
2. Give sternum as the reference point.
3. Tell the patient to close his eyes.
4. Start from the distal end and move proximally.

Tone:
a. Support the patient’s arm by holding their hand (Shake hand) and elbow.
b. Ask the patient to relax and go floppy.
c. Move the arm’s muscle groups through their full range of movements and check for tone
of shoulder, elbow, and wrist.
d. Check it in both the hands.
e. We are looking for movements if they are smooth or if there is some resistance.

Power:
Shoulder: Abduction and Adduction.
Elbow: Flexion and Extension
Wrist: Flexion and Extension
Finger Abduction: Spread your fingers, don’t let me squeeze.
Thumb Abduction: Point the thumb to the ceiling and don’t let me push them down.

Reflexes:
Upper limb reflexes: Biceps, triceps, and supinator

Cervical Movements:
Forward flexion: touch chin to chest
Backward: look up towards the ceiling.
Right rotation: look to the right
Left rotation: look to the left
Right lateral flexion: touch your ear to the shoulder
Left lateral flexion: touch your ear to the shoulder

NB:
Do neck x-ray before cervical movements.

www.aspire2plab.com 309
No real PLAB2 cases discussed. All discussed scenarios are fictitious and for educational purposes only.
Cerebellar Examination

Examination:

1. Speech
2. Nystagmus
a. Ask the patient to look straight ahead and examine the eyes. Look for any abnormal
movement such as nystagmus (To and fro movements)
b. Ask the patient to keep their head still and follow your finger with their eyes.
c. Move your finger throughout the various axis of vision (Horizontal and vertical axis)
d. We are looking for horizontal and vertical nystagmus.

3. Finger nose test:


a. Ask the patient to touch their nose with the tip of their index finger, then touch your
fingertip.
b. Position your finger at a distance so that the patient must fully outstretch their arm to
reach your finger.
c. Ask the patient to continue to do the finger to nose motion.
d. Then tell the patient to repeat it with their other hand.
e. We are looking for intentional tremor and past pointing or dysmetria.

NOTE:
Don’t confuse intentional tremor with action tremor.
Intentional Tremor: Tremor occurs when finger approaches the target.
Action tremor: Tremor occurs throughout the movements.

www.aspire2plab.com 310
No real PLAB2 cases discussed. All discussed scenarios are fictitious and for educational purposes only.
4. Dysdiadochokinesia

a. Demonstrate to the patient by patting the palm of your hand with the back/palm of your
other hand to the patient.
b. Ask the patient to do this rapid alternating movement.
c. Then tell the patient to repeat this movement on their other hand.
d. We are looking for irregular and slow movements.

5. Tone (Upper Limb)

6. Gait:
a. Ask the patient if she can stand independently.
b. A broad-based gait is present in cerebellar disease.
c. We are looking to see if the patient might veer towards the side of the lesion.

7. Tandem gait:
a. Ask the patient to walk in a straight line with their heels to their toes.
b. This test will exaggerate any kind of unsteadiness.

8. Romberg:
a. Ask the patient to put their feet together, keep their hands by their side.
b. Tell the patient to close the eyes.
c. It can differentiate sensory ataxia from cerebellar ataxia.

9. Heel to shin test:


a. Ask your patient to put the heel of one leg on the other knee, run it down the shin, lift it
off and repeat it again.
b. We are looking for incoordination.

Further assessments including cranial nerves, upper limb and lower limb examinations can
be done.

www.aspire2plab.com 311
No real PLAB2 cases discussed. All discussed scenarios are fictitious and for educational purposes only.
Meningitis

Glasgow Coma Scale

Eyes – open your eyes and keep them open for me.
For 2 – pain on sternum / supraorbital region
For 1 – No response.

Verbal
Do you know where you are?
5 - Hospital (Oriented)
4 - In a Park (a place but not correct one) (Confused)
3 - I have Cats (Inappropriate speech)
2 - No words / crying / moaning / mumble / incomprehensible words.
1 - No response

Motor:
6 - Raise your right arm, raise your left arm, raise your right leg, raise your left leg, Stick your
tongue out.
I’m going to squeeze you Thumb. (Give score according to response)

Neck stiffness (+/- examination)


It is Positive / Negative
Could you please touch your chin to the chest?
If no response, move to next.

Brudzinski’s (+/- examination)


Move Neck, Patient will bend Hip / Knees
Use both hands, look at legs to see the bending.
Positive with severe neck stiffness

Kernig’s sign (Knee)


Make 90 degrees with your right leg – slowly straighten it to check hamstring muscle – if he
moves the other leg in pain – positive.

Rash
Look for the rash on armpits, behind the ears, chest.
In dark skin patient, check on palms and soles.

Glass / Tumbler Test +/-


Blanching type rash (Disappear on pressure)

www.aspire2plab.com 312
No real PLAB2 cases discussed. All discussed scenarios are fictitious and for educational purposes only.
Eye Examination

www.aspire2plab.com 313
No real PLAB2 cases discussed. All discussed scenarios are fictitious and for educational purposes only.
Examination:

Hatpin Ishihara Chart Ophthalmoscope

NB.
1. Check for Glasses / Contact Lenses
- Check visual acuity with & without glasses both
- Check visual acuity without glasses only in case if patient is using reading glasses

2. Snellen Chart:
- Big chart - 20 feet distance.
- Small chart – 6 feet distance.
Instruct the patient to read Snellen’s chart from top to bottom with right and left eyes
individually.

Peripheral Visual field


Command: This is a white head pin, and I will make an imaginary X with the help of this pin.
You must look straight ahead on my nose and do not move your head. Let me know when
you see the tip of this pin and say Yes

Central Field of Vision.


It is a red pin. I am going to make a + sign. Look at my nose and do not move your head.
Cover your right eye. Say “disappear” when you can’t see it and “reappear” when you can.

Pupillary Reflexes:
Place the base of the thumb on the bridge of your nose (show) & look straight ahead.
Shine light on right eye – look at right eye (Direct).
Be at the same level as patient’s eyes when standing
Shine light on Right eye – look at Left eye (Consensual)

Tunnel Vision: It is a type of partial blindness where vision is restricted in a way that you can
only see in front of you.
Bi-Temporal Hemianopia: It is a type of partial blindness where vision is missing in the outer
half of both eyes.
Homonymous Hemianopia: It is a type of partial blindness where there is loss of half of the
field of view on the same side in both eyes.

www.aspire2plab.com 314
No real PLAB2 cases discussed. All discussed scenarios are fictitious and for educational purposes only.
Fundoscopy
Rule Out Glaucoma before doing Fundoscopy:
1. Have you ever been diagnosed with the medical condition called glaucoma?
2. Any pain at the back of your eyes?
3. Do you see any coloured halos around light?
4. Any red eye?

NB:
1. Do not drive or lift any heavy objects and sign any important documents after
putting dilating eye drops in the eyes.
2. Dim the light
3. To check the right eye of the patient, use your right hand and right eye.
To check the left eye of the patient, use your left hand and left eye.
4. Check the red reflex to see if the media is clear.

Fundus
Normal Fundus

Disc:
Clear, optic cup disc ratio is normal.
Central area of cup is pale.

Retina:
Normal, red/orange in colour.
Macula is dark with fovea as central pit

Vessels:
Originating from the disc are of normal
calibre (not tortuous).

Papilloedema
Diabetic Retinopathy

Disc:
Swollen, oedematous disc
Disc:Blurred disc margins
Elevated disc
Development of new vessels (NVD-new vessels
Paton’s
at the lines
disc, NVE-new vessels elsewhere)

Retina:
Retina:
Visiblehaemorrhages
Numerous small haemorrhages
are visible

Vessels:
Vessels:
Engorged
Dilated retinal and
veinstortuous retinal veins

www.aspire2plab.com 315
No real PLAB2 cases discussed. All discussed scenarios are fictitious and for educational purposes only.
Hypertensive Retinopathy

Disc:
Optic disc swelling (stage4/ malignant hypertension)

Retina:
Flame haemorrhages, hard exudates, cotton wool spots

Vessels:
Arteriolar narrowing, AV nicking, copper wiring, silver wiring

Stage 1-only diffuse AV narrowing


Stage 2- Stage 1 + focal AV nipping
Stage 3- Stage 2 + Haemorrhages
Stage 4- Stage 3 + hard exudates + retinal oedema + optic disc
swelling

Glaucoma

Disc
Enlarged cup to disc ratio (also called ‘cupping’), vertically
elongated, pale white in appearance, bayoneting of exiting
blood vessels

Retina
The retina looks normal

Vessels
The vessels look normal

Optic Nerve Atrophy

Disc
Well defined margins
Pale, chalky white in colour
Normal cup

Retina
The retina looks normal

Vessels
The retinal vessels are
normal

www.aspire2plab.com 316
No real PLAB2 cases discussed. All discussed scenarios are fictitious and for educational purposes only.
Diabetic Retinopathy

Disc:
Development of new vessels (NVD-new vessels
at the disc, NVE-new vessels elsewhere)

Retina:
Numerous haemorrhages are visible

Vessels:
Dilated retinal veins

CRAO

Disc
The disc appears normal

Retina
The retina looks pale and has a visible ‘cherry red’
spot

Vessels
Retinal blood vessels are attenuated and irregular.

CRVO

Disc
The disc is swollen and not easily visible due to haemorrhages.

Retina
Haemorrhages and cotton wool spots are visible.

Vessels
Retinal vessels are severely tortuous and dilated.

www.aspire2plab.com 317
No real PLAB2 cases discussed. All discussed scenarios are fictitious and for educational purposes only.
AMD

Disc
The disc appears normal.

Retina
Visible extensive yellow-white deposits known as ‘drusen’.

Vessels
The retinal vessels appear normal.

Retinitis Pigmentosa

Disc
The optic disc is pale (waxy pallor).
Retina
Retina is obscured by scattered pigmentation in
a bone spicule pattern (intraretinal pigmentary
migration).
Vessels
Retinal vessels are highly attenuated.

Image resource
https://www.hillrom.com/en/products/promotions/students/
(Educational material for medical students)

Fundoscopy Findings

Diabetic Retinopathy: 1-8


Normal Fundus: 9
Glaucoma: 10
Papilloedema: 11
Optic Atrophy: 12
Hypertensive Retinopathy: 14
CRVO: 15
CRAO: 16

www.aspire2plab.com 318
No real PLAB2 cases discussed. All discussed scenarios are fictitious and for educational purposes only.
Ear Examination

Normal

Tympanic membrane- pearly grey in colour, translucent, flat,


intact.

Light reflex/come of light- visible in the anterior inferior


quadrant.

Bony landmarks- short process, handle of malleus and umbo


visible.

NB:
1. Pull the Pinna: backward and upward (Downward, outward for children under 12 months)
2. Change the earpiece after examining one ear

www.aspire2plab.com 319
No real PLAB2 cases discussed. All discussed scenarios are fictitious and for educational purposes only.
Otitis Media

Bulging red T.M with a mucopurulent discharge in


the ear canal.

The TM may not be visible given the discharge and


associated ear canal swelling (secondary otitis
externa).

Air-fluid level visible, often with air bubbles.

Cone of light and bony landmarks diminished or


absent.

Cholesteatoma

Retraction pocket in the attic or postero-


superior quadrant of T.M (hallmark).

Accumulation of squamous debris in the


pocket created.

Bone destruction of adjacent bony ear


canal.

Otitis Externa

You may not be able to insert the otoscope in the ear at all
due to intense pain and swelling of the external ear canal.

Brighton Grading:
Grade 1:
Localised canal swelling, Mild pain, No hearing loss
Grade 2:
Debris in canal (not completely occluded), Erythematous ear
canal, T.M obscured partially
Grade 3:
Occluded canal, Oedematous & erythematous canal, T.M not
visible
Grade 4:
Pinna cellulitis, Perichondritis, Systemic signs

www.aspire2plab.com 320
No real PLAB2 cases discussed. All discussed scenarios are fictitious and for educational purposes only.
Perforation

Wax

Tympanosclerosis

Chalky white patches(plaques) on T.M


and middle ear structures

www.aspire2plab.com 321
No real PLAB2 cases discussed. All discussed scenarios are fictitious and for educational purposes only.
Foreign body

Can be visualised through the otoscope


Unsuccessful attempts to manually
remove the same can lead to T.M rupture
or otitis externa as well and result in
corresponding findings.

Keratosis obturans

Hyperdense, cerumen like material seen


in the external ear canal
Canal occluded
T.M not visible

Image resource
https://www.hillrom.com/en/products/promotions/students/
(Educational material for medical students)

Normal: 01 & 02
Ear Wax: 03
Acute Secretory OM: 08, 10 & 11
Resolving Secretory OM: 09
Perforation – ASOM: 12
Grommet: 16
Perforation: 19, 20, 21 & 22
Tympanosclerosis: 25
Cholesteatoma: 35 & 36
Foreign Bodies: 47

www.aspire2plab.com 322
No real PLAB2 cases discussed. All discussed scenarios are fictitious and for educational purposes only.
Thyroid Examination

Thyroid Examination
GPE - General appearance
- Hands (Palmer Erythema, Dry Skin, Sweating)
- Fine Tremors
- Pulse, Blood Pressure
Eye - Exophthalmos, Proptosis
- Lid Lag
- Eye Movements (H Test)
Thyroid - Inspection (Skin changes/Masses)
• Swallowing & Tongue Protrusion

- Palpation (Size/Consistency/Masses)
• Swallowing & Tongue Protrusion

- Percussion
- Auscultation

Special test - Pretibial Myxedema


- Proximal Myopathy
- Upper Limb Reflexes

Inspection:
General appearance:
- Anxiety, agitation

Hands:
Dry skin (hypothyroid), Increased sweating (hyperthyroid), Palmar erythema – reddening of
the palms at the thenar/ hypothenar eminences (hyperthyroidism)

Tremors:
Peripheral tremor can be a sign of hyperthyroidism.
1. Ask the patient to place their arms straight out in front of them.
2. Place a piece of paper across the backs of their hands
3. Observe for a tremor (the paper will quiver)

Pulse:
We are looking for bradycardia (Hypothyroidism) and tachycardia (Hyperthyroidism),
Rhythm – irregular (atrial fibrillation) – thyrotoxicosis

Eyes:
1. Inspect from the front, any redness / inflammation of the conjunctiva.

www.aspire2plab.com 323
No real PLAB2 cases discussed. All discussed scenarios are fictitious and for educational purposes only.
2. Inspect the sides (Exophthalmos: anterior displacement of the eye out of the orbit)
3. Inspect from behind (Proptosis)

Eye movements:
Diplopia – H Test
1. Ask the patient to keep their head still and follow your finger with their eyes only.
2. Move your finger in front of eye and make H.
3. Observe for restriction of eye movements and ask the patient to report any double
vision or pain

Lid lag:
1. Hold your finger high and ask the patient to follow it with their eyes, whilst keeping their
head still.
2. Move your finger downwards
3. Observe the upper eyelids as the patient follows your finger downwards

Bilateral exophthalmos is associated with Graves’ disease, caused by abnormal connective


tissue deposition in the orbit and extraocular muscles. Eye movement can be restricted in
Graves’ disease. Lid lag occurs because of the exophthalmos (Graves’ disease)

Thyroid:
Inspection
1. Skin changes, scars, masses, and swelling.
2. Note any swellings or masses in the area – assess size and shape
3. The normal thyroid gland should not be visible.
4. Ask the patient to swallow some water.
5. Ask patient to protrude their tongue.
6. Observe the movement of the mass

Masses embedded in the thyroid gland will move with swallowing. Thyroglossal cysts will
also move with swallowing. Lymph nodes will move very little.

Palpation
1. Stand behind the patient and ask them to slightly flex their neck (to relax the
sternocleidomastoids).
2. Place your hands either side of the neck.
3. Place the three middle fingers of each hand along the midline of the neck below the chin
4. Locate the upper edge of the thyroid cartilage (“Adam’s apple”)
5. Move inferiorly until you reach the cricoid cartilage/ ring
6. The first two rings of the trachea are located below the cricoid cartilage and the thyroid
isthmus overlies this area
7. Palpate the thyroid isthmus using the pads of your fingers Palpate each lobe of the
thyroid in turn by moving your fingers out laterally from the isthmus
8. Ask the patient to swallow some water, whilst you feel for symmetrical elevation of the
thyroid lobes (asymmetrical elevation may suggest a unilateral thyroid mass)
9. Ask the patient to protrude their tongue once more (if a mass is a thyroglossal cyst, it
will rise during tongue protrusion)

www.aspire2plab.com 324
No real PLAB2 cases discussed. All discussed scenarios are fictitious and for educational purposes only.
10. Note any deviation of the trachea – may be caused by a large thyroid mass

Lymph Nodes:
1. Submental lymph nodes.
2. Submandibular lymph nodes.
3. Anterior cervical chain (Tonsillar and deep cervical lymph nodes)
4. Posterior cervical chain
5. Pre auricular lymph nodes.
6. Post auricular lymph nodes.
7. Occipital lymph nodes.
8. Supraclavicular lymph nodes.

Percussion:
Percuss downwards from the sternal notch.
Retrosternal dullness may indicate a large thyroid mass, extending posterior to the
manubrium.

Auscultation:
Auscultate each lobe of the thyroid for a bruit.
A bruit would suggest increased vascularity, which occurs in Graves’ disease.

Special Tests
a. Upper limb reflexes: Biceps, triceps, and supinator
- (Biceps reflex – hyporeflexia is associated with hypothyroidism)

b. Pretibial myxoedema (Graves’ disease)


- Ask patients to roll up their trousers.
- Note for the raised, discoloured appearance over legs.

c. Proximal myopathy:
- Ask the patient to stand from a sitting position with arms crossed
- An inability to do this suggests proximal muscle wasting (Hyperthyroidism)

www.aspire2plab.com 325
No real PLAB2 cases discussed. All discussed scenarios are fictitious and for educational purposes only.
Cranial Nerve Examination

The 12 pairs of cranial nerves include: S = Sensory, M = Motor, B = Both


1. Olfactory Nerve (S)
2. Optic Nerve (S)
3. Oculomotor Nerve (M)
4. Trochlear Nerve (M)
5. Trigeminal Nerve (B)
6. Abducens Nerve (M)
7. Facial Nerve (B)
8. Vestibulocochlear Nerve (S)
9. Glossopharyngeal Nerve (B)
10. Vagus Nerve (B)
11. Accessory Nerve (M)
12. Hypoglossal Nerve (M)

Cranial Nerves: Only One Of The Two Athletes Felt Very Good, Victorious, And Healthy
Functions: Some Say Marry Money, But My Brother Says Big Brains Matter Most

Inspection:
General appearance – patient appears comfortable at rest
Facial asymmetries – there is no wasting of facial muscles, no asymmetry of angles of
mouth, nasolabial folds, or forehead wrinkles
Position of eyes – normal alignment, no ptosis (unilateral or bilateral)
Abnormality of speech or voice – there is no dysarthria
Signs around the bed – patient is not wearing/wearing any hearing aid/glasses

1) Olfactory
Taste & Smell
- Have you noticed any change in taste and smell?

2) Optic
Snellen Chart (Visual Acuity)
- Could you read these letters from Top to Bottom
Visual Field
- Use white head pin or wiggle finger
Pen Torch
- Direct/indirect
- Swinging light (MS)
Fundoscopy

3) Oculomotor + Trochlear and Abducens (3,4,6)


- Ptosis
- Diplopia (H test & Pain – Eye Movement)

5) Trigeminal (Sensory & Motor)


- Sensory - Fine Touch (Ocular, Maxillary, Mandibular) – do you feel the same on both sides?
- Motor - Mastication (Temporalis and Masseter) – clench your teeth – feel for muscle in
temporal area and masseter area)

www.aspire2plab.com 326
No real PLAB2 cases discussed. All discussed scenarios are fictitious and for educational purposes only.
7) Facial (Sensory & Motor)
- Sensory for taste
- Motor – Lower / Upper part of face
o Upper:
§ 1) raise eyebrows (surprise face),
§ 2) close eyes and don’t let me open them.
o Lower:
§ 1) Observation (nasolabial fold absent – palsy),
§ 2) cheeks inflate and deflate,
§ 3) smile (show me your teeth)

Facial nerve palsy caused by a lower motor neuron lesion presents with weakness of all
ipsilateral muscles of facial expression, due to the loss of innervation to all muscles on the
affected side. The most common cause of lower motor neuron facial palsy is Bell’s palsy.

Facial nerve palsy caused by an upper motor neuron lesion also presents with unilateral
facial muscle weakness, however, the upper facial muscles are partially spared because of
bilateral cortical representation (resulting in forehead/frontalis function being somewhat
maintained). The most common cause of upper motor neuron facial palsy is stroke.

8) Vestibular Cochlear
- Hearing – cover ears and rub your fingers
- Rinne and Weber

9&10) Glossopharyngeal & Vagus


- Torch and shine – central uvula = normal, if deviated = lesion on other side
- Say ahh - soft palate movement = normal, no movement = abnormal
- Gag reflex

11) Accessory
- Sternocleidomastoid – resist my hand (place on side)
- Trapezius – Shrug shoulder

12) Hypoglossal
- Stick out your tongue – deviation – to same side = same side lesion
- Atrophy & Fasciculation
- With your tongue push against my finger (cheek both sides – power check)

Further assessments including, upper limb and lower limb examinations can be done.

www.aspire2plab.com 327
No real PLAB2 cases discussed. All discussed scenarios are fictitious and for educational purposes only.
Testicular/ Inguino-scrotal Examination

Testicular Examination
Inspection - Penis, Groin, Scrotum, Abdomen
Palpation - Temperature
- Superficial & Deep Palpation
- Penis (Retract foreskin/ Glan penis)
- Epididymis - Posterior aspect (Tenderness = Epididymitis)
- Spermatic Cord - Superior aspect (Mass = Spermatocele)
Assess the mass - Site, Size, Shape, Tenderness, Consistency, Fluctuation, Transillumination, Cough
impulse, get above the swelling
Special test - Phrens test (Elevate testes – reduces pain = Epididymitis)
- Cremasteric Reflex (Lost in testicular torsion)
Varicocele - Palpable mass feels like a bag worms (usually on left side)

Examination
Inspection:
Inspection of genital region and the surrounding areas (Penis, Groin & Lower Abdomen)
There are no skin changes (rash, bruising, erythema, swelling), scars and obvious masses.
Inspection of the Scrotum: ask the patient to hold their penis out of the way to allow easier
inspection of the scrotum. Inspect the scrotum from the front and posterior sides.
There are normal scrotal rugosities, no skin changes, scar, obvious masses, swelling, sinuses
and necrotic tissue.

Palpation:
Temperature: compare both the testicles with the thigh.
(Palpate with thumb and index finger)
Superficial: check for tenderness in both the testicles.
Deep: palpate for spermatic cord, epididymis

Feel for any mass (site, size, shape, surface, consistency, contour, tenderness, mobile,
attached to underlying structure or not).

Special Tests
Cough Impulse:
Presence of cough impulse suggests hernia/varicocele.

Fluctuation Test:
Cystic, fluid filled masses fluctuate. Fluctuation is elicited by holding the mass firmly with
thumb and two fingers of both hands. Firmly press the mass with one finger while observing
for displacement of the other finger.

Transillumination Test:
Place a pen torch behind the scrotal swelling. (Transillumination suggests the mass is fluid
filled where there will be red glow – Hydrocele)

www.aspire2plab.com 328
No real PLAB2 cases discussed. All discussed scenarios are fictitious and for educational purposes only.
Phrens Test: If testicular pain is relieved by elevating the testes, this is suggestive of
Epididymitis.
Testicular Torsion (if pain is not relieved).

Cremasteric Reflex:
Stroke the patient’s medial thigh which leads to stimulation to cremaster reflex and elevate
the testicles. (Loss of Cremaster reflex may suggest Testicular Torsion).

I will complete the examination by doing Abdominal, Chest, Spine, Per-rectal and Lymph
Node Examination.

Inguino-scrotal Examination

4. The mid-inguinal point: It is halfway between the anterior superior iliac spine and the
pubic symphysis.

5. The midpoint of the inguinal ligament: It is halfway between the anterior superior
iliac spine and pubic tubercle.

6. The two openings to the inguinal canal are known as rings:

a) The deep (internal) ring is found above the midpoint of the inguinal ligament
which is lateral to the epigastric vessels.
b) The superficial (external) ring marks the end of the inguinal canal, and lies just
superior to the pubic tubercle.

www.aspire2plab.com 329
No real PLAB2 cases discussed. All discussed scenarios are fictitious and for educational purposes only.
www.aspire2plab.com 330
No real PLAB2 cases discussed. All discussed scenarios are fictitious and for educational purposes only.
Per rectal Examination

Per Rectal Examination


Inspection - Part the buttocks
- (Blood/ Haemorrhoids/ Fistulae/ Fissures/ Warts/ Skin Tags/
Excoriation)
- Rectal prolapse

Palpation - Lubricate the gloved Finger, Warn the patient before inserting the
finger.
- Hold your finger and pause until the Sphincter relaxes.
- Anal canal Ex (Do a 360* assessment to look for any masses/ Hard
stools/ Tenderness)
- Prostate Ex (Feel for prostate anteriorly and comment on size/
symmetry/ consistency)
- Anal Tone (Ask the patient to bear down on your finger)

- Withdraw your Finger and inspect for Blood and/or Mucous


- Clean the patient with paper towels or gauze and ask them to dress
up privately.

Explain the examination:


Rectal examination involves me inserting a gloved finger into your back passage to feel for
any abnormalities. It shouldn’t be painful, but it will feel a little uncomfortable. Could you
please lie down on your left side, with your knees lifted towards your chest? This is the
easiest position to examine your rectum. You can ask me to stop at any point.

BPH: Both the lobes or one lobe might be enlarged, and you may be able to feel the deep
sulcus.

Cancer: you might not be able median sulcus and one of the lobes might be enlarged.

www.aspire2plab.com 331
No real PLAB2 cases discussed. All discussed scenarios are fictitious and for educational purposes only.
Breast Examination

Inspection: While sitting:


1. Sit upright and put your hands on your thigh: Both the breasts are symmetrical, nipples
at the same level. There is no redness, swelling, visible mass, bruises and scars for any
surgery. There is no discharge, bleeding from the nipples. There is no peau d'orange.
2. Put your hands on your waist and bend forward: There is no visible mass on bending
forward.
3. Put your hands on the back of your head: There is no fullness in the supraclavicular and
axillary area.
4. Lift your breast with two fingers: There is no fungal infection in the infra-mammary area.
5. Squeeze your nipples with two fingers: There is no discharge or bleeding from the
nipples.

Palpation: While lying down at 45.


Temperature:
Compare all four quadrants of one breast with all the quadrants with the other breast.
Superficial Palpation:
Go anti-clockwise and check for any tenderness.
Deep palpation:
Again go anti-clockwise and find the mass and comment on the mass.
Periareolar Palpation:
Go anti-clockwise and use your thumb to find out the mass.

www.aspire2plab.com 332
No real PLAB2 cases discussed. All discussed scenarios are fictitious and for educational purposes only.
Antenatal Examination

I am here to examine your tummy for yours and your baby’s well-being. For the purpose of
examination, I want you to undress from below the breast to the mid-thigh. I will have a
chaperone with me throughout my examination to ensure your privacy. Please empty your
bladder before the examination. May I proceed?

Inspection:
Abdomen is uniformly distended. Umbilicus is centrally placed and everted. There are no
scars of any previous surgery, sinus, redness, swelling, distended veins and bruises. There
are no cutaneous signs of pregnancy like linea nigra and striae gravidarum.

Palpation:
Temperature: Compare 4 quadrants of the abdomen with the thigh.
Superficial palpation: Make S and look for any tenderness.
Grips:
Lateral grips: On the left lateral grip I can feel a curved structure which seems to be the back
of the baby. On the right lateral grip I can feel irregular structures which seem to be the
limbs of the baby.
Fundal grip: On fundal grip, I can feel the round, hard and globular structure which seems to
be the head of the baby.
Pelvic grip: On pelvic grip, I can feel round soft structures which seem to be buttocks of the
baby.
Fetal heart rate: I can feel fetal heart rate which seems to be regular.

www.aspire2plab.com 333
No real PLAB2 cases discussed. All discussed scenarios are fictitious and for educational purposes only.
Fundal height: Fundal height is 36 cm which corresponds to the period of gestation.

Comment on these 6 findings:


1. Lie
2. Presentation
3. Liquor Volume
4. No. of Fetus
5. Foetal Heart
6. Fundal Height

Findings: It is a single fetus, longitudinal lie, breech presentation and volume of liquor
seems to be adequate. I can feel fetal heart rate which seems to be regular. Fundal height is
36 cm which corresponds to the period of gestation.

www.aspire2plab.com 334
No real PLAB2 cases discussed. All discussed scenarios are fictitious and for educational purposes only.
Blood Sampling/ Blood Culture

Butterfly Needle

Blood Culture bottles:


Blue cap – Aerobic
Brown – Anaerobic Blood Culture Holder

www.aspire2plab.com 335
No real PLAB2 cases discussed. All discussed scenarios are fictitious and for educational purposes only.
Vacutainer Holder Vacutainer Needle

1. Explain the procedure


2. Sharp Scratch
3. Gentle / Quick
4. Repeat
5. Consent
6. Arm Preference
7. Arm Soreness
8. Vein Preference
9. Exposure

I am here to take a blood sample from one of the veins of your forearm with the help of a
needle. It will feel like a sharp scratch, I will be as gentle and as quick as possible. I may have
to repeat the procedure if I am unable to get in my first attempt. May I proceed? Do you
have any arm preference, vein preference or arm soreness? Could you please roll up your
sleeves?

www.aspire2plab.com 336
No real PLAB2 cases discussed. All discussed scenarios are fictitious and for educational purposes only.
IV Cannulation

I am here to insert a thin plastic tube in one of the veins of your forearm with the help of a
needle. It will feel like a sharp scratch, I will be as gentle and as quick as possible. I may have
to repeat the procedure if I fail in my first attempt. May I proceed. Do you have any arm
preference, vein preference or arm soreness? Please roll up your sleeves.

www.aspire2plab.com 337
No real PLAB2 cases discussed. All discussed scenarios are fictitious and for educational purposes only.
ABG Sampling

ABG Gloves
SAMPLING
Pre-heparinised arterial blood gas syringe and bung or cap

Arterial blood gas needle (23 G)

Alcohol wipe (70% isopropyl)

Gauze or cotton wool

Tape

Subcutaneous needle (25-27 G)

Sharps container

“I need to take a sample of blood from an artery in your wrist to accurately assess your
oxygen levels. The procedure will be a little painful, however, it should only take a short
amount of time. If you want me to stop at any point, just let me know. The procedure does
involve some risks which include bleeding, bruising, infection and very rarely permanent
damage to the artery being sampled from. “

If the patient is currently receiving oxygen therapy, note the oxygen delivery device and
flow rate.

www.aspire2plab.com 338
No real PLAB2 cases discussed. All discussed scenarios are fictitious and for educational purposes only.
Catheterisation

I am going to insert a thin flexible tube in your bladder through your penis to relieve your
retention. I will be as gentle and as quick as possible. To examine, I want you to undress
below your waist and I have a chaperone with me to ensure your privacy. I want you to lie
down on your back with your legs slightly apart. May I proceed?

www.aspire2plab.com 339
No real PLAB2 cases discussed. All discussed scenarios are fictitious and for educational purposes only.
Per Speculum and Pap Smear

www.aspire2plab.com 340
No real PLAB2 cases discussed. All discussed scenarios are fictitious and for educational purposes only.
Spacer

Small - Red: 0-18 months


Medium - Yellow: 1-5 years
Large - Blue: >5 years

www.aspire2plab.com 341
No real PLAB2 cases discussed. All discussed scenarios are fictitious and for educational purposes only.
Epi Pen

Sign of Anaphylaxis:

• Difficulty/noisy breathing
• Swelling of throat and mouth
• Difficulty swallowing and/or speaking
• Wheeze or persistent cough
• Loss of consciousness and/or collapse
• Pale and floppy (young children)
• Feeling lightheaded/faint/confused
• Fast heartbeat
• Confusion and anxiety
• Blue skin or lips
Epi Pen:
- Easy to read instructions
- Elliptical shape (for better handling)
- Never see needle
- Carry 2 Epi Pen
- Inspection window to check the medicine is clear and free
of particles
• If the medication is pink/brown/cloudy or contains any
solid particles means:
• the medication has decade and injector should not be
used even if it is not expired.

Remember:
1. Note down the time the Epi-pen was used, if there is no response within 5 mins use
a second Epi-pen if available.
2. Always carry 2 pens with you.
3. Discard the pen to pharmacy, ambulance or hospital staff.
4. If you are not sure that you are having anaphylaxis symptoms, then you can still take
the epi pen and follow the same procedure.

• Note the time the adrenaline auto injector was given.


• Lie the patient flat and legs raised. If vomiting lie on side. If breathing difficult allow
to sit. Do not allow to stand or walk.
• ANA-FIL-AX-IS
• Call parents or Emergency Contact
• If no improvement or there is a deterioration in condition give 2nd pen after 5 mins.
Commence CPR if there are no signs of life.
• If wheezy/asthmatic give inhaler via spacer if available.

www.aspire2plab.com 342
No real PLAB2 cases discussed. All discussed scenarios are fictitious and for educational purposes only.
Urine Dipstick

Initial assessment of urine


Sample bottle should have the correct details including name, date of birth and the hospital identification number.

Inspect the colour of the urine:


Normal Urine: Straw-coloured à indicates healthy hydrated person.
Dark concentrated urine: Indicates dehydration.
Red urine: indicates macroscopic haematuria, porphyria, and certain drugs like rifampicin and certain foods such as beetroot.
Brown urine: indicates jaundice due to presence of bile pigments and certain medications such as chloroquine.

Inspect the clarity of the urine:


Clear urine: indicates healthy hydrated person.
Cloudy urine: indicates urinary tract infection, renal stones, kidney issues (e.g. nephrotic syndrome).
Frothy urine: indicates significant proteinuria

Assessing urine odour:


Offensive odour: indicates urinary tract infection.
Sweet odour: indicates glycosuria (e.g. diabetes mellitus).

www.aspire2plab.com 343
No real PLAB2 cases discussed. All discussed scenarios are fictitious and for educational purposes only.
Sim Mannequin

www.aspire2plab.com 344
No real PLAB2 cases discussed. All discussed scenarios are fictitious and for educational purposes only.
Asthma

You are FY2 in A&E. James Rodriguez, aged 40, came with acute shortness of breath.
Please talk to the patient, assess his condition, examine him and discuss the initial plan of
management with the examiner.

D: What brought you to the hospital? P: I can’t breathe, my asthma is killing me.
D: When did it start? P: It started 2 hours ago
D: How did it start? P: I was sitting at home and it started suddenly.

Reassure
D: Let me quickly have a look at your vitals

Note: Saturations are decreased, and Respiratory rate is increased

D: Start ABCDE approach immediately

A - Airway Patient is talking to me. His airway is patent.


I would like to start my patient on high flow O2 – 15L/min
via non-rebreather mask.
B - Breathing Trachea and Chest Examination
Examination:
- Trachea is central and there is bilateral wheeze on
chest Examination
Management:
- I would like to give salbutamol and ipratropium
bromide nebulisers to my patient, preferably every 15-
20 mins driven by O2.
C - Circulation Peripheral signs of perfusion:
- Pulse,
- Cold peripheries,
- Cap refill,
- Cyanosis,
- Pallor,
- Active bleeding site.
Management:
- HR & BP stable. Insert one IV cannula and draw some
bloods at the same time for routine investigations
including blood sugars.
D - Disability Patients GCS seems to be 15/15.
E - Exposure Expose the patient fully for Head-to-Toe examination. Look
for any redness, rashes, swelling, temperature.

• Summarise your initial assessment: ABCDE – O2 and nebulisers. Ask the patient if he
feels any better.
• Give the diagnosis and definitive management for the diagnosis: Asthma exacerbation
and I would like to give steroids.

www.aspire2plab.com 345
No real PLAB2 cases discussed. All discussed scenarios are fictitious and for educational purposes only.
Start taking history NOW, if the patient’s vitals are stable.

Note: If the patient deteriorates while you are taking history, reassess the patient using ABCDE
approach and call your senior to review and consider further plan of management – IV
Magnesium, IV Salbutamol or IV Aminophylline.

Inform ITU if you suspect life threatening or near fatal asthma.

History to take:

D: Has it changed since it started? P: It is getting worse.


D: Do you have any other symptoms? P: No
D: Do you have a cough? P: No
D: Do you have chest tightness? P: No
D: Do you have wheeze? P: I don’t know.
D: Have you experienced any heavy pain in your chest? P: No (ACS)
D: Do you have any pain, redness, hotness or swelling in your legs? P: No (P.E)
D: Do you have any fever, flu-like symptoms? P: No (Pneumonia)

D: Has it happened before? P: No/Yes

D: When were you diagnosed with asthma? P: Since childhood.


D: How was it managed? P: I was taking blue and brown inhalers.

D: Did you use your inhaler to relieve your shortness of breath today?
P: I did, but it didn’t work.

D: Do you use it regularly as prescribed? P: Yes


D: How often do you use the blue inhaler? P: Once / twice a day recently.

D: Do you have any other medical conditions? P: No


D: Any high blood pressure, diabetes or any heart conditions? P: No
D: Are you taking any other medications other than blue and brown inhalers? P: No
D: Have you been using any over the counter painkillers such as ibuprofen? P: No
D: Do you have any allergies? P: No
D: Have you been hospitalized before? P: No
D: Have you had any surgeries in the past? P: No

D: Has any member of your family ever been diagnosed with any medical conditions? P: No

D: Do you smoke? P: No
D: Are you physically active? P: I am not quite active.
D: What do you do for a living? P: I am an estate agent.
D: Have you travelled recently? P: No.

www.aspire2plab.com 346
No real PLAB2 cases discussed. All discussed scenarios are fictitious and for educational purposes only.
D: How is your living status? P: It’s fine
D: Do you have any pets or carpets at home? P: No

Inspection:
Chest: symmetry of chest wall movements.
Palpation:
Chest: chest expansion
Percussion:
Hyper-resonance or Dullness
Auscultation:
Wheeze is present all over the chest.
PEFR:
D: I would like to do PEFR.

MANAGEMENT:
Severe Asthma if any of:

1. PEFR 33% - 50%.


2. Respiratory rate > 25
3. PR >110
4. Unable to complete a sentence in one breath.
5. Using accessory muscles.

Management:
1. High flow oxygen:
40% - 60% (6 L/min) up to 100% (15 L/min) to maintain saturation of 94% - 98%.
2. Nebulized salbutamol 2.5 – 5mg every 5 – 15 minutes,
3. Ipratropium bromide (Atrovent) 500 microgram.
4. Corticosteroids:
Prednisolone 40 mg PO or Hydrocortisone 1-200 mg iv.

Involve Senior:
1. Magnesium 2 gm iv over 20 minutes
2. Salbutamol 250 microgram iv bolus.
3. Salbutamol infusion 5 – 20 microgram / min.

Alternative:
1. Aminophylline 5mg/kg iv over 20 minutes loading (unless on oral therapy)
2. Aminophylline 0.5 – 0.7 mg/kg/hour
If patient is on oral aminophylline or theophylline, check blood levels on admission and daily
if infusion started.

www.aspire2plab.com 347
No real PLAB2 cases discussed. All discussed scenarios are fictitious and for educational purposes only.
Anaphylaxis

You are FY2 in surgery. Mr Kane West, aged 55, had undergone surgery on the abdomen for
burst appendix. Surgery team decided to give two units of blood. He was given one unit of
blood already. While he was receiving a second unit of blood, he experienced shortness of
breath. Your nurse colleague was concerned about the patient and asked you to talk to the
patient. Please talk to the patient, assess him and do relevant management.

Inside the cubicle, sim-man is lying down on a couch wearing a gown. You can see blood
attached to it. There are 2 masks, one with a bag attached and one with a tubing. Sim-man
is catheterised as well. There is a bag of blood that is connected, and transfusion is taking
place at the moment. There is also a bag of IV fluid, adrenaline, and colloid on the table
nearby. Sometimes you may find a wrist band showing penicillin allergy.

D: How can I help you?


P: Help me, I can’t breathe. (Breathing heavily).
D: Tell me what exactly happened?
P: I had a surgery for my appendix, and I was receiving blood after that. After that, my lips
and tongue have swollen up. My hands are itchy.
D: Since when you are having this problem? P: From last 20 minutes.
D: Has it changed ever since it started? P: It was getting worse.
D: Any other symptoms P: Yes, I feel my tummy is bloated.

Please look at the monitor:

Monitor findings:
ECG-normal
Oxygen- 85-88%
Pulse-92
BP- 90/50
RR->25

Note: Patient is hypotensive, tachycardic and tachypnoeic.

D: Examiner, I would like to stop blood because I suspect my patient has some reaction to it.
Examiner: Ok, stop it.

D: I would like to call for help and ask my colleague to arrange adrenaline immediately
whilst I proceed with ABCDE approach.

A:
- Patient is talking to me. His airway is patent.
- I would like to start my patient on high flow O2 – 15L/min via non-rebreather mask.
- Pick the oxygen mask with a reservoir bag attached. (Saturation will improve)

B:
- Trachea and Chest Ex

www.aspire2plab.com 348
No real PLAB2 cases discussed. All discussed scenarios are fictitious and for educational purposes only.
- Trachea is central and Chest Ex is Normal.
- Chest was also undressed showing rash which was basically red.

C:
- Peripheral signs of perfusion - Pulse, Cold peripheries, Cap refill, Cyanosis, Pallor, Active
bleeding site.
- Mx: HR & BP unstable. I would like to Insert TWO large bore IV cannulas and start
resuscitating my patient by giving IV Fluids. I would also like to draw some bloods at the
same time for routine investigations including blood cultures, lactate, and blood sugars.
- Fluid Challenge: Adults - 500 mL of warmed crystalloid solution (0.9% saline) in 5-10
minutes if the patient is normotensive or 1 L if the patient is hypotensive).

D:
- As my patient is talking to me and is obeying commands, her GCS seems to be 15/15.

E:
- Expose the patient fully for Head-to-Toe examination. Look for any redness, rashes,
swelling, temperature - 39. Start patient on IV paracetamol. (Might see a band on
patient’s wrist saying allergic to Penicillin – In that case ask if the patient has received
any Abx)

D: I would like to examine your tummy as well. P: Okay Dr.


D: How are you feeling now? P: Better doctor.

Summarise your initial assessment:


▪ ABCDE – O2 and IV Fluids. Ask the patient if he feels any better.

Give the diagnosis and definitive management for the diagnosis:


▪ You seem to be having an Anaphylactic reaction which is an allergic
reaction to the blood which you are receiving. I have stopped the
blood and given you some O2 and IV Fluids.

**I would like to immediately give adrenaline (0.5ml IM (500 micrograms) 1:1000 titration)
to my patient.

I would like to reassess my patient and give another dose of adrenaline if she doesn’t
improve.

I would like to send the blood you are receiving to the lab to cross match and see if you are
receiving the correct blood or not. If it is not the correct blood, then I will have to fill an
incident form.

EX: Ok.
D: Are you feeling better
P: Yes, Dr much better.

www.aspire2plab.com 349
No real PLAB2 cases discussed. All discussed scenarios are fictitious and for educational purposes only.
(If patient is not feeling better then we need to auscultate the chest and further treatment
with a bronchodilator, such as salbutamol (inhaled or IV), ipratropium (inhaled),
aminophylline (IV) or magnesium sulfate (IV - unlicensed indication). Magnesium is a
vasodilator and can compound hypotension and shock)

Start taking history NOW, if the patient’s vitals are stable.

D: Any other symptoms? P: No.

D: Have you been diagnosed with any medical condition in the past? P: No
D: Are you taking any medications including OTC or supplements?
P: Yes, in the hospital, I was given many medications.

D: Any allergies from any food or medications? P: Yes, I am allergic to penicillin.


D: Did you receive any penicillin in the last hour? P: I don’t know.
D: Did you receive any medication recently? P: Nurse gave some a couple of minutes ago.
D: Any previous hospital stays or surgeries? P: No
D: Has anyone in the family been diagnosed with any medical condition? P: No
D: Any other thing that you would like me to know? P: No

D: We are suspecting that you had a condition called anaphylaxis reaction. It is an allergic
reaction that happens when a foreign object enters the body and our body’s defence system
releases a substance to fight against it.

We will send Blood to the lab for further investigation, for Blood grouping and
incompatibility, FBC, LFT, U&Es, Creatinine, ABG, bleeding and Clotting screen. We will do an
ECG and keep an eye on the vitals.

We will also check for blood in the urine.

I will check your notes to see what kind of antibiotics were given to you after the surgery. If
it belongs to the penicillin group, then that could explain the symptoms. However, I need to
check if there has been any mismatch of blood as well.
I sincerely apologise for all you have been going through.

We have a system in our hospital in such situations, I will document about this incident in
your notes, I will inform my consultant and I will fill an adverse report form (Incident form)
to let the hospital authorities know about the incident. In this way, the hospital authorities
can act promptly to reduce the risk of further incidents and improve the service we provide
in the NHS. These incidents are reported nationally as well in order prevent them happening

www.aspire2plab.com 350
No real PLAB2 cases discussed. All discussed scenarios are fictitious and for educational purposes only.
Post UTI Sepsis Shock

You are an FY2 in the Medicine Dept. Mrs Rachel Thompson, aged 78, was admitted to the
hospital with UTI 3 days ago. Nurse has called you and says that the patient is feeling
poorly. She says that you are the only doctor available to see her. Talk to the patient,
assess her and manage her situation appropriately.

D: Hello, Mrs. Thompson, how are you doing today? (Introduce yourself)
P: I cannot breathe.

D: May I know since when? P: 2 hours


D: How did it start? P: It just started Dr.

D: I am sorry to hear Mrs. Thompson. Let me quickly examine you and see what’s going on.
P: Okay Dr.

D: I would like to check my patients’ vitals. (Look at the monitor)


RR 24
SpO2 88
Pulse 96
BP 105/65
Temp 39.0

Note: Patient is hypotensive and tachycardic. Her temperature is raised.

Start ABCDE approach immediately

A:
- Patient is talking to me. His airway is patent.
- I would like to start my patient on high flow O2 – 15L/min via non-rebreather mask.

B:
- Trachea and Chest Ex
- Trachea is central and Chest Ex is Normal. (You may find crackle sometimes).

C:
- Peripheral signs of perfusion - Pulse, Cold peripheries, Cap refill, Cyanosis, Pallor, Active
bleeding.
- Mx: HR & BP unstable. I would like to Insert TWO large bore IV cannulas and start
resuscitating my patient by giving IV Fluids. I would also like to draw some bloods at the
same time for routine investigations including blood cultures, lactate, and blood sugars.
- Fluid Challenge: Adults - 500 mL of warmed crystalloid solution (0.9% saline) in 5-10
minutes if the patient is normotensive or 1 L if the patient is hypotensive).

D:
- As my patient is talking to me and is obeying commands, her GCS seems to be 15/15.

www.aspire2plab.com 351
No real PLAB2 cases discussed. All discussed scenarios are fictitious and for educational purposes only.
E:
- Expose the patient fully for Head-to-Toe examination. Look for any redness, rashes,
swelling, temperature - 39. Start patient on IV paracetamol.

D: I would like to examine your tummy as well. P: Okay Dr.

Examiner: Abdominal Exam - Normal

D: How are you feeling now? P: Better doctor.

Summarise your initial assessment: ABCDE – O2 and IV Fluids. Ask the patient if he feels any
better.

Give the diagnosis and definitive management for the diagnosis: Septic Shock on background
of UTI which is a common complication. I would like to start SEPSIS 6 immediately.

Give 3 and Take 3. I would like to give broad spectrum antibiotics to my patient according to
hospital protocol and change the Abx according to culture and sensitivity report. I would also
like to monitor urine output and send urine for culture.

I have already sent for lactate and blood cultures. I would like to insert a urinary catheter to monitor
hourly urine output.

I would also send urine for culture and sensitivity and order a CXR to look for other sources of sepsis.

Start taking history NOW, if the patient’s vitals are stable.

D: You were admitted with a UTI, is that right? P: Yes


D: Do you have any burning sensation while passing urine? P: No
D: Any tummy pain? P: Yes/No
D: Any previous medical conditions? P: Yes/No
D: Any regular medications? P: Yes/No
D: Any allergies to any food or drugs? P: No

SEPSIS SCREEN: (Find out source of sepsis)


● Bloods for FBC, CRP, Renal and Liver function tests
● Bloods Culture
● Urine Dip and Culture
● CXR
● Sputum culture (If cough + phlegm present)

SEPSIS SIX:
GIVE: TAKE:
Oxygen Blood cultures
IV Fluids Lactate
IV Antibiotics Urine Output

www.aspire2plab.com 352
No real PLAB2 cases discussed. All discussed scenarios are fictitious and for educational purposes only.
D: From my assessment, I think you have a condition called Septic Shock. This means that
the infection from your urinary tract has spread to all over your body through blood.

D: I have given you O2 as your oxygen levels are low in your blood. I have given you fluids
and paracetamol through your vein as your BP is low and you have high temp.
I will arrange for some blood and urine tests and a Chest X-Ray.

D: I will have to give you stronger antibiotics through your vein for your condition. Is that
oaky with you?
P: Yes doctor.

D: I will inform my senior and Intensive Care team to come and review you.

www.aspire2plab.com 353
No real PLAB2 cases discussed. All discussed scenarios are fictitious and for educational purposes only.
UTI after TURP

You are FY2 in A&E. Mr Rodrey Pink, aged 75, was brought in the hospital by his wife as he
was feeling sick. Please talk to the patient and address his concerns.

Urinary catheter was attached to the Simman and he was attached to the monitor.

Monitor:
ECG - Normal
Pulse - 110/min
Blood Pressure - 100/70 mm hg,
SPO2 - 97%,
Temperature - 38 C

NOTE:
- If vitals are stable, go with History. Diagnose as UTI and admit and give management
plan for UTI.
- If the vitals are unstable and you suspect septic shock as patient’s temperature is high,
go with ABCDE approach.

SCENARIO 1: Patient NOT in shock

D: How can I help you? P: I am not feeling well.


D: Could you please tell me more?
P: I am feeling very tired. I haven’t passed urine and it was very uncomfortable.
D: For how long are you having this problem? P: Since yesterday

D: Do you have any other symptoms? P: Yes, I feel feverish from last few days.
D: Did you take anything for that? P: I tried PCM it helped a bit.
D: How much did you take? P: I took 2 yesterday.

D: Do you have any other symptoms? P: No

D: Did you have tummy pain? P: Yes doctor.


D: Was it continuous or comes and goes? P: It is continuous.
D: Was it sudden or gradual? P: it is gradual & getting worse
D: What type of pain is it? P: It is dull pain.
D: Does the pain go anywhere? P: No
D: Is there anything that makes the pain worse? P: It is getting worse.
D: Could you please score the pain on a scale of 1 to 10, where 1 being no pain and 10 being
the most severe pain you have ever experienced? P: around 5

D: Any changes in your urine colour or smell? P: Yes, it is smelly and cloudy
D: Any blood in it? P: No
D: Any nausea and vomiting? P: Yes, but I didn’t vomit.
D: Any cough? P: No
D: Any problem with the bowel? P: No

www.aspire2plab.com 354
No real PLAB2 cases discussed. All discussed scenarios are fictitious and for educational purposes only.
D: Have you noticed any weight loss? (Cancer) P: No
D: Any tiredness or SOB? P: No.
D: Someone your friends or family told you are losing weight? P: No
D: How is your appetite? P: It’s fine doctor.
D: Tell me about your diet? P: I try to eat healthy, mostly fruits and vegetables

D: Do you have to go to the loo more often these days?


P: I was going to the loo more since last few weeks. I had operation TURP 4 days ago for my
prostate.
D: How did the operation go?
P: It went well, I was in the hospital for two days and then I was sent home.
D: Any complications during the surgery? P: No

D: Have you been diagnosed with any other medical condition in the past? P: No
D: Any diabetes, high blood pressure, high cholesterol, kidney or heart disease P: No.
D: Are you taking any medications including OTC or supplements? P: No
D: Any long-term antibiotics or steroids? P: No
D: Any allergies from any food or medications? P: No
D: Has anyone in the family been diagnosed with any medical condition? P: No

D: Do you smoke? P: Yes/ No


D: Do you drink alcohol? P: Occasionally
D: Whom do you live with? P: I live with my wife.

D: I would like to check your vitals, examine your tummy. I will be having a chaperone with
me to ensure your privacy.

Examiner: Do it doctor.

Remove the hospital gown and the shorts. (Don’t forget to cover in the end)

Urine catheter attached to the urine bag showing yellowish turbid urine with pus collection
+/- Blood.

Abdomen: Tenderness over suprapubic area.

Examiner: What do you think is going on with the patient?

D: I am suspecting my patient is having urinary tract infection probably after TURP as it is


one of the most common complication of this surgery.

Examiner: What would you like to do for the patient?


D: I would like to send for some initial investigations routine blood test including kidney
function test, urine dip. We would like to take urine sample and send for culture and
sensitivity.

We will do ABG as well.

www.aspire2plab.com 355
No real PLAB2 cases discussed. All discussed scenarios are fictitious and for educational purposes only.
Examiner: Here is your ABG
ABG shows metabolic acidosis.
PH - low,
HCO3 - low
CO2 – Normal

Management:
(Follow SEPSIS 6 if patient is in septic shock)
We will give him some broad-spectrum antibiotics. We may also change antibiotics
according to the bug that is causing the infection.
We will give PCM and painkiller for the temperature and pain
We will give anti-sickness medication for nausea.
We will consider giving him IV fluids.
We will inform my seniors and I will arrange for USG of abdomen and prostate.
General advice:
Place a hot water bottle on your tummy, back or between your thighs
Rest and drink plenty of fluids – this helps your body to flush out the bacteria

NOTE:
Sometimes this patient is planned for the surgery and is having urine retention. Patient was
catheterized and posted for the surgery.

SCENARIO 2: Patient is IN shock

D: How can I help you? P: I am not feeling well.


D: Could you please tell me more?
P: I am feeling very tired. I haven’t passed urine and it was very uncomfortable.

D: For how long are you having this problem? P: Since yesterday

Reassure
D: Let me quickly have a look at your vitals

Note: Patient is hypotensive and tachycardic. His temperature is raised.

D: Start ABCDE approach immediately

A:
- Patient is talking to me. His airway is patent.
- I would like to start my patient on high flow O2 – 15L/min via non-rebreather mask.

B:
- Trachea and Chest Ex
- Trachea is central and Chest Ex is normal

C:

www.aspire2plab.com 356
No real PLAB2 cases discussed. All discussed scenarios are fictitious and for educational purposes only.
- Peripheral signs of perfusion - Pulse, Cold peripheries, Cap refill, Cyanosis, Pallor, Active
bleeding site.
- Mx: HR & BP unstable. I would like to Insert TWO large bore IV cannulas and start
resuscitating my patient by giving IV Fluids. I would also like to draw some bloods at the
same time for routine investigations including blood cultures, lactate, and blood sugars.
- Fluid Challenge: Adults - 500 mL of warmed crystalloid solution (0.9% Normal Saline) in
5-10 minutes if the patient is Hypotensive or 1 L if the patient is in shock). I would like to
reassess after the bolus and give further boluses of 500 ml fluids until a maximum of 2L
in the first one hour.
-

D:
- As my patient is talking to me and is obeying commands his GCS seems to be 15/15.

E:
- Expose the patient fully for Head-to-Toe examination. Look for any redness, rashes,
swelling, temperature – 38.9. I would like to start IV paracetamol.

Summarise your initial assessment: ABCDE – O2 and IV Fluids. Ask the patient if he feels any
better.

Give the diagnosis and definitive management for the diagnosis: Septic Shock on
background of UTI which is a common complication of TURP procedure. I would like to start
SEPSIS 6 immediately.

Give 3 and Take 3. I would like to give broad spectrum antibiotics to my patient according to
hospital protocol and change the Abx according to culture and sensitivity report. I would
also like to monitor urine output and also send urine for culture.

SEPSIS SCREEN: (Find out source of sepsis)


• Bloods for FBC, CRP, Renal and Liver function tests
• Blood Culture
• Urine Dip and Culture
• CXR
• Sputum culture (If cough + phlegm present)

SEPSIS SIX:
GIVE: TAKE:
Oxygen Blood cultures
IV Fluids Lactate
IV Antibiotics Urine Output

Start taking history NOW, if the patient’s vitals are stable.

www.aspire2plab.com 357
No real PLAB2 cases discussed. All discussed scenarios are fictitious and for educational purposes only.
Upper GI Bleed

You are an FY2 in A&E. Mr. John Taylor, aged 68, was brought into the A&E by his wife,
as he is feeling dizzy and faint. Talk to the patient, assess him, and discuss the initial
plan of management with him.

Both Sim-Man and Nurse are present inside the cubicle.

D: Who do we have in here? (Introduce yourself to the nurse)


Nurse: We have Mr. John here, who was brought in by his wife.

D: Do we know anything about why his wife brought him here today?
Nurse: No doctor.

D: Have you done anything so far?


Nurse: Yes, I have connected him to the monitor. Please have a look at the patient
doctor.

D: That’s fine, let me talk to patient first and assess him to see what’s going on.
Nurse: Okay doctor.

D: Hello Mr. John, what brought you to the hospital today?


P: I was feeling dizzy and faint doctor.

D: I am sorry to hear that Mr. John, could you tell me when did this start?
P: 2-3 days

D: How did it start?


P: It just started.

D: What were you doing?


P: Nothing in particular.

Let me quickly have a look at your vitals and then I will get back to you.

(Look at the monitor for vitals)


Vitals are stable.
If vitals deranged – Stabilise the patient first by ABCDE approach.

D: Nurse could you please keep an eye on the vitals and let me know if they are unstable.
Nurse: Okay.

D: Your vitals are stable Mr. John. Could you please tell me more about your dizziness and
fainting?
P: What do you want to know doctor?

D: What do you mean by dizziness? P: I feel lightheaded and loose balance.

www.aspire2plab.com 358
No real PLAB2 cases discussed. All discussed scenarios are fictitious and for educational purposes only.
D: Do you have this all the time? P: No, only when I walk for some time.
D: How many episodes did you have? P: 4-5
D: How long does each episode last? P: Few minutes
D: Is it getting worse? P: Yes
D: Does anything make it worse? P: When I do some activity.
D: Does anything make it better? P: When I rest

D: You told me that you this have feeling of fainting? Could you please tell me more
about it?
P: What do you want to know?

D: What do you mean by feeling faint?


P: I feel like I am about to fall and lose consciousness, but I don’t.

D: Did you lose consciousness anytime? P: No


D: Did you have a fall anytime? P: No
D: When did this start? P: Same time, 2-3 days
D: Is it getting worse? P: Yes

D: Do you have any other symptoms along with these? P: No

D: Any fever/flu like symptoms? P: No


D: Any nausea/vomiting? P: No
D: Any SOB? P: Yes/No
D: Any chest pain or discomfort? P: Yes, I feel some discomfort in my chest.

D: Can you tell me more about it?


P: It started with my symptoms & it’s all over my chest.

D: Any heart racing? P: Yes/No


D: Any ear pain or hearing problems? P: No
D: Any ringing sensation in your ear? P: No
D: Any headache? P: No
D: Any weakness in any part of your body? P: No
D: Any speech or vision problems? P: No
D: Any problems with your bowel or bladder? P: No
D: Any loose stools? (dehydration)? P: No
D: Any tummy pain or discomfort? P: No
D: Any bleeding from anywhere in your body? P: No
D: Any change in colour of your stool/any dark-coloured stools?
P: Yes/No, I didn’t look at my stools recently.

D: Is this the first time you have these symptoms? P: Yes


D: Have you been diagnosed with any medical condition in the past?
P: Yes, I have Osteoarthritis of my Knee.
D: How is it managed? P: I take pain killers for that.
D: Which pain killer and for how long? P: Diclofenac. Since few months.

www.aspire2plab.com 359
No real PLAB2 cases discussed. All discussed scenarios are fictitious and for educational purposes only.
D: Are you on any other medications apart from this? Any PPI’s (Gastric Tablets), steroids
or blood thinners? P: No

D: Are you diagnosed with any other medical problems? Any heart problems or blood
disorders? P: No

D: Do you have any allergy to any food or drug? P: No


D: Any medical conditions in the family? P: No

P: Sim-Man says, doctor I have soiled myself now.

D: Let me quickly have a look at your stool. DARK STOOL – MALEANA.


D: How are you feeling now? P: I feel short of breath doctor.
D: I think you have some bleeding from your gut. Let me quickly look at your vitals and
examine you.

LOOK at the monitor for vitals:


RR 26
SpO2 90
HR 98
BP 90/60
Temp 37.5
Note: Patient is hypotensive and tachycardic.

D: Start ABCDE approach immediately

A:
- Patient is talking to me. His airway is patent.
- I would like to start my patient on high flow O2 – 15L/min via non-rebreather mask.
- Pick the oxygen mask with a reservoir bag attached. (Saturation will improve)

B:
- Trachea and Chest Ex
- Trachea is central and Chest Ex is Normal.

C:
Peripheral signs of perfusion:
- Pulse, Cold peripheries, Cap refill, Cyanosis, Pallor, Active bleeding.
- Mx: HR & BP unstable. I would like to Insert TWO large bore IV cannulas and start resuscitating
my patient by giving IV Fluids. I would also like to draw some bloods at the same time for routine
investigations including group and cross match for blood transfusion.
- Fluid Challenge: Adults - 500 mL of warmed crystalloid solution (0.9% Normal Saline) in 5-10
minutes if the patient is Hypotensive or 1 L if the patient is in shock).
- I would like to reassess after the bolus and give further boluses of 500 ml fluids until a maximum
of 2L in the first one hour.

www.aspire2plab.com 360
No real PLAB2 cases discussed. All discussed scenarios are fictitious and for educational purposes only.
D:
- As my patient is talking to me and is obeying commands his GCS seems to be 15/15.

E:
- Expose the patient fully for Head-to-Toe examination. Look for any redness, rashes,
swelling, temperature. Undress his diaper and you can see dark coloured stools.

I would like to examine your tummy as well. P: Okay Dr.

D: How are you feeling now? P: Bit Better doctor.

Summarise your initial assessment: ABCDE – O2 and IV Fluids. Ask the patient if he feels
any better.

Give the diagnosis and definitive management for the diagnosis: You seem to be having
bleeding from your gut. That is why you are having all these symptoms. This is an
emergency. I need to refer you to a specialist. We will keep you Nil by Mouth and
continue fluids for now.

D: I will ask specialist (gastroenterologist) to review you.


P: Okay doctor.

D: The specialist will do a procedure called endoscopy (explain endoscopy) and try to find
out the source of bleeding and stop it.

D: I will review you regularly until you are seen by the specialist and get the procedure
done.
P: Okay doctor.

www.aspire2plab.com 361
No real PLAB2 cases discussed. All discussed scenarios are fictitious and for educational purposes only.
Post-Partum Haemorrhage

You are an FY2 in O&G. Mrs Samantha Jefferson, aged 35, multigravida, had her 5th delivery
an hour ago. She is bleeding. Nurse called you. Assess the patient and do the initial
management. There is a nurse in the cubicle.

Introduce yourself to the nurse.


D: How can I help you?
Nurse: Dr this lady had her 5th delivery, and she is bleeding now.

D: May I know at what time did the patient deliver the baby?
Nurse: An hour ago. (Bleeding <24hrs after delivery Primary PPH)
D: When did the bleeding start?
Nurse: Just now.

D: How are you feeling now? (Introduce yourself to the patient)


P: I can’t breathe. (or say hmm)

Look at the vitals on Monitor:


SpO2: 91
RR: 26
BP: 90/60
Pulse: >100
Note: Patient is hypotensive and tachycardic.

Start ABCDE approach immediately

A:
- Patient is talking to me. Her airway is patent.
- I would like to start my patient on high flow O2 – 15L/min via non-rebreather mask.
- Pick the oxygen mask with a reservoir bag attached. (Saturation will improve)

B:
- Trachea and Chest Ex.
- Trachea is central and Chest Ex is Normal.
- Let me examine you quickly. For the purpose of examination, I would like to undress you
above your waist, and I will keep a chaperone with me to ensure your privacy.

C:
- Peripheral signs of perfusion - Pulse, Cold peripheries, Cap refill, Cyanosis, Pallor, Active
bleeding site.

D: I can see bleeding from her vagina, how much blood has she lost until now?
Nurse: Shows you a bucket filled with blood.

Ask the nurse for the details and complications during the delivery.
D: Was the placental delivery complete? Nurse: Yes

www.aspire2plab.com 362
No real PLAB2 cases discussed. All discussed scenarios are fictitious and for educational purposes only.
D: Nurse, any vaginal tear during delivery? Nurse: Yes/No

I would like to examine your tummy as well to see if there is any uncontracted uterus.
P: Okay Dr.

- Mx: HR & BP unstable. I would like to Insert TWO large bore IV cannulas and start
resuscitating my patient by giving IV Fluids. I would also like to draw some bloods at the
same time for routine investigations including group and cross match for blood
transfusion.

Fluid Challenge: Adults - 500 mL of warmed crystalloid solution (0.9% Normal Saline)
in 5-10 minutes if the patient is Hypotensive or 1 L if the patient is in shock).

I would like to reassess after the bolus and give further boluses of 500 ml fluids until
a maximum of 2L in the first one hour.

(D: Nurse, could you please insert 2 Large Bore IV Cannulas (Grey 16G or Green 18G) and
start her on Fluids. We need to transfuse blood immediately. Could you please take some
blood for FBC, U&E’s, Clotting profile and for Grouping and Cross matching 4 Units of Blood.

Nurse: Okay Dr)

I would like to start uterine massage for my patient as I suspect uterine atony to be the
cause for bleeding. (Ask nurse colleague for help).

D:
- As my patient is talking to me and is obeying commands her GCS seems to be 15/15.
E:
- Expose the patient fully for Head-to-Toe examination. Look for any redness, rashes,
swelling, temperature.

D: I would like to examine your tummy as well. P: Okay Dr.


D: How are you feeling now? P: Bit Better doctor.

Summarise your initial assessment: ABCDE – O2 and IV Fluids. Ask the patient if she feels
any better.

Give the diagnosis and definitive management for the diagnosis: You seem to be having
bleeding from your front passage. This is called Post-partum haemorrhage (PPH). That is
why you are having all these symptoms. This is an emergency. I will involve my senior. I
need to do a manoeuvre called uterine massage to control the bleeding. If that doesn’t help,
then we will give you some medication to control your bleeding. Lastly, we might have to do
surgical intervention to control the bleeding.

D: Please insert a Catheter and Monitor the Vitals and Urine Output? Nurse: Yes
D: We may need to give Oxytocin 10 IU IV with the seniors help. Nurse: Okay

www.aspire2plab.com 363
No real PLAB2 cases discussed. All discussed scenarios are fictitious and for educational purposes only.
Start taking history NOW, if the patient’s vitals are stable.

D: How do you feel now? P: Better


D: How is your breathing? P: Better
D: Any pain? P: No Dr

D: How has your pregnancy been? P: Fine


D: Any problems during your pregnancy? P: No
D: Any problems with your previous pregnancies or deliveries? P: No
D: Have you been diagnosed with any medical condition in the past? P: No
D: Any Blood Disorder? P: No.
D: Are you taking any medications including OTC or supplements? P: No
D: Any blood thinners? P: No
D: Any allergies from any food or medications? P: No
D: Has anyone in the family been diagnosed with any medical condition? P: No

D: Was the placental delivery complete? Nurse: Yes


D: Nurse, any vaginal tear during delivery? Nurse: Yes/No
D: Please insert a Catheter and Monitor the Vitals and Urine Output. Nurse: Yes
D: We may need to give Oxytocin 10 IU IV with the senior’s help. Nurse: Okay

Talk to the Examiner: or Present case to Registrar - SBAR:


Ex: What happened?
Ex: What have you done so far?
Ex: Causes of PPH?
Ex: What else can be done to Control bleeding?

Postpartum haemorrhage (PPH) is a rare complication where a patient may bleed heavily
from the vagina after the baby's birth.
There are two types of PPH, depending on when the bleeding takes place:
- primary or immediate – bleeding that happens within 24 hours of birth.
- secondary or delayed – bleeding that happens after the first 24 hours and up to 6 weeks
after the birth.

Sometimes PPH happens because your womb doesn't contract strongly enough after the
birth. It can also happen because part of the placenta has been left in your womb or you
get an infection in the lining of the womb (endometritis). It can also happen in bleeding
disorders or trauma during delivery.

To help prevent PPH, you will be offered an injection of oxytocin (10 IU IM) as your baby is
being born. This stimulates contractions and helps to push the placenta out.

PPH Management:
Give immediate clinical treatment:
● emptying of the bladder and
● uterine massage and
● uterotonic drugs and

www.aspire2plab.com 364
No real PLAB2 cases discussed. All discussed scenarios are fictitious and for educational purposes only.
● intravenous fluids and
● controlled cord traction if the placenta has not yet been delivered

First-line treatment (can repeat another bolus):


Administer a bolus of one of the following as first-line treatment for postpartum
haemorrhage (uterotonic drugs):
• oxytocin (10 IU intravenous) or
• ergometrine (0.5 mg intramuscular) or
• combined oxytocin and ergometrine (5 IU/0.5 mg intramuscular).
If haemorrhage continues:
Consider oxytocin infusion or tranexamic acid

If the haemorrhage continues:


• perform examination under anaesthetic
• ensure that the uterus is empty and repair any trauma
• consider balloon tamponade before surgical options.

Surgical options: May include uterine artery ligation, ovarian artery ligation, internal iliac
artery ligation, selective arterial embolisation, B-lynch suture, dilatation and curettage and
hysterectomy.

Post resuscitation care:


It isn't a good idea to use tampons until after your 6-week postnatal check because they
could increase your chance of getting an infection.
You may notice the bleeding is red and heavy when you breastfeed. This happens
because breastfeeding makes your womb contract. You may also feel cramps similar to
period pains. The bleeding will carry on for a few weeks. It will gradually turn brownish in
colour and decrease until it finally stops.
If you're losing blood in large clots, tell your midwife. You may need some treatment.

SBAR Approach for referral or hand over or presenting a case:


Situation: Mrs Jefferson on O&G ward has massive vaginal bleeding after giving birth to a
healthy baby.
Background: She is 35 years old and had her 5th baby delivered an hour ago. She has no
history of medical problems and she is not on any blood thinners. There were no
complications during her previous deliveries.
Assessment: Mrs Jefferson was stable since admission for her delivery but suddenly her
vitals deteriorated to SpO2: 91, RR: 26, BP: 90/60, Pulse: >100.
I think she has PPH and she is going into circulatory shock.
Recommendation: I have resuscitated her by giving high flow O2, inserted 2 large bore IV
cannulas and started her on 2L warm crystalloids. I have sent her bloods for clotting and for
grouping and cross match 4 units of blood for transfusion. I would like you to review Mrs
Jefferson and consider Oxytocin 10 IU IV and balloon tamponade or surgical intervention if
needed.

www.aspire2plab.com 365
No real PLAB2 cases discussed. All discussed scenarios are fictitious and for educational purposes only.
Dizzy Spells

You are FY2 in A&E. Julia Robbins, aged 52, came with dizziness. Please talk to the patient,
assess his condition, examine him and discuss the initial plan of management with the
examiner. After 6 minutes, discuss the plan of management with the patient.

D: What brought you to the hospital? P: I have been feeling dizzy.


D: What do you mean by dizzy? P: I feel like I am about to fall. But, I don’t fall.
D: When did it start? P. It started a few weeks ago.

Reassure
D: Let me quickly have a look at your vitals

D. How did it start? P: Just started


D: What were you doing when it started? P: I don’t know. I wasn’t doing anything special.
D: Is it there all the time or does it come and go? P: I have this feeling from time to time.
D: How often do you get this feeling? P: I have had 4 episodes so far.
D: How long does each episode last? P: It lasts for a few seconds / minutes.
D: Is there anything which triggers your symptoms? P: No, I can’t think of anything.
D: is there anything which makes it better? P: Not really.
D: Has it changed since it started? P: What do you mean by that?
D: Are you experiencing it more often these days? P: I don’t think so.
D: Does each episode last longer than before? P: Not really.

D: Apart from what you told me, is there anything else you are experiencing?
P: I feel tired these days.

D: When did it start? P: It started about the same time as dizziness.


D: Has it changed since it started? P: I am getting more tired now.
D: Are you experiencing anything else? P: No doctor.
D: Any heart racing? P: I am not sure.
D: Are you feeling light headed? P: No
D. Are you short of breath?
P: Yes Doc. It started with dizziness. Whenever I feel dizzy, I have this shortness of breath.
D: Do you have any chest pain or chest discomfort? P: No (ACS)
D: Did you faint by any chance? P: No
D: Is there anything else you are experiencing? P: No

D: Has it happened before? P: No


D: Any fever or flu like symptoms? P: No
D: Any pain, swelling, redness or hotness in your legs? P: No
D: Any headaches recently? P: No.
D: Do you have any sweating? P: No
D: Do you feel hot when everyone around you is fine? P: No
D: Have you been losing weight? (Thyroid) P: No
D: Have you been diagnosed with any medical condition? P: No
D: How about any heart disease? P: No

www.aspire2plab.com 366
No real PLAB2 cases discussed. All discussed scenarios are fictitious and for educational purposes only.
D: How about high blood sugar or blood pressure? P: No
D: Any high cholesterol or lung diseases? P: No
D: Are you taking any medications regularly? P: No
D: Do you have any allergies? P: No
D: Has anyone in your family been diagnosed with any heart problems? P: No

D: Do you smoke? P: No
D: Do you drink alcohol? P: Yes/No
D: How is your diet? P: I pretty much eat healthy food.
D: Do you drink coffee or tea? P: Yes, 1 or 2 cups per day.
D: Are you physically active? P: I am quite active.
D: Do you have any stress in life? P: No
D: Do you use any recreational drugs? P: No

D: What do you do for a living? P: I work as a clerk / state agent.

In this station,
1. Look at the monitor carefully.
2. Check the patient's pulse for irregular pulse.
3. Auscultate the heart for murmur.

Monitor
BP: 110 / 70mmHg
HR: 70 – 100 bpm (Fluctuating)
RR: 20
ECG: AF.

Management
Note: At 6 mins bell, examiner will ask you:

E: What is wrong with this patient?


D: Based on the history and examination, the patient has AF.

E: Please summarise your findings of examination:


D:
1. The Patient's pulse is irregularly irregular.
2. On monitor – Look at all the vitals.

- ECG shows narrow complex and absent P waves.


- Pulse rate is fluctuating.
- Blood pressure is …..
- Temp is …..
- Respiratory rate is ……

On Auscultation, I can hear a murmur.

E: What are you going to do for this patient?

www.aspire2plab.com 367
No real PLAB2 cases discussed. All discussed scenarios are fictitious and for educational purposes only.
D: I will do some investigations.
I would like to do some blood tests (FBC, VBG, U&E, TFT, LFT, Troponin)
I would like to order a chest x-ray to identify any lung problem that may be causing AF.

E: How will you manage this patient? D: I will discuss the case with my senior.

I would like to give my patient a medication to control the rate of heartbeat as the first line
management such as beta blocker (metoprolol) or a calcium channel blocker (verapamil or
digoxin).

If symptoms continue after heart rate has been controlled or if the rate control strategy has
not been successful, rhythm control should be considered to restore a normal heart rhythm
(A) medication such as flecainide (B) cardioversion

Since my patient has not got any risk of stroke (based on CHADVAS and HAS-BLED scoring
system) there is no need of prescribing anti-coagulant.

I will discharge the patient after discussing with the senior.


I will do cardiology OPD referral urgently for further investigations.

E: Which investigation?
D:
- 1. Holter Monitor ECG (24 – 48 hrs)
- 2. Echocardiograph.

www.aspire2plab.com 368
No real PLAB2 cases discussed. All discussed scenarios are fictitious and for educational purposes only.
Acute Limb Ischaemia

You are an FY2 in A&E. Mrs Alexandra Stuey, 60-year-old, presented with a history of severe
pain in her right leg since this morning. Talk to her, take history, assess the patient, do
relevant examination and discuss the management.

D: What brought you to the hospital?


P: I have had severe pain in my right leg since this morning.

D: I am sorry to hear that, are you comfortable to talk? P: Yes Dr.

D: Let me quickly have a look at your vitals

Note: Patient has AF, HR is increased, and other vitals are normal. So, continue with History.
Keep monitoring the vitals while you are taking History.

D: Could you please elaborate this pain for me? P: What do you want to know?
D: Where exactly is the pain? P: It’s all over my leg.
D: How did the pain start? P: It just started suddenly .
D: What were you doing when the pain started? P: I was at home, not doing anything.
D: Is it continuous or intermittent? P: It is continuous.
D: Is it getting worse? P: Yes
D: Could you please describe this pain for me? P: It is very painful.
D: Does the pain go anywhere else? P: Just my leg.
D: Does the pain start at your back and travel down your legs? P: No.
D: Does anything make the pain worse? P: It is getting worse by itself.
D: Does anything make the pain better? P: No.
D: Could you please score the pain for me on a scale of 1-10, 1 being the lowest and 10
being the most severe pain you have ever experienced? P: 6/7/8

D: Do you have any other problems? P: No.

D: Is this the first time you are experiencing this kind of pain? P: Yes.
D: Have you experienced any pain in your legs recently after walking a certain distance?
P: No.
D: Do you have any fever or flu like symptoms? P: No.
D: Any swelling of the legs? P: No.
D: Any redness/swelling/hotness around the calf? P: No.
D: Any weakness in your legs? P: Yes Dr, I feel my right leg is weak.
D: When did this start? P: This morning.
D: Is it getting worse? P: Yes
D: Any tingling or numbness or loss of sensation in your legs? P: No.
D: Have you noticed any change in skin colour or ulcers in your legs? P: No.
D: Have you hurt your leg recently? P: No.
D: Do you have any chest pain or heart racing?
P: Dr I had a chest pain and my heart was racing 2 weeks back, but I am fine now.

www.aspire2plab.com 369
No real PLAB2 cases discussed. All discussed scenarios are fictitious and for educational purposes only.
D: Did you have any more episodes like that? P: No.

D: Have you ever been diagnosed with any medical conditions? P: No.
D: Any heart problems or diabetes or high blood pressure or high cholesterol? P: No.
D: Any regular medications or OTC drugs? P: No.
D: Any allergies to any food or drugs? P: No.
D: Any hospital admissions or surgeries recently? P: No.
D: Any family members diagnosed with any medical conditions? P: No.

D: Do you smoke? P: Yes.


D: How much and how often?
P: I have smoked 30 cigarettes a day from the last 30 years.
D: Do you drink alcohol? P: Yes
D: How much and how often? P: I drink occasionally.
D: Could you please tell me about your diet? P: I eat everything.
D: Are you physically active? P: I am not that active.
D: Do you have any stress in life? P: Yes/No
D: What do you do for a living? P: I have an office job.
D: Have you travelled anywhere recently? P: No.
D: Whom do you live with? P: I live alone.

D: I need to examine you, check your vitals, GPE and examine your legs.

Findings:
Right Limb: Pale/Bluish discolouration, cold compared to other leg, Extremely tender, No
dorsalis pedis pulse felt, Prolonged Cap Refill time.
Left Limb: Normal

D: I would like to examine your heart.


Tachycardic
S1 S2 heard
Murmur: Yes/no

D: From my assessment, I suspect that you have a condition called acute limb ischemia. This
is a condition in which there is disruption in blood supply due to blockage of one of your
arteries.

P: Why do I have this condition?


D: I think this is because of your heart beating very fast. We call this condition atrial
fibrillation. We have done tracing of your heart called ECG and it shows AF.

D: Sometimes when your heart is beating too fast, there might be some blood clot
formation which may dislodge into the circulation. When these clots get stuck in smaller
arteries, they obstruct the blood flow.

P: What are you going to do next Dr?

www.aspire2plab.com 370
No real PLAB2 cases discussed. All discussed scenarios are fictitious and for educational purposes only.
D: This is a serious condition, so we need to admit and treat you immediately. I am going to
arrange for some blood tests and an US scan of the arteries in your legs. I am going to refer
you to specialists called vascular surgeons. Depending upon your ultrasound scan result,
they might have to give you some medication to dissolve the clot or do a procedure to
remove the clot. Sometimes, they might have to do a bypass operation to restore the blood
flow to your legs. Is it OK with you?

P: Yes.

Management of Acute Limb Ischaemia:


- Oxygen
- Morphine
- Heparin
- US Doppler
- Referral to Vascular Surgeons.

www.aspire2plab.com 371
No real PLAB2 cases discussed. All discussed scenarios are fictitious and for educational purposes only.
DNAR

You are FY2 in Medicine. Mr. Marcus Tudor, aged 93, has been admitted in the hospital
with lung cancer. He has been receiving treatment for it. Nurse wants to talk to you about
Mr. Marcus Tudor. Talk to the nurse, assess the patient and write medical notes.

D: How can I help you?


Nurse: Mr. Tudor has been admitted in the hospital for the past few weeks. He is receiving
chemotherapy for lung cancer. He is not well for the last one week and now his condition is
deteriorating.

D: Hello, Hello. Tap on his shoulder


Assess airway, breathing and circulation. (The patient had no Pulse and was not breathing.)

D: He is in cardiac arrest. Please help me connect the monitor to confirm.


Nurse: Sorry doctor, there are no monitors here.

D: Ok then we must start CPR or he has DNAR.

Nurse: Yes doctor, Mr. Tudor had signed the DNAR form. Here, I have the form if you want
to have a look at it. (She will show you the form)

D: Mr. Tudor had wished for no resuscitation so we have to respect that.


Confirm patient death and write in notes.
Patient is lying in bed with eyes closed.

Note: You will get Neuro-pin, Pen Torch & Stethoscope

Death Confirmation:
1. Confirm the identity of the patient – check the wrist band
2. General inspection – skin colour / any obvious signs of life
3. Look for signs of respiratory effort
4. Does the patient respond to verbal stimuli? – Hello Mr Tudor, can you hear me?
5. Does the patient respond to pain? – press on fingernail / trapezius squeeze /
supraorbital pressure
6. Assess pupils using pen torch – after death they become fixed and dilated
7. Feel for a central pulse – carotid artery
8. Auscultation:
Listen for heart sounds for at least 2 minutes
Listen for respiratory sounds for at least 3 minutes
(It might differ according to the hospital policies)
9. Cold Peripheries
10. Rigor Mortis

Patient declared as deceased at (Date and time)

Inform nursing staff that you have confirmed the death:

www.aspire2plab.com 372
No real PLAB2 cases discussed. All discussed scenarios are fictitious and for educational purposes only.
a. They will then inform next of kin, if not already present
b. They will also contact the porters to arrange transfer of the body to the morgue
c. Discuss with the senior regarding cause of death before signing the form.

Consider if this death needs a referral to the coroner, as if this is the case a death certificate
cannot be issued – this will require discussion with the consultant responsible for the
patient.

Name:
Age:
Gender:
Date of Birth
NHS Number:

Date:

Death Confirmation Assessment:

Identity confirmed as Albert Tudor from wrist band


Patient in bed, eyes closed, no signs of life
No respiratory effort noted
No response to verbal stimuli
No response to painful stimuli
No carotid pulse palpable
Pupils fixed and dilated bilaterally
No heart sounds noted during 2 minutes of auscultation
No breathing sounds noted after 3 minutes of auscultation

Death Confirmed at __/__/__ at 09.30

www.aspire2plab.com 373
No real PLAB2 cases discussed. All discussed scenarios are fictitious and for educational purposes only.
www.aspire2plab.com 374
No real PLAB2 cases discussed. All discussed scenarios are fictitious and for educational purposes only.
Hypoglycaemia

You are FY2 in A&E. Mr Frank Escobar, a 50-year-old, was brought into the hospital by
ambulance as he was found unconscious. Please talk to the patient, assess the patient do
the initial management.

D: Try and TALK to the patient: Patient is not talking back


Tap on the shoulders: Patient is not responding again.

Patient is UNCONSCIOUS.

D: Now check whether the patient is breathing or not?


D: Look/ Listen/ Feel: Patient is breathing - his airway is patent.

D: I would like to check my patient’s vitals.

NOTE: Vitals are Stable.

As the patient is Unconscious and Breathing. Continue with ABCDE approach and keep an
eye on the vitals.

D: Start ABCDE approach immediately.

D: I would like to arrange for blood sugars as my patient is unconscious in the mean while I
would like to proceed with ABCDE.

A:
- Patient is breathing. There are no added sounds. His airway is patent.
- I would like to start my patient on high flow O2 – 15L/min via non-rebreather mask.
B:
- Trachea and Chest Ex
- Trachea is central and Chest Ex is Normal.
C:
- Peripheral signs of perfusion - Pulse, Cold peripheries, Cap refill, Cyanosis, Pallor,
Active bleeding site.
- Mx: HR & BP stable. Insert one IV cannula and draw some bloods at the same time
for routine investigations including blood sugars.
- Blood Glucose = 1.8
D:
- My patient has Hypoglycaemia, and I would like to give IM Glucagon as my patient is
unconscious and start him on IV Dextrose 20% in 100 mls NS bolus. I would like to
check his blood sugars every 10-15 minutes. I would like to check my patient’s GCS.
Examiner will give you the findings.

E:
- Expose the patient fully for Head-to-Toe examination. Look for any redness, rashes,
swelling, injuries, head trauma, temperature.

www.aspire2plab.com 375
No real PLAB2 cases discussed. All discussed scenarios are fictitious and for educational purposes only.
Summarise your initial assessment: ABCDE – O2 and Low Blood Sugars.

Give the diagnosis and definitive management for the diagnosis: Hypoglycaemia and I have
already given IM Glucagon as my patient is unconscious and started him on IV Dextrose 20%
in 100 mls NS. I would like to check his blood sugars every 15 minutes.

Start talking to the patient and take history if the patient gains consciousness.

Blood Glucose: 3.2

My patient’s blood sugars are improving. I would like to check his response.

- Start talking to the patient and take history if the patient gains Consciousness.

- If patient is not regaining his consciousness, start verbalising that I would like to
continue the same management plan after involving my seniors and keep checking
his blood sugars every 10-15 mins. Once the blood sugars are above 4 and Pt regains
his consciousness, I would like to give him a full meal and check his blood sugars until
they are normal. Then I would like to take a full detailed Hx of the event and PMHx
and refer him to Endocrinology. I would like to counsel him about management of
Diabetes before discharge.

- On the other hand, if the Pt doesn’t regain his consciousness even after his blood
sugars are improved, I would like to discuss with my senior and investigate for other
causes of unconsciousness (CT Head, Toxicology screen, etc).

www.aspire2plab.com 376
No real PLAB2 cases discussed. All discussed scenarios are fictitious and for educational purposes only.
Heart Failure

You are FY2 in A&E. Mrs Michelle Turner, aged 75, has come to the A&E with shortness of
breath and palpitations. Please talk to her, assess her, manage her, and discuss the
management plan with the her.

Talk to the patient. Patient is CONSCIOUS and oriented.

D: What brought you to the hospital? P: I can’t breathe.


D: When did it start? P: It started yesterday.
D: How did it start? P: Just started.
D: Has it changed since it started? P: It got worse, and I called the ambulance
D: Do you have any other symptoms? P: No

Reassure

D: Let me quickly have a look at your vitals

Note: Saturations are decreased. (You may also find AF on ECG).

D: Start ABCDE approach immediately

A:
- Patient is talking to me. Her airway is patent.
- I would like to start my patient on high flow O2 – 15L/min via non-rebreather mask.

B:
- Trachea and Chest Ex
- Trachea is central and there are bilateral crackles on chest Ex

C:
- Peripheral signs of perfusion - Pulse, Cold peripheries, Cap refill, Cyanosis, Pallor,
Active bleeding site.
- Mx: HR & BP stable. Insert one IV cannula and draw some bloods at the same time
for routine investigations including blood sugars.

D:
- As my patient is talking to me and is obeying commands her GCS seems to be 15/15.

E:
- Expose the patient fully for Head-to-Toe examination. Look for any redness, rashes,
swelling, temperature, pedal oedema. (Note: Examiner might say NO pedal oedema
or pedal oedema is present).

Summarise your initial assessment: ABCDE – O2. Ask the patient if she feels any better. Look
at the monitor and check if vitals have improved. Patient says she feels better, and the vitals
will be improved on the monitor.

www.aspire2plab.com 377
No real PLAB2 cases discussed. All discussed scenarios are fictitious and for educational purposes only.
Verbalise about further investigations: ECG, CXR and BNP. (Note: Examiner will give you an
ECG with AF and a CXR with findings of Heart Failure {Cardiomegaly and Pulmonary oedema
or Bilateral pleural effusions}).

Give the diagnosis and definitive management for the diagnosis: My patient seems to be
having heart Failure and I would like to give Diuretics to my patient. I will ask my senior to
review my patient.

I will arrange for an in-patient Echo to confirm the diagnosis and refer to a cardiologist.
I will also monitor my patient’s Renal function regularly.

Start taking history NOW, if the patient’s vitals are stable.

History to take:

D: Do you have any cough? P: No


D: Do you have any fatigue? P: Yes/No
D: Have you experienced any heavy pain in your chest? P: No

D: Has it happened before? P: Yes/No

D: Do you have any other medical conditions? P: Yes. HTN.


D: When was it diagnosed? P: 7 years
D: How is it managed? P: Ramipril
D: Is it well controlled? P: Yes
D: Are you taking any other medications other than Ramipril? P: No
D: Have you been hospitalized before? P: No
D: Have you had any surgeries in the past? P: No

D: Has any member of your family ever been diagnosed with any medical conditions? P: No

D: Do you smoke? P: Yes/No


D: Are you physically active? P: I am not quite active.
D: What do you do for a living? P: I am retired.
D: Have you travelled recently? P: No.

D: How is your living status? P: It’s fine

www.aspire2plab.com 378
No real PLAB2 cases discussed. All discussed scenarios are fictitious and for educational purposes only.
Haematemesis – Post Endoscopy

You are an FY2 in Gastroenterology. Mr Michael Turner, aged 55, was admitted to the
hospital for endoscopy. Endoscopy was done yesterday. He had haematemesis now. You
were asked to see him. Please talk to him, assess him, manage him, and discuss the
management plan with him.

D: How are you feeling now? (Introduce yourself to the patient)


P: I have vomited a lot of blood and I can’t breathe now.

Reassure

D: May I know how much blood you have vomited?


(There will be a bucket full of blood)

D: Is this the first time?


P: No Dr. I had few episodes before and I had endoscopy done yesterday.

D: Do you know the result of the procedure?


P: Yes/No

Let me quickly have a look at your vital signs.

Look at the vitals on Monitor:


SpO2: 91
RR: 26
BP: 90/60
Pulse: >100
Note: Patient is hypotensive and tachycardic.

D: Start ABCDE approach immediately

A:
- Patient is talking to me. His airway is patent.
- I would like to start my patient on high flow O2 – 15L/min via non-rebreather mask.
- Pick the oxygen mask with a reservoir bag attached. (Saturation will improve)

B:
- Trachea and Chest Ex.
- Trachea is central and Chest Ex is Normal.

C:
- Peripheral signs of perfusion - Pulse, Cold peripheries, Cap refill, Cyanosis, Pallor,
Active bleeding site.
- Mx: HR & BP unstable. I would like to Insert TWO large bore IV cannulas and start
resuscitating my patient by giving IV Fluids. I would also like to draw some bloods at

www.aspire2plab.com 379
No real PLAB2 cases discussed. All discussed scenarios are fictitious and for educational purposes only.
the same time for routine investigations including group and cross match for blood
transfusion.
- Fluid Challenge: Adults - 500 mL of warmed crystalloid solution (0.9% Normal Saline)
in 5-10 minutes if the patient is Hypotensive or 1 L if the patient is in shock).
- I would like to reassess after the bolus and give further boluses of 500 ml fluids until
a maximum of 2L in the first one hour.
- I will start the blood transfusion as soon as the blood is ready.

D:
- As my patient is talking to me and is obeying commands his GCS seems to be 15/15.

E:
- Expose the patient fully for Head-to-Toe examination. Look for any redness, rashes,
swelling, temperature.

I would like to examine your tummy as well.


(Abdominal Ex is Normal).

Summarise your initial assessment: ABCDE – O2 and IV Fluids. Ask the patient if he feels any
better.

Give the diagnosis and definitive management for the diagnosis: You seem to be having
bleeding from your gut. That is why you are having these symptoms. This is an emergency.

I will involve my senior and him to review you.

We will keep you Nil by Mouth and continue fluids for now. We need to give you blood as
well as you have lost blood.
P: Okay doctor.

D: We might have to do the endoscopy procedure again to find out the source of bleeding
and stop it.

D: I will review you regularly until you are seen by senior and get the procedure done.
P: Okay doctor.

Start taking history NOW, if the patient’s vitals are stable.

D: How do you feel now? P: Better


D: How is your breathing? P: Better
D: Any pain? P: No
D: Any other symptoms? P: No
D: Any heart racing? P: No
D: Do you feel lightheaded? P: No
D: Any swelling in your legs? P: No
D: Any change in colour of your skin? P: No

www.aspire2plab.com 380
No real PLAB2 cases discussed. All discussed scenarios are fictitious and for educational purposes only.
D: Did you notice any change in colour of your nails? P: No

D: Have you been diagnosed with any medical condition in the past? P: No
D: Any Blood Disorder? P: No.
D: Are you taking any medications including OTC or supplements? P: No
D: Any blood thinners? P: No
D: Any allergies from any food or medications? P: No
D: Has anyone in the family been diagnosed with any medical condition? P: No

D: Do you smoke? P: Yes/No


D: Do you drink alcohol? P: Yes/No
D: Tell me about your diet? P:
D: Are you physically active? P: I am not quite active.

www.aspire2plab.com 381
No real PLAB2 cases discussed. All discussed scenarios are fictitious and for educational purposes only.
Post CS – Pain Management

You are on FY2 in OBG. Mrs Mary Poppins had CS and had delivered twins 8 hours ago.
Both the babies are fine safe with the father in the other room. Talk to her and address
her concerns.

Additional information:

Surgery was done 8 hours ago.


Surgery was uneventful with delivery of twins.
Placenta was completely removed with blood loss of 250ml.
Surgery was done under spinal using diamorphine.

Vital Chart:
HR: 95, SP02: 99, RR: 19, BP: 130/70, T:37.4C, ECG: Sinus Rhythm

D: How can I help you?


P: I have pain at the surgical area.

Elaborate PAIN

Explore if she is breast feeding.

Exclude:
1. Urinary retention (See if the patients Bladder is full or is catheterised)
2. Constipation/Passed wind or not
3. Sepsis/Infections (Check the wound site and vitals)
4. Haemorrhage (Check the wound site and vitals for internal bleeding)

As the patient has received Diamorphine during the induction of anaesthesia, you can give
paracetamol and NSAIDS unless contraindicated. Counsel the patient for the pain as the
pain killer takes some time to show its affects. Offer oral morphine sulphate to women who
have received spinal or epidural anaesthesia for Caesarian birth. If oral is not tolerated, offer
subcutaneous, IV, IM morphine. In breast feeding women, you can use morphine,
dihydrocodiene, tramadol or oxycodone. At the lowest effective dose for the short duration
and not more than 3 days without close supervision. Consider laxatives and anti-emetics for
women taking opioids for the prevention of constipation and nausea/vomiting respectively.

Remember:

1) You can give co-dydramol (paracetamol and dihydrocodiene).


2) Do not offer co-codamol to women who are currently breastfeeding because it can
lead to serious neonatal sedation and respiratory depression.
3) When the patient is using paracetamol, NSAIDS, co-dydramol, prescribe them to be
taken regularly and not when needed for pain relief.

www.aspire2plab.com 382
No real PLAB2 cases discussed. All discussed scenarios are fictitious and for educational purposes only.
Prescription Writing

STEPS TO WRITE DRUG CHART

1 2 3 4 5 6 7

Fill the Fill ALLERGIES Write Antibiotics New Anticoagulants Regular & PRN
demographics Emergency medications & Insulins Medications
FIRST – Patient +
drugs first (Write at last)
Name/Age/DOB/H Type of Reaction
ospital or NHS No.,
Date, Drug Chart
No., Ward,
Consultant.

Aspire Education www.aspire2plab.com

Latin abbreviations
Directions should be in English without abbreviation. However, Latin abbreviations have
been used when prescribing.
The following is a list of appropriate abbreviations. It should be noted that the English
version is not always an exact translation.
• a. c. = ante cibum (before food)
• b. d. = bis die (twice daily)
• o. d. = omni die (every day)
• o. m. = omni mane (every morning)
• o. n. = omni nocte (every night)
• p. c. = post cibum (after food)
• p. r. n. = pro re nata (when required)
• q. d. s. = quater die sumendum (to be taken four times daily)
• q. q. h. = quarta quaque hora (every four hours)
• stat = immediately
• t. d. s. = ter die sumendum (to be taken three times daily)
• t.i.d. = ter in die (three times daily)

Drug chart in hospitals comprise of a booklet with multiple pages. There are different
sections in a drug chart as follows:

• - Patient Demographics
• - ALLERGY Box
• - Once Only Drugs
• - Thromboembolic Risk Ax – Prophylactic Heparin
• - Insulins

www.aspire2plab.com 383
No real PLAB2 cases discussed. All discussed scenarios are fictitious and for educational purposes only.
• - Anticoagulation – Warfarin/NOACS (Apixaban, Rivaroxaban)
• - Antimicrobial Drugs
• - Oxygen
• - Regular Drugs
• - PRN Drugs
• - Variable rate infusions
• - IV Fluids/ Blood Transfusions
• Timings to write on the Drug Chart:

The regular drug rounds in the hospitals happen four times during the day: Morning,
Midday, Evening and Bedtime. Please tick the appropriate box. For some sections of drugs,
you will have to write the timings yourself instead of ticking (E.g.: Antibiotics, once only
drugs). You will have to circle or tick the timings as per the drug chart.

- Always use 24-hour format.


- OD/OM – Once Daily/Once in Morning: Circle or tick the first dose of the day
- BD – Twice Daily: Circle or tick the first and last dose of the day
- TDS – Three times in a day: Circle or tick the first dose, afternoon and last dose of the
day
- QDS – Four times in a day: Circle or tick all four doses of the day on the drug chart
- 8 hourly – Space the doses 8 hours apart in a day. So, write new times on the drug chart
in the space provided
- ON – Once at Night: Circle or tick the last dose of the day
- PRN – Prescribe on the As Required side

Abbreviations for the route of administration:

- PO/O–Oral
- IV – Intravenous
- IM – Intramuscular
- S/C – Subcutaneous
- Top – Topical
- INH – Inhaled
- NEB - Nebulised
- PR – Per Rectal
- PV – Per Vaginal
- T – One Tablet
- TT – Two Tablets
- 1/2 - Puffs

Important points to note while writing the drug chart:


- Write the DRUG GENERIC NAME and not brand name in BLACK CAPITALS except for
approved names for few drugs like Inhalers, Insulins, Psychiatric and Epilepsy drugs, etc.
- Do not use Abbreviations for Drug Names.
- Write the DOSE and do not abbreviate Micrograms, Nanograms, Units.
- The unnecessary use of decimal points should be avoided, Eg: 3 mg, not 3.0 mg.

www.aspire2plab.com 384
No real PLAB2 cases discussed. All discussed scenarios are fictitious and for educational purposes only.
- TIMINGS should be mentioned clearly (OD, BD, TDS, QDS, etc.)
- ROUTE of administration should be clearly documented (PO, IV, IM, S/C, etc.)
- START DATE & END DATES (if applicable Eg: Antibiotics) should be documented.
- REVIEW DATE (if applicable Eg: Antibiotics, anticoagulants) should be mentioned clearly.
- Always use 24-hour format and clearly CIRCLE OR TICK the timings the medication to be
given.
- Always SIGN your name and write your GMC number.
- If you must stop any drug, strike it off cleanly and clearly on the drug chart – The whole
column to stop the drug.

- If you must hold a drug for few days, put an “X’’ in the boxes for specific dates to omit
the drugs.

Steps to write the drugs chart:


1. Demographics: First and foremost, fill the patient demographics to identify and confirm your
Pt. Patient Name/Age/DOB/Hospital or NHS No., Date, Drug Chart No., Ward, Consultant.
(Verbalise that you would like to fill the patient’s demographics first).
2. Allergies + Type of Reaction: Immediately document if your Pt has any allergies and
document the type of reaction they get when they take that drug. (Verbalise that I would
like to know my Pt’s full allergy status including drugs, OTC, and hospital equipment like
plaster, latex, etc. I would also like to document the type of reaction they experience when
they take the drug).
3. Emergency Drugs: Then write the emergency drugs that you want to give immediately.
These would be mostly once only drugs as stat doses. Eg: One dose of Adrenaline in
Anaphylaxis, One dose of Aspirin in ACS, etc.
4. Antibiotics: Prescribe Abx clearly with both START and END dates and verbalise the same. If
you are starting IV Abx, verbalise that you would like to review the Abx in 48-72 hours.
5. New Medications: Prescribe any new medications that you want to add in this admission.
6. Thromboembolic Risk Assessment: Verbalise that you would like to assess the patient’s
Thromboembolic risk and prescribe prophylactic Heparin if needed.
7. Anticoagulants and Insulins: Remember to write these drugs. These are very important and
commonly missed drugs in regular prescriptions.
8. Regular Drugs: Write all the regular drugs that the patient is taking from before this
admission. Keep a note to change the route if the Pt cannot take anything by mouth or Pt is
Nil by Mouth (NBM). In Palliative and terminally ill patients we tend to stop most of the
regular medications other than symptomatic medications.

www.aspire2plab.com 385
No real PLAB2 cases discussed. All discussed scenarios are fictitious and for educational purposes only.
9. As Required Drugs: Write the As required or PRN or SOS drugs on the AS REQUIRED section
of the drug chart. Write the indication, maximum dose in 24 hours and frequency of the
drug in this section.

Special Case Prescriptions:


• Inhalers: Write the inhaler drug name and the amount of drug in each puff in the DRUG BOX
and the number of puffs in the DOSE BOX. Eg: Ipratropium Bromide 20 micrograms (in DRUG
BOX) and 2 puffs (in DOSE BOX).
• Insulins
• Anticoagulants:
o Warfarin – Write the indication, desired INR range and duration of treatment in the
Anticoagulation section.
o Apixaban – Write in the regular side. Loading dose for 7 days and maintenance dose
as decided.
• IV Fluids: Prescribe on separate IV Fluids chart. Write the Type, Strength and Volume of fluid
and any medication to be added.
• Blood Transfusion: Prescribe on separate blood transfusion chart. Prescribe each unit to be
transfused separately. Write the duration to be transfused. 1 unit for 4-6 hours.

Drugs commonly prescribed:


Antibiotics:
- Amoxicillin à 500 mg TDS PO/1 gram TDS IV
- Co-Amoxiclav à 625 mg TDS PO/1.2gram TDS IV
- Clarithromycin à 500 mg BD PO
- Erythromycin à 500 mg QDS PO
- Metronidazole à 400 mg TDS PO
- Flucloxacillin à 1 gram QDS PO/2 gram QDS IV (if Severe infection)

Inhalers:
- Salbutamol 100 micrograms à 1-2 puffs PRN
- Ipratropium Bromide 40 micrograms à 2 puffs BD INH
- Seretide 250 Evohaler à 2 puffs BD INH
- Fostair 100/6 à 1 puff BD INH

Regular Drugs:
- Aspirin à 75 mg OD PO
- Clopidogrel à 75 mg OD PO
- Ramipril à 1.25 mg OD PO
- Losartan à 50 mg OD PO
- Bisoprolol à 1.25 mg OD PO
- Bendroflumethiazide à 2.5 mg OD PO
- Furosemide à 40 mg OD PO
- Atorvastatin à 10 mg OD PO
- Simvastatin à 10 mg OD PO
- Levothyroxine à 100 micrograms OD PO

Emergency Drugs:
- Oxygen à NRM 15L/min

www.aspire2plab.com 386
No real PLAB2 cases discussed. All discussed scenarios are fictitious and for educational purposes only.
- Aspirin à 300 mg Stat (ACS)
- Clopidogrel à 300 mg Stat (ACS)
- Adrenaline à 500 micrograms IM Stat (0.5 ml 1:1000 adrenaline injection) in
Anaphylaxis
- Salbutamol Nebs à 5 mg (Back-to-back in Asthma Exacerbation)
- Ipratropium Nebs à 500 micrograms (Back-to-back in Asthma Exacerbation)

Anticoagulants:
- Warfarin à As per INR
- Apixaban à 10 mg BD PO for 7 days loading, then 5 mg BD PO maintenance

Pain Killers:
- Paracetamol à 1 g QDS PO/1 g QDS IV
- Codeine à 30 mg PO QDS
- Ibuprofen à 400 mg QDS PO
- Morphine à 10 mg every 4 hours PO

Antiemetics:
- Metoclopramide à 10 mg TDS PO/IM/IV
- Cyclizine à 50 mg TDS PO/IM/IV

Anti-inflammatory:
- Ibuprofen à 400 mg QDS PO
- Prednisolone à 40 mg OD PO for 5 days (Asthma Exacerbation)
30 mg OD PO for 5 days (COPD Exacerbation)
PPIs:
- Omeprazole à 20 mg PO OD
- Lansoprazole à 15 mg PO OD

As Required:
Palliative:
- Morphine à 2.5 – 5 mg 4 hourly PRN S/C for Pain. Max dose in 24 hours 20 mg
- Midazolam à 2.5 – 5 mg 4 hourly PRN S/C for Agitation. Max dose in 24 hours 20 mg
- Cyclizine à 50 mg 8 hourly S/C PRN for Nausea/Vomiting. Max dose in 24 hours 150 mg
- Hyoscine Bromide à 400 micrograms 4 hourly PRN S/C for Secretions. Max dose in 24 hours 2.4
mg

www.aspire2plab.com 387
No real PLAB2 cases discussed. All discussed scenarios are fictitious and for educational purposes only.
Antibiotics Classification

Penicillin’s:
Penicillin, amoxicillin, co-amoxiclav, flucloxacillin and phenoxymethylpenicillin

Cephalosporins:
Cefalexin, Ceftriaxone, Cefotetan, Cephalothin

Aminoglycosides:
Gentamicin and Tobramycin

Tetracyclines:
Tetracycline, doxycycline and lymecycline

Macrolides:
Azithromycin, Erythromycin and Clarithromycin

Fluoroquinolones:
Ciprofloxacin and Levofloxacin

Carbapenems:
Meropenem, Imipenem, Ertapenem

www.aspire2plab.com 388
No real PLAB2 cases discussed. All discussed scenarios are fictitious and for educational purposes only.
MRSA

Where you are


You are an FY2 in MRSA ward.

Who the patient is


Mr David Halifax, aged 82, has been diagnosed with MRSA infection.

Other information you have about the patient


Patient was on the following medication:
Amlodipine 10mg OD
PCM PRN (max 4 gram)
Mupirocin Ointment BD 5 Days
Vancomycin
Weight is 80kg.
eGFR – normal
Consultant has decided to give ----ml of fluid.

What you must do


Write down the prescription for the above medications.
Check dose and write down vancomycin.
VANCOMYCIN IV INTERMITTENT
DOSING IN ADULTS
Special note
Patient is allergic to penicillin. After taking penicillin patient developed rash.
Uses:
Exclusions:

Step one: Loading Dose

Actual Body Weight Dose Volume of sodium chloride Duration of infusion


LOADING DOSE

(kg) 0.9% or glucose 5%

Step two: Maintenance Dose

Give first maintenance dose 12, 24 or 48 hours after start of loading dose according to dose interval in table below
www.aspire2plab.com 389
CrCl Dose Volume of Duration of Dose Interval (time Timing
(ml/min)
No real PLAB2 cases sodium
discussed. All discussed infusion
scenarios since
are fictitious andloading dose of levels
for educational purposes only.
chloride 0.9% and time between
or glucose 5% maintenance doses)
OSE
VTE Risk Assessment

Where you are


You are an FY2 in OBG.

Who the patient is


Miss Daniella Halifax, aged 42, had her 4th delivery yesterday.
She has lost 1200ml of blood.

Other information you have about the patient


No known allergy.
She has not been diagnosed with any past medical history.
Her weight is 60kg.

What you must do


Do DVT risk assessment and write down the anticoagulant according to the weight.

Special note
None

www.aspire2plab.com 390
No real PLAB2 cases discussed. All discussed scenarios are fictitious and for educational purposes only.
Palliative Care (1)

Where you are


You are an FY2 in Hospice care home.

Who the patient is


Mrs Victoria Yates, aged 80, is diagnosed metastatic pancreatic cancer.

Other information you have about the patient


Patient is terminal. Palliative care has been prescribed.
She has been referred from the hospital to hospice for the continuation of the palliative
care. She cannot eat or drink very well. Her list of medications can be found in the hospital
handover inside the cubicle.

What you must do


Write down the prescription for the above medications.

Special note
None

Hospital Handover Note

Patient name: Victoria Yates


Date of Birth: 25/03/1942
NHS Number: 123456
Hospital No: 1234
Allergy: Penicillin
Reaction: Breathlessness

Dear Doctor,
Mrs Victoria Yates, aged 80, has been admitted in the medical department with pancreatic
cancer. She is terminally ill and on palliative care. She cannot eat or drink well.

Plan: She will be discharged to hospice for palliative care with the following medications.

Other medications:
Morphine 2.5mg SC every 2 to 4 hours (max 6 doses) à indication pain
Cyclizine 50mg TDS SC à indication NV
Midazolam 2.5mg SC 2 Hourly (max 6 doses) à indication for agitation.
Hyoscine bromide 400micrograms SC 2 - 4 Hourly (Max 2.4mg) à indication for secretion.
PCM 1g PO every 4-6 hours (max 4g/day) à indication pain
Atorvastatin 10mg PO OD

www.aspire2plab.com 391
No real PLAB2 cases discussed. All discussed scenarios are fictitious and for educational purposes only.
Palliative Care (2)

Where you are


You are an FY2 in Hospice care home.

Who the patient is


Mrs Victoria Yates, aged 80, is diagnosed metastatic pancreatic cancer.

Other information you have about the patient


Patient is terminal. Palliative care has been prescribed.
She has been referred from the hospital to hospice for the continuation of the palliative
care. She cannot eat or drink very well. Her list of medications can be found in the hospital
handover inside the cubicle.

What you must do


Write down the prescription for the above medications.

Special note
None

Hospital Handover Note

Patient name: Victoria Yates


Date of Birth: 25/03/1942
NHS Number: 123456
Hospital No: 1234
Allergy: Penicillin
Reaction: Breathlessness

Dear Doctor,
Mrs Victoria Yates, aged 80, has been admitted in the medical department with pancreatic
cancer. She is terminally ill and on palliative care. She cannot eat or drink well.

Plan: She will be discharged to hospice for palliative care with the following medications.
She is prescribed morphine via syringe driver 30mg per 24 hours.

Other medications:
Morphine for breakthrough pain/SC
Cyclizine 50mg TDS SC indication NV
Midazolam 2.5mg SC 2 Hourly (max 6 doses) indication for agitation.
Hyoscine bromide 400micrograms SC 4 Hourly (Max 2.4mg) indication for secretion.
PCM 1g PO every 4-6 hours (max 4g/day for pain)

www.aspire2plab.com 392
No real PLAB2 cases discussed. All discussed scenarios are fictitious and for educational purposes only.
Community Acquired Pneumonia

Where you are


You are an FY2 in A&E.

Who the patient is


Mr Jamie Strauss, aged 55, was admitted with breathlessness and cough.
He was diagnosed with community acquired pneumonia in the hospital.
Consultant has reviewed the patient and planned to start him on clarithromycin 500mg PO
BD for 5 days.

Other information you have about the patient


Other regular medications:
Salbutamol 100 - 200 microgram (1-2 puff PRN) up to 4 times a day.
Seretide 250 Evohaler 1 puff BD
Ipratropium Bromide 20-40 microgram (1-2 puffs BD)
Prednisolone 30mg PO OD 7 days
Aspirin 75mg PO OD
Ramipril 2.5mg PO OD
Levothyroxine 50 microgram PO OD
Atorvastatin 20mg PO OD

What you must do


Write down the prescription for the above medications.

Special note
None

Clarithromycin is predicted to increase the exposure to atorvastatin.


Advised to avoid or adjust dose and monitor rhabdomyolysis.

www.aspire2plab.com 393
No real PLAB2 cases discussed. All discussed scenarios are fictitious and for educational purposes only.
Non-Valvular Atrial Fibrillation

Where you are


You are an FY2 in Medicine.

Who the patient is


Mrs Andrea Brighton, aged 81, was diagnosed with non-valvular atrial fibrillation.

Other information you have about the patient


Patient had CABG 10 years ago.
eGFR – 31
Patient was prescribed apixaban and atenolol.

What you must do


Write down the prescription for the above medications.

Special note
None

www.aspire2plab.com 394
No real PLAB2 cases discussed. All discussed scenarios are fictitious and for educational purposes only.
Find the information using the BNF

Atenolol

For people with AF alone


The usual dose is 50–100 mg a day.

With oral use


Max. 50 mg daily if eGFR 15–35 mL/minute/1.73 m2; max. 25 mg daily or 50 mg on alternate
days if eGFR less than 15 mL/minute/1.73 m2.

Apixaban

Prophylaxis of stroke and systemic embolism in non-valvular atrial fibrillation and at least
one risk factor (such as previous stroke or transient ischaemic attack, symptomatic heart
failure, diabetes mellitus, hypertension, or age 75 years and over)

By mouth
For Adult
5 mg twice daily, alternatively 2.5 mg twice daily, reduced dose used in patients with at least
two of the following characteristics:
- age 80 years and over,
- body weight 60 kg or less,
- serum creatinine 133 micromol/litre and over.

Reduce dose to 2.5 mg twice daily if creatinine clearance 15–29 mL/minute.

www.aspire2plab.com 395
No real PLAB2 cases discussed. All discussed scenarios are fictitious and for educational purposes only.
Drug interaction (Lithium)

Where you are


You are an FY2 in Psychiatry ward.

Who the patient is


Mr Robert Washington, aged 50, has hurt his ankle and is in severe pain.

Other information you have about the patient


He has been diagnosed with COPD.
Regular medications:
Ipratropium Bromide 20-40 microgram (1-2 puffs BD)
Atorvastatin 20mg PO OD
Lithium

What you must do


Nurse has asked you to give ibuprofen 400mg BD.

Special note
Patient is allergic to penicillin. After taking penicillin patient developed rash.

NB.
1. Ibuprofen increases the concentration of lithium. Advises monitor and adjust dose.
2. Both lithium and tramadol can increase the risk of serotonin syndrome.
3. Salbutamol and Salmeterol is predicted to cause hypokalaemia (potentially
increasing the risk of torsade de pointes) when given with lithium.

www.aspire2plab.com 396
No real PLAB2 cases discussed. All discussed scenarios are fictitious and for educational purposes only.
UTI (Nitrofurantoin)

Where you are


You are an FY2 in Medicine.

Who the patient is


Miss Rachel Trunk, aged 30, has come to you with burning micturition.

Other information you have about the patient


She was prescribed nitrofurantoin for suspected UTI.
Other medications:
- Amlodipine 10mg OD,
- PCM PRN max 4 grams

What you must do


Write down the prescription for the above medications.
Check dose and write down the antibiotic.

Special note
Patient is allergic to penicillin. After taking penicillin patient developed rash.
Antibiotic may be changed after urine culture.

www.aspire2plab.com 397
No real PLAB2 cases discussed. All discussed scenarios are fictitious and for educational purposes only.
Apixaban (1)

Treatment of deep-vein thrombosis, Treatment of pulmonary embolism


By mouth
For Adult
Initially 10 mg twice daily for 7 days, then maintenance 5 mg twice daily.

Prophylaxis of recurrent deep-vein thrombosis, Prophylaxis of recurrent pulmonary


embolism
By mouth
For Adult
2.5 mg twice daily, following completion of 6 months anticoagulant treatment.

www.aspire2plab.com 398
No real PLAB2 cases discussed. All discussed scenarios are fictitious and for educational purposes only.
Apixaban (2)

Where you are


You are an FY2 in Medicine.

Who the patient is


Miss Hannah Knowles, aged 60, has been admitted to the hospital because of
breathlessness.
She was diagnosed with pulmonary embolism and was managed in the hospital.

Other information you have about the patient


She is a diagnosed case of hypercholesterolemia.
She had a long-haul flight 10 days ago.
She was prescribed the following medications:
- Apixaban for 6 months
- Atorvastatin 20mg OD
- Aspirin 75mg OD
eGFR - 87
D-Dimer – increased

What you must do


Write down the prescription for the above medications.
Check dose and write down the apixaban.

Special note
Patient is allergic to codeine. She gets itching after taking codeine.

www.aspire2plab.com 399
No real PLAB2 cases discussed. All discussed scenarios are fictitious and for educational purposes only.
Peritonsillar Abscess

Where you are


You are an FY2 in Medicine.

Who the patient is


Miss Tracy Yates, aged 6 years, has been diagnosed with peritonsillar abscess (quinsy).
She was managed in the ER.
There are no signs of dehydration.
Weight of the child is 23kg

Other information you have about the patient


Consultant has advised the following medications.
Phenoxymethylpenicillin
Metronidazole
IV Fluids maintenance.

What you must do


Write down the prescription for the above medications.
Check doses with BNF.
Calculate the fluid dose.

Special note
Patient is allergic to clarithromycin. Patient had itching.

www.aspire2plab.com 400
No real PLAB2 cases discussed. All discussed scenarios are fictitious and for educational purposes only.
Metronidazole

For Child 3–6 years


100 mg every 12 hours for 3–7 days.

Phenoxymethylpenicillin

For Child 6–11 years


250 mg 4 times a day, alternatively 500 mg twice daily for 5–10 days.

www.aspire2plab.com 401
No real PLAB2 cases discussed. All discussed scenarios are fictitious and for educational purposes only.
Acute Pyelonephritis

Where you are


You are an FY2 in Medicine.

Who the patient is


Miss Sally Rude, aged 80, has been diagnosed with acute pyelonephritis.
She has a history of Rheumatoid Arthritis.
She has hypertension.
Weight of the patient is 65kg.

Other information you have about the patient


eGFR is 40.
Regular Medications:
Amlodipine 10mg OD
Methotrexate 7.5mg once weekly every Tuesday
Folic Acid 5mg daily except Tuesday

What you must do


Consultant has requested you to start her on Cefalexin.

Special note
Patient is allergic to clarithromycin. Patient had itching.

www.aspire2plab.com 402
No real PLAB2 cases discussed. All discussed scenarios are fictitious and for educational purposes only.
Cefalexin

For Adult
500 mg 2–3 times a day for 7 to 10 days; increased if necessary to 1–1.5 g 3–4 times a day,
increased dose used in severe infections.

www.aspire2plab.com 403
No real PLAB2 cases discussed. All discussed scenarios are fictitious and for educational purposes only.
Cellulitis (Prescription)

Where you are


You are an FY2 in Medicine.

Who the patient is


Mr Rio Fernandes, aged 50, has been admitted due to cellulitis (diabetic foot).

Other information you have about the patient


Patient has been diagnosed with diabetes 5 years ago.
He is on Metformin 500mg TDS.

What you must do


Consultant has requested you to start her on Co-trimoxazole with Gentamicin or
Metronidazole.

Special note
Patient is allergic to Penicillin. Patient had itching.

www.aspire2plab.com 404
No real PLAB2 cases discussed. All discussed scenarios are fictitious and for educational purposes only.
Alternative in penicillin allergy: co-trimoxazole [unlicensed] with or without intravenous
gentamicin and/or metronidazole.

Co-trimoxazole
By mouth
For Adult
960 mg twice daily.

By intravenous infusion
For Adult
960 mg every 12 hours, increased if necessary to 1.44 g every 12 hours.

Gentamicin
By intravenous infusion

For Adult
Initially 5–7 mg/kg once daily, subsequent doses adjusted according to serum-gentamicin
concentration.

Metronidazole
For Adult
400 mg every 8 hours for 7 days then review.

By intravenous infusion

For Adult
500 mg every 8 hours.

www.aspire2plab.com 405
No real PLAB2 cases discussed. All discussed scenarios are fictitious and for educational purposes only.
Facebook Page: Aspire Education

Facebook Group: PLAB2 by Dr Ankur Garg

YouTube: AnkurGargAspire

Instagram: aspire2education

Twitter: meankurgarg

E-mail: info@aspire2plab.com

Website: www.aspire2plab.com

www.aspire2plab.com 406
No real PLAB2 cases discussed. All discussed scenarios are fictitious and for educational purposes only.

You might also like